Sunteți pe pagina 1din 112

Law on NatRes (23-49) 1

SUSANA MENGUITO, EMELITA MENGUITO-MANALILI, 4. For Lot 6045-G -- in the name of Generoso Menguito, of
HELEN MARTA MENGUITO-LUNA, RENATO legal age, Filipino citizen, single, with residence and postal
MENGUITO, BERSAMIN MENGUITO, FROILAN address at T. Sulit, St., Pateros, Metro Manila;
MENGUITO and GENEROSO MENGUITO, petitioners,
vs. REPUBLIC OF THE PHILIPPINES, respondent.; 5. For Lot 6045-H -- in the name of Helen Marta Menguito,
G.R. No. 134308; December 14, 2000; PANGANIBAN, of legal age, Filipino citizen, single, with residence and
J.: postal address at T. Sulit, St., Pateros, Metro Manila;

Unless a piece of public land is shown to have been 6. For Lot 6046-I -- in the name of Froilan Menguito, of
classified as alienable and disposable, it remains part of legal age, Filipino citizen, married to Zenaida Carag, with
the inalienable public domain. Even assuming that such residence and postal address at T.Sulit St., Pateros, Metro
land has been classified as alienable, title thereto can be Manila;
registered only upon presentation of incontrovertible proof
of adverse, notorious and open possession in the concept 7. For Lot 6045-J -- in the name of Emelita Menguito, of
of owner for a period of thirty years. legal age, Filipino citizen, married to Luciano Manalili, with
residence and postal address at T. Sulit, St., Pateros,
The Case Metro Manila; and

Before us is a Petition for Review under Rule 45 of the 8. For Lot 6045-K -- in the name of Generoso Menguito, of
Rules of Court assailing the September 30, 1997 legal age, Filipino citizen, married to Luciano Manalili; and
Decision1 and the June 23, 1998 Resolution2 of the Court Froilan Menguito, of legal age, Filipino citizen, married to
of Appeals (CA) in CA-GR CV No. 39638. The decretal Zenaida Carag, all with residence and postal address at T.
portion of said Decision reads as follows: Sulit St., Pateros, Metro Manila.

"WHEREFORE, the decision appealed from is hereby Upon the finality of this Decision, let an Order be issued to
REVERSED and SET ASIDE. Accordingly, the appellees the Commissioner of Land Registration Authority for the
application for registration is hereby DISMISSED."3 issuance of the decree of registration and the
corresponding certificates of title in favor of the applicants
The Decision of the Regional Trial Court (RTC) of Pasig pursuant to Section 39 of PD No. 1529.
City (Branch 157),4 which was reversed by the appellate
court, granted petitioners application for registration in this SO ORDERED."
wise:5
The Facts
"WHEREFORE, the order of general default against the
whole world heretofore entered in this case is affirmed, The antecedents of the case are adequately summarized
and judgment is hereby rendered confirming the by the Court of Appeals as follows:
registerable title of the applicants to the land described in
"On November 10, 1987, in the Regional Trial Court at
their application under plan Swo-13-000227 and its
Pasig, Metro Manila an Application for Registration of Title
technical descriptions, situated in the Barrio of Ususan,
was filed by the following successors-in-interest of the
Municipality of Taguig, Metro Manila, and containing an
deceased spouses Cirilo Menguito and Juana Manalo-
aggregate area of 2,112 square meters; and individual and
Menguito, namely: SUSANA MENGUITO, EMELITA
separate certificates of titles to the lots comprising the said
MENGUITO-MANALILI, HELEN MARTA MENGUITO-
land are hereby ordered registered in the names of the
LUNA, RENATO MENGUITO, BERSAMIN MENGUITO,
applicants, as follows:
FROILAN MENGUITO and GENEROSO MENGUITO.
1. For lots 6045-A, 6045-B, 6045-C, and 6045-D in the Docketed in the said court as LRC Case No. N-10938, the
name of Susana Menguito, of legal age, widow, Filipino application reads:
citizen, with residence and postal address at T. Sulit, St.,
APPLICATION FOR REGISTRATION OF TITLE
Pater[o]s, Metro Manila;
The above-named applicants hereby apply to have the
2. For Lot 6045-E -- in the name of Renato Menguito, of
land hereinafter described brought under the operation of
legal age, married to Irene Toledo, Filipino citizen, with
the Land Registration Act as amended by the Property
residence and postal address at T. Sulit, St., Pateros,
Registration Decree No. 1529 and to have their title
Metro Manila;
thereto registered and confirmed,
3. For Lot 6045-F -- in the name of Bersamin Menguito, of
AND DECLARE:
legal age, Filipino citizen, single, with residence and postal
address at T. Sulit, St., Pateros, Metro Manila; 1. That the applicants are the owners in fee simple of
eleven (11) parcels of land situated in the Barrio of
Ususan, Municipality of Taguig, Metro Manila, and are
Law on NatRes (23-49) 2
bounded and described as shown on plan Swo-13-000227 8. That should the Land Registration Act invoked be not
(lot Nos. 6045-A, 6045-B, 6045-C, 6045-D, 6045-E, 6045- applicable in the instant case, the applicants hereby apply
F, 6045-G, 6045-H, 6045-I, 6045-J and 6045-K) and for the benefit of Chapter VIII of Commonwealth Act No.
corresponding technical descriptions, x x x; 141 as amended;

2. That said parcels of land are assessed for taxation for 9. That the following documents are attached hereto and
the current year at P5,910.00 as per Tax Declaration No. made part hereof:
B-11-01351 of the land record of Taguig, Metro Manila;
(a) Tracing cloth plan of Swo-13-000227
3. That to the best of applicants knowledge and belief, (b) Two (2) print copies of said plan Swo-13-000227
there is no mortgage or encumbrance of any kind (c) Three (3) copies each of the Technical Description of:
whatsoever affecting the said land nor any other persons Lot 6045-A
having any estate or interest therein, legal or equitable, in Lot 6045-B
possession, remainder, reversion or expectancy; Lot 6045-C
Lot 6045-D
4. That the applicants acquired the said parcels of land by Lot 6045-E
inheritance; Lot 6045-F
Lot 6045-G
5. That said parcels of land are occupied by the applicants Lot 6045-H
and their predecessors-in-interest have been in actual, Lot 6045-I
open, peaceful, continuous, and adverse possession, in Lot 6045-J
the concept of owners, of said parcels of land for more Lot 6045-K
than thirty years; (d) Three (3) copies of Engineers Certificate

6. That the names in full and addresses as far known to (e) Four (4) copies of Tax Declaration No. B-011-01351
the undersigned, of the owners of all adjoining properties
are as follows: xxx xxx xxx

a) Pilar Menguito (Amended Record on Appeal, pp. 1-5).


Pateros-Taguig Road
Ususan, Taguig "Acting on the foregoing application, the lower court issued
Metro Manila a Notice of Initial Hearing addressed to: the Solicitor
b) Andres Filemon General, the Director of the Land Management Bureau,
Pateros-Taguig Road the Secretary of the Department of Public Works and
Ususan, Taguig Highways, the Secretary of the Department of Agrarian
Metro Manila Reform, the Director of the Bureau of Forest Development,
c) Beatriz Dumagat and the owners of the adjacent properties as mentioned in
Pateros-Taguig Road the application, informing them that the application is
Ususan, Taguig scheduled for initial hearing on April 25, 1989. The
Metro Manila addressees were then ordered to present such claims as
d) Maura Cabanatan you may have to said lands or any portion thereof, and to
Pateros-Taguig Road submit evidence in support of such claims and unless you
Ususan, Taguig appear at said court at the time and place aforesaid, your
Metro Manila default will be recorded and the title to the lands will be
e) Pateros-Taguig Road adjudicated and determined in accordance with law and
c/o The District Engineer the evidence before the Court, and thereafter, you will
forever be barred from contesting said application or any
Pasig, Metro Manila decree entered thereon (Exhibit A).

7. That the applicants full name, age, citizenship, "Said notice of initial hearing was published in the April 5,
residence, and postal address, are as follows: 1989 issue of Abante, a daily tabloid (Exhs. C, C-1, C-1-
A).
SUSAN MENGUITO, widow; EMELITA M. MANALILI,
married to Luciano Manalili; HELEN MARTA M. LUNA, "Earlier, or on March 30, 1989, the Republic of the
married to Benjamin Luna, Jr.; RENATO MENGUITO, Philippines, through the Solicitor General, filed its
married to Irene Toledo; BERSAMIN MENGUITO, married Opposition to the application for registration contending:
to Elvira Salvacion; FROILAN MENGUITO, married to
Zenaida Carag; and GENEROSO MENGUITO, single; all 1. That neither the applicant nor his predecessors-in-
of legal age, Filipinos, and with residence and postal interest have been in open, continuous, exclusive and
address at T. Sulit St., Pateros, Metro Manila. notorious possession and occupation of the land in
Law on NatRes (23-49) 3
question since June 12, 1945 or prior thereto (Sec. 48 [b], It interposes no objection to the admission of Exhibits A,
C.A. 141, as amended by P.D. 1073). B, C, D, relative to jurisdictional requirements. It has no
objection to Exhibits E, F, F-1, to F-10 relating to the
2. That the muniments of title and tax payment receipts of plan and the technical description of the lots being applied
applicant, if any, attached to or alleged in the application, for and Exhibit G which is the Engineers certificate.
do not constitute competent and sufficient evidence of a
bona fide acquisition of the lands applied for or his open, It objects to Exhibits H, H-1 to H-2 the extrajudicial
continuous, exclusive and notorious possession and settlement and partition dated December 12, 1985 for
occupation thereof in the concept of owner, since June 12, being self serving. It objects to Exhibits I, J, K, L, M
1945, or prior thereto. Said muniments of title do not and N for being incompetent and insufficient proof of
appear to be genuine and indicate the pretended possession of the lot in question by applicants or their
possession of applicant to be of recent vintage. predecessors-in interest. In fact the said tax declarations
do not date back to at least June 12, 1945. It objects to
3. That the claim of ownership in fee simple on the basis of Exhibits O, P, Q, and R, the same being incompetent
Spanish title or grant can no longer be availed of by the and insufficient to prove possession since June 12, 1945.
applicant who has failed to file an appropriate application It objects to Exhibits O, P, Q, and R, the same being
for registration within the period of six (6) months from incompetent and insufficient to prove possession since
February 16, 1976 as required by Presidential Decree No. June 12, 1945. It objects to Exhibit S as being self-
892. From the records, it appears that the instant serving being a mere photocopy of the alleged Kasulatan
application was filed on July 31, 1990. ng Pagkakaloob dated May 7, 1989 executed by Cirilo
Menguito the same cannot be accepted in evidence,
4. That the parcel applied is part of the public domain applicants not having first laid the basis for the
belonging to the Republic of the Philippines not subject to presentation of secondary evidence. It objects to the first
private appropriation. (Amended Record on Appeal, pp. 5- page of Exhibit T, being self-serving and a mere
6). photocopy. Furthermore, page 2 of said exhibit, where the
supposed acknowledgment of the instrument appears,
"The Solicitor General therefore prayed for the denial of
refers to different parcels of land other than those being
the application for registration and for the declaration of
applied for.
the properties subject thereof as part of the public domain
belonging to the Republic of the Philippines. WHEREFORE, considering that the applicants have failed
to prove their title to the lands applied for, it is respectfully
"At the scheduled initial hearing of the case on April 25,
prayed that the application for registration be denied and
1989, a certain Jose Tangco, Jr. appeared and registered
that the land applied for be declared as part of the public
a verbal opposition to the application. On motion of
domain belonging to the Republic of the Philippines.
counsel for the applicants, the court issued an Order of
General Default against the whole world, except as Considering the above, oppositor respectfully manifests
against the oppositors Republic of the Philippines and that there is no need for it to submit evidence in support of
Jose Tangco, Jr., who was directed to file his written its opposition. (Amended Record on Appeal, pp. 11-13).
opposition but never did. Thereafter, trial on the merits
ensued. "On May 15, 1991, the lower court rendered its decision
disposing as follows:
"On June 13, 1990, the applicants filed their Formal Offer
of Evidence, submitting therewith the following WHEREFORE, the order of general default against the
documentary exhibits: (1) Plan Swo-13-000227 (Exh. F); whole world heretofore entered in this case is affirmed,
(2) technical descriptions of Lot Nos. 6045-A to 6045-J, and judgment is hereby rendered confirming the
inclusive (Exhs. F to F-10, inclusive); (3) Engineers registerable title of the applicants x x x
Certificate (Exh. G); (4) Extra-judicial Settlement and
Partition executed by the applicants dated December 12, "On June 11, 1991, the oppositor Republic, through the
1985 (Exh. H); (5) description of the land and the Solicitor General, moved for a reconsideration of the afore-
apportionment thereof among the applicants (Exhs.H-1 quoted decision, to which a written opposition was
and H-2, respectively); (6) Tax Declarations (Exhs. I, J, interposed by the applicants.
K, L, M, N and O) (7) Tax Receipts (Exhs. O, O-1,
P. P-1, Q and R); (8) Kasulatan ng Pagkakaloob dated "On July 8, 1991, the lower court issued an order denying
May 7, 1969 executed by Cirilo Menguito in favor of Pedro the motion for reconsideration for lack of merit."
Menguito (Exh. S); and (9) Deed of Partition dated
November 7, 1990 executed by the applicants (Exh. T). Ruling of the Court of Appeals

"On September 12, 1990, the oppositor Republic filed its The Court of Appeals agreed with respondent that the
Manifestation and Opposition to applicants formal offer of lower court had failed to consider the legal requirements
evidence. The said manifestation reads: for registration of imperfect titles; namely: (1) the land is
Law on NatRes (23-49) 4
alienable and disposable; and (2) the applicants and their Hence, as observed by the appellate court, petitioners
predecessors-in-interest have occupied and possessed were duty-bound to prove two legal requirements: (1) the
the land openly, continuously, exclusively, and adversely land applied for was alienable and disposable; and (2) the
since June 12, 1945. It was not convinced that the land in applicants and their predecessors-in-interest had occupied
question had been classified as alienable or disposable and possessed the land openly, continuously, exclusively,
and that petitioners or their predecessors-in-interest had and adversely since June 12, 1945.
been in possession of it since June 12, 1945.
The records show that petitioners failed to establish these
Hence, this Petition.7 two requisites.

The Issue Classification of the Land

In their Memorandum, petitioners submit a single issue for To prove that the land in question formed part of the
our consideration: alienable and disposable lands of the public domain,
petitioners relied on the printed words which read: "This
"Whether or not the court a quo erred in reversing the survey plan is inside Alienable and Disposable Land Area,
findings of facts of the trial court." Project No. 27-B as per L.C. Map No. 2623, certified by
the Bureau of Forestry on January 3, 1968," appearing on
In fine, the Court will resolve whether the CA erred in Exhibit "E" (Survey Plan No. Swo-13-000227).
rejecting petitioners application for the registration of their
respective titles. This proof is not sufficient. Section 2, Article XII of the
1987 Constitution, provides: "All lands of the public
The Courts Ruling domain, waters, minerals, coal, petroleum, and other
mineral oils, all forces of potential energy, fisheries, forests
The Petition is devoid of merit.
or timber, wildlife, flora and fauna, and other natural
Sole Issue: Registration of Petitioners Titles resources are owned by the State. x x x." (Emphasis
supplied.)
Section 48 of Commonwealth Act (CA) No. 141,9 as
amended, provides for the registration of imperfect titles to For the original registration of title, the applicant
lands of the public domain in this wise: (petitioners in this case) must overcome the presumption
that the land sought to be registered forms part of the
"SECTION 48. The following described citizens of the public domain.12 Unless public land is shown to have
Philippines, occupying lands of public domain or claiming been reclassified or alienated to a private person by the
to own any such lands or an interest thereon, but whose State, it remains part of the inalienable public domain.
titles have not been perfected or completed, may apply to Indeed, "occupation thereof in the concept of owner, no
the Court of First Instance of the province where the land matter how long, cannot ripen into ownership and be
is located for confirmation of their claims, and the issuance registered as a title."13 To overcome such presumption,
of a certificate of title therefor, under the Land Registration incontrovertible evidence must be shown by the
Act, to wit: applicant.14 Absent such evidence, the land sought to be
registered remains inalienable.
xxx xxx xxx
In the present case, petitioners cite a surveyor-geodetic
(b) those who by themselves or through their predecessor engineers notation in Exhibit "E" indicating that the survey
in-interest have been in open, continuous, exclusive and was inside alienable and disposable land. Such notation
notorious possession and occupation of agricultural lands does not constitute a positive government act validly
of the public domain, under a bona fide claim of acquisition changing the classification of the land in question. Verily, a
or ownership, for at least thirty years immediately mere surveyor has no authority to reclassify lands of the
preceding the filing of the application for confirmation of public domain. By relying solely on the said surveyors
title except when prevented by war or force majeure. They assertion, petitioners have not sufficiently proven that the
shall be conclusively presumed to have performed all the land in question has been declared alienable.
conditions essential to a Government grant and shall be
entitled to a certificate of title under the provisions of this Period of Possession
Chapter."
Even assuming arguendo that petitioners have been able
Presidential Decree (PD) No. 107310 clarified paragraph to prove that the land is alienable, their Petition for
"b" of the said provision by specifically declaring that it confirmation of their imperfect titles and registration
applied only to alienable and disposable lands of the thereof under the law will still be denied. The reason is that
public domain.11 they have failed to establish possession of the lots in
question -- openly, continuously, exclusively and adversely
Law on NatRes (23-49) 5
-- in the concept of owner for at least 30 years, since June policy of encouraging and promoting the distribution of
12, 1945. alienable public lands to spur economic growth and remain
true to the ideal of social justice, our hands are tied by the
Petitioners do not claim that they are the original laws stringent safeguards against registering imperfect
possessors of the lots in question, which had allegedly titles. In this case, we agree with the CA that petitioners
belonged to Cirilo Menguito before he donated it to his son have not presented sufficient proof of their compliance
Pedro. When Pedro died in 1978, these lots allegedly with the legal requirements for registration of imperfect
passed down to petitioners. titles.

Although petitioners can trace their possession of the land WHEREFORE, the Petition is DENIED and the assailed
from as far back as 1968 only, they would tack it to that of Decision AFFIRMED. Costs against petitioners. SO
their predecessors, who had supposedly been in ORDERED.
possession thereof even before the Second World War.
There is not enough convincing proof, however, to support G.R. No. L-14722 May 25, 1960
such claim.
IGNACIO MESINA, plaintiff-appellant, vs. EULALIA
Petitioners presented evidence that they had been paying PINEDA VDA. DE SONZA, ET AL., defendants.
real estate taxes since 1974.15 Their predecessors-in- EULALIA PINEDA VDA. DE SONZA, defendant-
interest, they claimed, have also been paying taxes on the appellee.
land for several years before them, and Cirilo Menguito
had declared the land for tax purposes in 1943.16 BAUTISTA ANGELO, J.:
However, they did not present any documents or any other
satisfactory proof to substantiate this claim. General Plaintiff brought this action before the Court of First
statements, which are mere conclusions of law and not Instance of Nueva Ecija praying that Original Certificate of
proofs of possession, are unavailing and cannot suffice. Title No. P-1137 of the Register of Deeds of Nueva Ecija
be ordered cancelled and that the registration case
Cirilos six children were not presented as witnesses by pending before the same court covering the property
petitioners during the hearing of their application for described therein be given due course and that
registration of the lots in question. In fact, of the six defendants be ordered to pay plaintiff P1,000.00 as
children, only Pilar Menguito was personally informed of attorney's fees and costs.
petitioners application. Still, she was not presented as a
witness.1wphi1 Defendants filed a motion to dismiss on the ground that
plaintiff's action is already barred by the statute of
There can be no question that Cirilos children were the limitations. The reasons advanced are: the complaint was
best witnesses, because they could have substantiated filed on March 25, 1958. The decree of registration or
petitioners claim that indeed the lots in question had been issuance of patent over the property was issued
donated to Pedro Menguito. Moreover, they may even "sometime on September 12, 1953 or thereabout", while
have in their possession documents that can adequately the transfer certificate of title covering the same was
support their supposed claim. Instead, petitioners issued on September 16, 1953. The present action which
presented only Raymunda Bautista, the alleged tenant of calls for the cancellation of said decree and title has,
Cirilo Menguito, who had tilled the land before petitioners therefore, been filed after the elapse of more than four
built their houses thereon. Neither Cirilos children nor the years, which cannot be done, because the title has already
documents that they might have had in their possession become indefeasible and incontrovertible. The court
were presented. sustained this motion and dismissed the complaint. Hence
the present appeal.
Furthermore, serious doubts are cast on petitioners claim
that their predecessors-in-interest have been in open, Plaintiff claims that he is the owner in fee simple of Lot No.
continuous, exclusive and adverse possession and 3259, with improvements thereon, situated in San Antonio,
occupation of the land. Because they are of recent Nueva Ecija; that he has been in actual possession thereof
vintage, the tax declarations (Exhs. "I" to "N"), tax receipts since 1914, publicly, openly, peacefully and against the
(Exhs. "O". "O1", "P", and "P-1") and the Municipal whole world and up to the present time he is the only one
Treasurers certifications of tax payments (Exhs. "Q" and who benefits from the produce thereof; that said lot is at
"R") presented in evidence are incompetent and present the subject of registration proceedings pending in
insufficient to prove petitioners and their predecessors-in- the same court known as Registration Case No. N-372,
interests possession of the lots in question. L.R.C. Cad. Record No. N-12238; that sometime in
September 12, 1953, the Director of Lands, without
Because the factual findings of the trial and the appellate exercising due care, and in spite of his knowledge that
courts were contrary to each other, we waded into the defendants had not complied with the knowledge that
records,18 but found no reason to modify the assailed CA defendants had not complied with the requirements of
Decision. Much as we want to conform to the States Commonwealth Act No. 141, issued a homestead patent
Law on NatRes (23-49) 6
in their favor as a consequence of which a certificate of requirements for a grant by the Government were
title was issued in their name by the register of deeds; that complied with, for he has been in actual and physical
said title was procured by defendants through frauds, possession, personally and through his predecessors, of
deception and misrepresentation since they knew that the an agricultural land of the public domain openly,
lot belonged to the plaintiff; and that the Director of Lands continuously, exclusively and publicly since July 26, 1894,
has no authority nor jurisdiction to issue a patent covering with a right to a certificate of title to said land under the
said land because it is a private property of plaintiff. For provisions of Chapter VIII of said Act. So that when Angela
these reasons, plaintiff prays that said decree and title be Razon applied for the grant in her favor, Valentin Susi had
cancelled. already acquired, by operation of law, not only a right to
grant, but a grant of the Government, for it is not
Republic Act No. 1942, which took effect on June 22, 1957 necessary that certificate of title should be issued in order
(amending Section 48-b of Commonwealth Act 141), that said grant may be sanctioned by the courts, an
provides: application therefor is sufficient, under the provisions of
section 47 of Act No. 2874. If by a legal fiction, Valentin
(b) Those who by themselves or through their Susi had acquired the land in question by a grant of the
predecessors in interest have been in open, continuous, State, it had already ceased to be of the public domain
exclusive and notorious possession and occupation of and had become private property, at least by presumption,
agricultural lands of the public domain, under a bona fide of Valentin Susi, beyond the control, of the Director of
claim of acquisition of ownership, for at least thirty years Lands. Consequently, in selling the land in question to
immediately preceeding the filing of the application for Angela Razon, the Director of Lands disposed of a land
confirmation of title except when prevented by war or force over which he had no longer any title or control, and the
majeure. These shall be conclusively presumed to have sale thus made was void and of no effect, and Angela
performed all the conditions essential to a Government Razon did not thereby acquire any right. (Emphasis
grant and shall be entitled to a certificate of title under the supplied)
provisions of this chapter.
Such is the situation in which the plaintiff claims to be in
In the case of Susi vs. Razon, et al., 48 Phil., 424, it was his complaint. He alleges that he is the owner in fee simple
observed that where all the necessary requirements for a of the lot in question, with the improvements thereon,
grant by the Government are complied with through actual situated in San Antonio, Nueva Ecija, and that he has
physical possession openly, continuously, and publicly, been in actual possession thereof since 1914, publicly,
with a right to a certificate of title to said land under the openly, peacefully and against the whole world, and that
provisions of Chapter VIII of Act No. 2874, amending Act up to the present time he is the only one who benefits from
No. 926 (carried over as Chapter VIII of Commonwealth the produce thereof. He further claims that said lot is
Act No. 141), the possessor is deemed to have already present the subject of a registration proceeding pending in
acquired by operation of law not only a right to a grant, but the same court, known as Registration Case No. N-372,
a grant of the Government, for it is not necessary that a L.R.C. Cad. Record No. N-12238. If by legal fiction, as
certificate of title be issued in order that said grant may be stated in the Susi case, plaintiff is deemed to have
sanctioned by the court an application therefor being acquired the lot by a grant of the State, it follows that the
sufficient under the provisions of Section 47 of Act No. same had ceased to be part of the public domain and had
2874 (reproduced as Section 50, Commonwealth Act No. become private property and, therefore, is beyond the
141). Thus, the following is what this Court said on the control of the Director of Lands. Consequently, the
matter: homestead patent and the original certificate of title
covering said lot issued by the Director of Lands in favor of
It clearly appears from the evidence that Valentin Susi has
the defendants can be said to be null and void, for having
been in possession of the land in question openly,
been issued through fraud, deceit and misrepresentation.
continuously, adversely and publicly, personally and
through his predecessors, since the year 1880, that is, for Considering that this case was dismissed by the trial court
about forty-five years. ... When on August 15, 1914, merely on a motion to dismiss on the ground that plaintiff's
Angela Razon applied for the purchase of said land, action is already barred by the statute of limitations, which
Valentin Susi had already been in possession thereof apparently is predicated on the theory that a decree of
personally and through his predecessors for thirty-forty registration can no longer be impugned on the ground of
years. And if it is taken into account that Nemesio Pinlac fraud one year after the issuance and entry of the decree,1
had already made said land a fish pond when he sold it on which theory does not apply here because the property
December 13, 1880, it can hardly be estimated when he involved is allegedly private in nature and has ceased to
began to possess and occupy it, the period of time being be part of the public domain, we are of the opinion that the
so long that it is beyond the reach of memory. ... In favor trial court erred in dismissing the case outright without
of Valentin Susi, there is, moreover the presumption juris giving plaintiff a chance to prove his claim. It would have
et de jure established paragraph (b) of section 45 of Act been more proper for the court to deny the motion on the
No. 2874, amending Act No. 926, that all the necessary ground that its object does not appear to be indubitable,
Law on NatRes (23-49) 7
rather than to have dismissed it, as was done by the trial it to Valentin Susi for the sum of P12, reserving the right to
court. repurchase it (Exhibit A). Before the execution of the deed
of sale, Valentin Susi had already paid its price and sown
Wherefore, the order appealed from is set aside. The case "bacawan" on said land, availing himself of the firewood
is remanded to the trial court for further proceedings. No gathered thereon, with the proceeds of the sale of which
costs. he had paid the price of the property. The possession and
occupation of the land in question, first, by Apolonio
VALENTIN SUSI, plaintiff-appellee,vs. ANGELA Garcia and Basilio Mendoza, and then by Valentin Susi
RAZON and THE DIRECTOR OF LANDS, defendants. has been open, continuous, adverse and public, without
THE DIRECTOR OF LANDS, appellant.; G.R. No. L- any interruption, except during the revolution, or
24066; December 9, 1925; VILLA-REAL, J.: disturbance, except when Angela Razon, on September
13, 1913, commenced an action in the Court of First
This action was commenced in the Court of First Instance Instance of Pampanga to recover the possession of said
of Pampanga by a complaint filed by Valentin Susi against land (Exhibit C), wherein after considering the evidence
Angela Razon and the Director of Lands, praying for introduced at the trial, the court rendered judgment in favor
judgment: (a) Declaring plaintiff the sole and absolute of Valentin Susi and against Angela Razon, dismissing the
owner of the parcel of land described in the second complaint (Exhibit E). Having failed in her attempt to obtain
paragraph of the complaint; (b) annulling the sale made by possession of the land in question through the court,
the Director of Lands in favor of Angela Razon, on the Angela Razon applied to the Director of Lands for the
ground that the land is a private property; (c) ordering the purchase thereof on August 15, 1914 (Exhibit C). Having
cancellation of the certificate of title issued to said Angela learned of said application, Valentin Susi filed and
Razon; and (d) sentencing the latter to pay plaintiff the opposition thereto on December 6, 1915, asserting his
sum of P500 as damages, with the costs. possession of the land for twenty-five years (Exhibit P).
After making the proper administrative investigation, the
For his answer to the complaint, the Director of Lands Director of Lands overruled the opposition of Valentin Susi
denied each and every allegation contained therein and, and sold the land to Angela Razon. By virtue of said grant
as special defense, alleged that the land in question was a the register of deeds of Pampanga, on August 31, 1921,
property of the Government of the United States under the issued the proper certificate of title to Angela Razon.
administration and control of the Philippine Islands before Armed with said document, Angela Razon required
its sale to Angela Razon, which was made in accordance Valentin Susi to vacate the land in question, and as he
with law. refused to do so, she brought and action for forcible entry
and detainer in the justice of the peace court of Guagua,
After trial, whereat evidence was introduced by both
Pampanga, which was dismissed for lack of jurisdiction,
parties, the Court of First Instance of Pampanga rendered
the case being one of title to real property (Exhibit F and
judgment declaring the plaintiff entitled to the possession
M). Valentin Susi then brought this action.
of the land, annulling the sale made by the Director of
Lands in favor of Angela Razon, and ordering the With these facts in view, we shall proceed to consider the
cancellation of the certificate of title issued to her, with the questions raised by the appellant in his assignments of
costs against Angela Razon. From this judgment the error.lawphi1.net
Director of Lands took this appeal, assigning thereto the
following errors, to wit: (1) The holding that the judgment It clearly appears from the evidence that Valentin Susi has
rendered in a prior case between the plaintiff and been in possession of the land in question openly,
defendant Angela Razon on the parcel of land in question continuously, adversely, and publicly, personally and
is controlling in this action; (2) the holding that plaintiff is through his predecessors, since the year 1880, that is, for
entitled to recover the possession of said parcel of land; about forty-five years. While the judgment of the Court of
the annulment of the sale made by the Director of Lands to First Instance of Pampanga against Angela Razon in the
Angela Razon; and the ordering that the certificate of title forcible entry case does not affect the Director of Lands,
issued by the register of deeds of the Province of yet it is controlling as to Angela Razon and rebuts her
Pampanga to Angela Razon by virtue of said sale be claim that she had been in possession thereof. When on
cancelled; and (3) the denial of the motion for new trial August 15, 1914, Angela Razon applied for the purchase
filed by the Director of Lands. of said land, Valentin Susi had already been in possession
thereof personally and through his predecessors for thirty-
The evidence shows that on December 18, 1880, Nemesio four years. And if it is taken into account that Nemesio
Pinlac sold the land in question, then a fish pond, tho Pinlac had already made said land a fish pond when he
Apolonio Garcia and Basilio Mendoza for the sum of P12, sold it on December 18, 1880, it can hardly be estimated
reserving the right to repurchase the same (Exhibit B). when he began to possess and occupy it, the period of
After having been in possession thereof for about eight time being so long that it is beyond the reach of memory.
years, and the fish pond having been destroyed, Apolonio These being the facts, the doctrine laid down by the
Garcia and Basilio Mendoza, on September 5, 1899, sold Supreme Court of the United States in the case of Cario
Law on NatRes (23-49) 8
vs. Government of the Philippine Islands (212 U. S., 449 parcel of land of Corazon Naguit (Naguit), the respondent
1), is applicable here. In favor of Valentin Susi, there is, herein.
moreover, the presumption juris et de jure established in
paragraph (b) of section 45 of Act No. 2874, amending Act The facts are as follows:
No. 926, that all the necessary requirements for a grant by
the Government were complied with, for he has been in On January 5, 1993, Naguit, a Filipino citizen, of legal age
actual and physical possession, personally and through and married to Manolito S. Naguit, filed with the MCTC of
his predecessors, of an agricultural land of the public Ibajay-Nabas, Aklan, a petition for registration of title of a
domain openly, continuously, exclusively and publicly parcel of land situated in Brgy. Union, Nabas, Aklan. The
since July 26, 1894, with a right to a certificate of title to parcel of land is designated as Lot No. 10049, Cad. 758-D,
said land under the provisions of Chapter VIII of said Act. Nabas Cadastre, AP 060414-014779, and contains an
So that when Angela Razon applied for the grant in her area of 31,374 square meters. The application seeks
favor, Valentin Susi had already acquired, by operation of judicial confirmation of respondents imperfect title over the
law, not only a right to a grant, but a grant of the aforesaid land.
Government, for it is not necessary that certificate of title
On February 20, 1995, the court held initial hearing on the
should be issued in order that said grant may be
application. The public prosecutor, appearing for the
sanctioned by the courts, an application therefore is
government, and Jose Angeles, representing the heirs of
sufficient, under the provisions of section 47 of Act No.
Rustico Angeles, opposed the petition. On a later date,
2874. If by a legal fiction, Valentin Susi had acquired the
however, the heirs of Rustico Angeles filed a formal
land in question by a grant of the State, it had already
opposition to the petition. Also on February 20, 1995, the
ceased to be the public domain and had become private
court issued an order of general default against the whole
property, at least by presumption, of Valentin Susi, beyond
world except as to the heirs of Rustico Angeles and the
the control of the Director of Lands. Consequently, in
government.
selling the land in question to Angela Razon, the Director
of Lands disposed of a land over which he had no longer The evidence on record reveals that the subject parcel of
any title or control, and the sale thus made was void and land was originally declared for taxation purposes in the
of no effect, and Angela Razon did not thereby acquire name of Ramon Urbano (Urbano) in 1945 under Tax
any right. Declaration No. 3888 until 1991.4 On July 9, 1992, Urbano
executed a Deed of Quitclaim in favor of the heirs of
The Director of Lands contends that the land in question
Honorato Maming (Maming), wherein he renounced all his
being of the public domain, the plaintiff-appellee cannot
rights to the subject property and confirmed the sale made
maintain an action to recover possession thereof.
by his father to Maming sometime in 1955 or 1956.5
If, as above stated, the land, the possession of which is in Subsequently, the heirs of Maming executed a deed of
dispute, had already become, by operation of law, private absolute sale in favor of respondent Naguit who thereupon
property of the plaintiff, there lacking only the judicial started occupying the same. She constituted Manuel
sanction of his title, Valentin Susi has the right to bring an Blanco, Jr. as her attorney-in-fact and administrator. The
action to recover possession thereof and hold it. administrator introduced improvements, planted trees,
such as mahogany, coconut and gemelina trees in
For the foregoing, and no error having been found in the addition to existing coconut trees which were then 50 to 60
judgment appealed from, the same is hereby affirmed in all years old, and paid the corresponding taxes due on the
its parts, without special pronouncement as to costs. So subject land. At present, there are parcels of land
ordered. surrounding the subject land which have been issued titles
by virtue of judicial decrees. Naguit and her predecessors-
REPUBLIC OF THE PHILIPPINES, petitioner, vs. THE in-interest have occupied the land openly and in the
HONORABLE COURT OF APPEALS and CORAZON concept of owner without any objection from any private
NAGUIT, respondents.; G.R. No. 144057 person or even the government until she filed her
January 17, 2005; TINGA, J.: application for registration.

This is a Petition for Review on Certiorari under Rule 45 of After the presentation of evidence for Naguit, the public
the 1997 Rules of Civil Procedure, seeking to review the prosecutor manifested that the government did not intend
Decision1 of the Sixth Division of the Court of Appeals to present any evidence while oppositor Jose Angeles, as
dated July 12, 2000 in CA-G.R. SP No. 51921. The representative of the heirs of Rustico Angeles, failed to
appellate court affirmed the decisions of both the Regional appear during the trial despite notice. On September 27,
Trial Court (RTC),2 Branch 8, of Kalibo, Aklan dated 1997, the MCTC rendered a decision ordering that the
February 26, 1999, and the 7th Municipal Circuit Trial subject parcel be brought under the operation of the
Court (MCTC)3 of Ibajay-Nabas, Aklan dated February 18, Property Registration Decree or Presidential Decree (P.D.)
1998, which granted the application for registration of a No. 1529 and that the title thereto registered and
confirmed in the name of Naguit.6
Law on NatRes (23-49) 9
The Republic of the Philippines (Republic), thru the Office exclusive and notorious possession and occupation of
of the Solicitor General (OSG), filed a motion for alienable and disposable lands of the public domain under
reconsideration. The OSG stressed that the land applied a bona fide claim of ownership since June 12, 1945, or
for was declared alienable and disposable only on October earlier.
15, 1980, per the certification from Regional Executive
Director Raoul T. Geollegue of the Department of (2) Those who have acquired ownership over private lands
Environment and Natural Resources, Region VI.7 by prescription under the provisions of existing laws.
However, the court denied the motion for reconsideration
in an order dated February 18, 1998.81awphi1.nt ....

Thereafter, the Republic appealed the decision and the There are three obvious requisites for the filing of an
order of the MCTC to the RTC, Kalibo, Aklan, Branch 8. application for registration of title under Section 14(1)
On February 26, 1999, the RTC rendered its decision, that the property in question is alienable and disposable
dismissing the appeal. land of the public domain; that the applicants by
themselves or through their predecessors-in-interest have
Undaunted, the Republic elevated the case to the Court of been in open, continuous, exclusive and notorious
Appeals via Rule 42 of the 1997 Rules of Civil Procedure. possession and occupation, and; that such possession is
On July 12, 2000, the appellate court rendered a decision under a bona fide claim of ownership since June 12, 1945
dismissing the petition filed by the Republic and affirmed in or earlier.
toto the Hence, the present petition for review raising a
pure question of law was filed by the Republic on Petitioner suggests an interpretation that the alienable and
September 4, 2000. disposable character of the land should have already been
established since June 12, 1945 or earlier. This is not
The OSG assails the decision of the Court of Appeals borne out by the plain meaning of Section 14(1). "Since
contending that the appellate court gravely erred in holding June 12, 1945," as used in the provision, qualifies its
that there is no need for the governments prior release of antecedent phrase "under a bonafide claim of ownership."
the subject lot from the public domain before it can be Generally speaking, qualifying words restrict or modify
considered alienable or disposable within the meaning of only the words or phrases to which they are immediately
P.D. No. 1529, and that Naguit had been in possession of associated, and not those distantly or remotely located.13
Lot No. 10049 in the concept of owner for the required Ad proximum antecedents fiat relation nisi impediatur
period.11 sentencia.

Hence, the central question for resolution is whether is Besides, we are mindful of the absurdity that would result
necessary under Section 14(1) of the Property if we adopt petitioners position. Absent a legislative
Registration Decree that the subject land be first classified amendment, the rule would be, adopting the OSGs view,
as alienable and disposable before the applicants that all lands of the public domain which were not declared
possession under a bona fide claim of ownership could alienable or disposable before June 12, 1945 would not be
even start. susceptible to original registration, no matter the length of
unchallenged possession by the occupant. Such
The OSG invokes our holding in Director of Lands v. interpretation renders paragraph (1) of Section 14 virtually
Intermediate Appellate Court12 in arguing that the inoperative and even precludes the government from
property which is in open, continuous and exclusive giving it effect even as it decides to reclassify public
possession must first be alienable. Since the subject land agricultural lands as alienable and disposable. The
was declared alienable only on October 15, 1980, Naguit unreasonableness of the situation would even be
could not have maintained a bona fide claim of ownership aggravated considering that before June 12, 1945, the
since June 12, 1945, as required by Section 14 of the Philippines was not yet even considered an independent
Property Registration Decree, since prior to 1980, the land state.
was not alienable or disposable, the OSG argues.
Instead, the more reasonable interpretation of Section
Section 14 of the Property Registration Decree, governing 14(1) is that it merely requires the property sought to be
original registration proceedings, bears close examination. registered as already alienable and disposable at the time
It expressly provides: the application for registration of title is filed. If the State, at
the time the application is made, has not yet deemed it
SECTION 14. Who may apply. The following persons proper to release the property for alienation or disposition,
may file in the proper Court of First Instance an application the presumption is that the government is still reserving
for registration of title to land, whether personally or the right to utilize the property; hence, the need to
through their duly authorized representatives: preserve its ownership in the State irrespective of the
length of adverse possession even if in good faith.
(1) those who by themselves or through their However, if the property has already been classified as
predecessors-in-interest have been in open, continuous, alienable and disposable, as it is in this case, then there is
Law on NatRes (23-49) 10
already an intention on the part of the State to abdicate its Sec. 48. The following described citizens of the
exclusive prerogative over the property. Philippines, occupying lands of the public domain or
claiming to own any such land or an interest therein, but
This reading aligns conformably with our holding in those titles have not been perfected or completed, may
Republic v. Court of Appeals .14 Therein, the Court noted apply to the Court of First Instance of the province where
that "to prove that the land subject of an application for the land is located for confirmation of their claims and the
registration is alienable, an applicant must establish the issuance of a certificate of title therefor, under the Land
existence of a positive act of the government such as a Registration Act, to wit:
presidential proclamation or an executive order; an
administrative action; investigation reports of Bureau of xxx xxx xxx
Lands investigators; and a legislative act or a statute."15
In that case, the subject land had been certified by the (b) Those who by themselves or through their
DENR as alienable and disposable in 1980, thus the Court predecessors in interest have been in open, continuous,
concluded that the alienable status of the land, exclusive, and notorious possession and occupation of
compounded by the established fact that therein agricultural lands of the public domain, under a bona fide
respondents had occupied the land even before 1927, claim of acquisition of ownership, for at least thirty years
sufficed to allow the application for registration of the said immediately preceding the filing of the application for
property. In the case at bar, even the petitioner admits that confirmation of title except when prevented by war or force
the subject property was released and certified as within majeure. These shall be conclusively presumed to have
alienable and disposable zone in 1980 by the DENR. performed all the conditions essential to a Government
grant and shall be entitled to a certificate of title under the
This case is distinguishable from Bracewell v. Court of provisions of this chapter.
Appeals,17 wherein the Court noted that while the
claimant had been in possession since 1908, it was only in When the Public Land Act was first promulgated in 1936,
1972 that the lands in question were classified as the period of possession deemed necessary to vest the
alienable and disposable. Thus, the bid at registration right to register their title to agricultural lands of the public
therein did not succeed. In Bracewell, the claimant had domain commenced from July 26, 1894. However, this
filed his application in 1963, or nine (9) years before the period was amended by R.A. No. 1942, which provided
property was declared alienable and that the bona fide claim of ownership must have been for
disposable.1awphi1.nt Thus, in this case, where the at least thirty (30) years. Then in 1977, Section 48(b) of
application was made years after the property had been the Public Land Act was again amended, this time by P.D.
certified as alienable and disposable, the Bracewell ruling No. 1073, which pegged the reckoning date at June 12,
does not apply. 1945. This new starting point is concordant with Section
14(1) of the Property Registration Decree.
A different rule obtains for forest lands,18 such as those
which form part of a reservation for provincial park Indeed, there are no material differences between Section
purposes19 the possession of which cannot ripen into 14(1) of the Property Registration Decree and Section
ownership.20 It is elementary in the law governing natural 48(b) of the Public Land Act, as amended. True, the Public
resources that forest land cannot be owned by private Land Act does refer to "agricultural lands of the public
persons. As held in Palomo v. Court of Appeals,21 forest domain," while the Property Registration Decree uses the
land is not registrable and possession thereof, no matter term "alienable and disposable lands of the public
how lengthy, cannot convert it into private property, unless domain." It must be noted though that the Constitution
such lands are reclassified and considered disposable and declares that "alienable lands of the public domain shall be
alienable.22 In the case at bar, the property in question limited to agricultural lands."24 Clearly, the subject lands
was undisputedly classified as disposable and alienable; under Section 48(b) of the Public Land Act and Section
hence, the ruling in Palomo is inapplicable, as correctly 14(1) of the Property Registration Decree are of the same
held by the Court of Appeals. type.

It must be noted that the present case was decided by the Did the enactment of the Property Registration Decree and
lower courts on the basis of Section 14(1) of the Property the amendatory P.D. No. 1073 preclude the application for
Registration Decree, which pertains to original registration registration of alienable lands of the public domain,
through ordinary registration proceedings. The right to file possession over which commenced only after June 12,
the application for registration derives from a bona fide 1945? It did not, considering Section 14(2) of the Property
claim of ownership going back to June 12, 1945 or earlier, Registration Decree, which governs and authorizes the
by reason of the claimants open, continuous, exclusive application of "those who have acquired ownership of
and notorious possession of alienable and disposable private lands by prescription under the provisions of
lands of the public domain. existing laws."

A similar right is given under Section 48(b) of the Public Prescription is one of the modes of acquiring ownership
Land Act, which reads: under the Civil Code.25 There is a consistent
Law on NatRes (23-49) 11
jurisprudential rule that properties classified as alienable doubt that she has acquired title thereto which may be
public land may be converted into private property by properly brought under the operation of the Torrens
reason of open, continuous and exclusive possession of at system. That she has been in possession of the land in the
least thirty (30) years.26 With such conversion, such concept of an owner, open, continuous, peaceful and
property may now fall within the contemplation of "private without any opposition from any private person and the
lands" under Section 14(2), and thus susceptible to government itself makes her right thereto undoubtedly
registration by those who have acquired ownership settled and deserving of protection under the law.
through prescription. Thus, even if possession of the
alienable public land commenced on a date later than WHEREFORE, foregoing premises considered, the
June 12, 1945, and such possession being been open, assailed Decision of the Court of Appeals dated July 12,
continuous and exclusive, then the possessor may have 2000 is hereby AFFIRMED. No costs. SO
the right to register the land by virtue of Section 14(2) of ORDERED.
the Property Registration Decree.
HEIRS OF MARIO MALABANAN, Petitioner, vs.
The land in question was found to be cocal in nature, it REPUBLIC OF THE PHILIPPINES, Respondent.; G.R.
having been planted with coconut trees now over fifty No. 179987 April 29, 2009; TINGA, J.:
years old.27 The inherent nature of the land but confirms
its certification in 1980 as alienable, hence agricultural. One main reason why the informal sector has not become
There is no impediment to the application of Section 14(1) formal is that from Indonesia to Brazil, 90 percent of the
of the Property Registration Decree, as correctly informal lands are not titled and registered. This is a
accomplished by the lower courts. generalized phenomenon in the so-called Third World.
And it has many consequences.
The OSG posits that the Court of Appeals erred in holding
that Naguit had been in possession in the concept of xxx
owner for the required period. The argument begs the
question. It is again hinged on the assertionshown The question is: How is it that so many governments, from
earlier to be unfoundedthat there could have been no Suharto's in Indonesia to Fujimori's in Peru, have wanted
bona fide claim of ownership prior to 1980, when the to title these people and have not been able to do so
subject land was declared alienable or disposable. effectively? One reason is that none of the state systems
in Asia or Latin America can gather proof of informal titles.
We find no reason to disturb the conclusion of both the In Peru, the informals have means of proving property
RTC and the Court of Appeals that Naguit had the right to ownership to each other which are not the same means
apply for registration owing to the continuous possession developed by the Spanish legal system. The informals
by her and her predecessors-in-interest of the land since have their own papers, their own forms of agreements,
1945. The basis of such conclusion is primarily factual, and their own systems of registration, all of which are very
and the Court generally respects the factual findings made clearly stated in the maps which they use for their own
by lower courts. Notably, possession since 1945 was informal business transactions.
established through proof of the existence of 50 to 60-year
old trees at the time Naguit purchased the property as well If you take a walk through the countryside, from Indonesia
as tax declarations executed by Urbano in 1945. Although to Peru, and you walk by field after field--in each field a
tax declarations and realty tax payment of property are not different dog is going to bark at you. Even dogs know what
conclusive evidence of ownership, nevertheless, they are private property is all about. The only one who does not
good indicia of the possession in the concept of owner for know it is the government. The issue is that there exists a
no one in his right mind would be paying taxes for a "common law" and an "informal law" which the Latin
property that is not in his actual or at least constructive American formal legal system does not know how to
possession. They constitute at least proof that the holder recognize.
has a claim of title over the property. The voluntary
declaration of a piece of property for taxation purposes - Hernando De Soto
manifests not only ones sincere and honest desire to
This decision inevitably affects all untitled lands currently
obtain title to the property and announces his adverse
in possession of persons and entities other than the
claim against the State and all other interested parties, but
Philippine government. The petition, while unremarkable
also the intention to contribute needed revenues to the
as to the facts, was accepted by the Court en banc in
Government. Such an act strengthens ones bona fide
order to provide definitive clarity to the applicability and
claim of acquisition of ownership.2
scope of original registration proceedings under Sections
Considering that the possession of the subject parcel of 14(1) and 14(2) of the Property Registration Decree. In
land by the respondent can be traced back to that of her doing so, the Court confronts not only the relevant
predecessors-in-interest which commenced since 1945 or provisions of the Public Land Act and the Civil Code, but
for almost fifty (50) years, it is indeed beyond any cloud of also the reality on the ground. The countrywide
phenomenon of untitled lands, as well as the problem of
Law on NatRes (23-49) 12
informal settlement it has spawned, has unfortunately On 3 December 2002, the RTC rendered judgment in
been treated with benign neglect. Yet our current laws are favor of Malabanan, the dispositive portion of which reads:
hemmed in by their own circumscriptions in addressing the
phenomenon. Still, the duty on our part is primarily to WHEREFORE, this Court hereby approves this application
decide cases before us in accord with the Constitution and for registration and thus places under the operation of Act
the legal principles that have developed our public land 141, Act 496 and/or P.D. 1529, otherwise known as
law, though our social obligations dissuade us from Property Registration Law, the lands described in Plan
casting a blind eye on the endemic problems. Csd-04-0173123-D, Lot 9864-A and containing an area of
Seventy One Thousand Three Hundred Twenty Four
I. (71,324) Square Meters, as supported by its technical
description now forming part of the record of this case, in
On 20 February 1998, Mario Malabanan filed an addition to other proofs adduced in the name of MARIO
application for land registration covering a parcel of land MALABANAN, who is of legal age, Filipino, widower, and
identified as Lot 9864-A, Cad-452-D, Silang Cadastre,2 with residence at Munting Ilog, Silang, Cavite.
situated in Barangay Tibig, Silang Cavite, and consisting
of 71,324 square meters. Malabanan claimed that he had Once this Decision becomes final and executory, the
purchased the property from Eduardo Velazco,3 and that corresponding decree of registration shall forthwith issue.
he and his predecessors-in-interest had been in open,
notorious, and continuous adverse and peaceful SO ORDERED.
possession of the land for more than thirty (30) years.
The Republic interposed an appeal to the Court of
The application was raffled to the Regional Trial Court of Appeals, arguing that Malabanan had failed to prove that
(RTC) Cavite-Tagaytay City, Branch 18. The Office of the the property belonged to the alienable and disposable land
Solicitor General (OSG) duly designated the Assistant of the public domain, and that the RTC had erred in finding
Provincial Prosecutor of Cavite, Jose Velazco, Jr., to that he had been in possession of the property in the
appear on behalf of the State.4 Apart from presenting manner and for the length of time required by law for
documentary evidence, Malabanan himself and his confirmation of imperfect title.
witness, Aristedes Velazco, testified at the hearing.
Velazco testified that the property was originally belonged On 23 February 2007, the Court of Appeals rendered a
to a twenty-two hectare property owned by his great- Decision8 reversing the RTC and dismissing the
grandfather, Lino Velazco. Lino had four sons Benedicto, application of Malabanan. The appellate court held that
Gregorio, Eduardo and Estebanthe fourth being under Section 14(1) of the Property Registration Decree
Aristedess grandfather. Upon Linos death, his four sons any period of possession prior to the classification of the
inherited the property and divided it among themselves. lots as alienable and disposable was inconsequential and
But by 1966, Estebans wife, Magdalena, had become the should be excluded from the computation of the period of
administrator of all the properties inherited by the Velazco possession. Thus, the appellate court noted that since the
sons from their father, Lino. After the death of Esteban and CENRO-DENR certification had verified that the property
Magdalena, their son Virgilio succeeded them in was declared alienable and disposable only on 15 March
administering the properties, including Lot 9864-A, which 1982, the Velazcos possession prior to that date could not
originally belonged to his uncle, Eduardo Velazco. It was be factored in the computation of the period of possession.
this property that was sold by Eduardo Velazco to This interpretation of the Court of Appeals of Section 14(1)
Malabanan. of the Property Registration Decree was based on the
Courts ruling in Republic v. Herbieto.
Assistant Provincial Prosecutor Jose Velazco, Jr. did not
cross-examine Aristedes Velazco. He further manifested Malabanan died while the case was pending with the
that he "also [knew] the property and I affirm the truth of Court of Appeals;10 hence, it was his heirs who appealed
the testimony given by Mr. Velazco."6 The Republic of the the decision of the appellate court. Petitioners, before this
Philippines likewise did not present any evidence to Court, rely on our ruling in Republic v. Naguit,11 which
controvert the application. was handed down just four months prior to Herbieto.
Petitioners suggest that the discussion in Herbieto cited by
Among the evidence presented by Malabanan during trial the Court of Appeals is actually obiter dictum since the
was a Certification dated 11 June 2001, issued by the Metropolitan Trial Court therein which had directed the
Community Environment & Natural Resources Office, registration of the property had no jurisdiction in the first
Department of Environment and Natural Resources place since the requisite notice of hearing was published
(CENRO-DENR), which stated that the subject property only after the hearing had already begun. Naguit,
was "verified to be within the Alienable or Disposable land petitioners argue, remains the controlling doctrine,
per Land Classification Map No. 3013 established under especially when the property in question is agricultural
Project No. 20-A and approved as such under FAO 4-1656 land. Therefore, with respect to agricultural lands, any
on March 15, 1982."7 possession prior to the declaration of the alienable
property as disposable may be counted in reckoning the
Law on NatRes (23-49) 13
period of possession to perfect title under the Public Land With respect to Section 14(2), petitioners submit that open,
Act and the Property Registration Decree. continuous, exclusive and notorious possession of an
alienable land of the public domain for more than 30 years
The petition was referred to the Court en banc,12 and on ipso jure converts the land into private property, thus
11 November 2008, the case was heard on oral placing it under the coverage of Section 14(2). According
arguments. The Court formulated the principal issues for to them, it would not matter whether the land sought to be
the oral arguments, to wit: registered was previously classified as agricultural land of
the public domain so long as, at the time of the application,
1. In order that an alienable and disposable land of the the property had already been "converted" into private
public domain may be registered under Section 14(1) of property through prescription. To bolster their argument,
Presidential Decree No. 1529, otherwise known as the petitioners cite extensively from our 2008 ruling in
Property Registration Decree, should the land be classified Republic v. T.A.N. Properties.19
as alienable and disposable as of June 12, 1945 or is it
sufficient that such classification occur at any time prior to The arguments submitted by the OSG with respect to
the filing of the applicant for registration provided that it is Section 14(2) are more extensive. The OSG notes that
established that the applicant has been in open, under Article 1113 of the Civil Code, the acquisitive
continuous, exclusive and notorious possession of the prescription of properties of the State refers to "patrimonial
land under a bona fide claim of ownership since June 12, property," while Section 14(2) speaks of "private lands." It
1945 or earlier? observes that the Court has yet to decide a case that
presented Section 14(2) as a ground for application for
2. For purposes of Section 14(2) of the Property registration, and that the 30-year possession period refers
Registration Decree may a parcel of land classified as to the period of possession under Section 48(b) of the
alienable and disposable be deemed private land and Public Land Act, and not the concept of prescription under
therefore susceptible to acquisition by prescription in the Civil Code. The OSG further submits that, assuming
accordance with the Civil Code? that the 30-year prescriptive period can run against public
lands, said period should be reckoned from the time the
3. May a parcel of land established as agricultural in
public land was declared alienable and disposable.
character either because of its use or because its slope is
below that of forest lands be registrable under Section Both sides likewise offer special arguments with respect to
14(2) of the Property Registration Decree in relation to the the particular factual circumstances surrounding the
provisions of the Civil Code on acquisitive prescription? subject property and the ownership thereof.

4. Are petitioners entitled to the registration of the subject II.


land in their names under Section 14(1) or Section 14(2) of
the Property Registration Decree or both? First, we discuss Section 14(1) of the Property
Registration Decree. For a full understanding of the
Based on these issues, the parties formulated their provision, reference has to be made to the Public Land
respective positions. Act.

With respect to Section 14(1), petitioners reiterate that the A.


analysis of the Court in Naguit is the correct interpretation
of the provision. The seemingly contradictory Commonwealth Act No. 141, also known as the Public
pronouncement in Herbieto, it is submitted, should be Land Act, has, since its enactment, governed the
considered obiter dictum, since the land registration classification and disposition of lands of the public domain.
proceedings therein was void ab initio due to lack of The President is authorized, from time to time, to classify
publication of the notice of initial hearing. Petitioners the lands of the public domain into alienable and
further point out that in Republic v. Bibonia,14 disposable, timber, or mineral lands.20 Alienable and
promulgated in June of 2007, the Court applied Naguit and disposable lands of the public domain are further classified
adopted the same observation that the preferred according to their uses into (a) agricultural; (b) residential,
interpretation by the OSG of Section 14(1) was patently commercial, industrial, or for similar productive purposes;
absurd. For its part, the OSG remains insistent that for (c) educational, charitable, or other similar purposes; or (d)
Section 14(1) to apply, the land should have been reservations for town sites and for public and quasi-public
classified as alienable and disposable as of 12 June 1945. uses.
Apart from Herbieto, the OSG also cites the subsequent
rulings in Buenaventura v. Republic,15 Fieldman May a private person validly seek the registration in his/her
Agricultural Trading v. Republic16 and Republic v. Imperial name of alienable and disposable lands of the public
Credit Corporation,17 as well as the earlier case of domain? Section 11 of the Public Land Act acknowledges
Director of Lands v. Court of Appeals. that public lands suitable for agricultural purposes may be
disposed of "by confirmation of imperfect or incomplete
titles" through "judicial legalization."22 Section 48(b) of the
Law on NatRes (23-49) 14
Public Land Act, as amended by P.D. No. 1073, supplies including lands of the public domain. It is Section 14(1)
the details and unmistakably grants that right, subject to that operationalizes the registration of such lands of the
the requisites stated therein: public domain. The provision reads:

Sec. 48. The following described citizens of the SECTION 14. Who may apply. The following persons
Philippines, occupying lands of the public domain or may file in the proper Court of First Instance an application
claiming to own any such land or an interest therein, but for registration of title to land, whether personally or
whose titles have not been perfected or completed, may through their duly authorized representatives:
apply to the Court of First Instance of the province where
the land is located for confirmation of their claims and the (1) those who by themselves or through their
issuance of a certificate of title therefor, under the Land predecessors-in-interest have been in open, continuous,
Registration Act, to wit: exclusive and notorious possession and occupation of
alienable and disposable lands of the public domain under
xxx a bona fide claim of ownership since June 12, 1945, or
earlier.
(b) Those who by themselves or through their
predecessors in interest have been in open, continuous, Notwithstanding the passage of the Property Registration
exclusive, and notorious possession and occupation of Decree and the inclusion of Section 14(1) therein, the
alienable and disposable lands of the public domain, under Public Land Act has remained in effect. Both laws
a bona fide claim of acquisition of ownership, since June commonly refer to persons or their predecessors-in-
12, 1945, or earlier, immediately preceding the filing of the interest who "have been in open, continuous, exclusive
application for confirmation of title except when prevented and notorious possession and occupation of alienable and
by war or force majeure. These shall be conclusively disposable lands of the public domain under a bona fide
presumed to have performed all the conditions essential to claim of ownership since June 12, 1945, or earlier." That
a Government grant and shall be entitled to a certificate of circumstance may have led to the impression that one or
title under the provisions of this chapter. the other is a redundancy, or that Section 48(b) of the
Public Land Act has somehow been repealed or mooted.
Section 48(b) of Com. Act No. 141 received its present That is not the case.
wording in 1977 when the law was amended by P.D. No.
1073. Two significant amendments were introduced by The opening clauses of Section 48 of the Public Land Act
P.D. No. 1073. First, the term "agricultural lands" was and Section 14 of the Property Registration Decree
changed to "alienable and disposable lands of the public warrant comparison:
domain." The OSG submits that this amendment restricted
the scope of the lands that may be registered.23 This is Sec. 48 [of the Public Land Act]. The following described
not actually the case. Under Section 9 of the Public Land citizens of the Philippines, occupying lands of the public
Act, "agricultural lands" are a mere subset of "lands of the domain or claiming to own any such land or an interest
public domain alienable or open to disposition." Evidently, therein, but whose titles have not been perfected or
alienable and disposable lands of the public domain are a completed, may apply to the Court of First Instance of the
larger class than only "agricultural lands." province where the land is located for confirmation of their
claims and the issuance of a certificate of title therefor,
Second, the length of the requisite possession was under the Land Registration Act, to wit:
changed from possession for "thirty (30) years immediately
preceding the filing of the application" to possession "since xxx
June 12, 1945 or earlier." The Court in Naguit explained:
Sec. 14 [of the Property Registration Decree]. Who may
When the Public Land Act was first promulgated in 1936, apply. The following persons may file in the proper Court
the period of possession deemed necessary to vest the of First Instance an application for registration of title to
right to register their title to agricultural lands of the public land, whether personally or through their duly authorized
domain commenced from July 26, 1894. However, this representatives:
period was amended by R.A. No. 1942, which provided
that the bona fide claim of ownership must have been for xxx
at least thirty (30) years. Then in 1977, Section 48(b) of
It is clear that Section 48 of the Public Land Act is more
the Public Land Act was again amended, this time by P.D.
descriptive of the nature of the right enjoyed by the
No. 1073, which pegged the reckoning date at June 12,
possessor than Section 14 of the Property Registration
1945. xxx
Decree, which seems to presume the pre-existence of the
It bears further observation that Section 48(b) of Com. Act right, rather than establishing the right itself for the first
No, 141 is virtually the same as Section 14(1) of the time. It is proper to assert that it is the Public Land Act, as
Property Registration Decree. Said Decree codified the amended by P.D. No. 1073 effective 25 January 1977, that
various laws relative to the registration of property, has primarily established the right of a Filipino citizen who
Law on NatRes (23-49) 15
has been "in open, continuous, exclusive, and notorious Section 14(1) of the Property Registration Decree or
possession and occupation of alienable and disposable Section 48(b) of the Public Land Act as amended. The
lands of the public domain, under a bona fide claim of absurdity of such an implication was discussed in Naguit.
acquisition of ownership, since June 12, 1945" to perfect
or complete his title by applying with the proper court for Petitioner suggests an interpretation that the alienable and
the confirmation of his ownership claim and the issuance disposable character of the land should have already been
of the corresponding certificate of title. established since June 12, 1945 or earlier. This is not
borne out by the plain meaning of Section 14(1). "Since
Section 48 can be viewed in conjunction with the afore- June 12, 1945," as used in the provision, qualifies its
quoted Section 11 of the Public Land Act, which provides antecedent phrase "under a bonafide claim of ownership."
that public lands suitable for agricultural purposes may be Generally speaking, qualifying words restrict or modify
disposed of by confirmation of imperfect or incomplete only the words or phrases to which they are immediately
titles, and given the notion that both provisions declare associated, and not those distantly or remotely located.25
that it is indeed the Public Land Act that primarily Ad proximum antecedents fiat relation nisi impediatur
establishes the substantive ownership of the possessor sentencia.
who has been in possession of the property since 12 June
1945. In turn, Section 14(a) of the Property Registration Besides, we are mindful of the absurdity that would result
Decree recognizes the substantive right granted under if we adopt petitioners position. Absent a legislative
Section 48(b) of the Public Land Act, as well provides the amendment, the rule would be, adopting the OSGs view,
corresponding original registration procedure for the that all lands of the public domain which were not declared
judicial confirmation of an imperfect or incomplete title. alienable or disposable before June 12, 1945 would not be
susceptible to original registration, no matter the length of
There is another limitation to the right granted under unchallenged possession by the occupant. Such
Section 48(b). Section 47 of the Public Land Act limits the interpretation renders paragraph (1) of Section 14 virtually
period within which one may exercise the right to seek inoperative and even precludes the government from
registration under Section 48. The provision has been giving it effect even as it decides to reclassify public
amended several times, most recently by Rep. Act No. agricultural lands as alienable and disposable. The
9176 in 2002. It currently reads thus: unreasonableness of the situation would even be
aggravated considering that before June 12, 1945, the
Section 47. The persons specified in the next following Philippines was not yet even considered an independent
section are hereby granted time, not to extend beyond state.
December 31, 2020 within which to avail of the benefits of
this Chapter: Provided, That this period shall apply only Accordingly, the Court in Naguit explained:
where the area applied for does not exceed twelve (12)
hectares: Provided, further, That the several periods of [T]he more reasonable interpretation of Section 14(1) is
time designated by the President in accordance with that it merely requires the property sought to be registered
Section Forty-Five of this Act shall apply also to the lands as already alienable and disposable at the time the
comprised in the provisions of this Chapter, but this application for registration of title is filed. If the State, at the
Section shall not be construed as prohibiting any said time the application is made, has not yet deemed it proper
persons from acting under this Chapter at any time prior to to release the property for alienation or disposition, the
the period fixed by the President.24 presumption is that the government is still reserving the
right to utilize the property; hence, the need to preserve its
Accordingly under the current state of the law, the ownership in the State irrespective of the length of adverse
substantive right granted under Section 48(b) may be possession even if in good faith. However, if the property
availed of only until 31 December 2020. has already been classified as alienable and disposable,
as it is in this case, then there is already an intention on
B. the part of the State to abdicate its exclusive prerogative
over the property.
Despite the clear text of Section 48(b) of the Public Land
Act, as amended and Section 14(a) of the Property The Court declares that the correct interpretation of
Registration Decree, the OSG has adopted the position Section 14(1) is that which was adopted in Naguit. The
that for one to acquire the right to seek registration of an contrary pronouncement in Herbieto, as pointed out in
alienable and disposable land of the public domain, it is Naguit, absurdly limits the application of the provision to
not enough that the applicant and his/her predecessors-in- the point of virtual inutility since it would only cover lands
interest be in possession under a bona fide claim of actually declared alienable and disposable prior to 12 June
ownership since 12 June 1945; the alienable and 1945, even if the current possessor is able to establish
disposable character of the property must have been open, continuous, exclusive and notorious possession
declared also as of 12 June 1945. Following the OSGs under a bona fide claim of ownership long before that
approach, all lands certified as alienable and disposable date.
after 12 June 1945 cannot be registered either under
Law on NatRes (23-49) 16
Moreover, the Naguit interpretation allows more only in 1980. Ceniza cited Bracewell, quoted extensively
possessors under a bona fide claim of ownership to avail from it, and following the mindset of the dissent, the
of judicial confirmation of their imperfect titles than what attempt at registration in Ceniza should have failed. Not
would be feasible under Herbieto. This balancing fact is so.
significant, especially considering our forthcoming
discussion on the scope and reach of Section 14(2) of the To prove that the land subject of an application for
Property Registration Decree. registration is alienable, an applicant must establish the
existence of a positive act of the government such as a
Petitioners make the salient observation that the presidential proclamation or an executive order; an
contradictory passages from Herbieto are obiter dicta administrative action; investigation reports of Bureau of
since the land registration proceedings therein is void ab Lands investigators; and a legislative act or a statute.
initio in the first place due to lack of the requisite
publication of the notice of initial hearing. There is no need In this case, private respondents presented a certification
to explicitly overturn Herbieto, as it suffices that the Courts dated November 25, 1994, issued by Eduardo M. Inting,
acknowledgment that the particular line of argument used the Community Environment and Natural Resources
therein concerning Section 14(1) is indeed obiter. Officer in the Department of Environment and Natural
Resources Office in Cebu City, stating that the lots
It may be noted that in the subsequent case of involved were "found to be within the alienable and
Buenaventura,26 the Court, citing Herbieto, again stated disposable (sic) Block-I, Land Classification Project No.
that "[a]ny period of possession prior to the date when the 32-A, per map 2962 4-I555 dated December 9, 1980." This
[s]ubject [property was] classified as alienable and is sufficient evidence to show the real character of the land
disposable is inconsequential and should be excluded subject of private respondents application. Further, the
from the computation of the period of possession" That certification enjoys a presumption of regularity in the
statement, in the context of Section 14(1), is certainly absence of contradictory evidence, which is true in this
erroneous. Nonetheless, the passage as cited in case. Worth noting also was the observation of the Court
Buenaventura should again be considered as obiter. The of Appeals stating that:
application therein was ultimately granted, citing Section
14(2). The evidence submitted by petitioners therein did [n]o opposition was filed by the Bureaus of Lands and
not establish any mode of possession on their part prior to Forestry to contest the application of appellees on the
1948, thereby precluding the application of Section 14(1). ground that the property still forms part of the public
It is not even apparent from the decision whether domain. Nor is there any showing that the lots in question
petitioners therein had claimed entitlement to original are forestal land....
registration following Section 14(1), their position being
that they had been in exclusive possession under a bona Thus, while the Court of Appeals erred in ruling that mere
fide claim of ownership for over fifty (50) years, but not possession of public land for the period required by law
before 12 June 1945. would entitle its occupant to a confirmation of imperfect
title, it did not err in ruling in favor of private respondents
Thus, neither Herbieto nor its principal discipular ruling as far as the first requirement in Section 48(b) of the
Buenaventura has any precedental value with respect to Public Land Act is concerned, for they were able to
Section 14(1). On the other hand, the ratio of Naguit is overcome the burden of proving the alienability of the land
embedded in Section 14(1), since it precisely involved subject of their application.
situation wherein the applicant had been in exclusive
possession under a bona fide claim of ownership prior to As correctly found by the Court of Appeals, private
12 June 1945. The Courts interpretation of Section 14(1) respondents were able to prove their open, continuous,
therein was decisive to the resolution of the case. Any exclusive and notorious possession of the subject land
doubt as to which between Naguit or Herbieto provides the even before the year 1927. As a rule, we are bound by the
final word of the Court on Section 14(1) is now settled in factual findings of the Court of Appeals. Although there are
favor of Naguit. exceptions, petitioner did not show that this is one of
them.29
We noted in Naguit that it should be distinguished from
Bracewell v. Court of Appeals27 since in the latter, the Why did the Court in Ceniza, through the same eminent
application for registration had been filed before the land member who authored Bracewell, sanction the registration
was declared alienable or disposable. The dissent though under Section 48(b) of public domain lands declared
pronounces Bracewell as the better rule between the two. alienable or disposable thirty-five (35) years and 180 days
Yet two years after Bracewell, its ponente, the esteemed after 12 June 1945? The telling difference is that in
Justice Consuelo Ynares-Santiago, penned the ruling in Ceniza, the application for registration was filed nearly six
Republic v. Ceniza,28 which involved a claim of (6) years after the land had been declared alienable or
possession that extended back to 1927 over a public disposable, while in Bracewell, the application was filed
domain land that was declared alienable and disposable nine (9) years before the land was declared alienable or
disposable. That crucial difference was also stressed in
Law on NatRes (23-49) 17
Naguit to contradistinguish it from Bracewell, a difference original registration under Section 14(2). Specifically, it is
which the dissent seeks to belittle. Article 1113 which provides legal foundation for the
application. It reads:
III.
All things which are within the commerce of men are
We next ascertain the correct framework of analysis with susceptible of prescription, unless otherwise provided.
respect to Section 14(2). The provision reads: Property of the State or any of its subdivisions not
patrimonial in character shall not be the object of
SECTION 14. Who may apply. The following persons prescription.
may file in the proper Court of First Instance an application
for registration of title to land, whether personally or It is clear under the Civil Code that where lands of the
through their duly authorized representatives: public domain are patrimonial in character, they are
susceptible to acquisitive prescription. On the other hand,
xxx among the public domain lands that are not susceptible to
acquisitive prescription are timber lands and mineral lands.
(2) Those who have acquired ownership over private lands
The Constitution itself proscribes private ownership of
by prescription under the provisions of existing laws.
timber or mineral lands.
The Court in Naguit offered the following discussion
There are in fact several provisions in the Civil Code
concerning Section 14(2), which we did even then
concerning the acquisition of real property through
recognize, and still do, to be an obiter dictum, but we
prescription. Ownership of real property may be acquired
nonetheless refer to it as material for further discussion,
by ordinary prescription of ten (10) years,32 or through
thus:
extraordinary prescription of thirty (30) years.33 Ordinary
Did the enactment of the Property Registration Decree and acquisitive prescription requires possession in good
the amendatory P.D. No. 1073 preclude the application for faith,34 as well as just title.
registration of alienable lands of the public domain,
When Section 14(2) of the Property Registration Decree
possession over which commenced only after June 12,
explicitly provides that persons "who have acquired
1945? It did not, considering Section 14(2) of the Property
ownership over private lands by prescription under the
Registration Decree, which governs and authorizes the
provisions of existing laws," it unmistakably refers to the
application of "those who have acquired ownership of
Civil Code as a valid basis for the registration of lands.
private lands by prescription under the provisions of
The Civil Code is the only existing law that specifically
existing laws."
allows the acquisition by prescription of private lands,
Prescription is one of the modes of acquiring ownership including patrimonial property belonging to the State.
under the Civil Code.[30 ] There is a consistent Thus, the critical question that needs affirmation is
jurisprudential rule that properties classified as alienable whether Section 14(2) does encompass original
public land may be converted into private property by registration proceedings over patrimonial property of the
reason of open, continuous and exclusive possession of at State, which a private person has acquired through
least thirty (30) years.[31 ] With such conversion, such prescription.
property may now fall within the contemplation of "private
The Naguit obiter had adverted to a frequently reiterated
lands" under Section 14(2), and thus susceptible to
jurisprudence holding that properties classified as
registration by those who have acquired ownership
alienable public land may be converted into private
through prescription. Thus, even if possession of the
property by reason of open, continuous and exclusive
alienable public land commenced on a date later than
possession of at least thirty (30) years.36 Yet if we
June 12, 1945, and such possession being been open,
ascertain the source of the "thirty-year" period, additional
continuous and exclusive, then the possessor may have
complexities relating to Section 14(2) and to how exactly it
the right to register the land by virtue of Section 14(2) of
operates would emerge. For there are in fact two distinct
the Property Registration Decree.
origins of the thirty (30)-year rule.
Naguit did not involve the application of Section 14(2),
The first source is Rep. Act No. 1942, enacted in 1957,
unlike in this case where petitioners have based their
which amended Section 48(b) of the Public Land Act by
registration bid primarily on that provision, and where the
granting the right to seek original registration of alienable
evidence definitively establishes their claim of possession
public lands through possession in the concept of an
only as far back as 1948. It is in this case that we can
owner for at least thirty years.
properly appreciate the nuances of the provision.
The following-described citizens of the Philippines,
A.
occupying lands of the public domain or claiming to own
The obiter in Naguit cited the Civil Code provisions on any such lands or an interest therein, but whose titles have
prescription as the possible basis for application for not been perfected or completed, may apply to the Court
Law on NatRes (23-49) 18
of First Instance of the province where the land is located Then, with the repeal of Rep. Act No. 1942, the thirty-year
for confirmation of their claims and the issuance of a possession period as basis for original registration became
certificate of title therefor, under the Land Registration Act, Section 14(2) of the Property Registration Decree, which
to wit: entitled those "who have acquired ownership over private
lands by prescription under the provisions of existing laws"
xxxxxxxxx to apply for original registration. Again, the thirty-year
period is derived from the rule on extraordinary
(b) Those who by themselves or through their prescription under Article 1137 of the Civil Code. At the
predecessors in interest have been in open, continuous, same time, Section 14(2) puts into operation the entire
exclusive and notorious possession and occupation of regime of prescription under the Civil Code, a fact which
agricultural lands of the public domain, under a bona fide does not hold true with respect to Section 14(1).
claim of acquisition of ownership, for at least thirty years
immediately preceding the filing of the application for B.
confirmation of title, except when prevented by war or
force majeure. These shall be conclusively presumed to Unlike Section 14(1), Section 14(2) explicitly refers to the
have performed all the conditions essential to a principles on prescription under existing laws. Accordingly,
Government grant and shall be entitled to a certificate of we are impelled to apply the civil law concept of
title under the provisions of this Chapter. (emphasis prescription, as set forth in the Civil Code, in our
supplied)37 interpretation of Section 14(2). There is no similar demand
on our part in the case of Section 14(1).
This provision was repealed in 1977 with the enactment of
P.D. 1073, which made the date 12 June 1945 the The critical qualification under Article 1113 of the Civil
reckoning point for the first time. Nonetheless, applications Code is thus: "[p]roperty of the State or any of its
for registration filed prior to 1977 could have invoked the subdivisions not patrimonial in character shall not be the
30-year rule introduced by Rep. Act No. 1942. object of prescription." The identification what consists of
patrimonial property is provided by Articles 420 and 421,
The second source is Section 14(2) of P.D. 1529 itself, at which we quote in full:
least by implication, as it applies the rules on prescription
under the Civil Code, particularly Article 1113 in relation to Art. 420. The following things are property of public
Article 1137. Note that there are two kinds of prescription dominion:
under the Civil Codeordinary acquisitive prescription and
extraordinary acquisitive prescription, which, under Article (1) Those intended for public use, such as roads, canals,
1137, is completed "through uninterrupted adverse rivers, torrents, ports and bridges constructed by the State,
possession for thirty years, without need of title or of banks, shores, roadsteads, and others of similar character;
good faith."
(2) Those which belong to the State, without being for
Obviously, the first source of the thirty (30)-year period public use, and are intended for some public service or for
rule, Rep. Act No. 1942, became unavailable after 1977. the development of the national wealth.
At present, the only legal basis for the thirty (30)-year
period is the law on prescription under the Civil Code, as Art. 421. All other property of the State, which is not of the
mandated under Section 14(2). However, there is a character stated in the preceding article, is patrimonial
material difference between how the thirty (30)-year rule property
operated under Rep. Act No. 1942 and how it did under
It is clear that property of public dominion, which generally
the Civil Code.
includes property belonging to the State, cannot be the
Section 48(b) of the Public Land Act, as amended by Rep. object of prescription or, indeed, be subject of the
Act No. 1942, did not refer to or call into application the commerce of man.39 Lands of the public domain, whether
Civil Code provisions on prescription. It merely set forth a declared alienable and disposable or not, are property of
requisite thirty-year possession period immediately public dominion and thus insusceptible to acquisition by
preceding the application for confirmation of title, without prescription.
any qualification as to whether the property should be
Let us now explore the effects under the Civil Code of a
declared alienable at the beginning of, and continue as
declaration by the President or any duly authorized
such, throughout the entire thirty-(30) years. There is
government officer of alienability and disposability of lands
neither statutory nor jurisprudential basis to assert Rep.
of the public domain. Would such lands so declared
Act No. 1942 had mandated such a requirement,38 similar
alienable and disposable be converted, under the Civil
to our earlier finding with respect to the present language
Code, from property of the public dominion into patrimonial
of Section 48(b), which now sets 12 June 1945 as the
property? After all, by connotative definition, alienable and
point of reference.
disposable lands may be the object of the commerce of
man; Article 1113 provides that all things within the
Law on NatRes (23-49) 19
commerce of man are susceptible to prescription; and the entitled "An Act Accelerating The Conversion Of Military
same provision further provides that patrimonial property Reservations Into Other Productive Uses, etc.," is more
of the State may be acquired by prescription. commonly known as the BCDA law. Section 2 of the law
authorizes the sale of certain military reservations and
Nonetheless, Article 422 of the Civil Code states that portions of military camps in Metro Manila, including Fort
"[p]roperty of public dominion, when no longer intended for Bonifacio and Villamor Air Base. For purposes of effecting
public use or for public service, shall form part of the the sale of the military camps, the law mandates the
patrimonial property of the State." It is this provision that President to transfer such military lands to the Bases
controls how public dominion property may be converted Conversion Development Authority (BCDA)40 which in
into patrimonial property susceptible to acquisition by turn is authorized to own, hold and/or administer them.41
prescription. After all, Article 420 (2) makes clear that The President is authorized to sell portions of the military
those property "which belong to the State, without being camps, in whole or in part.42 Accordingly, the BCDA law
for public use, and are intended for some public service or itself declares that the military lands subject thereof are
for the development of the national wealth" are public "alienable and disposable pursuant to the provisions of
dominion property. For as long as the property belongs to existing laws and regulations governing sales of
the State, although already classified as alienable or government properties."43
disposable, it remains property of the public dominion if
when it is "intended for some public service or for the From the moment the BCDA law was enacted the subject
development of the national wealth". military lands have become alienable and disposable.
However, said lands did not become patrimonial, as the
Accordingly, there must be an express declaration by the BCDA law itself expressly makes the reservation that
State that the public dominion property is no longer these lands are to be sold in order to raise funds for the
intended for public service or the development of the conversion of the former American bases at Clark and
national wealth or that the property has been converted Subic.44 Such purpose can be tied to either "public
into patrimonial. Without such express declaration, the service" or "the development of national wealth" under
property, even if classified as alienable or disposable, Article 420(2). Thus, at that time, the lands remained
remains property of the public dominion, pursuant to property of the public dominion under Article 420(2),
Article 420(2), and thus incapable of acquisition by notwithstanding their status as alienable and disposable. It
prescription. It is only when such alienable and disposable is upon their sale as authorized under the BCDA law to a
lands are expressly declared by the State to be no longer private person or entity that such lands become private
intended for public service or for the development of the property and cease to be property of the public dominion.
national wealth that the period of acquisitive prescription
can begin to run. Such declaration shall be in the form of a C.
law duly enacted by Congress or a Presidential
Proclamation in cases where the President is duly Should public domain lands become patrimonial because
authorized by law. they are declared as such in a duly enacted law or duly
promulgated proclamation that they are no longer intended
It is comprehensible with ease that this reading of Section for public service or for the development of the national
14(2) of the Property Registration Decree limits its scope wealth, would the period of possession prior to the
and reach and thus affects the registrability even of lands conversion of such public dominion into patrimonial be
already declared alienable and disposable to the detriment reckoned in counting the prescriptive period in favor of the
of the bona fide possessors or occupants claiming title to possessors? We rule in the negative.
the lands. Yet this interpretation is in accord with the
Regalian doctrine and its concomitant assumption that all The limitation imposed by Article 1113 dissuades us from
lands owned by the State, although declared alienable or ruling that the period of possession before the public
disposable, remain as such and ought to be used only by domain land becomes patrimonial may be counted for the
the Government. purpose of completing the prescriptive period. Possession
of public dominion property before it becomes patrimonial
Recourse does not lie with this Court in the matter. The cannot be the object of prescription according to the Civil
duty of the Court is to apply the Constitution and the laws Code. As the application for registration under Section
in accordance with their language and intent. The remedy 14(2) falls wholly within the framework of prescription
is to change the law, which is the province of the under the Civil Code, there is no way that possession
legislative branch. Congress can very well be entreated to during the time that the land was still classified as public
amend Section 14(2) of the Property Registration Decree dominion property can be counted to meet the requisites
and pertinent provisions of the Civil Code to liberalize the of acquisitive prescription and justify registration.
requirements for judicial confirmation of imperfect or
incomplete titles. Are we being inconsistent in applying divergent rules for
Section 14(1) and Section 14(2)? There is no
The operation of the foregoing interpretation can be inconsistency. Section 14(1) mandates registration on the
illustrated by an actual example. Republic Act No. 7227, basis of possession, while Section 14(2) entitles
Law on NatRes (23-49) 20
registration on the basis of prescription. Registration under good faith involves the application of Articles 526, 527,
Section 14(1) is extended under the aegis of the Property and 528, as well as Article 1127 of the Civil Code,45
Registration Decree and the Public Land Act while provisions that more or less speak for themselves.
registration under Section 14(2) is made available both by
the Property Registration Decree and the Civil Code. On the other hand, the concept of just title requires some
clarification. Under Article 1129, there is just title for the
In the same manner, we can distinguish between the purposes of prescription "when the adverse claimant came
thirty-year period under Section 48(b) of the Public Land into possession of the property through one of the modes
Act, as amended by Rep. Act No. 1472, and the thirty-year recognized by law for the acquisition of ownership or other
period available through Section 14(2) of the Property real rights, but the grantor was not the owner or could not
Registration Decree in relation to Article 1137 of the Civil transmit any right." Dr. Tolentino explains:
Code. The period under the former speaks of a thirty-year
period of possession, while the period under the latter Just title is an act which has for its purpose the
concerns a thirty-year period of extraordinary prescription. transmission of ownership, and which would have actually
Registration under Section 48(b) of the Public Land Act as transferred ownership if the grantor had been the owner.
amended by Rep. Act No. 1472 is based on thirty years of This vice or defect is the one cured by prescription.
possession alone without regard to the Civil Code, while Examples: sale with delivery, exchange, donation,
the registration under Section 14(2) of the Property succession, and dacion in payment.
Registration Decree is founded on extraordinary
prescription under the Civil Code. The OSG submits that the requirement of just title
necessarily precludes the applicability of ordinary
It may be asked why the principles of prescription under acquisitive prescription to patrimonial property. The major
the Civil Code should not apply as well to Section 14(1). premise for the argument is that "the State, as the owner
Notwithstanding the vaunted status of the Civil Code, it and grantor, could not transmit ownership to the possessor
ultimately is just one of numerous statutes, neither before the completion of the required period of
superior nor inferior to other statutes such as the Property possession."47 It is evident that the OSG erred when it
Registration Decree. The legislative branch is not bound to assumed that the grantor referred to in Article 1129 is the
adhere to the framework set forth by the Civil Code when it State. The grantor is the one from whom the person
enacts subsequent legislation. Section 14(2) manifests a invoking ordinary acquisitive prescription derived the title,
clear intent to interrelate the registration allowed under whether by sale, exchange, donation, succession or any
that provision with the Civil Code, but no such intent exists other mode of the acquisition of ownership or other real
with respect to Section 14(1). rights.

IV. Earlier, we made it clear that, whether under ordinary


prescription or extraordinary prescription, the period of
One of the keys to understanding the framework we set possession preceding the classification of public dominion
forth today is seeing how our land registration procedures lands as patrimonial cannot be counted for the purpose of
correlate with our law on prescription, which, under the computing prescription. But after the property has been
Civil Code, is one of the modes for acquiring ownership become patrimonial, the period of prescription begins to
over property. run in favor of the possessor. Once the requisite period
has been completed, two legal events ensue: (1) the
The Civil Code makes it clear that patrimonial property of patrimonial property is ipso jure converted into private
the State may be acquired by private persons through land; and (2) the person in possession for the periods
prescription. This is brought about by Article 1113, which prescribed under the Civil Code acquires ownership of the
states that "[a]ll things which are within the commerce of property by operation of the Civil Code.
man are susceptible to prescription," and that [p]roperty of
the State or any of its subdivisions not patrimonial in It is evident that once the possessor automatically
character shall not be the object of prescription." becomes the owner of the converted patrimonial property,
the ideal next step is the registration of the property under
There are two modes of prescription through which the Torrens system. It should be remembered that
immovables may be acquired under the Civil Code. The registration of property is not a mode of acquisition of
first is ordinary acquisitive prescription, which, under ownership, but merely a mode of confirmation of
Article 1117, requires possession in good faith and with ownership.
just title; and, under Article 1134, is completed through
possession of ten (10) years. There is nothing in the Civil Looking back at the registration regime prior to the
Code that bars a person from acquiring patrimonial adoption of the Property Registration Decree in 1977, it is
property of the State through ordinary acquisitive apparent that the registration system then did not fully
prescription, nor is there any apparent reason to impose accommodate the acquisition of ownership of patrimonial
such a rule. At the same time, there are indispensable property under the Civil Code. What the system
requisitesgood faith and just title. The ascertainment of accommodated was the confirmation of imperfect title
Law on NatRes (23-49) 21
brought about by the completion of a period of possession period for the acquisition of property of the public dominion
ordained under the Public Land Act (either 30 years begin to run.
following Rep. Act No. 1942, or since 12 June 1945
following P.D. No. 1073). (a) Patrimonial property is private property of the
government. The person acquires ownership of
The Land Registration Act49 was noticeably silent on the patrimonial property by prescription under the Civil Code is
requisites for alienable public lands acquired through entitled to secure registration thereof under Section 14(2)
ordinary prescription under the Civil Code, though it of the Property Registration Decree.
arguably did not preclude such registration.50 Still, the gap
was lamentable, considering that the Civil Code, by itself, (b) There are two kinds of prescription by which
establishes ownership over the patrimonial property of patrimonial property may be acquired, one ordinary and
persons who have completed the prescriptive periods other extraordinary. Under ordinary acquisitive
ordained therein. The gap was finally closed with the prescription, a person acquires ownership of a patrimonial
adoption of the Property Registration Decree in 1977, with property through possession for at least ten (10) years, in
Section 14(2) thereof expressly authorizing original good faith and with just title. Under extraordinary
registration in favor of persons who have acquired acquisitive prescription, a persons uninterrupted adverse
ownership over private lands by prescription under the possession of patrimonial property for at least thirty (30)
provisions of existing laws, that is, the Civil Code as of years, regardless of good faith or just title, ripens into
now. ownership.

V. B.

We synthesize the doctrines laid down in this case, as We now apply the above-stated doctrines to the case at
follows: bar.

(1) In connection with Section 14(1) of the Property It is clear that the evidence of petitioners is insufficient to
Registration Decree, Section 48(b) of the Public Land Act establish that Malabanan has acquired ownership over the
recognizes and confirms that "those who by themselves or subject property under Section 48(b) of the Public Land
through their predecessors in interest have been in open, Act. There is no substantive evidence to establish that
continuous, exclusive, and notorious possession and Malabanan or petitioners as his predecessors-in-interest
occupation of alienable and disposable lands of the public have been in possession of the property since 12 June
domain, under a bona fide claim of acquisition of 1945 or earlier. The earliest that petitioners can date back
ownership, since June 12, 1945" have acquired ownership their possession, according to their own evidencethe
of, and registrable title to, such lands based on the length Tax Declarations they presented in particularis to the
and quality of their possession. year 1948. Thus, they cannot avail themselves of
registration under Section 14(1) of the Property
(a) Since Section 48(b) merely requires possession since Registration Decree.
12 June 1945 and does not require that the lands should
have been alienable and disposable during the entire Neither can petitioners properly invoke Section 14(2) as
period of possession, the possessor is entitled to secure basis for registration. While the subject property was
judicial confirmation of his title thereto as soon as it is declared as alienable or disposable in 1982, there is no
declared alienable and disposable, subject to the competent evidence that is no longer intended for public
timeframe imposed by Section 47 of the Public Land use service or for the development of the national
Act.51 evidence, conformably with Article 422 of the Civil Code.
The classification of the subject property as alienable and
(b) The right to register granted under Section 48(b) of the disposable land of the public domain does not change its
Public Land Act is further confirmed by Section 14(1) of status as property of the public dominion under Article
the Property Registration Decree. 420(2) of the Civil Code. Thus, it is insusceptible to
acquisition by prescription.
(2) In complying with Section 14(2) of the Property
Registration Decree, consider that under the Civil Code, VI.
prescription is recognized as a mode of acquiring
ownership of patrimonial property. However, public domain A final word. The Court is comfortable with the correctness
lands become only patrimonial property not only with a of the legal doctrines established in this decision.
declaration that these are alienable or disposable. There Nonetheless, discomfiture over the implications of todays
must also be an express government manifestation that ruling cannot be discounted. For, every untitled property
the property is already patrimonial or no longer retained for that is occupied in the country will be affected by this
public service or the development of national wealth, ruling. The social implications cannot be dismissed lightly,
under Article 422 of the Civil Code. And only when the and the Court would be abdicating its social responsibility
property has become patrimonial can the prescriptive
Law on NatRes (23-49) 22
to the Filipino people if we simply levied the law without For our consideration and resolution are the motions for
comment. reconsideration of the parties who both assail the decision
promulgated on April 29, 2009, whereby we upheld the
The informal settlement of public lands, whether declared ruling of the Court of Appeals (CA) denying the application
alienable or not, is a phenomenon tied to long-standing of the petitioners for the registration of a parcel of land
habit and cultural acquiescence, and is common among situated in Barangay Tibig, Silang, Cavite on the ground
the so-called "Third World" countries. This paradigm that they had not established by sufficient evidence their
powerfully evokes the disconnect between a legal system right to the registration in accordance with either Section
and the reality on the ground. The law so far has been 14(1) or Section 14(2) of Presidential Decree No. 1529
unable to bridge that gap. Alternative means of acquisition (Property Registration Decree).
of these public domain lands, such as through homestead
or free patent, have Antecedents

proven unattractive due to limitations imposed on the The property subject of the application for registration is a
grantee in the encumbrance or alienation of said parcel of land situated in Barangay Tibig, Silang Cavite,
properties.52 Judicial confirmation of imperfect title has more particularly identified as Lot 9864-A, Cad-452-D, with
emerged as the most viable, if not the most attractive an area of 71,324-square meters. On February 20, 1998,
means to regularize the informal settlement of alienable or applicant Mario Malabanan, who had purchased the
disposable lands of the public domain, yet even that property from Eduardo Velazco, filed an application for
system, as revealed in this decision, has considerable land registration covering the property in the Regional Trial
limits. Court (RTC) in Tagaytay City, Cavite, claiming that the
property formed part of the alienable and disposable land
There are millions upon millions of Filipinos who have of the public domain, and that he and his predecessors-in-
individually or exclusively held residential lands on which interest had been in open, continuous, uninterrupted,
they have lived and raised their families. Many more have public and adverse possession and occupation of the land
tilled and made productive idle lands of the State with their for more than 30 years, thereby entitling him to the judicial
hands. They have been regarded for generation by their confirmation of his title.
families and their communities as common law owners.
There is much to be said about the virtues of according To prove that the property was an alienable and
them legitimate states. Yet such virtues are not for the disposable land of the public domain, Malabanan
Court to translate into positive law, as the law itself presented during trial a certification dated June 11, 2001
considered such lands as property of the public dominion. issued by the Community Environment and Natural
It could only be up to Congress to set forth a new phase of Resources Office (CENRO) of the Department of
land reform to sensibly regularize and formalize the Environment and Natural Resources (DENR), which
settlement of such lands which in legal theory are lands of reads:
the public domain before the problem becomes insoluble.
This could be accomplished, to cite two examples, by This is to certify that the parcel of land designated as Lot
liberalizing the standards for judicial confirmation of No. 9864 Cad 452-D, Silang Cadastre as surveyed for Mr.
imperfect title, or amending the Civil Code itself to ease Virgilio Velasco located at Barangay Tibig, Silang, Cavite
the requisites for the conversion of public dominion containing an area of 249,734 sq. meters as shown and
property into patrimonial. described on the Plan Ap-04-00952 is verified to be within
the Alienable or Disposable land per Land Classification
Ones sense of security over land rights infuses into every Map No. 3013 established under Project No. 20-A and
aspect of well-being not only of that individual, but also to approved as such under FAO 4-1656 on March 15, 1982.2
the persons family. Once that sense of security is
deprived, life and livelihood are put on stasis. It is for the After trial, on December 3, 2002, the RTC rendered
political branches to bring welcome closure to the long judgment granting Malabanans application for land
pestering problem. registration, disposing thusly:

WHEREFORE, the Petition is DENIED. The Decision of WHEREFORE, this Court hereby approves this application
the Court of Appeals dated 23 February 2007 and for registration and thus places under the operation of Act
Resolution dated 2 October 2007 are AFFIRMED. No 141, Act 496 and/or P.D. 1529, otherwise known as
pronouncement as to costs. SO ORDERED. Property Registration Law, the lands described in Plan
Csd-04-0173123-D, Lot 9864-A and containing an area of
HEIRS OF MARIO MALABANAN, (Represented by Seventy One Thousand Three Hundred Twenty Four
Sally A. Malabanan), Petitioners, vs. REPUBLIC (71,324) Square Meters, as supported by its technical
OF THE PHILIPPINES, Respondent.; G.R. No. 179987 description now forming part of the record of this case, in
September 3, 2013; BERSAMIN, J.: addition to other proofs adduced in the name of MARIO
MALABANAN, who is of legal age, Filipino, widower, and
with residence at Munting Ilog, Silang, Cavite.
Law on NatRes (23-49) 23
Once this Decision becomes final and executory, the previously classified as agricultural land of the public
corresponding decree of registration shall forthwith issue. domain.

SO ORDERED. As earlier stated, we denied the petition for review on


certiorari because Malabanan failed to establish by
The Office of the Solicitor General (OSG) appealed the sufficient evidence possession and occupation of the
judgment to the CA, arguing that Malabanan had failed to property on his part and on the part of his predecessors-in
prove that the property belonged to the alienable and interest since June 12, 1945, or earlier.
disposable land of the public domain, and that the RTC
erred in finding that he had been in possession of the Petitioners Motion for Reconsideration
property in the manner and for the length of time required
by law for confirmation of imperfect title. In their motion for reconsideration, the petitioners submit
that the mere classification of the land as alienable or
On February 23, 2007, the CA promulgated its decision disposable should be deemed sufficient to convert it into
reversing the RTC and dismissing the application for patrimonial property of the State. Relying on the rulings in
registration of Malabanan. Citing the ruling in Republic v. Spouses De Ocampo v. Arlos,7 Menguito v. Republic8
Herbieto (Herbieto),4 the CA declared that under Section and Republic v. T.A.N. Properties, Inc.,9 they argue that
14(1) of the Property Registration Decree, any period of the reclassification of the land as alienable or disposable
possession prior to the classification of the land as opened it to acquisitive prescription under the Civil Code;
alienable and disposable was inconsequential and should that Malabanan had purchased the property from Eduardo
be excluded from the computation of the period of Velazco believing in good faith that Velazco and his
possession. Noting that the CENRO-DENR certification predecessors-in-interest had been the real owners of the
stated that the property had been declared alienable and land with the right to validly transmit title and ownership
disposable only on March 15, 1982, Velazcos possession thereof; that consequently, the ten-year period prescribed
prior to March 15, 1982 could not be tacked for purposes by Article 1134 of the Civil Code, in relation to Section
of computing Malabanans period of possession. 14(2) of the Property Registration Decree, applied in their
favor; and that when Malabanan filed the application for
Due to Malabanans intervening demise during the appeal registration on February 20, 1998, he had already been in
in the CA, his heirs elevated the CAs decision of February possession of the land for almost 16 years reckoned from
23, 2007 to this Court through a petition for review on 1982, the time when the land was declared alienable and
certiorari. disposable by the State.

The petitioners assert that the ruling in Republic v. Court The Republics Motion for Partial Reconsideration
of Appeals and Corazon Naguit5 (Naguit) remains the
controlling doctrine especially if the property involved is The Republic seeks the partial reconsideration in order to
agricultural land. In this regard, Naguit ruled that any obtain a clarification with reference to the application of the
possession of agricultural land prior to its declaration as rulings in Naguit and Herbieto.
alienable and disposable could be counted in the
reckoning of the period of possession to perfect title under Chiefly citing the dissents, the Republic contends that the
the Public Land Act (Commonwealth Act No. 141) and the decision has enlarged, by implication, the interpretation of
Property Registration Decree. They point out that the Section 14(1) of the Property Registration Decree through
ruling in Herbieto, to the effect that the declaration of the judicial legislation. It reiterates its view that an applicant is
land subject of the application for registration as alienable entitled to registration only when the land subject of the
and disposable should also date back to June 12, 1945 or application had been declared alienable and disposable
earlier, was a mere obiter dictum considering that the land since June 12, 1945 or earlier.
registration proceedings therein were in fact found and
declared void ab initio for lack of publication of the notice Ruling
of initial hearing.
We deny the motions for reconsideration.
The petitioners also rely on the ruling in Republic v. T.A.N.
In reviewing the assailed decision, we consider to be
Properties, Inc.6 to support their argument that the
imperative to discuss the different classifications of land in
property had been ipso jure converted into private property
relation to the existing applicable land registration laws of
by reason of the open, continuous, exclusive and
the Philippines.
notorious possession by their predecessors-in-interest of
an alienable land of the public domain for more than 30 Classifications of land according to ownership
years. According to them, what was essential was that the
property had been "converted" into private property Land, which is an immovable property,10 may be
through prescription at the time of the application without classified as either of public dominion or of private
regard to whether the property sought to be registered was ownership.11 Land is considered of public dominion if it
either: (a) is intended for public use; or (b) belongs to the
Law on NatRes (23-49) 24
State, without being for public use, and is intended for agricultural.24 A positive act of the Government is
some public service or for the development of the national necessary to enable such reclassification,25 and the
wealth.12 Land belonging to the State that is not of such exclusive prerogative to classify public lands under
character, or although of such character but no longer existing laws is vested in the Executive Department, not in
intended for public use or for public service forms part of the courts.26 If, however, public land will be classified as
the patrimonial property of the State.13 Land that is other neither agricultural, forest or timber, mineral or national
than part of the patrimonial property of the State, park, or when public land is no longer intended for public
provinces, cities and municipalities is of private ownership service or for the development of the national wealth,
if it belongs to a private individual. thereby effectively removing the land from the ambit of
public dominion, a declaration of such conversion must be
Pursuant to the Regalian Doctrine (Jura Regalia), a legal made in the form of a law duly enacted by Congress or by
concept first introduced into the country from the West by a Presidential proclamation in cases where the President
Spain through the Laws of the Indies and the Royal is duly authorized by law to that effect.27 Thus, until the
Cedulas,14 all lands of the public domain belong to the Executive Department exercises its prerogative to classify
State.15 This means that the State is the source of any or reclassify lands, or until Congress or the President
asserted right to ownership of land, and is charged with declares that the State no longer intends the land to be
the conservation of such patrimony.16 used for public service or for the development of national
wealth, the Regalian Doctrine is applicable.
All lands not appearing to be clearly under private
ownership are presumed to belong to the State. Also, Disposition of alienable public lands
public lands remain part of the inalienable land of the
public domain unless the State is shown to have Section 11 of the Public Land Act (CA No. 141) provides
reclassified or alienated them to private persons.17 the manner by which alienable and disposable lands of the
public domain, i.e., agricultural lands, can be disposed of,
Classifications of public lands according to alienability to wit:

Whether or not land of the public domain is alienable and Section 11. Public lands suitable for agricultural purposes
disposable primarily rests on the classification of public can be disposed of only as follows, and not otherwise:
lands made under the Constitution. Under the 1935
Constitution,18 lands of the public domain were classified (1) For homestead settlement;
into three, namely, agricultural, timber and mineral.19
Section 10, Article XIV of the 1973 Constitution classified (2) By sale;
lands of the public domain into seven, specifically,
agricultural, industrial or commercial, residential, (3) By lease; and
resettlement, mineral, timber or forest, and grazing land,
(4) By confirmation of imperfect or incomplete titles;
with the reservation that the law might provide other
classifications. The 1987 Constitution adopted the (a) By judicial legalization; or
classification under the 1935 Constitution into agricultural,
forest or timber, and mineral, but added national parks.20 (b) By administrative legalization (free patent).
Agricultural lands may be further classified by law
according to the uses to which they may be devoted.21 The core of the controversy herein lies in the proper
The identification of lands according to their legal interpretation of Section 11(4), in relation to Section 48(b)
classification is done exclusively by and through a positive of the Public Land Act, which expressly requires
act of the Executive Department.22 possession by a Filipino citizen of the land since June 12,
1945, or earlier, viz:
Based on the foregoing, the Constitution places a limit on
the type of public land that may be alienated. Under Section 48. The following-described citizens of the
Section 2, Article XII of the 1987 Constitution, only Philippines, occupying lands of the public domain or
agricultural lands of the public domain may be alienated; claiming to own any such lands or an interest therein, but
all other natural resources may not be. whose titles have not been perfected or completed, may
apply to the Court of First Instance of the province where
Alienable and disposable lands of the State fall into two the land is located for confirmation of their claims and the
categories, to wit: (a) patrimonial lands of the State, or issuance of a certificate of title thereafter, under the Land
those classified as lands of private ownership under Article Registration Act, to wit:
425 of the Civil Code,23 without limitation; and (b) lands of
the public domain, or the public lands as provided by the xxxx
Constitution, but with the limitation that the lands must only
be agricultural. Consequently, lands classified as forest or (b) Those who by themselves or through their
timber, mineral, or national parks are not susceptible of predecessors-in-interest have been in open, continuous,
alienation or disposition unless they are reclassified as exclusive, and notorious possession and occupation of
Law on NatRes (23-49) 25
alienable and disposable lands of the public domain, under The dissent stresses that the classification or
a bona fide claim of acquisition of ownership, since June reclassification of the land as alienable and disposable
12, 1945, or earlier, immediately preceding the filing of the agricultural land should likewise have been made on June
applications for confirmation of title, except when 12, 1945 or earlier, because any possession of the land
prevented by war or force majeure. These shall be prior to such classification or reclassification produced no
conclusively presumed to have performed all the legal effects. It observes that the fixed date of June 12,
conditions essential to a Government grant and shall be 1945 could not be minimized or glossed over by mere
entitled to a certificate of title under the provisions of this judicial interpretation or by judicial social policy concerns,
chapter. (Bold emphasis supplied) and insisted that the full legislative intent be respected.

Note that Section 48(b) of the Public Land Act used the We find, however, that the choice of June 12, 1945 as the
words "lands of the public domain" or "alienable and reckoning point of the requisite possession and occupation
disposable lands of the public domain" to clearly signify was the sole prerogative of Congress, the determination of
that lands otherwise classified, i.e., mineral, forest or which should best be left to the wisdom of the lawmakers.
timber, or national parks, and lands of patrimonial or Except that said date qualified the period of possession
private ownership, are outside the coverage of the Public and occupation, no other legislative intent appears to be
Land Act. What the law does not include, it excludes. The associated with the fixing of the date of June 12, 1945.
use of the descriptive phrase "alienable and disposable" Accordingly, the Court should interpret only the plain and
further limits the coverage of Section 48(b) to only the literal meaning of the law as written by the legislators.
agricultural lands of the public domain as set forth in
Article XII, Section 2 of the 1987 Constitution. Bearing in Moreover, an examination of Section 48(b) of the Public
mind such limitations under the Public Land Act, the Land Act indicates that Congress prescribed no
applicant must satisfy the following requirements in order requirement that the land subject of the registration should
for his application to come under Section 14(1) of the have been classified as agricultural since June 12, 1945,
Property Registration Decree,28 to wit: or earlier. As such, the applicants imperfect or incomplete
title is derived only from possession and occupation since
1. The applicant, by himself or through his predecessor-in- June 12, 1945, or earlier. This means that the character of
interest, has been in possession and occupation of the the property subject of the application as alienable and
property subject of the application; disposable agricultural land of the public domain
determines its eligibility for land registration, not the
2. The possession and occupation must be open, ownership or title over it.
continuous, exclusive, and notorious;
Alienable public land held by a possessor, either
3. The possession and occupation must be under a bona personally or through his predecessors-in-interest, openly,
fide claim of acquisition of ownership; continuously and exclusively during the prescribed
statutory period is converted to private property by the
4. The possession and occupation must have taken place mere lapse or completion of the period.29 In fact, by virtue
since June 12, 1945, or earlier; and of this doctrine, corporations may now acquire lands of the
public domain for as long as the lands were already
5. The property subject of the application must be an
converted to private ownership, by operation of law, as a
agricultural land of the public domain.
result of satisfying the requisite period of possession
Taking into consideration that the Executive Department is prescribed by the Public Land Act.30 It is for this reason
vested with the authority to classify lands of the public that the property subject of the application of Malabanan
domain, Section 48(b) of the Public Land Act, in relation to need not be classified as alienable and disposable
Section 14(1) of the Property Registration Decree, agricultural land of the public domain for the entire
presupposes that the land subject of the application for duration of the requisite period of possession.
registration must have been already classified as
To be clear, then, the requirement that the land should
agricultural land of the public domain in order for the
have been classified as alienable and disposable
provision to apply. Thus, absent proof that the land is
agricultural land at the time of the application for
already classified as agricultural land of the public domain,
registration is necessary only to dispute the presumption
the Regalian Doctrine applies, and overcomes the
that the land is inalienable.
presumption that the land is alienable and disposable as
laid down in Section 48(b) of the Public Land Act. The declaration that land is alienable and disposable also
However, emphasis is placed on the requirement that the serves to determine the point at which prescription may
classification required by Section 48(b) of the Public Land run against the State. The imperfect or incomplete title
Act is classification or reclassification of a public land as being confirmed under Section 48(b) of the Public Land
agricultural. Act is title that is acquired by reason of the applicants
possession and occupation of the alienable and
disposable agricultural land of the public domain. Where
Law on NatRes (23-49) 26
all the necessary requirements for a grant by the incomplete title. By legal fiction, the land has already
Government are complied with through actual physical, ceased to be part of the public domain and has become
open, continuous, exclusive and public possession of an private property.
alienable and disposable land of the public domain, the
possessor is deemed to have acquired by operation of law (b) Lands of the public domain subsequently classified or
not only a right to a grant, but a grant by the Government, declared as no longer intended for public use or for the
because it is not necessary that a certificate of title be development of national wealth are removed from the
issued in order that such a grant be sanctioned by the sphere of public dominion and are considered converted
courts.31 into patrimonial lands or lands of private ownership that
may be alienated or disposed through any of the modes of
If one follows the dissent, the clear objective of the Public acquiring ownership under the Civil Code. If the mode of
Land Act to adjudicate and quiet titles to unregistered acquisition is prescription, whether ordinary or
lands in favor of qualified Filipino citizens by reason of extraordinary, proof that the land has been already
their occupation and cultivation thereof for the number of converted to private ownership prior to the requisite
years prescribed by law32 will be defeated. Indeed, we acquisitive prescriptive period is a condition sine qua non
should always bear in mind that such objective still in observance of the law (Article 1113, Civil Code) that
prevails, as a fairly recent legislative development bears property of the State not patrimonial in character shall not
out, when Congress enacted legislation (Republic Act No. be the object of prescription.
10023)33 in order to liberalize stringent requirements and
procedures in the adjudication of alienable public land to To reiterate, then, the petitioners failed to present
qualified applicants, particularly residential lands, subject sufficient evidence to establish that they and their
to area limitations. predecessors-in-interest had been in possession of the
land since June 12, 1945. Without satisfying the requisite
On the other hand, if a public land is classified as no character and period of possession - possession and
longer intended for public use or for the development of occupation that is open, continuous, exclusive, and
national wealth by declaration of Congress or the notorious since June 12, 1945, or earlier - the land cannot
President, thereby converting such land into patrimonial or be considered ipso jure converted to private property even
private land of the State, the applicable provision upon the subsequent declaration of it as alienable and
concerning disposition and registration is no longer disposable. Prescription never began to run against the
Section 48(b) of the Public Land Act but the Civil Code, in State, such that the land has remained ineligible for
conjunction with Section 14(2) of the Property Registration registration under Section 14(1) of the Property
Decree.35 As such, prescription can now run against the Registration Decree. Likewise, the land continues to be
State. ineligible for land registration under Section 14(2) of the
Property Registration Decree unless Congress enacts a
To sum up, we now observe the following rules relative to law or the President issues a proclamation declaring the
the disposition of public land or lands of the public domain, land as no longer intended for public service or for the
namely: development of the national wealth.

(1) As a general rule and pursuant to the Regalian WHEREFORE, the Court DENIES the petitioners' Motion
Doctrine, all lands of the public domain belong to the State for Reconsideration and the respondent's Partial Motion
and are inalienable. Lands that are not clearly under for Reconsideration for their lack of merit.SO ORDERED.
private ownership are also presumed to belong to the
State and, therefore, may not be alienated or disposed; JAMES R. BRACEWELL, petitioner, vs. HONORABLE
COURT OF APPEALS and REPUBLIC OF THE
(2) The following are excepted from the general rule, to
PHILIPPINES, respondents.; G.R. No. 107427
wit:
January 25, 2000; YNARES-SANTIAGO, J.:
(a) Agricultural lands of the public domain are rendered
Before us is a petition to affirm the Order of the Regional
alienable and disposable through any of the exclusive
Trial Court of Makati, Branch 58, in LRC Case No. M-77,1
modes enumerated under Section 11 of the Public Land
which was reversed by respondent Court of Appeals in its
Act. If the mode is judicial confirmation of imperfect title
Decision dated June 29, 1992 in CA-G.R. CV No. 26122.2
under Section 48(b) of the Public Land Act, the agricultural
Petitioner's Motion for Reconsideration was denied by
land subject of the application needs only to be classified
respondent court on September 30, 1992.3
as alienable and disposable as of the time of the
application, provided the applicants possession and The controversy involves a total of nine thousand six
occupation of the land dated back to June 12, 1945, or hundred fifty-seven (9,657) square meters of land located
earlier. Thereby, a conclusive presumption that the in Las Pias, Metro Manila. The facts show that sometime
applicant has performed all the conditions essential to a in 1908, Maria Cailles, married to James Bracewell, Sr.,
government grant arises,36 and the applicant becomes acquired the said parcels of land from the Dalandan and
the owner of the land by virtue of an imperfect or
Law on NatRes (23-49) 27
Jimenez families of Las Pias; after which corresponding declaration by the Bureau of Forestry in March 27, 1972,
Tax Declarations were issued in the name of Maria the parcels of land sought to be registered by Applicant
Cailles. On January 16, 1961, Maria Cailles sold the said was part of the forest land or forest reserves.
parcels of land to her son, the petitioner, by virtue of a
Deed of Sale which was duly annotated and registered III. The Honorable Court of Appeal ERRED and failed to
with the Registry of Deeds of Pasig, Rizal. Tax consider VESTED RIGHTS of the applicant-appellant and
Declarations were thereafter issued in the name of his predecessors-in-interest land occupied from 1908.12
petitioner, cancelling the previous Tax Declarations issued
to Maria Cailles. The controversy is simple. On one hand, petitioner asserts
his right of title to the subject land under Section 48 (b) of
On September 19, 1963, petitioner filed before the then Commonwealth Act No. 141, having by himself and
Court of First Instance of Pasig, Rizal an action for through his predecessors-in-interest been in open,
confirmation of imperfect title under Section 48 of continuous, exclusive and notorious possession and
Commonwealth Act No. 141.4 The case was docketed as occupation of the subject parcels of land, under a bona
L.R.C. Case No. 4328. On February 21, 1964, the Director fide claim of acquisition or ownership, since 1908. On the
of Lands, represented by the Solicitor General, opposed other hand, it is the respondents' position that since the
petitioner's application on the grounds that neither he nor subject parcels of land were only classified as alienable or
his predecessors-in-interest possessed sufficient title to disposable on March 27, 1972,13 petitioner did not have
the subject land nor have they been in open, continuous, any title to confirm when he filed his application in 1963.
exclusive and notorious possession and occupation of the Neither was the requisite thirty years possession met.
same for at least thirty (30) years prior to the application,
and that the subject land is part of the public domain.5 We agree with respondents.

The registration proceedings were meanwhile suspended In Republic vs. Doldol,14 the requisites to acquire title to
on account of an action filed by Crescencio Leonardo public land were laid down, as follows
against Maria Cailles before the then Court of First
. . . . The original Section 48(b) of C.A. No. 141 provided
Instance of Pasig, Rizal. The case was finally disposed of
for possession and occupation of lands of the public
by this Court in G.R. No. 51263 where the rights of Maria
domain since July 26, 1894. This was superseded by R.A.
Cailles were upheld over those of the oppositor
No. 1942 which provided for a simple thirty-year
Leonardo.6
prescriptive period of occupation by an applicant for
On March 26, 1985, the entire records of the registration judicial confirmation of imperfect title. The same, however,
case were forwarded to the Makati Regional Trial Court7 has already been amended by Presidential Decree No.
where it was docketed as Land Registration Case No. M- 1073, approved on January 25, 1977. As amended,
77. The Solicitor General resubmitted his opposition to the Section 48(b) now reads:
application on July 22, 1985,8 this time alleging the
(b) Those who by themselves or through their
following additional grounds: (1) the failure of petitioner to
predecessors-in-interest have been in open, continuous,
prosecute his action for an unreasonable length of time;
exclusive and notorious possession and occupation of
and (2) that the tax declarations attached to the complaint
agricultural lands of the public domain, under a bona fide
do not constitute acquisition of the lands applied for.
claim of acquisition or ownership, since June 12, 1945, or
On May 3, 1989, the lower court issued an Order granting earlier, immediately preceding the filing of the application
the application of petitioner.9 The Solicitor General for confirmation of title, except when prevented by wars or
promptly appealed to respondent Court which, on June 29, force majeure. Those shall be conclusively presumed to
1992, reversed and set aside the lower court's Order.10 It have performed all the conditions essential to a
also denied petitioner's Motion for Reconsideration in its Government grant and shall be entitled to a certificate of
Resolution of September 30, 1992.11 title under the provisions of this chapter. (emphasis in the
original).
Hence, the instant Petition anchored upon the following
grounds Thus, in the aforecited Republic vs. CA case, we stated
that the Public Land Act requires that the applicant must
I. The Honorable Court of Appeals ERRED in finding that prove (a) that the land is alienable public land and (b) that
the commencement of thirty (30) year period mandated his open, continuous, exclusive and notorious possession
under Sec. 48 (b) shall commence only on March 27, 1972 and occupation of the same must be since time
in accordance with the classification made by the Bureau immemorial or for the period prescribed in the Public Land
of Forestry in First (1st) Indorsement dated August 20, Act. When the conditions set by law are complied with, the
1986. possessor of the land, by operation of law, acquires a right
to a grant, a government grant, without the necessity of a
II. The Honorable Court of Appeals committed an ERROR certificate of title being issued.
in DRAWING conclusion and inference that prior to the
Law on NatRes (23-49) 28
Clear from the above is the requirement that the applicant the rules on the confirmation of imperfect title do not apply
must prove that the land is alienable public land. On this unless and until the land classified as forest land is
score, we agree with respondents that petitioner failed to released in an official proclamation to that effect so that it
show that the parcels of land subject of his application are may form part of the disposable agricultural lands of the
alienable or disposable. On the contrary, it was public domain.19
conclusively shown by the government that the same were
only classified as alienable or disposable on March 27, Neither has petitioner shown proof that the subject
1972. Thus, even granting that petitioner and his Forestry Administrative Order recognizes private or vested
predecessors-in-interest had occupied the same since rights under which his case may fall. We only find on
1908, he still cannot claim title thereto by virtue of such record the Indorsement of the Bureau of Forest
possession since the subject parcels of land were not yet Development20 from which no indication of such
alienable land at that time nor capable of private exemption may be gleaned.
appropriation. The adverse possession which may be the
basis of a grant of title or confirmation of an imperfect title Having found petitioner to have no cause of action for his
refers only to alienable or disposable portions of the public application for confirmation of imperfect title, we see no
domain.15 need to discuss the other errors raised in this petition.

A similar situation in the case of Reyes v. Court of WHEREFORE, premises considered, the instant Petition
Appeals,16 where a homestead patent issued to the is hereby DENIED for lack of merit. No pronouncement as
petitioners' predecessor-in-interest was cancelled on the to costs. SO ORDERED.
ground that at the time it was issued, the subject land was
still part of the public domain. In the said case, this Court REPUBLIC OF THE PHILIPPINES, Petitioner, v. THE
ruled as follows HON. COURT OF APPEALS, and EMILIO BERNABE,
SR., EMILIO BERNABE, JR., LUZ BERNABE, AMPARO
Under the Regalian doctrine, all lands of the public domain BERNABE, and ELISA BERNABE, Respondents.; [G.R.
belong to the State, and that the State is the source of any No. L-40402. March 16, 1987.]; PARAS, J.:
asserted right to ownership in land and charged with the
conservation of such patrimony. This same doctrine also This is a petition for review on certiorari seeking a reversal
states that all lands not otherwise appearing to be clearly of the decision of Respondent Court of Appeals 1 dated
within private ownership are presumed to belong to the February 5, 1975 in CA-G.R. No. 50076-R, entitled
State (Director of Lands vs. Intermediate Appellate Court, "EMILIO BERNABE, SR., Et. Al. v. REPUBLIC OF THE
219 SCRA 340). PHILIPPINES," affirming the order of the Court of First
Instance of Bataan dated August 14, 1971 in Cadastral
Hence, the burden of proof in overcoming the presumption Case No. 19, LRC Cadastral Record No. 1097, which
of State ownership of lands of the public domain is on the dismissed petitioner Republics petition for review of the
person applying for registration. The applicant must show decrees of registration issued pursuant to the decision
that the land subject of the application is alienable or rendered on December 17, 1968 adjudicating in favor of
disposable. This petitioners failed to do.1wphi1.nt the private Respondents herein, the lots applied for by
them, and the Resolution of Respondent Court dated
We have stated earlier that at the time the homestead March 19, 1975 denying herein Petitioners motion for
patent was issued to petitioners' predecessor-in-interest, reconsideration.
the subject land belong to the inalienable and
undisposable portion of the public domain. Thus, any title The undisputed facts are as follows:
issued in their name by mistake or oversight is void ab
initio because at the time the homestead patent was Lot No. 622 of the Mariveles Cadastre was declared public
issued to petitioners, as successors-in-interest of the land in a decision rendered before the last war in
original patent applicant, the Director of Lands was not Cadastral Case No. 19, LRC Cadastral Record No. 1097.
then authorized to dispose of the same because the area
was not yet classified as disposable public land. On July 6, 1965, Lot 622 was segregated from the forest
Consequently, the title issued to herein petitioners by the zone and released and certified by the Bureau of Forestry
Bureau of Lands is void ab initio. as an agricultural land for disposition under the Public
Land Act (Record on Appeal, p. 7).
Prior to March 27, 1972, when the subject parcels of land
were classified as inalienable or indisposable, therefore, On April 26, 1967, Respondents filed in the Court of First
the same could not be the subject of confirmation of Instance of Bataan a petition to reopen Cadastral Case
imperfect title. There can be no imperfect title to be No. 19, LRC Cadastral Record No. 1097, under Republic
confirmed over lands not yet classified as disposable or Act 931, as amended by Republic Act 2061, concerning a
alienable.17 In the absence of such classification, the land portion of Lot No. 622 Lot Nos. 792, 793, 794, 795, 796,
remains unclassified public land until released therefrom 797, 798 and a portion of Lot No. 324 Lot Nos. 791
and open to disposition.18 Indeed, it has been held that and 799 more particularly identified and delineated in
Law on NatRes (23-49) 29
the segregation plans of Sgs-3343, Sgs-3440, Sgs-3340, Sgs-3440 Sgs-799 64,052. sq. m.
Sgs-3341, Sgs-3342 and Sgs-3339, approved by the
Director of Lands, to perfect their rights and register their and upon this decision having become final, the
titles to said lots, having allegedly acquired ownership and Commissioner of Land Registration is hereby directed to
possession of said parcels of land by purchase from the issue the corresponding decrees of registration therefor."
original owners thereof, whose possession of the same
including that of the herein Respondents, has always been Pursuant to the aforecited decision, the Commissioner of
continuous, open, active, exclusive, public, adverse, and in Land Registration issued Decrees Nos. N-124813-124818,
the concept of owners thereof for more than 30 years all dated May 7, 1969 (Record on Appeal, pp. 20-25).
(Record on Appeal, pp. 3-5 and 11).
On May 7, 1979, petitioner Republic of the Philippines,
On May 17, 1967, the lower court issued an Order setting acting in its behalf and in behalf of the Director of Lands
the petition for hearing and directing that the Republic of and the Director of Forestry, through the Solicitor-General,
the Philippines be notified thereof by furnishing the filed a petition for review of the decrees of registration
Solicitor-General, the Director of Lands and the Director of under Section 38, of Act No. 496, as amended, and the
Forestry, a copy of said Order together with Respondents corresponding decision of the lower court, on the grounds
petition by registered mail (Record on Appeal, p. 6). that the entire proceeding was vitiated by lack of notice to
the Solicitor General of the subsequent hearings of the
On August 24, 1967, the Director of Forestry filed an petition for re-opening of the cadastral proceedings; that
opposition to the petition praying for the denial of the the parcels of land subject matter of the petition to re-open
petition once the area involved is found to be within the cadastral proceedings are portions of the public domain,
timberland and therefore inalienable under the Constitution admittedly within the unclassified public forest of
(Record on Appeal, p. 7). Upon verification, however, the Mariveles, Bataan, opened for disposition only on or about
Director of Forestry found the area to be the portion of the July 6, 1965; that subsequently, respondents do not have
timberland already released by the government from the a registerable title to the land subject matter of the
mass of public forests and promptly withdrew his proceedings; and the lower court, without jurisdiction to
Opposition (Record on Appeal, p. 8). decree the confirmation of registerable title to respondents
over portions of the public domain, as respondents do not
On September 1, 1967, the Acting Provincial Fiscal of qualify under the provisions of Section 48(b) of CA 141, as
Bataan, for and in behalf of the Director of Lands, filed his amended, and that under the circumstances, respondents
opposition to the petition alleging that the land is still, in employed actual fraud in procuring title over the parcels of
truth and in fact, public land and as such cannot be the land (Record on Appeal, p. 25).
subject of a land registration proceeding under Act 496.
On May 29, 1979, respondents moved to dismiss the
The lower court found that the petitioners have complied Petition for Review on the grounds that: (1) The trial court
with all the terms and conditions which would entitle them has no jurisdiction over the nature of the action or suit as
to a grant. Thus, the dispositive portion of its decision there is no fraud to justify the setting aside on review of a
dated December 17, 1968 (Record on Appeal, p. 19), decree of registration. If the Solicitor General was not
reads: notified of the subsequent hearings, it was because he
delegated his appearance to the Provincial Fiscal of
"WHEREFORE, the segregation plans, Sgs-3340, Sgs- Bataan. Besides the setting aside or review was filed out
3339, Sgs-3341, Sgs-3342, Sgs-3343 and Sgs-3340 and of time. (2) The petition states no cause of action, the
their technical descriptions are hereby APPROVED, and parcels of land involved in the actions having been already
pursuant to Sec. 11 of Act 2259, the court hereby transferred to innocent purchasers for value long before
adjudicates in favor of petitioners Emilio Bernabe, Sr., the Solicitor-General even filed the petition for review
married; Emilio Bernabe, Jr., married; Luz Bernabe, single; (Record on Appeal, pp. 27-40).
Amparo Bernabe, single and Elisa Bernabe, single, all
Filipinos and residents of Balanga, Bataan, the lots herein Their motion to dismiss having been held in abeyance until
applied for as follows: the hearing of the merits of the case which was set for
August 16, 1970, respondents filed their answer to the
Luz Bernabe Sgs-791 82,771 sq. m. Petition for Review on August 4, 1970. In their answer,
3339 respondents reiterated their grounds in their motion to
Elisa Bernabe Sgs-793 71.596 sq. m. dismiss (Record on Appeal, pp. 40-44).
3341
Amparo Bernabe Sgs-794 43,399 sq. m. On November 12, 1970, Petitioner filed an amended
3342 795 100,439 sq. m. Petition for Review, with the additional allegation that after
Josefina Bernabe Sgs-796 69,355 sq. m. having fraudulently secured title over the parcels of land
3343 797 75,100 sq. m. involved, the petitioners executed simulated deeds of sale
Emilio Bernabe, Jr. Sgs-798 100,183 sq. m. purporting to convey various lots composing portions of
the parcels involved to third parties for fictitious
Law on NatRes (23-49) 30
considerations in an obvious attempt to remove the III. THE RESPONDENT COURT ERRED IN NOT
parcels of land involved from the coverage of Section 38 of HOLDING THAT THE ALLEGED TRANSFER OF THE
Act 496, but in truth, the aforementioned third parties are LOTS IN QUESTION BY PRIVATE RESPONDENTS TO
not innocent purchasers for value, being mere dummies of THIRD PARTIES WHEN THEIR TITLES WERE STILL
the petitioners, holding the parcels of land involved only in SUBJECT TO THE ONE-YEAR PERIOD OF REVIEW
trust for the petitioners. On November 23, 1970, CONSTITUTES FRAUD SCHEMED BY THE
respondents filed their answer to the Amended Petition for TRANSFERORS AS A MEANS OF FRUSTRATING ANY
Review (Record on Appeal, p. 56). ACTION AIMED AT NULLIFYING THEIR TITLES
THERETO.
On August 14, 1971, the lower court issued its Order
denying petitioners Amended Petition for Review (Record The governments cause is meritorious.
on Appeal, p. 56).
I
On appeal to the Court of Appeals on September 20,
1971, the questioned Order of the Court of First Instance It is evident from the facts of the case at bar that private
of Bataan, Branch I was affirmed (Rollo, p. 33). respondents did file a claim for Lot No. 622 of the
Mariveles Cadastre and in fact a decision was rendered
On February 25, 1975, Petitioner filed a Motion for before the last war in Cadastral Case No. 19 LRC
Reconsideration which was denied by the Court of Cadastral Record No. 1097, declaring the lot in question
Appeals for lack of merit, in the Resolution of a special as public land. It must be stressed that said lot was
Division of Five, promulgated on March 19, 1975. declared public land by virtue of a court decision which
has become final and as held by the Supreme Court
Hence this petition. aforesaid decision is res judicata. (Republic v. Estenzo,
120 SCRA 222 [1983]). It is therefore beyond question that
Without giving due course to the Petition, the Court, the trial court has no jurisdiction to reopen the cadastral
through its First Division, resolved on May 5, 1975 to proceeding under R.A. 931 as amended by R.A. 2061 and
require the respondents to comment thereon. On May 30, the decision therein rendered is null and void ab initio.
1975, respondents filed their comment, alleging that the
decision of respondent Court and the questioned Furthermore, it is undisputed that aforesaid Lot No. 622
resolution were not rendered without or in excess of its was released as an agricultural land for disposition under
jurisdiction. Neither was the discretion exercised by Public Land Act only on July 6, 1965. The lower court
respondent Court arbitrary or despotic. ordered the issuance of the corresponding decrees of
registration for the lots, pursuant to Sec. 48(b), C.A. 141,
In its Resolution dated June 4, 1975, the Court resolved to otherwise known as the Public Land Act, as amended by
give due course to the Petition and denied the urgent Republic Act No. 1942, providing for the confirmation of
motion of respondents for leave to file a supplemental imperfect or incomplete titles, which reads:
and/or amended comment. Petitioners filed its Brief on
November 29, 1975; respondents, on March 2, 1976. "(b) Those who by themselves or through their
Petitioner filed its Reply Brief on March 25, 1976 and on predecessors in interest have been in open, continuous,
May 5, 1976, the case was deemed submitted for exclusive, and notorious possession and occupation of
decision. agricultural lands of the public domain, under a bona fide
claim of acquisition of ownership, for at least thirty years
Petitioner assigns the following errors: immediately preceding the filing of the application for
confirmation of title except when prevented by war or force
I. THE RESPONDENT COURT OF APPEALS
majeure. These shall be conclusively presumed to have
COMMITTED A REVERSIBLE ERROR IN TOTALLY
performed all the conditions essential to a Government
DISREGARDING THE UNDISPUTED FACT THAT THE
grant and shall be entitled to a certificate of title under the
LOTS CLAIMED BY HEREIN PRIVATE RESPONDENTS
provisions of this chapter."
BECAME AGRICULTURAL ONLY ON JULY 6, 1965
WHEN THE SAME WERE RELEASED FROM THE As pointed out by petitioner, the question is whether or not
FOREST ZONE AND THAT CONSEQUENTLY THEY the lots claimed by respondents could legally be the
LACK THE REQUISITE THIRTY (30) YEARS subject of a judicial confirmation of title under the
POSSESSION TO ENTITLE THEM TO A GRANT. aforequoted provisions of the Public Land Act, as
amended.
II. THE RESPONDENT COURT OF APPEALS
COMMITTED A REVERSIBLE ERROR IN NOT HOLDING The answer is in the negative.
THAT THE ENTIRE PROCEEDING FOR REOPENING
OF THE CADASTRAL CASE OVER THE LOTS IN Section 48(b) of C.A. No. 141, as amended, applies
QUESTION WAS VITIATED BY LACK OF NOTICE TO exclusively to public agricultural land. Forest lands or
THE SOLICITOR-GENERAL. areas covered with forests are excluded. They are
Law on NatRes (23-49) 31
incapable of registration and their inclusion in a title, service on the Solicitor-General, and added that technical
whether such title be one issued during the Spanish transgressions relative to the filing and service may be
sovereignty or under the present Torrens system of brushed aside when the adverse party (this time the
registration, nullifies the title (Li Seng Giap v. Director of Director of Lands and Forestry and their counsel, the
Lands, 55 Phil. 693 [1931]; Director of Lands v. Reyes, 68 Solicitor-General) is aware of the matter which his
SCRA 177 [1975]). Thus, possession of forest lands, adversary would want the court to act upon. Once it
however long, cannot ripen into private ownership (Vano v. appears that the party is already informed by one means
Government, 41 Phil. 161 [1920]; Adorable v. Director of or another of what he is to be notified, the required service
Forestry, 107 Phil. 401 [1960]; Director of Forestry v. becomes an empty gesture and strict observance thereof
Muoz, 23 SCRA 1183 [1968]; Director of Lands v. is considered waived. (Citing Estrada v. Sto. Domingo, 28
Abanzado, 65 SCRA 5 [1975]). A parcel of forest land is SCRA 890 [1969]).
within the exclusive jurisdiction of the Bureau of Forestry
and beyond the power and jurisdiction of the cadastral In the case at bar, it does not appear that the Solicitor
court to register under the Torrens System (Republic v. General was so apprised of the decision of the lower court
Court of Appeals, 89 SCRA 648 [1979]; Republic v. Vera in question as there is no proof that the Provincial Fiscal of
(120 SCRA 210 [1983]; Director of Lands v. Court of Bataan ever sent the Solicitor-General a copy thereof.
Appeals, 129 SCRA 689 [1984]. Furthermore, after the 3rd Assistant Provincial Fiscal filed
a notice of appeal from the decision of the trial court, the
Thus, even if the reopening of the cadastral proceedings Provincial Fiscal on March 21, 1969 manifested that he
was at all possible, private respondents have not qualified was withdrawing the appeal upon the intervention of the
for a grant under Sec. 48(b) of Commonwealth Act 141, District Forester. (Respondents Brief, p. 44).
the facts being that private respondents could only be
credited with 1 year, 9 months and 20 days possession It will be observed however that later decisions of the
and occupation of the lots involved, counted from July 6, Supreme Court tend to be more strict in the matter of
1965, the date when the land area in sitio San Jose, barrio giving notice to the Solicitor General. In a more recent
Cabcaban, Mariveles, Bataan, known as Bataan PMD No. case, Republic v. Court of Appeals, 135 SCRA 161 [1985],
267, which includes the lots claimed by respondents, had it was established that the Solicitor-General is the only
been segregated from the forest zone and released by the legal counsel of the government in land registration cases
Bureau of Forestry as an agricultural land for disposition and as such, he alone may withdraw the Governments
under the Public Land Act. (Record on Appeal, p. 19). appeal with binding effect on the latter. He is entitled to be
Consequently, under the above mentioned jurisprudence, furnished copies of all court orders, notices and decisions
neither private respondents nor their predecessors-in- and as held the reglementary thirty-day period for appeal
interest could have possessed the lots for the requisite should be reckoned from the time the Solicitor-Generals
period of thirty (30) years as disposable agricultural land. Office is apprised of the 1970 order of denial and not from
the time the special counsel or the fiscal was served with
II that order. Thus, representatives of the Solicitor General in
the case at bar, had no power to decide whether or not an
Petitioner argues that the government, being a necessary appeal should be made. They should have referred the
party in the cadastral case, as reopened, its counsel, the matter to the Solicitor-General and without copies of court
Solicitor-General, should have been furnished copies of all orders, notices and decisions, having been provided by
court orders, notices and decisions, as in ordinary cases, either the trial court or the Provincial Fiscal of Bataan to
in order to bind the government. Failure to give such the Solicitor-General, the assailed decision has no binding
notice deprives the State of its day in Court, and renders effect on the government.
the decision void. (Brief for Petitioner, pp. 16-17).
III
The records show that the Solicitor-General was duly
notified of the initial hearing on the petition to reopen The petition for review of Decrees Nos. N-124813 to N-
Cadastral Case No. 19 but thereafter, notice of 124818 under Sec. 38 of Act No. 496 as amended was
subsequent hearings as well as a copy of the decision filed by the Solicitor General on May 7, 1970 in
itself promulgated by the lower court on December 19, representation of the Republic of the Philippines, in the
1968 was sent instead to the Provincial Fiscal of Bataan, same Cadastral Case No. 19, LRC Cadastral Record No.
admittedly the duly authorized representative of the 1097, exactly a year after the issuance of aforesaid
Solicitor-General in the cadastral proceeding as shown in decrees of registration, on the ground of actual fraud.
a telegram dated January 19, 1968. (Record on Appeal, p. (Record on Appeal, pp. 43-44).
47).
The basic elements for the allowance of the reopening or
In the case of Republic v. Director of Lands (71 SCRA 426 review of a decree, are: (1) that the petitioner has real or
[1976], the Supreme Court, applying the time-honored dominical right; (2) that he has been deprived thereof
principle of agency ruled that the service of the questioned through fraud; (3) that the petition is filed within one year
decision on the Provincial Fiscal must necessarily be from the issuance of the decree and (4) that the property
Law on NatRes (23-49) 32
has not as yet been transferred to an innocent purchaser. lands for disposition, which release is tantamount to
(Libudan v. Gil, 45 SCRA 27 [1972]; Rubico, Et. Al. v. qualifying the latter to a grant on said lands while they
Orellana, 30 SCRA 513 [1969]). It has been held however were still non-disposable. Thus, under the foregoing
that the action to annul a judgment, upon the ground of rulings, even assuming that the transferees are innocent
fraud would be unavailing unless the fraud be extrinsic or purchasers for value, their titles to said lands derived from
collateral and the facts upon which it is based have not the titles of private respondents which were not validly
been controverted or resolved in the case where the issued as they cover lands still a part of the public domain,
judgment sought to be annulled was rendered. (Libudan v. may be cancelled.
Gil, supra). Review of the decree demands a showing of
actual (not constructive) fraud, i.e. actual malice. (Rublico PREMISES CONSIDERED, the assailed decision of the
v. Orellana, supra). Court of Appeals and the decision of the Court of First
Instance are hereby SET ASIDE and REVERSED,
In the case at bar, it cannot be said that private because the lots in question still form part of the public
respondents employed actual fraud in procuring titles over domain. The certificates of title issued over them are
parcels of land of the public domain as it is a matter of hereby ordered CANCELLED. SO ORDERED.
record that the land in question was opened for disposition
and alienation only on July 6, 1965. The matter was REPUBLIC OF THE PHILIPPINES, Petitioner vs.
threshed out in the lower court and the decision of the APOLONIO BAUTISTA, JR., Respondent.; G.R. No.
latter was affirmed by the Court of Appeals. Actual malice 166890; June 28, 2016; BERSAMIN, J.:
is therefore absent.
The applicant for judicial confirmation of imperfect title
However, it has been held that, if a decree issued in must trace his possession of the subject land to June 12,
pursuance of a valid decision, obtained by fraud, may be 1945, or earlier. Any length of possession that does not
annulled within one (1) year from entry of said decree, comply with the requirement cannot support the
there is more reason to hold that the same is true if application, which must be then dismissed for failure to
entered in compliance with a decision suffering from a fatal comply with Commonwealth Act No. 141 (Public Land Act)
infirmity, such as want of due process, (Vda. de Cuaycong and Presidential Decree No. 1529 (Property Registration
v. Vda. de Sangbengoo, 110 Phil. 118 [1960] or lack of Decree).
jurisdiction of the court that decided the cadastral case.
(Republic v. De Kalintas, 25 SCRA 720 [1969]). Thus, on The Case
both counts, the case at bar can properly be the subject of
review, it having been shown that the Solicitor-General The Government appeals the adverse judgment
was not properly furnished the requisite notices and copy promulgated on September 30, 2004, 1 whereby the Court
of the assailed decision but more importantly, the lower of Appeals (CA) affirmed the decision of the Municipal
court as previously stated had no jurisdiction to re-open Trial Court (MTC) of Subic, Zambales rendered on
the cadastral proceeding under Republic Act 931 as November 17, 1998 in LRC Case No. N-12-10-96 entitled
amended by R.A. No. 2061. In Re: Application for Land Registration of Lot 17078 of
Cad. 547-D, Subic Cadastre2granting the application of
IV respondent Apolonio Bautista, Jr. for the judicial
confirmation of title of Lot 17078 of Cad. 547-0, Subic
As to whether or not the transferees of the lot in question Cadastre.
are innocent purchasers for value, it is a well settled rule
that a purchaser cannot close his eyes to facts which Antecedents
should put a reasonable man upon his guard, and then
claim that he acted in good faith under the belief that there After acquiring Lot 17078 of Cad. 547-D, Subic Cadastre,
was no defect in the title of the vendor. (Leung Yee v. F.L. located in Capisanan, Subic, Zambales from Mario Jardin
Strong Machiner Co., Et Al., 37 Phil. 651[1918]. Without on February 15, 1971 and Cornelia Villanueva on May 25,
the needed verification, he cannot claim to be an innocent 1973, Apolonio, Sr. had the property declared for taxation
purchaser for value in contemplation of law. purposes. He had been the sole and exclusive possessor
and occupant from the time of acquisition until his death,
Moreover, it is well-settled that a certificate of title is void, with no party questioning his possession and ownership,
when it covers property of public domain classified as or staking any adverse claim against him thereon.3 He
forest or timber and mineral lands. Any title issued on non- died in 1987, and was succeeded by his children, namely:
disposable lots even in the hands of an alleged innocent respondent Apolonio, Jr. and his siblings. Apolonio, Sr.'s
purchaser for value, shall be cancelled. (Lepanto children executed an extra-judicial settlement of their
Consolidated Mining Company v. Dumyung, 89 SCRA 540 father's estate, whereby Apolonio, Jr.'s brothers and
[1979] underscoring supplied). In the case at bar, it will be sisters waived their rights in his favor. Thus, the property
noted that in granting titles to the land in dispute, the lower was declared for taxation purposes in Apolonio, Jr.'s name
court counted the period of possession of private under Tax Declaration No. 014-0432A of the Municipality
respondents before the same were released as forest
Law on NatRes (23-49) 33
of Subic, Zambales. There were no arrears in real estate on hearsay evidence showed his unfitness to own the
taxes.4 The declared value was 73,040.00. 5 land.

On October 21, 1996, Apolonio Jr. commenced LRC Case In response, Apolonio Jr. insists that he had duly
No. N-12-10-96 in the MTC. He later on testified that his established his lawful occupation of the land as owner in
father had been in actual possession since 1969, and had fee simple; that the Government did not timely object to his
eventually acquired the land from Jardin and Villanueva testimony, and did not also controvert his evidence; that
through the notarized Deeds of Absolute Sale dated the property had been properly identified; and that the
February 15, 1971, and May 25, 1973; and that his father lower courts had observed the legal safeguards and
had paid taxes on the land. guidelines in granting his application for judicial
confirmation of his ownership in fee simple.
The Government did not interpose any timely objection to
the testimony of Apolonia, Jr. It did not also object to the Ruling of the Court
documentary evidence (i.e., the deeds of absolute sale
and tax declarations) offered by him.1wphi1 Hence, the We reverse.
MTC admitted all the evidence presented by Apolonia, Jr.
The Government has correctly insisted that the requisite
In due course, the MTC granted Apolonia, Jr.'s application, period of possession of the property should conform to
and declared him as the owner in fee simple of the land,6 that provided for in Section 48(b) of the Public Land Act,
and confirmed his ownership thereof. 7 as amended by Presidential Decree No. 1073, which has
limited the right to apply for judicial confirmation to citizens
The Government appealed the decision to the Court of of the Philippines "who by themselves or through their
Appeals (CA), which, on September 30, 2004, predecessors in interest have been in open, continuous,
promulgated its assailed decision affirming the ruling of the exclusive, and notorious possession and occupation of
MTC.8 The CA pointed out that the Government did not alienable and disposable lands of the public domain, under
present evidence against the claim of Apolonio Jr.; and a bona fide claim of acquisition of ownership, since June
that the Government did not timely object to his testimony 12, 1945, or earlier, immediately preceding the filing of the
on the ground of its being hearsay. application for confirmation of title except when prevented
by war or force majeure. x x x" The provision is reprised by
Issue Section 14(1) of Presidential Decree No. 1529 (Property
Registration Decree), adopting the length of possession
In this appeal, the Government reiterates that the and occupation of alienable and disposable lands of the
testimony of Apolonio, Jr. on possession, being hearsay, public domain under a bona fide claim of ownership since
had no probative value; that the alienation of public land June 12, 1945, or earlier.
should always undergo careful scrutiny; and that the Court
should carefully re-examine the factual issues that could We note that in its amendment of the Public Land Act that
alter the result of the case. 10 took effect on January 25, 1977, Presidential Decree No.
1073 changed the length of the requisite possession from
The Government points out that Apolonio, Jr. had given "thirty (30) years immediately preceding the filing of the
only general statements pertaining to the open, application" to possession "since June 12, 1945, or
continuous, exclusive and notorious possession of his earlier." Republic v. Naguit15has explained this change
father since 1971; that such statements were mere thusly:
conclusions of law, and did not prove the alleged
possession; that because the application for judicial When the Public Land Act was first promulgated in
confirmation of imperfect title was filed on October 21, 1936, the period of possession deemed necessary to vest
1996, the applicable law was Section 48(b) of the right to register their title to agricultural lands of the
Commonwealth Act No. 141 (Public land Act), as public domain commenced from July 26, 1894. However,
amended by Presidential Decree No. 1073; that, this period was amended by R.A. No. 1942, which
accordingly, the required period of possession must be provided that the bona fide claim of ownership must have
"since June 12, 1945 or earlier," as stated in Republic v. been for at least thirty (30) years. Then in 1977, Section
Doldol, 11 a more stringent requirement the non- 48(b) of the Public Land Act was again amended, this time
compliance with which was fatal to his cause. by P.D. No. 1073, which pegged the reckoning date at
June 12, 1945. xxx
Lastly, the Government points out that tax declarations or
tax receipts did not suffice to prove ownership of land in Based on the records before us, Apolonia, Jr. presented
fee simple; that although it was the State's policy to only himself to establish the possession and ownership of
encourage and promote distribution of alienable public his father, Apolonia, Sr., who was his immediate
lands as an ideal of social justice, stringent safeguards predecessor-in-interest. He did not present as witnesses
must be adopted and applied to prevent the lands from during the trial either of the transferors of Apolonia, Sr. -
going to the wrong hands; and that Apolonio, Jr.'s reliance that is, Mario Jardin or Cornelia Villanueva - to establish
Law on NatRes (23-49) 34
the requisite length of the possession of the predecessors- Executive Order Nos. 9091 and 2982 amended the MIAA
in-interest of the applicant that would be tacked to his own. Charter.
His personal incompetence to attest to the possession of
the property within the time required by law underscored As operator of the international airport, MIAA administers
the weakness of the evidence on possession, particularly the land, improvements and equipment within the NAIA
as it has not been denied that the applicant had arrived in Complex. The MIAA Charter transferred to MIAA
the Philippines only on November 28, 1987. Considering approximately 600 hectares of land,3 including the
that the possession and occupation of the property in runways and buildings ("Airport Lands and Buildings") then
question by Apolonia, Jr. and his predecessors-in-interest under the Bureau of Air Transportation.4 The MIAA
were not shown in the records to have been "since June Charter further provides that no portion of the land
12, 1945, or earlier," the application must be rejected. transferred to MIAA shall be disposed of through sale or
any other mode unless specifically approved by the
We should stress that only the title of those who had President of the Philippines.5
possessed and occupied alienable and disposable lands
of the public domain within the requisite period could be On 21 March 1997, the Office of the Government
judicially confirmed. Indeed, alienable public land held by a Corporate Counsel (OGCC) issued Opinion No. 061. The
possessor, either personally or through his predecessors- OGCC opined that the Local Government Code of 1991
in-interest, openly, continuously and exclusively during the withdrew the exemption from real estate tax granted to
prescribed statutory period is converted to private property MIAA under Section 21 of the MIAA Charter. Thus, MIAA
by the mere lapse or completion of the period. 16 negotiated with respondent City of Paraaque to pay the
real estate tax imposed by the City. MIAA then paid some
That the Government did not timely object to the of the real estate tax already due.
admission of the testimony of Apolonia, Jr., or of the other
evidence presented by him was of no consequence to the On 28 June 2001, MIAA received Final Notices of Real
success of the application. If he had no personal Estate Tax Delinquency from the City of Paraaque for the
knowledge of the facts establishing the possession of taxable years 1992 to 2001. MIAA's real estate tax
property for the requisite period, no court can give any delinquency is broken down as follows:
value to his assertion, particularly as it was conceded by
him no less that he had no personal or direct competence 1992-1997 RPT was paid on Dec. 24, 1997 as per
to know the truth of his assertion. It was one thing for the O.R.#9476102 for P4,207,028.75
trial court to admit the evidence, but quite another to give it
#9476101 for P28,676,480.00
any worth for purposes of judicial adjudication.
#9476103 for P49,115.00
WHEREFORE, the Court GRANTS the petition for review
on certiorari; REVERSES and SETS ASIDE the decision On 17 July 2001, the City of Paraaque, through its City
promulgated on September 30, 2004; DISMISSES the Treasurer, issued notices of levy and warrants of levy on
application of respondent Apolonia Bautista, Jr. for the the Airport Lands and Buildings. The Mayor of the City of
judicial confirmation of his imperfect title in LRC Case No. Paraaque threatened to sell at public auction the Airport
N-12-10-96; and ORDERS Apolonia Bautista, Jr. to pay Lands and Buildings should MIAA fail to pay the real
the costs of suit.SO ORDERED. estate tax delinquency. MIAA thus sought a clarification of
OGCC Opinion No. 061.
MANILA INTERNATIONAL AIRPORT AUTHORITY,
petitioner, vs. COURT OF APPEALS, CITY OF On 9 August 2001, the OGCC issued Opinion No. 147
PARAAQUE, CITY MAYOR OF PARAAQUE, clarifying OGCC Opinion No. 061. The OGCC pointed out
SANGGUNIANG PANGLUNGSOD NG PARAAQUE, that Section 206 of the Local Government Code requires
CITY ASSESSOR OF PARAAQUE, and CITY persons exempt from real estate tax to show proof of
TREASURER OF PARAAQUE, respondents.; G.R. No. exemption. The OGCC opined that Section 21 of the MIAA
155650; July 20, 2006; CARPIO, J.: Charter is the proof that MIAA is exempt from real estate
tax.
The Antecedents
On 1 October 2001, MIAA filed with the Court of Appeals
Petitioner Manila International Airport Authority (MIAA) an original petition for prohibition and injunction, with
operates the Ninoy Aquino International Airport (NAIA) prayer for preliminary injunction or temporary restraining
Complex in Paraaque City under Executive Order No. order. The petition sought to restrain the City of
903, otherwise known as the Revised Charter of the Paraaque from imposing real estate tax on, levying
Manila International Airport Authority ("MIAA Charter"). against, and auctioning for public sale the Airport Lands
Executive Order No. 903 was issued on 21 July 1983 by and Buildings. The petition was docketed as CA-G.R. SP
then President Ferdinand E. Marcos. Subsequently, No. 66878.
Law on NatRes (23-49) 35
On 5 October 2001, the Court of Appeals dismissed the MIAA also points out that Section 21 of the MIAA Charter
petition because MIAA filed it beyond the 60-day specifically exempts MIAA from the payment of real estate
reglementary period. The Court of Appeals also denied on tax. MIAA insists that it is also exempt from real estate tax
27 September 2002 MIAA's motion for reconsideration and under Section 234 of the Local Government Code
supplemental motion for reconsideration. Hence, MIAA because the Airport Lands and Buildings are owned by the
filed on 5 December 2002 the present petition for review.7 Republic. To justify the exemption, MIAA invokes the
principle that the government cannot tax itself. MIAA
Meanwhile, in January 2003, the City of Paraaque posted points out that the reason for tax exemption of public
notices of auction sale at the Barangay Halls of Barangays property is that its taxation would not inure to any public
Vitalez, Sto. Nio, and Tambo, Paraaque City; in the advantage, since in such a case the tax debtor is also the
public market of Barangay La Huerta; and in the main tax creditor.
lobby of the Paraaque City Hall. The City of Paraaque
published the notices in the 3 and 10 January 2003 issues Respondents invoke Section 193 of the Local Government
of the Philippine Daily Inquirer, a newspaper of general Code, which expressly withdrew the tax exemption
circulation in the Philippines. The notices announced the privileges of "government-owned and-controlled
public auction sale of the Airport Lands and Buildings to corporations" upon the effectivity of the Local Government
the highest bidder on 7 February 2003, 10:00 a.m., at the Code. Respondents also argue that a basic rule of
Legislative Session Hall Building of Paraaque City. statutory construction is that the express mention of one
person, thing, or act excludes all others. An international
A day before the public auction, or on 6 February 2003, at airport is not among the exceptions mentioned in Section
5:10 p.m., MIAA filed before this Court an Urgent Ex-Parte 193 of the Local Government Code. Thus, respondents
and Reiteratory Motion for the Issuance of a Temporary assert that MIAA cannot claim that the Airport Lands and
Restraining Order. The motion sought to restrain Buildings are exempt from real estate tax.
respondents the City of Paraaque, City Mayor of
Paraaque, Sangguniang Panglungsod ng Paraaque, Respondents also cite the ruling of this Court in Mactan
City Treasurer of Paraaque, and the City Assessor of International Airport v. Marcos8 where we held that the
Paraaque ("respondents") from auctioning the Airport Local Government Code has withdrawn the exemption
Lands and Buildings. from real estate tax granted to international airports.
Respondents further argue that since MIAA has already
On 7 February 2003, this Court issued a temporary paid some of the real estate tax assessments, it is now
restraining order (TRO) effective immediately. The Court estopped from claiming that the Airport Lands and
ordered respondents to cease and desist from selling at Buildings are exempt from real estate tax.
public auction the Airport Lands and Buildings.
Respondents received the TRO on the same day that the The Issue
Court issued it. However, respondents received the TRO
only at 1:25 p.m. or three hours after the conclusion of the This petition raises the threshold issue of whether the
public auction. Airport Lands and Buildings of MIAA are exempt from real
estate tax under existing laws. If so exempt, then the real
On 10 February 2003, this Court issued a Resolution estate tax assessments issued by the City of Paraaque,
confirming nunc pro tunc the TRO. and all proceedings taken pursuant to such assessments,
are void. In such event, the other issues raised in this
On 29 March 2005, the Court heard the parties in oral petition become moot.
arguments. In compliance with the directive issued during
the hearing, MIAA, respondent City of Paraaque, and the The Court's Ruling
Solicitor General subsequently submitted their respective
Memoranda. We rule that MIAA's Airport Lands and Buildings are
exempt from real estate tax imposed by local
MIAA admits that the MIAA Charter has placed the title to governments.
the Airport Lands and Buildings in the name of MIAA.
However, MIAA points out that it cannot claim ownership First, MIAA is not a government-owned or controlled
over these properties since the real owner of the Airport corporation but an instrumentality of the National
Lands and Buildings is the Republic of the Philippines. The Government and thus exempt from local taxation. Second,
MIAA Charter mandates MIAA to devote the Airport Lands the real properties of MIAA are owned by the Republic of
and Buildings for the benefit of the general public. Since the Philippines and thus exempt from real estate tax.
the Airport Lands and Buildings are devoted to public use
and public service, the ownership of these properties 1. MIAA is Not a Government-Owned or Controlled
remains with the State. The Airport Lands and Buildings Corporation
are thus inalienable and are not subject to real estate tax
Respondents argue that MIAA, being a government-
by local governments.
owned or controlled corporation, is not exempt from real
Law on NatRes (23-49) 36
estate tax. Respondents claim that the deletion of the Clearly, under its Charter, MIAA does not have capital
phrase "any government-owned or controlled so exempt stock that is divided into shares.
by its charter" in Section 234(e) of the Local Government
Code withdrew the real estate tax exemption of Section 3 of the Corporation Code10 defines a stock
government-owned or controlled corporations. The deleted corporation as one whose "capital stock is divided into
phrase appeared in Section 40(a) of the 1974 Real shares and x x x authorized to distribute to the holders of
Property Tax Code enumerating the entities exempt from such shares dividends x x x." MIAA has capital but it is not
real estate tax. divided into shares of stock. MIAA has no stockholders or
voting shares. Hence, MIAA is not a stock corporation.
There is no dispute that a government-owned or controlled
corporation is not exempt from real estate tax. However, MIAA is also not a non-stock corporation because it has
MIAA is not a government-owned or controlled no members. Section 87 of the Corporation Code defines
corporation. Section 2(13) of the Introductory Provisions of a non-stock corporation as "one where no part of its
the Administrative Code of 1987 defines a government- income is distributable as dividends to its members,
owned or controlled corporation as follows: trustees or officers." A non-stock corporation must have
members. Even if we assume that the Government is
SEC. 2. General Terms Defined. x x x x considered as the sole member of MIAA, this will not make
MIAA a non-stock corporation. Non-stock corporations
(13) Government-owned or controlled corporation refers to cannot distribute any part of their income to their
any agency organized as a stock or non-stock corporation, members. Section 11 of the MIAA Charter mandates MIAA
vested with functions relating to public needs whether to remit 20% of its annual gross operating income to the
governmental or proprietary in nature, and owned by the National Treasury.11 This prevents MIAA from qualifying
Government directly or through its instrumentalities either as a non-stock corporation.
wholly, or, where applicable as in the case of stock
corporations, to the extent of at least fifty-one (51) percent Section 88 of the Corporation Code provides that non-
of its capital stock: x x x. (Emphasis supplied) stock corporations are "organized for charitable, religious,
educational, professional, cultural, recreational, fraternal,
A government-owned or controlled corporation must be literary, scientific, social, civil service, or similar purposes,
"organized as a stock or non-stock corporation." MIAA is like trade, industry, agriculture and like chambers." MIAA
not organized as a stock or non-stock corporation. MIAA is is not organized for any of these purposes. MIAA, a public
not a stock corporation because it has no capital stock utility, is organized to operate an international and
divided into shares. MIAA has no stockholders or voting domestic airport for public use.
shares. Section 10 of the MIAA Charter9 provides:
Since MIAA is neither a stock nor a non-stock corporation,
SECTION 10. Capital. The capital of the Authority to be MIAA does not qualify as a government-owned or
contributed by the National Government shall be increased controlled corporation. What then is the legal status of
from Two and One-half Billion (P2,500,000,000.00) Pesos MIAA within the National Government?
to Ten Billion (P10,000,000,000.00) Pesos to consist of:
MIAA is a government instrumentality vested with
(a) The value of fixed assets including airport facilities, corporate powers to perform efficiently its governmental
runways and equipment and such other properties, functions. MIAA is like any other government
movable and immovable[,] which may be contributed by instrumentality, the only difference is that MIAA is vested
the National Government or transferred by it from any of with corporate powers. Section 2(10) of the Introductory
its agencies, the valuation of which shall be determined Provisions of the Administrative Code defines a
jointly with the Department of Budget and Management government "instrumentality" as follows:
and the Commission on Audit on the date of such
contribution or transfer after making due allowances for SEC. 2. General Terms Defined. x x x x
depreciation and other deductions taking into account the
loans and other liabilities of the Authority at the time of the (10) Instrumentality refers to any agency of the National
takeover of the assets and other properties; Government, not integrated within the department
framework, vested with special functions or jurisdiction by
(b) That the amount of P605 million as of December 31, law, endowed with some if not all corporate powers,
1986 representing about seventy percentum (70%) of the administering special funds, and enjoying operational
unremitted share of the National Government from 1983 to autonomy, usually through a charter. x x x (Emphasis
1986 to be remitted to the National Treasury as provided supplied)
for in Section 11 of E. O. No. 903 as amended, shall be
converted into the equity of the National Government in When the law vests in a government instrumentality
the Authority. Thereafter, the Government contribution to corporate powers, the instrumentality does not become a
the capital of the Authority shall be provided in the General corporation. Unless the government instrumentality is
Appropriations Act. organized as a stock or non-stock corporation, it remains a
Law on NatRes (23-49) 37
government instrumentality exercising not only When local governments invoke the power to tax on
governmental but also corporate powers. Thus, MIAA national government instrumentalities, such power is
exercises the governmental powers of eminent domain,12 construed strictly against local governments. The rule is
police authority13 and the levying of fees and charges.14 that a tax is never presumed and there must be clear
At the same time, MIAA exercises "all the powers of a language in the law imposing the tax. Any doubt whether a
corporation under the Corporation Law, insofar as these person, article or activity is taxable is resolved against
powers are not inconsistent with the provisions of this taxation. This rule applies with greater force when local
Executive Order."15 governments seek to tax national government
instrumentalities.
Likewise, when the law makes a government
instrumentality operationally autonomous, the Another rule is that a tax exemption is strictly construed
instrumentality remains part of the National Government against the taxpayer claiming the exemption. However,
machinery although not integrated with the department when Congress grants an exemption to a national
framework. The MIAA Charter expressly states that government instrumentality from local taxation, such
transforming MIAA into a "separate and autonomous exemption is construed liberally in favor of the national
body"16 will make its operation more "financially viable."17 government instrumentality. As this Court declared in
Maceda v. Macaraig, Jr.:
Many government instrumentalities are vested with
corporate powers but they do not become stock or non- The reason for the rule does not apply in the case of
stock corporations, which is a necessary condition before exemptions running to the benefit of the government itself
an agency or instrumentality is deemed a government- or its agencies. In such case the practical effect of an
owned or controlled corporation. Examples are the Mactan exemption is merely to reduce the amount of money that
International Airport Authority, the Philippine Ports has to be handled by government in the course of its
Authority, the University of the Philippines and Bangko operations. For these reasons, provisions granting
Sentral ng Pilipinas. All these government instrumentalities exemptions to government agencies may be construed
exercise corporate powers but they are not organized as liberally, in favor of non tax-liability of such agencies.
stock or non-stock corporations as required by Section
2(13) of the Introductory Provisions of the Administrative There is, moreover, no point in national and local
Code. These government instrumentalities are sometimes governments taxing each other, unless a sound and
loosely called government corporate entities. However, compelling policy requires such transfer of public funds
they are not government-owned or controlled corporations from one government pocket to another.
in the strict sense as understood under the Administrative
Code, which is the governing law defining the legal There is also no reason for local governments to tax
relationship and status of government entities. national government instrumentalities for rendering
essential public services to inhabitants of local
A government instrumentality like MIAA falls under Section governments. The only exception is when the legislature
133(o) of the Local Government Code, which states: clearly intended to tax government instrumentalities for the
delivery of essential public services for sound and
SEC. 133. Common Limitations on the Taxing Powers of compelling policy considerations. There must be express
Local Government Units. Unless otherwise provided language in the law empowering local governments to tax
herein, the exercise of the taxing powers of provinces, national government instrumentalities. Any doubt whether
cities, municipalities, and barangays shall not extend to such power exists is resolved against local governments.
the levy of the following:
Thus, Section 133 of the Local Government Code states
xxxx that "unless otherwise provided" in the Code, local
governments cannot tax national government
(o) Taxes, fees or charges of any kind on the National instrumentalities. As this Court held in Basco v. Philippine
Government, its agencies and instrumentalities and local Amusements and Gaming Corporation:
government units.(Emphasis and underscoring supplied)
The states have no power by taxation or otherwise, to
Section 133(o) recognizes the basic principle that local retard, impede, burden or in any manner control the
governments cannot tax the national government, which operation of constitutional laws enacted by Congress to
historically merely delegated to local governments the carry into execution the powers vested in the federal
power to tax. While the 1987 Constitution now includes government. (MC Culloch v. Maryland, 4 Wheat 316, 4 L
taxation as one of the powers of local governments, local Ed. 579)
governments may only exercise such power "subject to
such guidelines and limitations as the Congress may This doctrine emanates from the "supremacy" of the
provide."18 National Government over local governments.
Law on NatRes (23-49) 38
"Justice Holmes, speaking for the Supreme Court, made and Buildings constitute a "port" constructed by the State.
reference to the entire absence of power on the part of the Under Article 420 of the Civil Code, the MIAA Airport
States to touch, in that way (taxation) at least, the Lands and Buildings are properties of public dominion and
instrumentalities of the United States (Johnson v. thus owned by the State or the Republic of the Philippines.
Maryland, 254 US 51) and it can be agreed that no state
or political subdivision can regulate a federal The Airport Lands and Buildings are devoted to public use
instrumentality in such a way as to prevent it from because they are used by the public for international and
consummating its federal responsibilities, or even to domestic travel and transportation. The fact that the MIAA
seriously burden it in the accomplishment of them." collects terminal fees and other charges from the public
(Antieau, Modern Constitutional Law, Vol. 2, p. 140, does not remove the character of the Airport Lands and
emphasis supplied) Buildings as properties for public use. The operation by
the government of a tollway does not change the character
Otherwise, mere creatures of the State can defeat of the road as one for public use. Someone must pay for
National policies thru extermination of what local the maintenance of the road, either the public indirectly
authorities may perceive to be undesirable activities or through the taxes they pay the government, or only those
enterprise using the power to tax as "a tool for regulation" among the public who actually use the road through the
(U.S. v. Sanchez, 340 US 42). toll fees they pay upon using the road. The tollway system
is even a more efficient and equitable manner of taxing the
The power to tax which was called by Justice Marshall as public for the maintenance of public roads.
the "power to destroy" (Mc Culloch v. Maryland, supra)
cannot be allowed to defeat an instrumentality or creation The charging of fees to the public does not determine the
of the very entity which has the inherent power to wield it. character of the property whether it is of public dominion or
20 not. Article 420 of the Civil Code defines property of public
dominion as one "intended for public use." Even if the
2. Airport Lands and Buildings of MIAA are Owned by the government collects toll fees, the road is still "intended for
Republic public use" if anyone can use the road under the same
terms and conditions as the rest of the public. The
a. Airport Lands and Buildings are of Public Dominion charging of fees, the limitation on the kind of vehicles that
can use the road, the speed restrictions and other
The Airport Lands and Buildings of MIAA are property of
conditions for the use of the road do not affect the public
public dominion and therefore owned by the State or the
character of the road.
Republic of the Philippines. The Civil Code provides:
The terminal fees MIAA charges to passengers, as well as
ARTICLE 419. Property is either of public dominion or of
the landing fees MIAA charges to airlines, constitute the
private ownership.
bulk of the income that maintains the operations of MIAA.
ARTICLE 420. The following things are property of public The collection of such fees does not change the character
dominion: of MIAA as an airport for public use. Such fees are often
termed user's tax. This means taxing those among the
(1) Those intended for public use, such as roads, canals, public who actually use a public facility instead of taxing all
rivers, torrents, ports and bridges constructed by the State, the public including those who never use the particular
banks, shores, roadsteads, and others of similar character; public facility. A user's tax is more equitable a principle
of taxation mandated in the 1987 Constitution.
(2) Those which belong to the State, without being for
public use, and are intended for some public service or for The Airport Lands and Buildings of MIAA, which its
the development of the national wealth. (Emphasis Charter calls the "principal airport of the Philippines for
supplied) both international and domestic air traffic,"22 are
properties of public dominion because they are intended
ARTICLE 421. All other property of the State, which is not for public use. As properties of public dominion, they
of the character stated in the preceding article, is indisputably belong to the State or the Republic of the
patrimonial property. Philippines.

ARTICLE 422. Property of public dominion, when no b. Airport Lands and Buildings are Outside the Commerce
longer intended for public use or for public service, shall of Man
form part of the patrimonial property of the State.
The Airport Lands and Buildings of MIAA are devoted to
No one can dispute that properties of public dominion public use and thus are properties of public dominion. As
mentioned in Article 420 of the Civil Code, like "roads, properties of public dominion, the Airport Lands and
canals, rivers, torrents, ports and bridges constructed by Buildings are outside the commerce of man. The Court
the State," are owned by the State. The term "ports" has ruled repeatedly that properties of public dominion are
includes seaports and airports. The MIAA Airport Lands outside the commerce of man. As early as 1915, this Court
Law on NatRes (23-49) 39
already ruled in Municipality of Cavite v. Rojas that the auction sale of the 600-hectare runway of the MIAA for
properties devoted to public use are outside the commerce non-payment of real estate tax.
of man, thus:
Before MIAA can encumber26 the Airport Lands and
According to article 344 of the Civil Code: "Property for Buildings, the President must first withdraw from public
public use in provinces and in towns comprises the use the Airport Lands and Buildings. Sections 83 and 88 of
provincial and town roads, the squares, streets, fountains, the Public Land Law or Commonwealth Act No. 141, which
and public waters, the promenades, and public works of "remains to this day the existing general law governing the
general service supported by said towns or provinces." classification and disposition of lands of the public domain
other than timber and mineral lands,"27 provide:
The said Plaza Soledad being a promenade for public use,
the municipal council of Cavite could not in 1907 withdraw SECTION 83. Upon the recommendation of the Secretary
or exclude from public use a portion thereof in order to of Agriculture and Natural Resources, the President may
lease it for the sole benefit of the defendant Hilaria Rojas. designate by proclamation any tract or tracts of land of the
In leasing a portion of said plaza or public place to the public domain as reservations for the use of the Republic
defendant for private use the plaintiff municipality of the Philippines or of any of its branches, or of the
exceeded its authority in the exercise of its powers by inhabitants thereof, in accordance with regulations
executing a contract over a thing of which it could not prescribed for this purposes, or for quasi-public uses or
dispose, nor is it empowered so to do. purposes when the public interest requires it, including
reservations for highways, rights of way for railroads,
The Civil Code, article 1271, prescribes that everything hydraulic power sites, irrigation systems, communal
which is not outside the commerce of man may be the pastures or lequas communales, public parks, public
object of a contract, and plazas and streets are outside of quarries, public fishponds, working men's village and other
this commerce, as was decided by the supreme court of improvements for the public benefit.
Spain in its decision of February 12, 1895, which says:
"Communal things that cannot be sold because they are SECTION 88. The tract or tracts of land reserved under
by their very nature outside of commerce are those for the provisions of Section eighty-three shall be non-
public use, such as the plazas, streets, common lands, alienable and shall not be subject to occupation, entry,
rivers, fountains, etc." (Emphasis supplied) sale, lease, or other disposition until again declared
alienable under the provisions of this Act or by
Again in Espiritu v. Municipal Council, the Court declared proclamation of the President. (Emphasis and
that properties of public dominion are outside the underscoring supplied)
commerce of man:
Thus, unless the President issues a proclamation
xxx Town plazas are properties of public dominion, to be withdrawing the Airport Lands and Buildings from public
devoted to public use and to be made available to the use, these properties remain properties of public dominion
public in general. They are outside the commerce of man and are inalienable. Since the Airport Lands and Buildings
and cannot be disposed of or even leased by the are inalienable in their present status as properties of
municipality to private parties. While in case of war or public dominion, they are not subject to levy on execution
during an emergency, town plazas may be occupied or foreclosure sale. As long as the Airport Lands and
temporarily by private individuals, as was done and as was Buildings are reserved for public use, their ownership
tolerated by the Municipality of Pozorrubio, when the remains with the State or the Republic of the Philippines.
emergency has ceased, said temporary occupation or use
must also cease, and the town officials should see to it The authority of the President to reserve lands of the
that the town plazas should ever be kept open to the public domain for public use, and to withdraw such public
public and free from encumbrances or illegal private use, is reiterated in Section 14, Chapter 4, Title I, Book III
constructions.24 (Emphasis supplied) of the Administrative Code of 1987, which states:

The Court has also ruled that property of public dominion, SEC. 14. Power to Reserve Lands of the Public and
being outside the commerce of man, cannot be the subject Private Domain of the Government. (1) The President
of an auction sale.25 shall have the power to reserve for settlement or public
use, and for specific public purposes, any of the lands of
Properties of public dominion, being for public use, are not the public domain, the use of which is not otherwise
subject to levy, encumbrance or disposition through public directed by law. The reserved land shall thereafter remain
or private sale. Any encumbrance, levy on execution or subject to the specific public purpose indicated until
auction sale of any property of public dominion is void for otherwise provided by law or proclamation;
being contrary to public policy. Essential public services
will stop if properties of public dominion are subject to x x x x. (Emphasis supplied)
encumbrances, foreclosures and auction sale. This will
happen if the City of Paraaque can foreclose and compel
Law on NatRes (23-49) 40
There is no question, therefore, that unless the Airport SECTION 22. Transfer of Existing Facilities and Intangible
Lands and Buildings are withdrawn by law or presidential Assets. All existing public airport facilities, runways,
proclamation from public use, they are properties of public lands, buildings and other property, movable or
dominion, owned by the Republic and outside the immovable, belonging to the Airport, and all assets,
commerce of man. powers, rights, interests and privileges belonging to the
Bureau of Air Transportation relating to airport works or air
c. MIAA is a Mere Trustee of the Republic operations, including all equipment which are necessary
for the operation of crash fire and rescue facilities, are
MIAA is merely holding title to the Airport Lands and hereby transferred to the Authority. (Emphasis supplied)
Buildings in trust for the Republic. Section 48, Chapter 12,
Book I of the Administrative Code allows instrumentalities SECTION 25. Abolition of the Manila International Airport
like MIAA to hold title to real properties owned by the as a Division in the Bureau of Air Transportation and
Republic, thus: Transitory Provisions. The Manila International Airport
including the Manila Domestic Airport as a division under
SEC. 48. Official Authorized to Convey Real Property. the Bureau of Air Transportation is hereby abolished.
Whenever real property of the Government is authorized
by law to be conveyed, the deed of conveyance shall be x x x x.
executed in behalf of the government by the following:
The MIAA Charter transferred the Airport Lands and
(1) For property belonging to and titled in the name of the Buildings to MIAA without the Republic receiving cash,
Republic of the Philippines, by the President, unless the promissory notes or even stock since MIAA is not a stock
authority therefor is expressly vested by law in another corporation.
officer.
The whereas clauses of the MIAA Charter explain the
(2) For property belonging to the Republic of the rationale for the transfer of the Airport Lands and Buildings
Philippines but titled in the name of any political to MIAA, thus:
subdivision or of any corporate agency or instrumentality,
by the executive head of the agency or instrumentality. WHEREAS, the Manila International Airport as the
(Emphasis supplied) principal airport of the Philippines for both international
and domestic air traffic, is required to provide standards of
In MIAA's case, its status as a mere trustee of the Airport airport accommodation and service comparable with the
Lands and Buildings is clearer because even its executive best airports in the world;
head cannot sign the deed of conveyance on behalf of the
Republic. Only the President of the Republic can sign such WHEREAS, domestic and other terminals, general
deed of conveyance.28 aviation and other facilities, have to be upgraded to meet
the current and future air traffic and other demands of
d. Transfer to MIAA was Meant to Implement a aviation in Metro Manila;
Reorganization
WHEREAS, a management and organization study has
The MIAA Charter, which is a law, transferred to MIAA the indicated that the objectives of providing high standards of
title to the Airport Lands and Buildings from the Bureau of accommodation and service within the context of a
Air Transportation of the Department of Transportation and financially viable operation, will best be achieved by a
Communications. The MIAA Charter provides: separate and autonomous body; and

SECTION 3. Creation of the Manila International Airport WHEREAS, under Presidential Decree No. 1416, as
Authority. x x x x amended by Presidential Decree No. 1772, the President
of the Philippines is given continuing authority to
The land where the Airport is presently located as well as reorganize the National Government, which authority
the surrounding land area of approximately six hundred includes the creation of new entities, agencies and
hectares, are hereby transferred, conveyed and assigned instrumentalities of the Government[.] (Emphasis supplied)
to the ownership and administration of the Authority,
subject to existing rights, if any. The Bureau of Lands and The transfer of the Airport Lands and Buildings from the
other appropriate government agencies shall undertake an Bureau of Air Transportation to MIAA was not meant to
actual survey of the area transferred within one year from transfer beneficial ownership of these assets from the
the promulgation of this Executive Order and the Republic to MIAA. The purpose was merely to reorganize
corresponding title to be issued in the name of the a division in the Bureau of Air Transportation into a
Authority. Any portion thereof shall not be disposed separate and autonomous body. The Republic remains the
through sale or through any other mode unless specifically beneficial owner of the Airport Lands and Buildings. MIAA
approved by the President of the Philippines. (Emphasis itself is owned solely by the Republic. No party claims any
supplied)
Law on NatRes (23-49) 41
ownership rights over MIAA's assets adverse to the tax exemption only if the "beneficial use thereof has been
Republic. granted, for consideration or otherwise, to a taxable
person." MIAA, as a government instrumentality, is not a
The MIAA Charter expressly provides that the Airport taxable person under Section 133(o) of the Local
Lands and Buildings "shall not be disposed through sale or Government Code. Thus, even if we assume that the
through any other mode unless specifically approved by Republic has granted to MIAA the beneficial use of the
the President of the Philippines." This only means that the Airport Lands and Buildings, such fact does not make
Republic retained the beneficial ownership of the Airport these real properties subject to real estate tax.
Lands and Buildings because under Article 428 of the Civil
Code, only the "owner has the right to x x x dispose of a However, portions of the Airport Lands and Buildings that
thing." Since MIAA cannot dispose of the Airport Lands MIAA leases to private entities are not exempt from real
and Buildings, MIAA does not own the Airport Lands and estate tax. For example, the land area occupied by
Buildings. hangars that MIAA leases to private corporations is
subject to real estate tax. In such a case, MIAA has
At any time, the President can transfer back to the granted the beneficial use of such land area for a
Republic title to the Airport Lands and Buildings without consideration to a taxable person and therefore such land
the Republic paying MIAA any consideration. Under area is subject to real estate tax. In Lung Center of the
Section 3 of the MIAA Charter, the President is the only Philippines v. Quezon City, the Court ruled:
one who can authorize the sale or disposition of the Airport
Lands and Buildings. This only confirms that the Airport Accordingly, we hold that the portions of the land leased to
Lands and Buildings belong to the Republic. private entities as well as those parts of the hospital
leased to private individuals are not exempt from such
e. Real Property Owned by the Republic is Not Taxable taxes. On the other hand, the portions of the land occupied
by the hospital and portions of the hospital used for its
Section 234(a) of the Local Government Code exempts patients, whether paying or non-paying, are exempt from
from real estate tax any "[r]eal property owned by the real property taxes.29
Republic of the Philippines." Section 234(a) provides:
3. Refutation of Arguments of Minority
SEC. 234. Exemptions from Real Property Tax. The
following are exempted from payment of the real property The minority asserts that the MIAA is not exempt from real
tax: estate tax because Section 193 of the Local Government
Code of 1991 withdrew the tax exemption of "all persons,
(a) Real property owned by the Republic of the Philippines whether natural or juridical" upon the effectivity of the
or any of its political subdivisions except when the Code. Section 193 provides:
beneficial use thereof has been granted, for consideration
or otherwise, to a taxable person; SEC. 193. Withdrawal of Tax Exemption Privileges
Unless otherwise provided in this Code, tax exemptions or
x x x. (Emphasis supplied) incentives granted to, or presently enjoyed by all persons,
whether natural or juridical, including government-owned
This exemption should be read in relation with Section
or controlled corporations, except local water districts,
133(o) of the same Code, which prohibits local
cooperatives duly registered under R.A. No. 6938, non-
governments from imposing "[t]axes, fees or charges of
stock and non-profit hospitals and educational institutions
any kind on the National Government, its agencies and
are hereby withdrawn upon effectivity of this Code.
instrumentalities x x x." The real properties owned by the
(Emphasis supplied)
Republic are titled either in the name of the Republic itself
or in the name of agencies or instrumentalities of the The minority states that MIAA is indisputably a juridical
National Government. The Administrative Code allows real person. The minority argues that since the Local
property owned by the Republic to be titled in the name of Government Code withdrew the tax exemption of all
agencies or instrumentalities of the national government. juridical persons, then MIAA is not exempt from real estate
Such real properties remain owned by the Republic and tax. Thus, the minority declares:
continue to be exempt from real estate tax.
It is evident from the quoted provisions of the Local
The Republic may grant the beneficial use of its real Government Code that the withdrawn exemptions from
property to an agency or instrumentality of the national realty tax cover not just GOCCs, but all persons. To
government. This happens when title of the real property is repeat, the provisions lay down the explicit proposition that
transferred to an agency or instrumentality even as the the withdrawal of realty tax exemption applies to all
Republic remains the owner of the real property. Such persons. The reference to or the inclusion of GOCCs is
arrangement does not result in the loss of the tax only clarificatory or illustrative of the explicit provision.
exemption. Section 234(a) of the Local Government Code
states that real property owned by the Republic loses its
Law on NatRes (23-49) 42
The term "All persons" encompasses the two classes of any of the exempt entities under Section 193. (Emphasis
persons recognized under our laws, natural and juridical supplied)
persons. Obviously, MIAA is not a natural person. Thus,
the determinative test is not just whether MIAA is a GOCC, The minority's theory directly contradicts and completely
but whether MIAA is a juridical person at all. (Emphasis negates Section 133(o) of the Local Government Code.
and underscoring in the original) This theory will result in gross absurdities. It will make the
national government, which itself is a juridical person,
The minority posits that the "determinative test" whether subject to tax by local governments since the national
MIAA is exempt from local taxation is its status whether government is not included in the enumeration of exempt
MIAA is a juridical person or not. The minority also insists entities in Section 193. Under this theory, local
that "Sections 193 and 234 may be examined in isolation governments can impose any kind of local tax, and not
from Section 133(o) to ascertain MIAA's claim of only real estate tax, on the national government.
exemption."
Under the minority's theory, many national government
The argument of the minority is fatally flawed. Section 193 instrumentalities with juridical personalities will also be
of the Local Government Code expressly withdrew the tax subject to any kind of local tax, and not only real estate
exemption of all juridical persons "[u]nless otherwise tax. Some of the national government instrumentalities
provided in this Code." Now, Section 133(o) of the Local vested by law with juridical personalities are: Bangko
Government Code expressly provides otherwise, Sentral ng Pilipinas,30 Philippine Rice Research
specifically prohibiting local governments from imposing Institute,31 Laguna Lake
any kind of tax on national government instrumentalities.
Section 133(o) states: Development Authority,32 Fisheries Development
Authority,33 Bases Conversion Development Authority,34
SEC. 133. Common Limitations on the Taxing Powers of Philippine Ports Authority,35 Cagayan de Oro Port
Local Government Units. Unless otherwise provided Authority,36 San Fernando Port Authority,37 Cebu Port
herein, the exercise of the taxing powers of provinces, Authority,38 and Philippine National Railways.39
cities, municipalities, and barangays shall not extend to
the levy of the following: The minority's theory violates Section 133(o) of the Local
Government Code which expressly prohibits local
xxxx governments from imposing any kind of tax on national
government instrumentalities. Section 133(o) does not
(o) Taxes, fees or charges of any kinds on the National distinguish between national government instrumentalities
Government, its agencies and instrumentalities, and local with or without juridical personalities. Where the law does
government units. (Emphasis and underscoring supplied) not distinguish, courts should not distinguish. Thus,
Section 133(o) applies to all national government
By express mandate of the Local Government Code, local instrumentalities, with or without juridical personalities. The
governments cannot impose any kind of tax on national determinative test whether MIAA is exempt from local
government instrumentalities like the MIAA. Local taxation is not whether MIAA is a juridical person, but
governments are devoid of power to tax the national whether it is a national government instrumentality under
government, its agencies and instrumentalities. The taxing Section 133(o) of the Local Government Code. Section
powers of local governments do not extend to the national 133(o) is the specific provision of law prohibiting local
government, its agencies and instrumentalities, "[u]nless governments from imposing any kind of tax on the national
otherwise provided in this Code" as stated in the saving government, its agencies and instrumentalities.
clause of Section 133. The saving clause refers to Section
234(a) on the exception to the exemption from real estate Section 133 of the Local Government Code starts with the
tax of real property owned by the Republic. saving clause "[u]nless otherwise provided in this Code."
This means that unless the Local Government Code
The minority, however, theorizes that unless exempted in grants an express authorization, local governments have
Section 193 itself, all juridical persons are subject to tax by no power to tax the national government, its agencies and
local governments. The minority insists that the juridical instrumentalities. Clearly, the rule is local governments
persons exempt from local taxation are limited to the three have no power to tax the national government, its
classes of entities specifically enumerated as exempt in agencies and instrumentalities. As an exception to this
Section 193. Thus, the minority states: rule, local governments may tax the national government,
its agencies and instrumentalities only if the Local
x x x Under Section 193, the exemption is limited to (a)
Government Code expressly so provides.
local water districts; (b) cooperatives duly registered under
Republic Act No. 6938; and (c) non-stock and non-profit The saving clause in Section 133 refers to the exception to
hospitals and educational institutions. It would be the exemption in Section 234(a) of the Code, which makes
belaboring the obvious why the MIAA does not fall within the national government subject to real estate tax when it
gives the beneficial use of its real properties to a taxable
Law on NatRes (23-49) 43
entity. Section 234(a) of the Local Government Code between the grant of power and the withholding of power.
provides: The grantee of the power simply cannot exercise the
power on matters withheld from its power.
SEC. 234. Exemptions from Real Property Tax The
following are exempted from payment of the real property Second, Section 133 is entitled "Common Limitations on
tax: the Taxing Powers of Local Government Units." Section
133 limits the grant to local governments of the power to
(a) Real property owned by the Republic of the Philippines tax, and not merely the exercise of a delegated power to
or any of its political subdivisions except when the tax. Section 133 states that the taxing powers of local
beneficial use thereof has been granted, for consideration governments "shall not extend to the levy" of any kind of
or otherwise, to a taxable person. tax on the national government, its agencies and
instrumentalities. There is no clearer limitation on the
x x x. (Emphasis supplied) taxing power than this.

Under Section 234(a), real property owned by the Since Section 133 prescribes the "common limitations" on
Republic is exempt from real estate tax. The exception to the taxing powers of local governments, Section 133
this exemption is when the government gives the logically prevails over Section 193 which grants local
beneficial use of the real property to a taxable entity. governments such taxing powers. By their very meaning
and purpose, the "common limitations" on the taxing
The exception to the exemption in Section 234(a) is the
power prevail over the grant or exercise of the taxing
only instance when the national government, its agencies
power. If the taxing power of local governments in Section
and instrumentalities are subject to any kind of tax by local
193 prevails over the limitations on such taxing power in
governments. The exception to the exemption applies only
Section 133, then local governments can impose any kind
to real estate tax and not to any other tax. The justification
of tax on the national government, its agencies and
for the exception to the exemption is that the real property,
instrumentalities a gross absurdity.
although owned by the Republic, is not devoted to public
use or public service but devoted to the private gain of a Local governments have no power to tax the national
taxable person. government, its agencies and instrumentalities, except as
otherwise provided in the Local Government Code
The minority also argues that since Section 133 precedes
pursuant to the saving clause in Section 133 stating
Section 193 and 234 of the Local Government Code, the
"[u]nless otherwise provided in this Code." This exception
later provisions prevail over Section 133. Thus, the
which is an exception to the exemption of the Republic
minority asserts:
from real estate tax imposed by local governments
x x x Moreover, sequentially Section 133 antecedes refers to Section 234(a) of the Code. The exception to the
Section 193 and 234. Following an accepted rule of exemption in Section 234(a) subjects real property owned
construction, in case of conflict the subsequent provisions by the Republic, whether titled in the name of the national
should prevail. Therefore, MIAA, as a juridical person, is government, its agencies or instrumentalities, to real
subject to real property taxes, the general exemptions estate tax if the beneficial use of such property is given to
attaching to instrumentalities under Section 133(o) of the a taxable entity.
Local Government Code being qualified by Sections 193
The minority also claims that the definition in the
and 234 of the same law. (Emphasis supplied)
Administrative Code of the phrase "government-owned or
The minority assumes that there is an irreconcilable controlled corporation" is not controlling. The minority
conflict between Section 133 on one hand, and Sections points out that Section 2 of the Introductory Provisions of
193 and 234 on the other. No one has urged that there is the Administrative Code admits that its definitions are not
such a conflict, much less has any one presenteda controlling when it provides:
persuasive argument that there is such a conflict. The
SEC. 2. General Terms Defined. Unless the specific
minority's assumption of an irreconcilable conflict in the
words of the text, or the context as a whole, or a particular
statutory provisions is an egregious error for two reasons.
statute, shall require a different meaning:
First, there is no conflict whatsoever between Sections
xxxx
133 and 193 because Section 193 expressly admits its
subordination to other provisions of the Code when The minority then concludes that reliance on the
Section 193 states "[u]nless otherwise provided in this Administrative Code definition is "flawed."
Code." By its own words, Section 193 admits the
superiority of other provisions of the Local Government The minority's argument is a non sequitur. True, Section 2
Code that limit the exercise of the taxing power in Section of the Administrative Code recognizes that a statute may
193. When a provision of law grants a power but withholds require a different meaning than that defined in the
such power on certain matters, there is no conflict Administrative Code. However, this does not automatically
Law on NatRes (23-49) 44
mean that the definition in the Administrative Code does The contention of the minority is seriously flawed. It is not
not apply to the Local Government Code. Section 2 of the in accord with the Constitution and existing legislations. It
Administrative Code clearly states that "unless the specific will also result in gross absurdities.
words x x x of a particular statute shall require a different
meaning," the definition in Section 2 of the Administrative First, the Administrative Code definition of the phrase
Code shall apply. Thus, unless there is specific language "government-owned or controlled corporation" does not
in the Local Government Code defining the phrase distinguish between one incorporated under the
"government-owned or controlled corporation" differently Corporation Code or under a special charter. Where the
from the definition in the Administrative Code, the law does not distinguish, courts should not distinguish.
definition in the Administrative Code prevails.
Second, Congress has created through special charters
The minority does not point to any provision in the Local several government-owned corporations organized as
Government Code defining the phrase "government- stock corporations. Prime examples are the Land Bank of
owned or controlled corporation" differently from the the Philippines and the Development Bank of the
definition in the Administrative Code. Indeed, there is Philippines. The special charter40 of the Land Bank of the
none. The Local Government Code is silent on the Philippines provides:
definition of the phrase "government-owned or controlled
corporation." The Administrative Code, however, expressly SECTION 81. Capital. The authorized capital stock of
defines the phrase "government-owned or controlled the Bank shall be nine billion pesos, divided into seven
corporation." The inescapable conclusion is that the hundred and eighty million common shares with a par
Administrative Code definition of the phrase "government- value of ten pesos each, which shall be fully subscribed by
owned or controlled corporation" applies to the Local the Government, and one hundred and twenty million
Government Code. preferred shares with a par value of ten pesos each, which
shall be issued in accordance with the provisions of
The third whereas clause of the Administrative Code Sections seventy-seven and eighty-three of this Code.
states that the Code "incorporates in a unified document (Emphasis supplied)
the major structural, functional and procedural principles
and rules of governance." Thus, the Administrative Code is Likewise, the special charter41 of the Development Bank
the governing law defining the status and relationship of of the Philippines provides:
government departments, bureaus, offices, agencies and
SECTION 7. Authorized Capital Stock Par value. The
instrumentalities. Unless a statute expressly provides for a
capital stock of the Bank shall be Five Billion Pesos to be
different status and relationship for a specific government
divided into Fifty Million common shares with par value of
unit or entity, the provisions of the Administrative Code
P100 per share. These shares are available for
prevail.
subscription by the National Government. Upon the
The minority also contends that the phrase "government- effectivity of this Charter, the National Government shall
owned or controlled corporation" should apply only to subscribe to Twenty-Five Million common shares of stock
corporations organized under the Corporation Code, the worth Two Billion Five Hundred Million which shall be
general incorporation law, and not to corporations created deemed paid for by the Government with the net asset
by special charters. The minority sees no reason why values of the Bank remaining after the transfer of assets
government corporations with special charters should and liabilities as provided in Section 30 hereof.
have a capital stock. Thus, the minority declares:
Other government-owned corporations organized as stock
I submit that the definition of "government-owned or corporations under their special charters are the Philippine
controlled corporations" under the Administrative Code Crop Insurance Corporation,42 Philippine International
refer to those corporations owned by the government or its Trading Corporation,43 and the Philippine National
instrumentalities which are created not by legislative Bank44 before it was reorganized as a stock corporation
enactment, but formed and organized under the under the Corporation Code. All these government-owned
Corporation Code through registration with the Securities corporations organized under special charters as stock
and Exchange Commission. In short, these are GOCCs corporations are subject to real estate tax on real
without original charters. properties owned by them. To rule that they are not
government-owned or controlled corporations because
xxxx they are not registered with the Securities and Exchange
Commission would remove them from the reach of Section
It might as well be worth pointing out that there is no point 234 of the Local Government Code, thus exempting them
in requiring a capital structure for GOCCs whose full from real estate tax.
ownership is limited by its charter to the State or Republic.
Such GOCCs are not empowered to declare dividends or Third, the government-owned or controlled corporations
alienate their capital shares. created through special charters are those that meet the
two conditions prescribed in Section 16, Article XII of the
Law on NatRes (23-49) 45
Constitution. The first condition is that the government- government-owned or controlled corporations, which
owned or controlled corporation must be established for derive their income to meet operating expenses solely
the common good. The second condition is that the from commercial transactions in competition with the
government-owned or controlled corporation must meet private sector. The intent of the Constitution is to prevent
the test of economic viability. Section 16, Article XII of the the creation of government-owned or controlled
1987 Constitution provides: corporations that cannot survive on their own in the market
place and thus merely drain the public coffers.
SEC. 16. The Congress shall not, except by general law,
provide for the formation, organization, or regulation of Commissioner Blas F. Ople, proponent of the test of
private corporations. Government-owned or controlled economic viability, explained to the Constitutional
corporations may be created or established by special Commission the purpose of this test, as follows:
charters in the interest of the common good and subject to
the test of economic viability. MR. OPLE: Madam President, the reason for this concern
is really that when the government creates a corporation,
The Constitution expressly authorizes the legislature to there is a sense in which this corporation becomes exempt
create "government-owned or controlled corporations" from the test of economic performance. We know what
through special charters only if these entities are required happened in the past. If a government corporation loses,
to meet the twin conditions of common good and then it makes its claim upon the taxpayers' money through
economic viability. In other words, Congress has no power new equity infusions from the government and what is
to create government-owned or controlled corporations always invoked is the common good. That is the reason
with special charters unless they are made to comply with why this year, out of a budget of P115 billion for the entire
the two conditions of common good and economic government, about P28 billion of this will go into equity
viability. The test of economic viability applies only to infusions to support a few government financial
government-owned or controlled corporations that perform institutions. And this is all taxpayers' money which could
economic or commercial activities and need to compete in have been relocated to agrarian reform, to social services
the market place. Being essentially economic vehicles of like health and education, to augment the salaries of
the State for the common good meaning for economic grossly underpaid public employees. And yet this is all
development purposes these government-owned or going down the drain.
controlled corporations with special charters are usually
organized as stock corporations just like ordinary private Therefore, when we insert the phrase "ECONOMIC
corporations. VIABILITY" together with the "common good," this
becomes a restraint on future enthusiasts for state
In contrast, government instrumentalities vested with capitalism to excuse themselves from the responsibility of
corporate powers and performing governmental or public meeting the market test so that they become viable. And
functions need not meet the test of economic viability. so, Madam President, I reiterate, for the committee's
These instrumentalities perform essential public services consideration and I am glad that I am joined in this
for the common good, services that every modern State proposal by Commissioner Foz, the insertion of the
must provide its citizens. These instrumentalities need not standard of "ECONOMIC VIABILITY OR THE ECONOMIC
be economically viable since the government may even TEST," together with the common good.45
subsidize their entire operations. These instrumentalities
are not the "government-owned or controlled corporations" Father Joaquin G. Bernas, a leading member of the
referred to in Section 16, Article XII of the 1987 Constitutional Commission, explains in his textbook The
Constitution. 1987 Constitution of the Republic of the Philippines: A
Commentary:
Thus, the Constitution imposes no limitation when the
legislature creates government instrumentalities vested The second sentence was added by the 1986
with corporate powers but performing essential Constitutional Commission. The significant addition,
governmental or public functions. Congress has plenary however, is the phrase "in the interest of the common
authority to create government instrumentalities vested good and subject to the test of economic viability." The
with corporate powers provided these instrumentalities addition includes the ideas that they must show capacity to
perform essential government functions or public services. function efficiently in business and that they should not go
However, when the legislature creates through special into activities which the private sector can do better.
charters corporations that perform economic or Moreover, economic viability is more than financial viability
commercial activities, such entities known as but also includes capability to make profit and generate
"government-owned or controlled corporations" must benefits not quantifiable in financial terms.
meet the test of economic viability because they compete
in the market place. Clearly, the test of economic viability does not apply to
government entities vested with corporate powers and
This is the situation of the Land Bank of the Philippines performing essential public services. The State is
and the Development Bank of the Philippines and similar obligated to render essential public services regardless of
Law on NatRes (23-49) 46
the economic viability of providing such service. The non- All these agencies of government perform government
economic viability of rendering such essential public functions essential to the operation of an international
service does not excuse the State from withholding such airport.
essential services from the public.
MIAA performs an essential public service that every
However, government-owned or controlled corporations modern State must provide its citizens. MIAA derives its
with special charters, organized essentially for economic revenues principally from the mandatory fees and charges
or commercial objectives, must meet the test of economic MIAA imposes on passengers and airlines. The terminal
viability. These are the government-owned or controlled fees that MIAA charges every passenger are regulatory or
corporations that are usually organized under their special administrative fees47 and not income from commercial
charters as stock corporations, like the Land Bank of the transactions.
Philippines and the Development Bank of the Philippines.
These are the government-owned or controlled MIAA falls under the definition of a government
corporations, along with government-owned or controlled instrumentality under Section 2(10) of the Introductory
corporations organized under the Corporation Code, that Provisions of the Administrative Code, which provides:
fall under the definition of "government-owned or
controlled corporations" in Section 2(10) of the SEC. 2. General Terms Defined. x x x x
Administrative Code.
(10) Instrumentality refers to any agency of the National
The MIAA need not meet the test of economic viability Government, not integrated within the department
because the legislature did not create MIAA to compete in framework, vested with special functions or jurisdiction by
the market place. MIAA does not compete in the market law, endowed with some if not all corporate powers,
place because there is no competing international airport administering special funds, and enjoying operational
operated by the private sector. MIAA performs an autonomy, usually through a charter. x x x (Emphasis
essential public service as the primary domestic and supplied)
international airport of the Philippines. The operation of an
The fact alone that MIAA is endowed with corporate
international airport requires the presence of personnel
powers does not make MIAA a government-owned or
from the following government agencies:
controlled corporation. Without a change in its capital
1. The Bureau of Immigration and Deportation, to structure, MIAA remains a government instrumentality
document the arrival and departure of passengers, under Section 2(10) of the Introductory Provisions of the
screening out those without visas or travel documents, or Administrative Code. More importantly, as long as MIAA
those with hold departure orders; renders essential public services, it need not comply with
the test of economic viability. Thus, MIAA is outside the
2. The Bureau of Customs, to collect import duties or scope of the phrase "government-owned or controlled
enforce the ban on prohibited importations; corporations" under Section 16, Article XII of the 1987
Constitution.
3. The quarantine office of the Department of Health, to
enforce health measures against the spread of infectious The minority belittles the use in the Local Government
diseases into the country; Code of the phrase "government-owned or controlled
corporation" as merely "clarificatory or illustrative." This is
4. The Department of Agriculture, to enforce measures fatal. The 1987 Constitution prescribes explicit conditions
against the spread of plant and animal diseases into the for the creation of "government-owned or controlled
country; corporations." The Administrative Code defines what
constitutes a "government-owned or controlled
5. The Aviation Security Command of the Philippine corporation." To belittle this phrase as "clarificatory or
National Police, to prevent the entry of terrorists and the illustrative" is grave error.
escape of criminals, as well as to secure the airport
premises from terrorist attack or seizure; To summarize, MIAA is not a government-owned or
controlled corporation under Section 2(13) of the
6. The Air Traffic Office of the Department of Introductory Provisions of the Administrative Code
Transportation and Communications, to authorize aircraft because it is not organized as a stock or non-stock
to enter or leave Philippine airspace, as well as to land on, corporation. Neither is MIAA a government-owned or
or take off from, the airport; and controlled corporation under Section 16, Article XII of the
1987 Constitution because MIAA is not required to meet
7. The MIAA, to provide the proper premises such as the test of economic viability. MIAA is a government
runway and buildings for the government personnel, instrumentality vested with corporate powers and
passengers, and airlines, and to manage the airport performing essential public services pursuant to Section
operations. 2(10) of the Introductory Provisions of the Administrative
Code. As a government instrumentality, MIAA is not
Law on NatRes (23-49) 47
subject to any kind of tax by local governments under includes public airports and seaports, as properties of
Section 133(o) of the Local Government Code. The public dominion and owned by the Republic. As properties
exception to the exemption in Section 234(a) does not of public dominion owned by the Republic, there is no
apply to MIAA because MIAA is not a taxable entity under doubt whatsoever that the Airport Lands and Buildings are
the Local Government Code. Such exception applies only expressly exempt from real estate tax under Section
if the beneficial use of real property owned by the Republic 234(a) of the Local Government Code. This Court has also
is given to a taxable entity. repeatedly ruled that properties of public dominion are not
subject to execution or foreclosure sale.
Finally, the Airport Lands and Buildings of MIAA are
properties devoted to public use and thus are properties of WHEREFORE, we GRANT the petition. We SET ASIDE
public dominion. Properties of public dominion are owned the assailed Resolutions of the Court of Appeals of 5
by the State or the Republic. Article 420 of the Civil Code October 2001 and 27 September 2002 in CA-G.R. SP No.
provides: 66878. We DECLARE the Airport Lands and Buildings of
the Manila International Airport Authority EXEMPT from
Art. 420. The following things are property of public the real estate tax imposed by the City of Paraaque. We
dominion: declare VOID all the real estate tax assessments,
including the final notices of real estate tax delinquencies,
(1) Those intended for public use, such as roads, canals, issued by the City of Paraaque on the Airport Lands and
rivers, torrents, ports and bridges constructed by the State, Buildings of the Manila International Airport Authority,
banks, shores, roadsteads, and others of similar character; except for the portions that the Manila International Airport
Authority has leased to private parties. We also declare
(2) Those which belong to the State, without being for
VOID the assailed auction sale, and all its effects, of the
public use, and are intended for some public service or for
Airport Lands and Buildings of the Manila International
the development of the national wealth. (Emphasis
Airport Authority. No costs. SO ORDERED.
supplied)

The term "ports x x x constructed by the State" includes ROMAN CATHOLIC BISHOP OF KALIBO, AKLAN,
airports and seaports. The Airport Lands and Buildings of represented by BISHOP JUAN N. NILMAR, vs.
MIAA are intended for public use, and at the very least MUNICIPALITY OF BURUANGA, AKLAN, represented
intended for public service. Whether intended for public by the HON. PROTACIO S. OBRIQUE; G.R. No. 149145
use or public service, the Airport Lands and Buildings are March 31, 2006; CALLEJO, SR., J.:
properties of public dominion. As properties of public
Before the Court is the petition for review on certiorari filed
dominion, the Airport Lands and Buildings are owned by
by the Roman Catholic Bishop1 of Kalibo, Aklan, seeking
the Republic and thus exempt from real estate tax under
the partial review of the Decision2 dated January 31, 2001
Section 234(a) of the Local Government Code.
of the Court of Appeals in CA-G.R. CV No. 52626.
4. Conclusion Likewise sought to be reviewed is the Resolution dated
July 18, 2001 of the appellate court denying the
Under Section 2(10) and (13) of the Introductory petitioners motion for partial reconsideration.
Provisions of the Administrative Code, which governs the
legal relation and status of government units, agencies Factual and Procedural Antecedents
and offices within the entire government machinery, MIAA
Some time in 1990, the Roman Catholic Bishop of Kalibo,
is a government instrumentality and not a government-
Aklan, filed with the Regional Trial Court (RTC) thereof a
owned or controlled corporation. Under Section 133(o) of
complaint for declaration of ownership and quieting of title
the Local Government Code, MIAA as a government
to land with prayer for preliminary injunction against the
instrumentality is not a taxable person because it is not
Municipality of Buruanga, Aklan. The case was docketed
subject to "[t]axes, fees or charges of any kind" by local
as Civil Case No. 4164 and raffled to Branch 1 of the said
governments. The only exception is when MIAA leases its
RTC.
real property to a "taxable person" as provided in Section
234(a) of the Local Government Code, in which case the
The complaint alleged, among others, that the Roman
specific real property leased becomes subject to real
Catholic Bishop of Kalibo is the lawful owner and
estate tax. Thus, only portions of the Airport Lands and
possessor of a parcel of residential and commercial land
Buildings leased to taxable persons like private parties are
(Cadastral Lot No. 138) located at the poblacion of the
subject to real estate tax by the City of Paraaque.
Municipality of Buruanga, Aklan. The said lot, with an area
of 9,545 square meters, is a block bounded by four streets
Under Article 420 of the Civil Code, the Airport Lands and
on all sides. It is more particularly described as follows:
Buildings of MIAA, being devoted to public use, are
properties of public dominion and thus owned by the State
A parcel of commercial and residential land known as
or the Republic of the Philippines. Article 420 specifically
Cadastral Lot No. 138, GSS-06-00012, located at
mentions "ports x x x constructed by the State," which
Poblacion, Buruanga, Aklan, containing an area of NINE
Law on NatRes (23-49) 48
THOUSAND FIVE HUNDRED FORTY- FIVE (9,545) On March 12, 1990, the Roman Catholic Bishop of Kalibo
SQUARE METERS, more or less. Bounded on the North wrote the Department of Public Works and Highways of
by Viven Ostan Street; on the East by the Provincial Road; the said province requesting the said office not to issue
on the South by Nitoy Sualog Street; and on the West by any building permit to the Municipal Mayor and/or the
Emilio Ostan Street, and declared for taxation purposes in Municipality of Buruanga in connection with the
the name of the Roman Catholic Church, Buruanga, construction of its municipal building on the land owned by
Aklan, under Tax Declaration No. 6339 (1985) and the Roman Catholic Bishop of Kalibo.6
assessed at P23,850.00, including the improvements
thereon.3 These letters went unheeded as the construction of the
new municipal building on the same site proceeded.
In 1894, the Roman Catholic Church was built in the Consequently, the Roman Catholic Bishop of Kalibo filed
middle portion of the said lot and has been in existence the complaint a quo and prayed that it be declared the
since then up to the present. lawful owner and possessor of Lot 138. It likewise prayed
that a temporary restraining order be issued to enjoin the
The complaint further alleged that some time in 1978, 4 the said municipality and its authorized representatives from
Municipality of Buruanga constructed its municipal building constructing the new municipal building thereon and that
on the northeastern portion of the subject lot after it the latter be directed to pay damages to the Roman
obtained the permission of Fr. Jesus Patio, then parish Catholic Bishop of Kalibo.
priest of Buruanga. The municipality promised to
In its Answer,7 the Municipality of Buruanga, represented
remove all the improvements it constructed thereon if and by Mayor Protacio Obrique, denied that the Roman
when the Roman Catholic Bishop of Kalibo needed the Catholic Bishop of Kalibo ever acquired ownership and
said land. possession over the land subject of the complaint. It raised
as affirmative defenses that the said lot was surveyed as
In October 1989, the said municipal building was razed by property of the municipality on February 3, 1909 in
fire allegedly perpetrated by members of the New Peoples accordance with Section 58 of Act 926 by A.W. Bushell
Army. On November 25, 1989, the Roman Catholic Bishop and approved by the Bureau of Lands on May 15, 1909. 8
of Kalibo, through its counsel, wrote to the Municipal Thereafter, a decree was issued on March 14, 1919 in
Mayor of Buruanga requesting the officials of the said favor of the Municipality of Buruanga under Case No.
municipality to refrain from constructing its new building on 12871 of then Court of Land Registration, Bureau of
the same site because it is the property of the church. Lands.
Further, it needed the said land for its social action
projects. The letter reads in part: It was further alleged that the said land was again
surveyed in the name of the Municipality of Buruanga
I am writing you on behalf of my client THE ROMAN under Act No. 2259 and denominated as Lot No. 138
CATHOLIC BISHOP OF KALIBO, AKLAN, a corporation GSS-06-00012 from the approved cadastral map and that
sole and represented by Bishop Juan N. Nilmar requesting the said municipality alone had possessed the said land
you and the Honorable Members of the Municipal Council under the claim of title exclusively for over fifty (50) years,
(Sangguniang Bayan) to refrain from constructing your exclusive of all other rights and adverse to all other
new Municipal Building on the same site where your old claimants.
Municipal Building was burned down because it is
constructed on the property of the Church. The Municipality of Buruanga urged the court a quo to
dismiss the complaint and, instead, declare it the absolute
Please be informed that the land of the Church is needed and exclusive owner of the disputed lot.
for its social action projects and additional building, hence,
kindly relocate your New Municipal Building in your own On November 29, 1990, the court a quo issued the Order9
land located along Emilio Ostan Street, known as appointing Geodetic Engineer Rodrigo Santiago of the
Cadastral Lot No. 87. Bureau of Lands as

With respect to your other public buildings such as the Commissioner and directing him to identify and delineate
Rural Hospital, Buruanga Community Medicare Hospital, the lot in question.
the Basketball Court and the Grandstand which are all
occupying the Church property, you can continue using In compliance therewith, Engr. Santiago submitted the
the same land subject to your recognition of the true Commissioners Report and Sketch stating in part:
ownership of the property of the Church The Roman
Catholic Church of Buruanga, Aklan, under the Roman That as per order of the court dated November 29, 1990 to
Catholic Bishop of Kalibo, Aklan, the lawful administrator delineate the land[in] question, the undersigned court
of all church properties in the Province of Aklan.5 commissioner notified both parties and the schedule of
survey was January 12, 1991 but it was postponed and
Law on NatRes (23-49) 49
moved to January 15 as requested by the representative At the pre-trial, the parties stipulated on the following facts:
from the Municipality of Buruanga.
1. The identity of the lot in question which is Lot 138
That the land in question involved was pointed to me by consisting of Lots 138-A, 138-B and 138-C as reflected in
the Honorable Mayor of the Municipality of Buruanga, the commissioners sketch with an area of 9,544 square
identified on the plan as [L]ot 138 located at Poblacion meters and subdivided as follows:
Buruanga with survey no. GSS-06-00012 approved by the
Director Lands last February 19, 1985, listed as Public Lot 138-A 2,319 square meters
Plaza on file in the CENR Office Land Management
Sector, Kalibo, Aklan. Lot 138-B 3,836 square meters

That the Honorable Mayor of the Municipality of Buruanga Lot 138-C 3,389 square meters
pointed also the boundary between the Public Plaza and
2. Lot 138-B is the present site of the Roman Catholic
the Roman Catholic Church.
Church of Buruanga.11
The Technical Descriptions are as follows:
The parties also agreed that the sole issue for resolution is
Lot 138-A (Public Plaza) who between the Roman Catholic Bishop of Kalibo and
the Municipality of Buruanga is the owner of Lot 138.
corner 1-2 S86 - 03E 65.54 m.
After due trial, the court a quo rendered its Decision dated
2-3 S03 - 17E 32.36 m. October 30, 1995 declaring the Roman Catholic Bishop of
Kalibo as the lawful owner and possessor of Lot 138-B
3-4 N88 - 54W 71.31 m and the Municipality of Buruanga as the lawful owner and
possessor of Lots 138-A and 138-C, the said lots being
4-1 N06 - 33E 35.68 m. public plaza for public use.

containing an area of 2,319 square meters The court a quo found that of the various tax declarations 12
presented by the Roman Catholic Bishop of Kalibo to
Lot 138-B (Roman Catholic Church) support its claim, only one referred to a portion of Lot 138.
Said tax declaration13 covered the church site and the
1-2 S86 - 03E 65.54 m.
parish house situated within Lot 138-B. The other pieces
2-3 S03 - 17E 32.36 m. of evidence14 could not be relied upon because they
contained hearsay information relating to the disputed lot
3-4 N88 - 54W 71.31 m. that occurred before the affiants were born. The affidavit
executed by Fr. Jesus Patio15 stating that he was the one
4-1 N06 - 33E 35.68 m. who gave verbal permission to then Municipal Mayor
Pedro Omugtong to construct the municipal building on the
containing an area of 3,836 square meters vacant lot owned by the church was not accorded any
evidentiary value because he (Fr. Patio) did not testify
Lot 138-C (Public Plaza) during the trial.

1-2 N81 - 19W 87.70 m. On the other hand, the court a quo did not give credence
to the Municipality of Buruangas Exhibit "1," a microfilm
2-3 N06 - 33E 38.90 m.
enlargement of a plan showing that the land consisting of
12,615 square meters was subject of Land Registration
3-4 S83 - 17E 80.35 m.
Case No. 12871. The plan showed that the survey was
4-1 S03 - 17E 42.57 m. approved on May 15, 1909 and the notations therein
indicated that a decree was issued on March 14, 1919. But
containing an area of 3,389 square meters no such decree was shown. It was further found by the
court a quo that the plan was requested from the Bureau
Consistent with the above technical description, the sketch of Lands Survey Division on December 22, 1976.
submitted by Engr. Santiago showed the delineation of Lot However, the same was not duly certified by the issuing
138 into three parts: Lots 138-A, 138-B and 138-C. The government agency. Even assuming that the disputed lot
municipal building stood on Lot 138-A; the Roman Catholic was indeed subject of a land registration proceeding and a
Church stood on Lot 138-B and the municipal health decree had been issued therefor in March 1919, the
center and the Buruanga Community Medicare building Municipality of Buruanga, despite lapse of decades, failed
stood on Lot 138-C. It also showed that portions of Lots to take the necessary judicial steps for the issuance of a
138-A and 138-C were being used as public plaza. title in its name based on the decree. Neither did it take
Law on NatRes (23-49) 50
any other course of action that would render its title thereto The Roman Catholic Bishop of Kalibo seasonably filed its
indefeasible. appeal with the Court of Appeals. It sought the reversal of
that portion of the court a quos judgment adjudicating the
The court a quo, however, gave probative weight to the ownership of Lots 138-A and 138-C to the Municipality of
testimony of Manuel Sualog, Chief of the Lands Buruanga.
Management Section of the Department of Environment
and National Resources, who was presented by the During the pendency of the case in the appellate court, the
Municipality of Buruanga. Sualog testified that the disputed Roman Catholic Bishop of Kalibo moved to submit
lot was the public plaza of the said municipality. Standing additional evidence to support its claim of ownership over
thereon are the Roman Catholic Church and its parish the entire Lot 138. The additional evidence consisted of
house, the new municipal hall, the rural health center, the affidavits of old residents of Buruanga stating that the
barangay community hospital and a basketball court. municipal building was constructed on the disputed lot only
in the late 1950s. Prior thereto, the municipal building
During the court a quos ocular inspection conducted on stood at a place called Sunset Park, a block totally
May 7, 1992, the town was celebrating its town fiesta. It different from the disputed lot. The said motion was denied
observed that the public was using the whole plaza (in by the appellate court on the ground that the Roman
Lots 138-A and 138-C) for the festivities. Also, the Catholic Bishop of Kalibo had already been accorded full
existence of the health centers, basketball court and the opportunity to present its evidence in the court a quo.
municipal hall showed that portions of the disputed lot
were being used by the public. The Municipality of Buruanga did not file its appellees
brief with the CA. On January 31, 2001, the appellate court
Upon inspection of the church, the court a quo further rendered the assailed Decision affirming with modification
observed that it was indeed an old stone structure and the decision of the court a quo. The CA affirmed the
probably built in 1894, the year carved on its left side ownership of the Roman Catholic Bishop of Kalibo over
entrance. It described the church as "vintage turn-of-the Lot 138-B but reversed the court a quos ruling relative to
century colonial Filipino church architecture. Moss and the ownership of Lots 138-A and 138-C. The appellate
ficus grow out of its wall crevices. The age of the church court declared the said lots property of public dominion,
shows that it has been occupying that particular space for hence, not owned by either of the parties.
almost one hundred (100) years long enough for the
plaintiff to have possessed it in the concept of owner The CA stated that the court a quo correctly relied on the
continuously, adversely and publicly against the whole ruling in Harty, which was reiterated in Bishop of Calbayog
world."16 v. Director of Lands,20 where the Court held that the public
plaza and public thoroughfare were not subject to
The court a quo held that the facts of the present case registration by the church. In the latter case, it was ruled
were similar to those in Harty v. Municipality of Victoria, 17 that since neither the Church nor the municipality
where the Court ruled that: presented positive proof of ownership or exclusive
possession for an appreciable period of time, and the only
For the above reasons, x x x it should be held, as we do indubitable fact was the free and continuous use of the lot
hereby hold, that the whole of the land not occupied by the in question by the residents of the town, which had no
church of the town of Victoria and its parish house, is a other public plaza to speak of other than the disputed lot,
public plaza of said town, of public use and that in there was a strong presumption that the same had been
consequence thereof, the defendant is absolved of the segregated as a public plaza upon the founding of the
complaint without any special ruling as to the costs of both municipality therein.
instances.18
As mentioned earlier, the appellate court reversed that
The dispositive portion of the court a quos decision reads: portion of the court a quos judgment declaring the
Municipality of Buruanga as the owner of Lots 138-A and
WHEREFORE, judgment is hereby rendered as follows:
138-C which form part of the public plaza. Citing Articles
1. The Roman Catholic Bishop of Kalibo, Aklan, is 41921 and 42022 of the Civil Code, the appellate court
declared the lawful owner and possessor of Lot 138-B with classified these lots as property of public dominion; hence,
an area of 3,836 square meters in the Commissioners not susceptible to private ownership by the Municipality of
Report as against the defendant; Buruanga. The said lots are merely under its jurisdiction
and administration. Being intended for the common and
2. Defendant Municipality of Buruanga is declared the public welfare, they could not be appropriated either by the
lawful owner and possessor of Lot 138-A with an area of State or by private persons.
2,319 square meters and Lot 138-C with an area of 3,389
square meters in the Commissioners Report, said lots The dispositive portion of the assailed CA decision reads:
being public plaza destined for public use.19
WHEREFORE, upon the premises, the appealed decision
is AFFIRMED with the MODIFICATION that Lots 138-A
Law on NatRes (23-49) 51
and 138-C are declared property of public dominion not parish house were situated. Rather, the Court in Harty
owned by either of the parties.23 referred to the entire lot or block (bounded by a street on
each of the four sides) on which the church and its parish
The Roman Catholic Bishop of Kalibo moved for a partial house were erected.
reconsideration of the appellate courts ruling that Lots
138-A and 138-C, being the public plaza, are property of The petitioner asserts that the following facts that have
public dominion. The Roman Catholic Bishop of Kalibo been established support its claim of ownership over the
averred that the appellate court erred in affirming the entire Lot 138 as against the claim of the Municipality of
finding of the court a quo that these lots comprise the Buruanga (respondent municipality):
public plaza. It maintained that it owned the entire Lot 138.
[a] The church is built in the middle of Lot 138 (which is
The appellate court denied the motion for partial now Lot 138-B). It was built therein in 1894. The church
reconsideration, hence, the recourse to this Court by the was almost 100 years old (at the time the case was
Roman Catholic Bishop of Kalibo (the petitioner). instituted with the trial court in 1990). x x x

The Petitioners Arguments [b] The Municipality of Buruanga is an old municipality


constituted or created during the colonial period, when the
The present petition for review on certiorari alleges that: Philippine Islands was under the Spanish sovereignty. x x
x
I. THE COURT OF APPEALS GRIEVOUSLY ERRED IN
RELYING UPON THE CASES OF HARTY V. [c] No building was built on Lot 138 earlier than or at about
MUNICIPALITY OF VICTORIA, TARLAC (13 Phil. 152 the same time as the church. No municipal building was
[1909]) and BISHOP OF CALBAYOG V. DIRECTOR OF built around the church for many decades after 1894. x x x
LANDS (45 SCRA 418 [1972]) TO SUPPORT ITS
CONCLUSION THAT THE PETITIONER IS NOT THE [d] The municipal hall of Buruanga was built on what is
OWNER OF LOTS 138-A AND 138-C. now Lot 138-A only in the late 1950s. x x x

II. THE COURT OF APPEALS GRIEVOUSLY ERRED IN [e] It was not controverted by the private respondent that
FINDING THAT LOTS 138-A AND 138-C [WHICH ARE then Mayor Omugtong of Buruanga sought and obtained
WITHIN THE ORIGINAL LOT 138] ARE PROPERTIES the permission of the then parish priest, Fr. Jesus Patino,
OF THE PUBLIC [DOMAIN] AND NOT SUSCEPTIBLE TO to allow the municipal government to build its municipal
PRIVATE OWNERSHIP BY THE PETITIONER. hall on Lot 138-A in the late 1950s only. x x x

III. THE COURT OF APPEALS GRIEVOUSLY ERRED [f] No evidence was adduced by private respondent that it
WHEN IT REFUSED TO RECOGNIZE THAT had obtained title of Lot 138-A or 138-C from the church
PETITIONERS OWNERSHIP OF THE ENTIRE LOT 138 (the owner of these lots) or that its possession of any
WAS GRANTED AND RECOGNIZED UNDER SPANISH portion of Lot 138 was adverse to that of the church. x x x
LAW, AND AFFIRMED IN THE TREATY OF PARIS.24
[g] When the municipal hall was burned down by the NPA
The petitioner clarifies that it is seeking a partial review of rebels in 1989 the church asked the municipal government
the appellate courts Decision dated January 31, 2001 to relocate the municipal hall elsewhere since it (church)
classifying Lots 138-A and 138-C as property of public needed the lot for itself.
dominion and not susceptible to private ownership and
that the petitioner is not entitled to the possession and [h] Because the municipal government resisted and for the
ownership thereof. It is not contesting the portion of the CA first time exhibited a possession adverse to the church, the
decision upholding its ownership over Lot 138-B. petitioner promptly filed the instant suit before the lower
court for quieting of title to the subject lot (the entire Lot
The petitioner opines that the case of Harty, relied upon by 138) and to be declared the owner of such property.
the court a quo and the CA to hold that Lots 138-A and
138-C comprise the public plaza, are inapplicable because [i] The church has been in continuous, open, adverse,
the facts therein are not similar to those of the present notorious possession of the entire Lot 138 in the concept
case. The petitioner points out that the public plaza of owner since at least 1894 until the late 1950s. x x x
referred to in Harty was the lot across the street from the
church lot. It was not referring to the land surrounding the [j] No evidence has been shown that Lots 138-A and 138-
Catholic church and the convent of the town of Victoria, C were devoted for public use or for use as a public plaza
which was bounded by streets on each of its four sides. before 1894 or even at about the time the church was built
Thus, when the Court in Harty limited the ownership of the on Lot 138. x x x
church to the land "occupied by the church of the town of
{k] The only evidence as to the supposed character of Lot
Victoria and its parish house," it was not confining the
138-C as a public plaza is a survey plan allegedly
ownership to a portion of the lot on which the church and
approved on 15 May 1909 denominated as [GSS]-06-
Law on NatRes (23-49) 52
00012, Buruanga Settlement Project, approved only in Harty is not applicable, the petitioner expounds, because it
1984. Petitioner was not notified of this survey. x x x was indubitably established therein that the "plaza was
used without let or hindrance by the public and the
[l] The real property tax declaration presented by private residents of Victoria ever since its creation." In contrast, in
respondent to establish its supposed possession (Exhs. 4, the present case, there was allegedly no evidence to show
4-a, and 4-b, Record, pages 45-47) covered the year 1992 that Lots 138-A and 138-C were set aside as the public
only. plaza, or for any public purpose, when the Municipality of
Buruanga was created during the Spanish period. The
[m] Witness Jaime S. Prado, Sr. (who was born on 17 evidence, in fact, show that the entire Lot 138, bounded on
December 1905 and coming to the age of reason when he all its four sides by streets, belonged to the church and it
was about 10 years old) testified that as far as he can had continuous use and occupation thereof since 1894
remember (since he was grade 1) he was brought to mass when it constructed its church in the middle of Lot 138. No
by his elders at the church of Buruanga, which was the such use of Lot 138-A and 138-C as the public plaza for
very same church as of the time he testified in 1992, and the same length of time or from 1894 had been shown.
was active in church activities in that church (e.g., tsn, 9
January 1992, pages 5, 16); that the property of the The petitioner assails the reliance by the appellate court
church was bounded on all four sides by the very same on the court a quos statement during its ocular inspection
streets that bounded it at the time he testified (ibid., at on Lot 138 in 1992 that it observed that the property was
page 6-8). occupied by the Roman Catholic Church, a parish house,
the municipal hall and three of its municipal edifices, and a
[n] Private respondent indirectly judicially admitted that it basketball court. Based on this observation, the court a
has no title (Torrens or otherwise) to the subject properties quo concluded, and the appellate court affirmed, that Lots
when its star witness (the incumbent Mayor Protacio 138-A and 138-C comprise the public plaza. The petitioner
Obrique of Buruanga) testified that the properties in the objects to this conclusion stating that the same cannot
poblacion of Buruanga are not covered by any title (tsn, 27 overcome the evidence in favor of the church as to its
July 1992, page 5).25 ownership over these lots traced back to 1894 when it
constructed the church in the middle of Lot 138 or what is
The petitioner contends that the pronouncement in Bishop
now Lot 138-B.
of Calbayog, cited by the appellate court, does not support
its decision. Instead, it actually supports the petitioners It reiterates that under the Laws of the Indies, when a
claim of ownership over Lot 138, including Lots 138-A and municipality was created, the church was assigned a
138-C. In the said case, the lot (Lot 2) that was declared property consisting of a parcel of land bounded on all its
by the Court as plaza was a separate and distinct lot four sides by streets, and that the public plaza was
separated from the church lot (Lot 1) by a provincial road. situated not on the same parcel of land assigned to the
Lot 1, held to be owned by the church in the said case, church but on a distinct lot separated by a street from that
included not only the space occupied by the church, belfry, assigned to the church.
convent, parish school and nuns residence, but also the
empty space which only had concrete benches as The petitioner likewise argues that even if it, as the owner
improvements thereon and which was used as a public of the entire Lot 138, allowed respondent municipality to
playground. build its municipal hall on what is now Lot 138-A in the late
1950s by mere tolerance of the parish priest, it does not
The petitioner also cites Roman Catholic Bishop of Jaro v. necessarily follow that Lot 138-A had become property of
Director of Lands,26 where the Court recognized that under public dominion. It does not allegedly lose its possession
the Laws of the Indies (Leyes de las Indias), the law in or ownership over the property if the possession or use by
force in the Philippine Islands during the Spanish regime, another of the same is by mere tolerance.
the property of the church in the pueblos consisted of one
parcel of land which meant "not only the two buildings but Respondent municipality, through its Mayor Protacio
also the land adjacent and contiguous to said buildings, Obrique during his testimony, allegedly admitted that
that is, the parcel which by itself constitutes one whole respondent municipalitys lot was located in a portion
piece of land bounded on its four sides by streets, and designated as Lot 2 in its Exhibit "1." The said lot was
within which said buildings, the church and the convent, along the beach and separated from Lot 138 by Emilio
are situated."27 Ostan Street. The alleged import of this admission is that
the entire Lot 138 (designated as Lot 1 in Exhibit "1") was
According to the petitioner, the appellate court erred in assigned solely to the church since a different lot was
affirming the finding of the court a quo that Lots 138-A and assigned to respondent municipality.
138-C comprise the public plaza. Unlike in Harty, no
evidence was allegedly adduced to show that from the The petitioner avers that Buruanga is an ancient Spanish
time respondent municipality was created these two lots town and that when it was created the Spanish authorities
had been set aside for the public. assigned a distinct and separate lot for its municipal
government or pueblo where it could build its municipal
Law on NatRes (23-49) 53
hall or casa real. It could thus be assumed that the casa conclusively presumed to have performed all the
real of respondent municipality would be built at about the conditions essential to a government grant and to have
same time as the church or around 1894. The petitioner received the same, and shall be entitled to a certificate of
contends that nothing in the evidence suggests that the title to such land under the provisions of this chapter.
casa real was built on Lot 138 during the said period. It
was only in the late 1950s that the municipal hall was built It is allegedly clear that as early as July 26, 1904, when
thereon upon the permission granted by the parish priest. Act No. 926 took effect, the petitioner had already
acquired a government grant, a vested title, to Lot 138.
Refuting respondent municipalitys view that it is
unthinkable that the church would be given a bigger Subsection b of Section 45 of Act No. 2874, approved on
property than the municipal government, the petitioner November 9, 1919, which amended Act No. 926, is
submits that such notion is not far fetched considering that similarly cited by the petitioner. It provided that:
the primary aim of the Spaniards at the time was to spread
the Catholic faith to the colonies. (b) Those who by themselves or their predecessors in
interest have been in the open, continuous, exclusive and
That the entire Lot 138 belonged to the petitioner is notorious possession and occupation of agricultural lands
allegedly supported by the practice during the Spanish of the public domain, under a bona fide claim of acquisition
period, as shown by the layout of the church convent and of ownership, except as against the Government, since
church plaza in practically all the old towns in the July twenty-sixth, eighteen hundred and ninety-four,
Philippines and the early cases28 decided by the Court, to except when prevented by war or force majeure. These
invariably provide the church with spacious grounds shall be conclusively presumed to have performed all the
bounded by the four principal streets of the town. conditions essential to a government grant and shall be
entitled to a certificate of title under the provisions of this
Even without any document or certificate of title thereto, chapter.
the petitioner bases its claim of ownership over Lot 138
under the Spanish Law as recognized and affirmed under On the basis of the foregoing provisions, a land
the Treaty of Paris. It cites Roman Catholic Apostolic registration proceeding instituted would, according to the
Church v. Municipality of Placer29 where the Court petitioner, "in truth be little more than a formality, at the
recognized that the church is entitled not only to most limited to ascertaining whether the possession
possession of its properties but to ownership thereof. claimed is of the required character and length of time,
Bishop of Jaro was again invoked by the petitioner as the and registration thereunder would not confer title, but
Court explained therein that it did not find it strange that simply recognize a title already vested."
the church was unable to exhibit a written title to its
property since the Laws of the Indies in force during the In addition to its arguments on the merits of the case, the
Spanish regime dictated the layout of the towns and petitioner assails the appellate courts denial of its motion
assigned the locations of the church, square and to submit additional evidence which would have showed
government administration buildings. The provisions of the that the casa real of respondent municipality, together with
Laws of the Indies pertaining thereto were held to be its plaza (Sunset Park Plaza), was located on a distinct lot
sufficient to secure the registration in the name of the (Lot 2) separated from Lot 138 by Emilio Ostan Street.
church of its land.
The petitioner urges the Court to reverse and set aside the
The petitioner asserts that even granting arguendo that Lot portion of the appellate courts decision declaring Lots
138 was not assigned to it during the Spanish regime or is 138-A and 138-C as property of public dominion and to
not owned by it pursuant to the Laws of the Indies, still, it declare the petitioner the absolute owner of the entire Lot
had acquired ipso jure or by operation of law a 138. In the alternative, the petitioner prays that it be
government grant, a vested title, to the disputed lot by allowed to submit additional evidence of its ownership over
virtue of its open, continuous, exclusive and notorious Lots 138-A and 138-C.
possession and occupation thereof since 1894. In support
Respondent Municipalitys Counter-arguments
of this contention, the petitioner cites Subsection 6 of
Section 54 of Act No. 926, which became effective on July For its part, respondent municipality contends that, except
26, 1904, and which provided that: for the figures 1894 etched on the left wall of the church,
the petitioner has not presented any evidence to show that
6. All persons who by themselves or their predecessors in
it had continuous possession of the entire Lot 138 since
interest have been in the open, continuous, exclusive and
the turn of the twentieth century. The petitioner is allegedly
notorious possession and occupation of agricultural public
of the mistaken belief that because it possessed Lot 138-
lands, as defined by said Act of Congress of July first,
B, it must have likewise possessed Lots 138-A and 138-C.
nineteen hundred and two, under a bona fide claim of
Respondent municipality claims that it is the one that has
ownership except as against the Government, for a period
been exercising acts of exclusive ownership over the
of ten years next preceding the taking effect of this Act,
disputed lot.
except when prevented by war or force majeure, shall be
Law on NatRes (23-49) 54
The petitioner has allegedly misread Harty and Bishop of Respondent municipality distinguishes the present case
Calbayog in claiming that in cases involving the church, from those cases in that there is an open contest over the
the lot adjudicated to it invariably consisted of the entire ownership and possession of Lots 138-A and 138-C and
block, bounded by a street on each of the four sides, and respondent municipality has in its favor actual and adverse
the public plazas were situated in separate blocks. While it possession thereof. It emphasizes that there is nothing in
may true that many church properties occupy an entire fact and in law that would support the petitioners bare
block in certain municipalities, it is allegedly equally true claim of ownership and possession over Lots 138-A and
that other church properties occupy only portions thereof 138-C. On the contrary, there is allegedly strong evidence
depending on the exigencies of the locality at the time showing respondent municipalitys exercise of proprietary
when the church was being established. and governmental rights over the said lots where it has
constructed permanent structures, e.g., municipal building,
In those instances that the Court allegedly adjudicated an community hospital, health center, social hall/basketball
open space in favor of the church, the local government court, and where public functions are openly conducted.
was not shown to have exercised dominion over the
property and the church has consistently established some Respondent municipality urges the Court to dismiss the
control over it, like the putting up of a religious monument petition and, instead, to affirm the decision of the court a
thereon. On the other hand, in the present case, quo declaring it the lawful owner and possessor of Lots
respondent municipality insists that it has laid adverse 138-A and 138-C.
claim over Lot 138 as early as 1909 when it applied for title
over it and was even issued a decree over the said lot. Issue
Respondent municipality places its actual, public and
adverse possession of Lot 138 at the latest in 1958 when it The substantive issue to be resolved is whether the
built its old municipal hall on the said site. Its occupation appellate court correctly declared Lots 138-A and 138-C
prior thereto could also be allegedly presumed from its as property of public dominion, hence, not susceptible to
actual possession thereof. ownership by either the petitioner and respondent
municipality.
The petitioner has allegedly failed to establish that the
construction of the old municipal building in 1958 was by Since respondent municipality no longer sought the review
mere tolerance on its part. Respondent municipality harps of the assailed decision of the appellate court, the Court
on the failure of the petitioner to present as its witness Fr. shall mainly resolve the merits of the petitioners claim of
Patio, the parish priest who supposedly gave respondent ownership over Lots 138-A and 138-C vis--vis the
municipality permission to construct its municipal building appellate courts holding that they are of public dominion,
on the disputed lot. Respondent municipality denies ever hence, not susceptible to private ownership.
seeking such permission. Further, the tax declaration
The Courts Ruling
(Exhibit "B") of the petitioner only pertained to Lot 138-B
proving that its ownership was limited to the said lot and The petition is denied.
did not extend to Lots 138-A and 138-C.
The Laws of the Indies and the cases cited
Respondent municipality avers that it is already contented
with the decision of the appellate court although the latter by the petitioner do not support its claim
allegedly erred in concluding that Lots 138-A and 138-C
are property of public dominion without taking into of ownership over Lots 138-A and 138-C
consideration that respondent municipality applied for the
issuance of title covering the disputed lot and was issued a The petitioner anchors its claim of ownership over Lots
decree thereto in 1919. The admission of Mayor Obrique, 138-A and 138-C on its theory that the entire Lot 138,
during his testimony, that respondent municipality owned bounded on all its four sides by streets, was assigned to it
the lot along the beach (Lot 2) and situated across the as far back as 1894 when the church was built in the
street from Lot 138 could not be allegedly taken to mean middle of the said lot. The cases it cited allegedly stand for
that the Mayor was disclaiming ownership over Lot 138. the proposition that "under the Laws of the Indies, when a
municipality was created, the church was assigned a
Respondent municipality theorizes that in those cases 30 property consisting of a parcel of land bounded on all its
that the Court upheld the ownership of the church over a (four) sides by streets, and that the public plaza was
subject property, the same have ever since remained the situated not on the same parcel of land assigned to the
property of the church and have been in its peaceful church but on a distinct parcel of land separated from the
possession. Further, there were no adverse claimants and parcel of land assigned to the church by a street."31
the primary issue being resolved was whether, despite
non-compliance with procedural requirements, title may be This allegation fails to persuade. The pertinent provision of
granted in favor of the church. the Laws of the Indies relating to the designation of a
parcel of land for the church upon the establishment of a
town or pueblo during the Spanish regime reads:
Law on NatRes (23-49) 55
Ley viij. Que se fabriquen el Templo principal en el sitio, y "Whoever wishes to undertake to establish a new town in
disposicion, que se ordena, y otras Iglesias, y the manner provided for, of not more than thirty nor less
Monasterios. than ten residents, shall be granted the time and territory
necessary for the purpose and under the same
En lugares Mediterrneos no se fabrique el Templo en la conditions."
plaza, sino algo distante de ella, donde est separado de
otro qualquier edificio, que no pertenezca su comodidad It may be affirmed that years afterwards all the modern
y ordenato, y porque de todas partes sea visto, y mejor pueblos of the Archipelago were formed by taking as a
venerado, est algo levantado de suelo, de forma que se basis for their establishment the barrios already populated
haya de entrar por gradas, y entre la plaza mayor, y by a large number of residents who, under the agreement
Templo se edifiquen las Casas Reales, Cabildo, to build the church of the new pueblo, the court-house and
Concejo, Aduana, y Atarazana, en tal distancia, que afterwards the schoolhouse, obtained from the General
autoricen al Templo, y no le embaracen, y en caso de Government the administrative separation of their barrio
necesidad se puedan socorrer, y si la poblacion fuere en from the pueblo on which it depended and in whose
Costa, dispngase de forma que en saliendo de Mar sea territory it was previously comprised. In such cases
visto, y su fbrica como defensa del Puerto, sealando procedure analogous to that prescribed by the Laws of the
solares cerca de l, y no su continuacion, en que se Indies was observed.
fabriquen Casas Reales, y tiendas en la plaza para
propios, imponiendo algun For the establishment, then, of new pueblos, the
administrative authority of the province, in representation
moderado tributo en las mercaderas: y asmismo sitios en of the Governor-General, designated the territory for their
otras plazas menores para Iglesias Parroquiales, y location and extension and the metes and bounds of the
Monasterios donde sean convenientes.32 same; and before the allotting the lands among the new
settlers, a special demarcation was made of the places
The above provision prescribed that the church be built at which were to serve as the public square of the pueblo, for
some distance from the square, separate from other the erection of the church, and as sites for the public
buildings in order that it may be better seen and buildings, among others, the municipal building or the casa
venerated, and raised from the ground with steps leading real, as well as of the lands which were to constitute the
to it. It decreed that government administration buildings, commons, pastures, and propios of the municipality and
including casas reales, be built between the main square the streets and roads which were to intersect the new town
and the church and at such distance as not to shut the were laid out, as may be seen by the following laws:
church from view. In cases of coastal towns, the church
was to be constructed in such location as to be seen by Law 7, title 7, book 4, of the Recompilation of the Laws of
those coming from the sea and serve for the defense of the Indies, provides:
the port.
"The district or territory to be given for settlement by
The other provisions of the Laws of the Indies on the composition shall be allotted in the following manner:
establishment of new towns or pueblos in the archipelago, There shall first be set apart the portion required for the
including the designation of lands for the church, casa lots of the pueblo, the exido or public lands, and pastures
reales (municipal buildings) and public squares, had been amply sufficient for the stock which the residents may
discussed by the Court in this wise: have, and as much more as propios del lugar or common
lands of the locality; the rest of the territory and district
xxx shall be divided into four parts one of them, of his
choice, shall be for him who takes upon himself the
The executive authorities and other officials who then obligation to found the pueblo, and the other three shall be
represented the Spanish Government in these Islands apportioned equally among the settlers."
were obliged to adjust their procedure, in the fulfillment of
their duties with regard to the establishment and laying out Law 8, of the same title and book, prescribes, among other
of new towns, to the Laws of the Indies, which determined things:
the course that they were to pursue for such purposes, as
may be seen by the following: "That, between the main square and the church, there
shall be constructed the casas reales or municipal
Law 6, title 5, book 4, of the Recompilation of the Laws of buildings, the cabildo, concejo, customs buildings," etc.
the Indies, provides, among other things:
Law 14 of the said title and book, also directs among other
"That within the boundaries which may be assigned to it, things:
there must be at least thirty residents, and each one of
them must have a house," etc. "That the viceroys shall have set aside such lands as to
them appear suitable as the common lands (propios) of
Law 7 of the same title and book contains this provision: the pueblos that have none, therewith to assist in the
Law on NatRes (23-49) 56
payment of the salaries of the corregidores, and sufficient was organized by a Fr. Ranera and which used to give
public lands (exidos) and pasture lands as provided for musical performances on the bandstand. On the feast of
and prescribed by law." Corpus Christi the parishioners would construct an altar on
this lot and hold the procession there.35
Law 1, title 13 of the aforesaid book, provides the
following: It is apparent that the Court adjudicated to the church the
ownership of Lot 1 (except a portion thereof which was a
"Such viceroys and governors as have due authority shall public thoroughfare) because the latter was able to
designate to each villa and lugar newly founded and establish that it had exercised acts of possession or
settled the lands and lots which they may need and may ownership over the same including over its empty space.
be given to them, without detriment to a third party, as In particular, the empty space was used for religious
propios, and a statement shall be sent to us of what was functions, such as the Feast of Corpus Christi and the
designated and given to each, in order that we may have procession held on the occasion and the church did not
such action approved."33 ask for any permit from the local authorities whenever it
used the said space for such activities.
Nowhere in the above provisions was it stated that the
parcel of land designated for the church of the town or In the present case, the petitioner has not shown that, at
pueblo was, in all cases, to be an entire block or bounded one time after the church was built in 1894 in the middle of
on all its four sides by streets. The petitioner thus Lot 138 (now Lot 138-B), it exercised acts of ownership or
erroneously asseverates that the said ancient laws sustain possession over Lots 138-A and 138-C as well.
its claim of ownership over the entire Lot 138.
It must be emphasized that the petitioners allegation that
Neither can it find support in the cases that it cited. A it merely tolerated the construction of not only the
careful review of these cases reveal that, in those municipal building but also the other improvements
instances where the Court upheld the claim of the church thereon, e.g., the rural health center, Buruanga community
over a parcel of land vis--vis that of the municipality or Medicare hospital, basketball court, Rizal monument and
national government, the ownership and possession by grandstand, has remained unsubstantiated. The affidavit
the church of the same had been indubitably established of Fr. Patio was correctly not given any credence since
by its exclusive exercise thereon of proprietary acts or acts he was not presented on the witness stand; thus,
of dominion. considered hearsay. Hearsay evidence is generally
excluded because the party against whom it is presented
For example, in Bishop of Calbayog v. Director of Lands, 34 is deprived of his right and opportunity to cross-examine
which according to the petitioner supports its case, the the person to whom the statement or writing is attributed.36
Court adjudicated in favor of the church the ownership of
Lot 1 (except the portion thereof occupied by a public The testimony of Mr. Jaime S. Prado, Sr., an octogenarian
thoroughfare) including not only the space occupied by the and resident of Buruanga, cannot likewise be given any
church, belfry, convent, parish school and nuns residence, credence because it consisted only of a bare assertion
but also the empty space which only had concrete that the church building and the land on which it was built,
benches as improvements thereon. bounded by streets on all its four sides, were the
petitioners property.37 He based this statement on the fact
With respect to the empty space (eastern portion of Lot 1), that as a child he heard masses at the church with his
the Court noted the following: parents. This assertion, without more or without any
corroborative evidence, is not sufficient to establish the
x x x The eastern portion of Lot 1, the area in contention,
petitioners ownership over Lots 138-A and 138-C
is an empty space except for concrete benches along the
especially in light of the fact that Mr. Prado is not
perimeter. A partly cemented path runs across this lot from
competent to testify on the matter because he had no
east to west leading up to the front or entrance of the
actual personal knowledge with respect to any
church and appears to be an extension of Anunciacion St.,
transactions involving Lot 138:
which runs from the bank of the Catarman river up to
Mendiola St. In the middle of this path, half-way between FISCAL DEL ROSARIO:
Mendiola St. and the church, is a statue of the Sacred
Heart of Jesus. Q Now, when you were President of the Parish Council,
have you access of any documents relating to church
xxx properties in Buruanga, Aklan?

x x x The Roman Catholic Church had made no A Never.


improvements on this eastern portion of Lot 1, which at
present is being used as a public playground, although a Q Now, you have stated that the boundaries of the
bandstand stood there for about three years after it was property of the[R]oman [C]atholic [C]hurch in Buruanga,
constructed in 1926 by the members of an orchestra which Aklan has previous names thus, the present name of
Law on NatRes (23-49) 57
Viven Ostan, Nitoy Sualog and Emilio Ostan is at present the swamp, men are hauling stones[,] and women [,] sand
now. My question is, what was the previous names of and gravel;
these roads?
That after so many years of hard labor the swamp was
A Of Nitoy Sualog, that was Malilipayon Street. The filled up and then the friars build a church in the center of
Provincial road street, before that was Kaaganhon street, the town;
West, Emilio Ostan, before that was Kahaponanon Street
and instead of Viven Ostan, that was Kabulakan Street. That as far as we are concerned the site where the church
now stand and the surrounding area and the site where
Q So, you will agree with me Mr. Witness so, that previous the present Municipal building now stands is even the part
names is not in any way related to the names of former of the property of the church and not the property of the
parish priest[s] of Buruanga, Aklan or saints, am I right? municipality as allege (sic) by the Mayor;

A No, sir. That we execute this affidavit with our own free act and
voluntary deed.39
Q You know very well Rev. Jesus Patinio and Mayor
Pedro Omugtong? The information proffered by these affidavits could not
have been based on the personal knowledge of the
A Yes, sir. affiants because they obviously were not yet born when
the events they narrated took place.40
Q Now, do you remember if there was any transaction of
them during your lifetime? Like in Bishop of Calbayog, the Court in Hacbang v.
Director of Lands41 adjudicated to the church the
A This Padre Patinio and I were close friends. Mr. ownership of two parcels of land, designated as Lots 1 and
Omugtong met Patinio [,] talked together about the land 2 despite the opposition of the Director of Lands who
they agreed [,] and I dont know what is there (sic) claimed that a portion on the eastern part of Lot 1 and the
agreement but the building was constructed then. entire Lot 2 were public plazas. The Court affirmed the
ownership of the church over these lots upon the following
Q Were you present during the talked (sic) of Rev. Jesus
findings:
Patinio and Mayor Pedro Omugtong?
It is inferred from the foregoing facts which are held to
A No, sir.
have been indisputably established by the evidence, that
Q You identified in this Exhibit "F" as Municipal Hall, Rural the disputed portion of lot No. 1 as well as the entire lot
Health Unit Hospital, the Buruanga Community Hospital, No. 2 belongs to the Roman Catholic Apostolic Church of
Basketball Court and the Grandstand. Now, my question the Diocese of Samar and Leyte. It cannot be denied that
is, are these buildings constructed by the [R]oman said church, for more than half a century, was in the
[C]atholic [C]hurch? possession of said lands together with the church, belfry
and convent which existed first on lot No. 1 and later
A Not one.38 provisionally on lot No. 2. The fact that the catholic
cemetery was located on lot No. 2 and that the stone posts
Even the affidavits of the other residents of Buruanga, and pillars were later erected thereon, thereby converting
which were also properly considered hearsay, made no it into a place for the celebration of the Way of the Cross,
mention of any instance where the petitioner exercised conclusively proves that the property belonged to the
acts of dominion over Lots 138-A and 138-C. These church and that the latter's possession has constantly
affidavits uniformly stated: been under claim of ownership.

That we have been residing in this Municipality since birth x x x [I]t must be presumed upon these facts that said
and that we have full knowledge of the site where the portion formed part of the parcels of land assigned and
church now stands; adjudicated by the authorities to the Roman Catholic
Apostolic Church in said town for the erection of the
That during the Pre-Spanish time, the site of the town church, belfry, convent and cemetery, all of which, as
proper was swampy, fishermen used to fish in the swamp, everybody knows, are necessary for the practice and
trees of different kinds grows (sic) along the beach; celebration of the cults of said religion.42

That when the Spaniards arrived in our town, they The proprietary acts exercised by the church over the
introduced education and religion; disputed lots consisted of the construction thereon of the
church, belfry, convent and cemetery. Moreover, it
That because there was no site for the church, the conducted thereon the Way of the Cross and other
Spaniards forced the inhabitants to work for the filling up of religious celebrations.
Law on NatRes (23-49) 58
Unlike in the Bishop of Calbayog and Hacbang, in the The Universal Encyclopedia defines an adjacent angle as
present case, the petitioner has not shown that it "an angle contiguous to another, so that one side is
exercised proprietary acts or acts of dominion over Lots common to both angles."
138-A and 138-C, to the exclusion of others, to buttress its
claim of ownership over these lots. In the case of Miller v. Cabell (81 Ky., 184) it was held that
where a change of venue was taken to an adjacent county
Neither can the petitioner rely on Roman Catholic Bishop it must be taken to an adjoining county.
of Jaro v. Director of Lands43 where the Court categorically
made the finding that the lot in question (Lot 3) had been In the case of Camp Hill Borough (142 Penn. State, 517),
in the possession of the church, as owner, for a time it was held that the word "adjacent" meant adjoining or
sufficiently long for purposes of prescription. In a prior contiguous.
case44 involving the said lot, the Court adjudged that the
church was "entitled to the possession of the following In the case of In re Municipality, etc. (7 La. Ann., 76), the
property situated in the Municipality of Sibalom: The court said: "We think the word adjacent, applied to lots, is
Church of Sibalom, the convent, contiguous to the same, synonymous with the word "contiguous."
and the land occupied by these two buildings."
In the case of the People v. Schemerhorne (19 Barber
The Court interpreted the phrase "land occupied by the [N.Y.], 576), the court said: "The interpretations given to
church and its convent" to mean "not only the two the adjacent by Walker are lying close, bordering upon
buildings, but also the land adjacent and contiguous to something."46
said buildings, that is, the parcel which by itself constitutes
Blacks Law Dictionary defines "contiguous" as "in close
one whole piece of land bounded on its four sides by
proximity; neighboring; adjoining; near in succession; in
streets, and within which said buildings, the church and
actual close contact; touching in at a point or along a
the convent, are situated."45
boundary; bounded or traversed by."47
Significantly, the parcel of land that was adjudicated in
Applying the foregoing definitions, the "land adjacent and
favor of the church was the "land adjacent and contiguous
contiguous" to the church and the parish house in the
to said buildings," i.e., church and convent. The word
present case is the land comprising Lot 138-B. On the
"adjacent" has been defined as follows:
other hand, Lots 138-A and 138-C are the lands adjacent
The word "adjacent" is of Latin derivation. An examination and contiguous to the municipal building, rural health
of its original use clearly indicates that in order that things center, Buruanga community Medicare hospital, basketball
shall be adjacent they shall be thrown near together. court, Rizal monument and grandstand thereon.

Webster in his International Dictionary defines "adjacent" Roman Catholic Bishop of Jaro therefore is not squarely
as "lying near, close or contiguous; neighboring; bordering applicable to the present case because of significant
upon;" and gives as synonyms the words "adjoining, factual differences. Specifically, in the former, the buildings
contiguous, near." or structures on the disputed land all belonged to the
church; hence, this fact was construed by the Court in
Roque Barcia in his "Diccionario General Etimolgico de la favor of the church as constituting its exercise of acts of
Lengua Espaola," in defining the word "adjacent," uses dominion over the land adjacent and contiguous to these
as synonyms "inmediato, junto, prximo." Things cannot buildings. On the other hand, the municipal building, rural
be "inmediatas, juntas, prximas" where other objects health center, Buruanga community Medicare hospital,
intervene. basketball court, Rizal monument and grandstand, all
standing on Lots 138-A and 138-C, are not owned by the
Vicente Salva in his "Nuevo Diccionario Francs-Espaol" petitioner. Moreover, the petitioner has not shown that it
defines the word "adjacent as "qui est situ aupres, aux had, at any time, exercised acts of dominion over these
environs." lots. Consequently, given its tenuous claim of ownership,
Lots 138-A and 138-C,
Black in his Law Dictionary defines "adjacent" as "lying
near or close to; contiguous. The difference between the lands adjacent and contiguous to the buildings and
adjacent and adjoining seems to be that the former implies improvements which admittedly do not belong to the
that two objects are not widely separated, though they petitioner, cannot be adjudicated to the latter under the
may not actually touch." circumstances.

Harpers Latin Dictionary as revised by Lewis and Short, in Seminary of San Carlos v. Municipality of Cebu,48 cited in
defining the word "ad-jaceo," which is equivalent to the Roman Catholic Bishop of Jaro, is also unavailing to the
English word "adjacent," says it means "to lie at or near, to petitioner because the Court, among others, simply
be contiguous to, to border upon." explained therein that the word "church" refers to the land
upon which the church stands, and not to the church
Law on NatRes (23-49) 59
building itself. In the present case, the petitioners It must be assumed that the principal residents of the old
ownership has not been limited by the court a quo and the barrio, being interested in the conversion of the barrio into
appellate court to the church structure itself but also as a civil town, arranged in such a way that the barrio, as the
including Lot 138-B, on which it stands. center of the future town which was subsequently called
Victoria, should have streets and a public plaza with its
In Roman Catholic Apostolic Church v. Municipality of church and parish house, and also a tribunal or building
Placer,49 the Court definitively recognized the juridical destined for the use of the municipality and the local
personality and proprietary rights of the church citing the official at the time called the gobernadorcillo and later on
Treaty of Paris50 and other pertinent Spanish laws. It held capitan municipal, as has occurred in the foundation of all
therein that the church not only was entitled to the the towns in these Islands, under the old administrative
possession of the church, convent and cemetery of Placer laws.
but was also the lawful owner thereof.
It may be true that the father of the witness Casimiro
It bears stressing that the crux in the foregoing cases, Taedo, who owned the space of land where the church
particularly Bishop of Calbayog, Hacbang and Jaro, is that and parish house were erected, had voluntarily donated it
the church had indubitably established its exercise of to the Catholic Church, the only one known at the time, but
exclusive proprietary acts on the lots that were subject of proper proof is lacking that the donation affirmed by the
the controversy. The same cannot be said with respect to said Taedo comprehended the whole of the large tract
the petitioner in relation to Lots 138-A and 138-C. In fact, which at the present time constitutes the plaza of the town.
not one of the enumeration ([a] to ([n]) made by the
petitioner in its Petition for Review as allegedly showing its It was a custom observed by all the towns established
ownership over Lots 138-A and 138-C categorically administratively in these Islands under the old Laws of the
establishes that it exercised thereon exclusive proprietary Indies, that on their creation, a certain amount of land was
acts or acts of dominion. always reserved for plazas, commons, and special and
communal property, and as it is unquestionable that the
The ruling in Harty v. Municipality of said large space of land was left vacant in the center of the
town of Victoria when it was constituted as a civil town,
Victoria is applicable to the present case more than twelve years prior to the appointment of a
permanent curate therein, there are good grounds to
Contrary to the stance taken by the petitioner, the ruling in
suppose that the late Vicente Taedo donated the land
Harty v. Municipality of Victoria51 is applicable to the
now occupied by the church and parish house in said
present case. The said case involved the dispute between
municipality for religious purposes, or to the church, but
the church and the Municipality of Victoria over the parcel
not to the parish curate, because at the time there was no
of land that surrounded the parish church of the said town,
curate at the new town of Victoria.
and which was called the public plaza of the same. The
Court therein held that "the whole of the land not occupied Even though all the remaining space of land which now
by the church of the town of Victoria and its parish house, forms the great plaza of the town of Victoria had been
is a public plaza of the said town, of public use." It justified owned by the said Taedo, it must be presumed that he
its conclusion, thus: waived his right thereto for the benefit of the townspeople,
since from the creation or establishment of the town, down
xxx
to the present day, all the residents, including the curate of
From the evidence presented by both parties it appears said town, have enjoyed the free use of said plaza; it has
that the town of Victoria, which was formerly only a barrio not been satisfactorily shown that the municipality or the
of the town of Tarlac and known as Canarum, was principales of the town of Victoria had donated the whole
converted into a town in 1855, and named Victoria; to this of said land to the curate of Victoria or to the Catholic
end they must have laid out the streets and the plaza of Church, as alleged, nor could it have been so donated, it
the town, in the center of which were situated the church being a public plaza destined to public use and was not
and parish house from the commencement, and at the private ownership, or patrimony of the town of Victoria, or
expiration of about twelve years the parish of said town of the Province of Tarlac.
was constituted and the priest, who was to perform the
It should be noted that, among other things, plazas
office of curate, was appointed; that from the very
destined to the public use are not subject to prescription.
beginning, the large tract of land that surrounds the church
(Art. 1936, Civil Code.) That both the curates and the
and the parish house was known as a public plaza,
gobernadorcillos of the said town procured fruit trees and
destined to the use of all the residents of the recently
plants to be set out in the plaza, does not constitute an act
founded town; public performances and religious
of private ownership, but evidences the public use thereof,
processions were held thereon without hindrance either on
or perhaps the intention to improve and embellish the said
the part of the local authorities or of the curate of said
plaza for the benefit of the townspeople.
town.
Law on NatRes (23-49) 60
Certain it is that the plaintiff has not proven that the Spanish regime or it is not the owner thereof pursuant to
Catholic Church or the parish of Victoria was the owner or the Laws of the Indies, its open, continuous, exclusive and
proprietor of the said extensive piece of land which now notorious possession and occupation of Lot 138 since
forms the public plaza of said town, nor that it was in 1894 and for many decades thereafter vests ipso jure or
possession thereof under the form and conditions required by operation of law upon the petitioner a government
by law, inasmuch as it has been fully proven that said grant, a vested title, to the subject property. It cites
plaza has been used without let or hindrance by the public Subsection 6 of Section 54 of Act No. 926 54 and
and the residents of the town of Victoria ever since its Subsection b of Section 45 of Act No. 2874.55
creation. For the above reasons, it is our opinion that the
judgment appealed from should be reversed, and that it This contention is likewise not persuasive.
should be held, as we do hereby hold, that the whole of
the land not occupied by the church of the town of Victoria One of the important requisites for the application of the
and its parish house, is a public plaza of the said town, of pertinent provisions of Act No. 926 and Act No. 2874 is the
public use, and that in consequence thereof, the defendant "open, continuous, exclusive and notorious possession
is absolved of the complaint without any special ruling as and occupation" of the land by the applicant. Actual
to the costs of both instances.52 possession of land consists in the manifestation of acts of
dominion over it of such a nature as a party would
The petitioner argues against the applicability of Harty as it naturally exercise over his own property.56 The phrase
makes much of the fact that the disputed lot therein was "possession and occupation" was explained as follows:
situated across the street from the church lot. When the
Court therein limited the ownership of the church to the It must be underscored that the law speaks of "possession
land "occupied by the church of the town of Victoria and its and occupation." Since these words are separated by the
conjunction and, the clear intention of the law is not to
parish house," it did not allegedly confine its ownership to make one synonymous with the order [sic]. Possession is
a portion of the lot on which the church and parish house broader than occupation because it includes constructive
were situated but to the block occupied by these possession. When, therefore, the law adds the word
structures. occupation, it seeks to delimit the all-encompassing effect
of constructive possession. Taken together with the words
To the Courts mind, however, whether the disputed lot open, continuous, exclusive and notorious, the word
was on the same block as the church or separated occupation serves to highlight the fact that for one to
therefrom by a street was not the crucial factor which qualify under paragraph (b) of the aforesaid section, his
constrained the Court in Harty to rule against the churchs possession of the land must not be mere fiction. As this
claim of ownership over the said property. Rather, it was Court stated, through then Mr. Justice Jose P. Laurel, in
the fact that the church was not able to prove its Lasam v. The Director of Lands:
ownership or possession thereof. The ruling on this point
is reiterated below: x x x Counsel for the applicant invokes the doctrine laid
down by us in Ramos v. Director of Lands. But it should be
Certain it is that the plaintiff has not proven that the observed that the application of the doctrine of
Catholic Church or the parish of Victoria was the owner or constructive possession in that case is subject to certain
proprietor of the said extensive piece of land which now qualifications, and this court was careful to observe that
forms the public plaza of said town, nor that it was in among these qualifications is "one particularly relating to
possession thereof under the form and conditions required the size of the tract in controversy with reference to the
by law, x x x53 portion actually in possession of the claimant." While,
therefore, "possession in the eyes of the law does not
As applied to the present case, that Lots 138-A and 138-C mean that a man has to have his feet on every square
are on the same block as the lot on which the church and meter of ground before it can be said that he is in
its parish house stand do not necessarily make them (Lots possession," possession under paragraph 6 of section 54
138-A and 138-C) also the property of the petitioner of Act No. 926, as amended by paragraph (b) of section 45
absent any evidence that its ownership or possession of Act No. 2874, is not gained by mere nominal claim. The
extended to these lots and under the conditions required mere planting of a sign or symbol of possession cannot
by law. justify a Magellan-like claim of dominion over an immense
tract of territory. Possession as a means of acquiring
Contrary to its submission, the petitioner had ownership, while it may be constructive, is not a mere
fiction. x x x
not acquired ipso jure or by operation of law
xxx
a government grant or title to the entire Lot 138
Possession is open when it is patent, visible, apparent,
The petitioner submits that even granting arguendo that
notorious and not clandestine. It is continuous when
the entire Lot 138 was not assigned to it during the
uninterrupted, unbroken and not intermittent or occasional;
Law on NatRes (23-49) 61
exclusive when the adverse possessor can show exclusive was a strong presumption that the same had been
dominion over the land and an appropriation of it to his segregated as a public plaza upon the founding of the
own use and benefit; and notorious when it is so municipality of Catarman. x x x59
conspicuous that it is generally known and talked of by the
public or the people in the neighborhood. As can be gleaned, the above discussion principally
pertained to Lot 2, a public plaza the ownership of which
Use of land is adverse when it is open and notorious.57 was disputed by the Bishop of Calbayog and the
Municipality of Catarman.
Indisputably, the petitioner has been in open, continuous,
exclusive and notorious possession and occupation of Lot The appellate court correctly cited Bishop of Calabayog.
138-B since 1894 as evidenced by the church structure However, the ruling therein pertaining to a portion of Lot 1
built thereon. However, the record is bereft of any occupied by a public thoroughfare is more apropos to the
evidence that would tend to show that such possession present case. To recall, in the said case, the application of
and occupation extended to Lots 138-A and 138-C the Bishop of Calabayog as to the eastern portion of Lot 1
beginning the same period. No single instance of the was also being opposed by the Municipality of Catarman
exercise by the petitioner of proprietary acts or acts of on the ground that it was part of the public plaza. As
dominion over these lots was established. Its mentioned earlier, the Court upheld the ownership of the
unsubstantiated claim that the construction of the church over Lot 1 including not only the space occupied by
municipal building as well as the subsequent the church, belfry, convent, parish school and nuns
improvements thereon, e.g., the rural health center, residence, but also the empty space which only had some
Buruanga community Medicare hospital, basketball court, benches as improvements thereon. Significantly, the
Rizal monument and grandstand, was by its tolerance portion of Lot 1 occupied by a public thoroughfare
does not constitute proof of possession and occupation on (Nalazon Street) was ordered excluded from the
its (the petitioners) part. application for registration filed by the church. The Court
therein made the following findings with respect to the
Absent the important requisite of open, continuous, public thoroughfare:
exclusive and notorious possession and occupation
thereon since 1894, no government grant or title to Lots Admittedly Nalazon St. was originally merely a trail used
138-A and 138-C had vested upon the petitioner ipso jure by the parishioners in going to and from the church. But
or by operation of law. Possession under paragraph 6 of since 1910, when it was opened and improved as a public
section 54 of Act No. 926, as amended by paragraph (b) of thoroughfare by the municipality, it had been continuously
section 45 of Act No. 2874, is not gained by mere nominal used as such by the townspeople of Catarman without
claim.58 objection from the Church authorities. The acacia trees
along both sides of the street were planted by the
Lots 138-A and 138-C comprise the public municipality in 1920, although these trees

plaza and are property of public dominion; were cut down recently upon order of the priest. There is
no proof that the Church merely tolerated and limited the
hence, not susceptible to private ownership use of this street for the benefit of its parishioners,
considering that the street traverses the entire length of
by the petitioner or respondent municipality
the poblacion from south to north and that Lot 1, on which
The appellate court correctly declared that Lots 138-A and the church stands, is located almost at the center of the
138-C comprise the public plaza and are property of public poblacion. The street does not stop on Lot 1 but extends
dominion; hence, may not be the object of appropriation north toward the sea, passing along the lot occupied by
either by the petitioner or respondent municipality. In the Central Elementary School and the Northern Samar
support thereof, it cited Bishop of Calbayog ratiocinating: General Hospital. Thus, it is clear that Nalazon St. inside
Lot 1 is used by the residents not only in going to the
This ruling [referring to Harty] was, in fact, reiterated in church but to the public school and the general hospital
Bishop of Calbayog v. Director of Lands (45 SCRA 418) north of Lot 1.60
involving the same question of ownership of the land
which surrounded the parish church of the town. The In the present case, the following improvements now stand
Supreme Court therein declared that the public plaza and on Lots 138-A and 138-C: the municipal building, rural
public thoroughfare are not subject to registration by the health center, Buruanga community Medicare hospital,
church; that since neither the Church nor the municipality basketball court, Rizal monument and grandstand. Except
presented positive proof of ownership or exclusive for the construction of the municipal building, the other
possession for an appreciable period of time, and the only improvements were made on Lots 138-A and 138-C, and
indubitable fact is the free and continuous use of Lot 2 by continuously used by the public without the petitioners
residents of Catarman, and the town had no public plaza objection. Further, there is no proof that the petitioner
to speak of other than the disputed parcel of land, there merely tolerated the construction of these improvements.
On the other hand, the free and continuous use by the
Law on NatRes (23-49) 62
public of Lots 138-A and 138-C, as found by the court a substantiate its claim of ownership over the entire Lot 138.
quo and affirmed by the appellate court, incontrovertibly In no sense, therefore, may it be argued that it was denied
establishes that they are property for public use. due process of law.67

On this point, Articles 420, quoted anew below, and 424 of With the reality that those documents were never
the Civil Code are applicable: presented and formally offered during the trial in the court
a quo, their belated admission for purposes of having them
Art. 420. The following things are property of public duly considered in the resolution of the case on appeal
dominion: would certainly collide with Section 34, Rule 132 of the
Rules of Court which reads:
(1) Those intended for public use, such as roads, canals,
rivers, torrents, ports and bridges constructed by the State, SECTION 34. Offer of Evidence. The court shall
banks, shores, roadsteads, and others of similar character; consider no evidence which has not been formally offered.
The purpose for which the evidence is offered must be
specified.68

Art. 424. Property for public use, in the provinces, cities, In any case, as correctly stated by the appellate court,
and municipalities, consist of the provincial roads, city these affidavits would not directly establish the petitioners
streets, municipal streets, the squares, fountains, public ownership over Lots 138-A and 138-C.
waters, promenades, and public works for public service
paid for by said provinces, cities or municipalities. WHEREFORE, premises considered, the petition is
DENIED. The Decision dated January 31, 2001 of the
Property for public use of provinces and towns are Court of Appeals and its Resolution dated July 18, 2001 in
governed by the same principles as property of public CA-G.R. CV No. 52626 are AFFIRMED in toto. SO
dominion of the same character.61 The ownership of such ORDERED.
property, which has the special characteristics of a
collective ownership for the general use and enjoyment, by
HEIRS OF JOSE AMUNATEGUI, Petitioners, v.
virtue of their application to the satisfaction of the
DIRECTOR OF FORESTRY, Respondent.; [G.R. No. L-
collective needs, is in the social group, whether national,
27873. November 29, 1983.]
provincial, or municipal.62 Their purpose is not to serve the
State as a juridical person, but the citizens; they are
ROQUE BORRE and ENCARNACION DELFIN,
intended for the common and public welfare, and so they
Petitioners, v. ANGEL ALPASAN, HEIRS OF
cannot be the object of appropriation, either by the State
MELQUIADES BORRE, EMETERIO BEREBER and
or by private persons.63
HEIRS OF JOSE AMUNATEGUI and THE CAPIZ
COURT OF FIRST INSTANCE, Respondents.; [G.R. No.
The appellate court committed no reversible error in
L-30035. November 29, 1983.]
denying the petitioners motion for reception of
evidence
1. CIVIL LAW; PUBLIC LAND ACT; FOREST LAND;
CLASSIFICATION NOT LOST EVEN IF IT HAS BEEN
In denying the petitioners motion for reception evidence,
STRIPPED OF FOREST COVER; UNLESS RELEASED
the appellate court reasoned that based on the records,
IN AN OFFICIAL PROCLAMATION AS DISPOSABLE
the petitioner was already accorded the full opportunity to
LANDS, RULES ON CONFIRMATION OF IMPERFECT
present its evidence in the court a quo and that the
TITLE DO NOT APPLY. A forested area classified as
evidence to be introduced in the desired hearing would not
forest land of the public domain does not lose such
directly establish its ownership of the disputed lots. 64
classification simply because loggers or settlers may have
The petitioners motion for reception of evidence filed with stripped it of its forest cover. Parcels of land classified as
the appellate court stated that the additional evidence that forest land may actually be covered with grass or planted
it sought to submit consisted of affidavits of old residents to crops by kaingin cultivators or other farmers. "Forest
of Buruanga attesting to the fact that "the old municipal lands" do not have to be on mountains or in out of the way
building was in fact at a place called Sunset Park prior to places. Swampy areas covered by mangrove trees, nipa
its transfer to the present site."65 These affidavits would palms, and other tress growing in brackish or sea water
allegedly establish that respondent municipality could not may also be classified as forest land. The classification is
be the owner of Lots 138-A and 138-C which it had neither descriptive of its legal nature or status and does not have
possessed nor occupied.66 to be descriptive of what the land actually looks like.
Unless and until the land classified as "forest" is released
The appellate court did not err in denying the petitioners in an official proclamation to that effect so that it may form
motion for reception of evidence. Indeed, the petitioner part of the disposable agricultural lands of the public
was already given full opportunity during the trial in the domain, the rules on confirmation of imperfect title do not
court a quo to adduce any and all relevant evidence to apply.
Law on NatRes (23-49) 63
swamp which was still classified as forest land and part of
2. ID.; ID.; FOREST LANDS; ACQUISITIVE OWNERSHIP the public domain.
NOT ACQUIRED. This Court ruled in the leading case
of Director of Forestry v. Muoz (23 SCRA 1184) that Another oppositor, Emeterio Bereber filed his opposition
possession of forest lands, no matter how long, cannot insofar as a portion of Lot No. 885 containing 117,956
ripen into private ownership. And in Republic v. Animas square meters was concerned and prayed that title to said
(56 SCRA 499), we granted the petition on the ground that portion be confirmed and registered in his name.
the ares covered by the patent and title was not
disposable public land, it being a part of the forest zone During the progress of the trial, applicant-petitioner Roque
and any patent and title to said area is void ab initio. It Borre sold whatever rights and interests he may have on
bears emphasizing that a positive act of Government is Lot No. 885 to Angel Alpasan. The latter also filed an
needed to declassify land which is classified as forest and opposition, claiming that he is entitled to have said lot
to convert it into alienable or disposable land for registered in his name.
agricultural or other purposes.
After trial, the Court of First Instance of Capiz adjudicated
3. ID.; ID.; CONFIRMATION, OF IMPERFECT TITLE 117,956 square meters to Emeterio Bereber and the rest
CASES; BURDEN OF PROVING THAT THE of the land containing 527,747 square meters was
REQUIREMENTS OF THE LAW HAVE BEEN MET, adjudicated in the proportion of 5/6 share to Angel Alpasan
RESTS ON THE APPLICANT. In confirmation of and 1/6 share to Melquiades Borre.
imperfect title cases, the applicant shoulders the burden of
proving that he meets the requirements of Section 48, Only the Heirs of Jose Amunategui and the Director of
Commonwealth Act No. 141, as amended by Republic Act Forestry filed their respective appeals with the Court of
No. 1942. He must overcome the presumption that the Appeals, The case was docketed as CA-G.R. No. 34190-
land he is applying for is part of the public domain but that R.
he has an interest therein sufficient to warrant registration
in his name because of an imperfect title such as those In its decision, the Court of Appeals
derived from old Spanish grants or that he has had held:jgc:chanrobles.com.ph
continuous, open, and notorious possession and
occupation of agricultural lands of the public domain under ". . . the conclusion so far must have to be that as to the
a bona fide claim of acquisition of ownership for at least private litigants that have been shown to have a better
thirty (30) years preceding the filing of his application. right over Lot 885 are, as to the northeastern portion of a
little less than 117,956 square meters, it was Emeterio
Bereber and as to the rest of 527,747 square meters, it
GUTIERREZ, JR., J.: was the heirs of Jose Amunategui; but the last question
that must have to be considered is whether after all, the
The two petitions for review on certiorari before us title that these two (2) private litigants have shown did not
question the decision of the Court of Appeals which amount to a registerable one in view of the opposition and
declared the disputed property as forest land, not subject evidence of the Director of Forestry; . . .
to titling in favor of private persons.
". . . turning back the clock thirty (30) years from 1955
These two petitions have their genesis in an application for when the application was filed which would place it at
confirmation of imperfect title and its registration filed with 1925, the fact must have to be accepted that during that
the Court of First Instance of Capiz. The parcel of land period, the land was a classified forest land so much so
sought to be registered is known as Lot No. 885 of the that timber licenses had to be issued to certain licensee
Cadastral Survey of Pilar, Capiz, and has an area of before 1926 and after that; that even Jose Amunategui
645,703 square meters.cralawnad himself took the trouble to ask for a license to cut timber
within the area; and this can only mean that the Bureau of
Roque Borre, petitioner in G.R. No, L-30035, and Forestry had stood and maintained its ground that it was a
Melquiades Borre, filed the application for registration. In forest land as indeed the testimonial evidence referred to
due time, the heirs of Jose Amunategui, petitioners in G.R. above persuasively indicates, and the only time when the
No. L-27873 filed an opposition to the application of Roque property was converted into a fishpond was sometime
and Melquiades Borre. At the same time, they prayed that after 1950; or a bare five (5) years before the filing of the
the title to a portion of Lot No. 885 of Pilar Cadastre application; but only after there had been a previous
containing 527,747 square meters be confirmed and warning by the District Forester that that could not be done
registered in the names of said Heirs of Jose Amunategui. because it was classified as a public forest; so that having
these in mind and remembering that even under Republic
The Director of Forestry, through the Provincial Fiscal of Act 1942 which came into effect in 1957, two (2) years
Capiz, also filed an opposition to the application for after this case had already been filed in the lower Court, in
registration of title claiming that the land was mangrove order for applicant to be able to demonstrate a registerable
Law on NatRes (23-49) 64
title he must have shown.
The need for resolving the questions raised by Roque
"open, continuous, exclusive and notorious possession Borre and Encarnacion Delfin in their petition depends on
and occupation of agricultural lands of the public domain the issue raised by the Heirs of Jose Amunategui, that is,
under a bona fide claim of acquisition of ownership for at whether or not Lot No. 885 is public forest land, not
least thirty (30) years, preceding the filing of the capable of registration in the names of the private
application; applicants.

the foregoing details cannot but justify the conclusion that The Heirs of Jose Amunategui maintain that Lot No. 885
not one of the applicants or oppositors had shown that cannot be classified as forest land because it is not thickly
during the required period of thirty (30) years prescribed forested but is a "mangrove swamp." Although conceding
by Republic Act 1942 in order for him to have shown a that a "mangrove swamp" is included in the classification
registerable title for the entire period of thirty (30) years of forest land in accordance with Section 1820 of the
before filing of the application, he had been in Revised Administrative Code, the petitioners argue that no
big trees classified in Section 1821 of said Code as first,
"open, continuous, exclusive and notorious possession second and third groups are found on the land in question.
and occupation of agricultural lands of the public domain, Furthermore, they contend that Lot 885, even if it is a
mangrove swamp, is still subject to land registration
it is evident that the Bureau of Forestry had insisted on its proceedings because the property had been in actual
claim all throughout that period of thirty (30) years and possession of private persons for many years, and
even before and applicants and their predecessors had therefore, said land was already "private land" better
made implicit recognition of that; the result must be to adapted and more valuable for agricultural than for forest
deny all these applications; this Court stating that it had purposes and not required by the public interests to be
felt impelled notwithstanding, just the same to resolve the kept under forest classification.
conflicting positions of the private litigants among
themselves as to who of them had demonstrated a better The petition is without merit.
right to possess because this Court foresees that this
litigation will go all the way to the Supreme Court and it is A forested area classified as forest land of the public
always better that the findings be as complete as possible domain does not lose such classification simply because
to enable the Highest Court to pass final judgment; loggers or settlers may have stripped it of its forest cover.
Parcels of land classified as forest land may actually be
"IN VIEW WHEREOF, the decision must have to be as it is covered with grass or planted to crops by kaingin
hereby reversed; the application as well as all the cultivators or other farmers. "Forest lands" do not have to
oppositions with the exception of that of the Director of be on mountains or in out of the way places. Swampy
Forestry which is hereby sustained are dismissed; no areas covered by mangrove trees, nipa palms, and other
more pronouncement as to costs."cralaw virtua1aw library trees growing in brackish or sea water may also be
classified as forest land. The classification is descriptive of
A petition for review on certiorari was filed by the Heirs of its legal nature or status and does not have to be
Jose Amunategui contending that the disputed lot had descriptive of what the land actually looks like. Unless and
been in the possession of private persons for over thirty until the land classified as "forest" is released in an official
years and therefore in accordance with Republic Act No. proclamation to that effect so that it may form part of the
1942, said lot could still be the subject of registration and disposable agricultural lands of the public domain, the
confirmation of title in the name of a private person in rules on confirmation of imperfect title do not apply.
accordance with Act No. 496 known as the Land
Registration Act. On the other hand, another petition for This Court ruled in the leading case of Director of Forestry
review on certiorari was filed by Roque Borre and v. Muoz (23 SCRA 1184) that possession of forest lands,
Encarnacion Delfin, contending that the trial court no matter how long, cannot ripen into private ownership.
committed grave abuse of discretion in dismissing their And in Republic v. Animas (56 SCRA 499), we granted the
complaint against the Heirs of Jose Amunategui. The petition on the ground that the area covered by the patent
Borre complaint was for the annulment of the deed of and title was not disposable public land, it being a part of
absolute sale of Lot No. 885 executed by them in favor of the forest zone and any patent and title to said area is void
the Heirs of Amunategui. The complaint was dismissed on ab initio. It bears emphasizing that a positive act of
the basis of the Court of Appeals decision that the Government is needed to declassify land which is
disputed lot is part of the public domain. The petitioners classified as forest and to convert it into alienable or
also question the jurisdiction of the Court of Appeals in disposable land for agricultural or other purposes.
passing upon the relative rights of the parties over the
disputed lot when its final decision after all is to declare The findings of the Court of Appeals are particularly well-
said lot a part of the public domain classified as forest grounded in the instant petition.
land.chanrobles law library : red
Law on NatRes (23-49) 65
The fact that no trees enumerated in Section 1821 of the period thereof may have extended, never confers title
Revised Administrative Code are found in Lot No. 885 thereto upon the possessor because the statute of
does not divest such land of its being classified as forest limitations with regard to public land does not operate
land, much less as land of the public domain. The against the State, unless the occupant can prove
appellate court found that in 1912, the land must have possession and occupation of the same under claim of
been a virgin forest as stated by Emeterio Berebers ownership for the required number of years to constitute a
witness Deogracias Gavacao, and that as late as 1926, it grant from the State. (Director of Lands v. Reyes, 68
must have been a thickly forested area as testified by SCRA 177, 195)."cralaw virtua1aw library
Jaime Bertolde. The opposition of the Director of Forestry
was strengthened by the appellate courts finding that We, therefore, affirm the finding that the disputed property
timber licenses had to be issued to certain licensees and Lot No. 885 is part of the public domain, classified as
even Jose Amunategui himself took the trouble to ask for a public forest land. There is no need for us to pass upon
license to cut timber within the area. It was only sometime the other issues raised by petitioners Roque Borre and
in 1950 that the property was converted into fishpond but Encarnacion Delfin, as such issues are rendered moot by
only after a previous warning from the District Forester that this finding.chanrobles virtual lawlibrary
the same could not be done because it was classified as
"public forest." chanrobles.com:cralaw:red WHEREFORE, the petitions in G. R. No. L-30035 and G.
R. No. L-27873 are DISMISSED for lack of merit. Costs
In confirmation of imperfect title cases, the applicant against the petitioners.
shoulders the burden of proving that he meets the SO ORDERED.
requirements of Section 48, Commonwealth Act No. 141,
as amended by Republic Act No. 1942. He must overcome J. H. ANKRON,petitioner-appellee, vs. THE
the presumption that the land he is applying for is part of GOVERNMENT OF THE PHILIPPINE ISLANDS,
the public domain but that he has an interest therein objector-appellant.; G.R. No. L-14213 August 23,
sufficient to warrant registration in his name because of an 1919;JOHNSON, J.:
imperfect title such as those derived from old Spanish
grants or that he has had continuous, open, and notorious This action was commenced in the Court of First Instance
possession and occupation of agricultural lands of the of the Province of Davao, Department of Mindanao and
public domain under a bona fide claim of acquisition of Sulu. Its purpose was to have registered, under the
ownership for at least thirty (30) years preceding the filing Torrens system, a certain piece or parcel of land situated,
of his application. bounded and particularly described in the plan and
technical description attached to the complaint and made
The decision of the appellate court is not based merely on a part thereof.
the presumptions implicit in Commonwealth Act No. 141
as amended. The records show that Lot No. 88S never The only opposition which was presented was on the part
ceased to be classified as forest land of the public domain. of the Director of Lands. The oppositor[objector] alleged
that the land in question was the property of the
In Republic v. Gonong (118 SCRA 729) we Government of the United States under the control and
ruled:jgc:chanrobles.com.ph administration of the Government of the Philippine Islands.

"As held in Oh Cho v. Director of Lands, 75 Phil. 890, all During the trial of the cause two witnesses only were
lands that were not acquired from the Government, either presented by the petitioner. No proof whatever was offered
by purchase or by grant, belong to the public domain. An by the oppositor. After hearing and considering the
exception to the rule would be any land that should have evidence, the Honorable Francisco Soriano, judge,
been in the possession of an occupant and of his reached the following conclusions of fact:
predecessors in-interests since time immemorial, for such
1. That the land sought to be registered consists of one
possession would justify the presumption that the land had
parcel of land as marked and indicated on the plan and
never been part of the public domain or that it had been a
technical description presented;
private property even before the Spanish conquest."cralaw
virtua1aw library 2. That all of said land, with the exception of a small part at
the north, the exact description and extension of which
In the instant petitions, the exception in the Oh Cho case does not appear, has been cultivated and planted for more
does not apply. The evidence is clear that Lot No. 885 had than forty-four years prior to the date of this decision;
always been public land classified as forest.
3. That said land was formerly occupied, cultivated and
Similarly, in Republic v. Vera (120 SCRA 210), we planted by Moros, Mansacas and others, under a claim of
ruled:jgc:chanrobles.com.ph ownership, and that they lived thereon and had their
houses thereon, and that portion of the land which was not
". . . The possession of public land however long the
Law on NatRes (23-49) 66
planted or cultivated was used as pasture land whereon occupation of the same under a bona fide claim of
they pastured their carabaos, cattle, and horses; ownership for a period of ten years next preceding the
taking effect of said Act.
4. That all of said Moros and Mansacas sold, transferred
and conveyed all their right, title and interest in said land to In the present case the applicant proved, and there was no
the applicant, J. H. Ankron, some eleven years past, at effort to dispute said proof, that the land in question was
which time all of the said former owners moved o n to agricultural land and that he and his predecessors in
adjoining lands where they now reside; interest had occupied the same as owners in good faith for
a period of more than forty years prior to the
5. That the possession under claim of ownership of the commencement of the present action. No question is
applicant and his predecessors in interest was shown to raised nor discussed by the appellant with reference to the
have been open, notorious, actual, public and continuous right of the Moros to acquire the absolute ownership and
for more than forty-four years past, and that their claim dominion of the land which they have occupied openly,
was exclusive of any other right adverse to all other notoriously, peacefully and adversely for a long period of
claims; years. (Cario vs. Insular Government, 7 Phil. Rep., 132
[212 U. S., 449].)
6. That the applicant now has some one hundred fifty
(150) hills of hemp, some eight thousand (8,000) cocoanut Accepting the undisputed proof, we are of the opinion that
trees, a dwelling house, various laborers' quarters, store- said paragraph 6 of section 54 of Act No. 926 has been
building, large camarin (storehouse of wood, a galvanized fully complied with and that the petitioner, so far as the
iron and other buildings and improvements on said land. second assignment of error is concerned, is entitled to
have his land registered under the Torrens system.
Upon the foregoing facts the lower court ordered and
decreed that said parcel of land be registered in the name Under the third assignment of error the appellant contends
of the said applicant, J. H. Ankron, subject, however, to that portions of said land cannot be registered in
the right of the Government of the Philippine Islands to accordance with the existing Land Registration Law for the
open a road thereon in the manner and conditions reason that they are manglares. That question is not
mentioned in said decision. The conditions mentioned with discussed in the present brief. The appellant, however.,
reference to the opening of the road, as found in said refers the court to his discussion of that question in the
decision, are that the applicant give his consent, which he case of Jocson vs. Director of Forestry(39 Phil. Rep., 560).
has already done, to the opening of said road which By reference to the argument in the brief in the case, it is
should be fifteen (15) meters wide and should follow found that the appellant relied upon the provisions of
approximately the line of the road as it now exists subject section 3 of Act No. 1148 in relation with section 1820 of
to the subsequent survey to be made by the engineer of Act No. 2711 (second Administrative Code). Section 3 of
the province of Davao. Act No. 1148 provides that "the public forests shall include
all unreserved lands covered with trees of whatever age."
From that decree the Director of Lands appealed to this Said section 1820 (Act No. 2711) provides that "for the
court. purpose of this chapter 'public forest' includes, except as
otherwise specially indicated, all unreserved public land,
The appellant argues, first, that the applicant did not
including nipa and mangrove swamps, and all forest
sufficiently identify the land in question. In reply to that
reserves of whatever character."
argument, the record shows that a detained and technical
description of the land was made a part of the record. The In the case of Mapa vs. Insular Government(10 Phil. Rep.,
evidence shows that the boundaries of the land in question 175), which decision has been follows in numerous other
were marked by monuments built of cement. The decision, the phrase "agricultural public lands" as defined
oppositor neither presented the question of the failure of by Act of Congress of July 1, 1902, was held to mean
proper identification of the land in the lower court nor "those public lands acquired from Spain which are neither
presented any proof whatever to show that said cement mineral nor timber lands" (forestry lands).
monuments did not exist.
Paragraph 6 of section 54 of Act No. 926 only permits the
The appellant, in his second assignment of error, contends registration, under the conditions therein mentioned, of
that the appellant failed to prove his possession and "public agricultural lands." It must follow, therefore, that the
occupation in accordance with the provisions of paragraph moment that it appears that the land is not agricultural, the
6 of section 54 of Act No. 926. The important prerequisites petition for registration must be denied. If the evidence
for registration of land imposed by said section 54, shows that it is public forestry land or public mineral land,
paragraph 6, are (a) that the land shall be agricultural the petition for registration must be denied. Many
publicland as defined by the Act of Congress of July 1, definitions have been given for "agricultural," "forestry,"
1902; (b) that the petitioner, by himself or his and "mineral" lands. These definitions are valuable so far
predecessors in interest, shall have been in the open, as they establish general rules. In this relation we think the
continuous, exclusive and notorious possession and executive department of the Government, through the
Law on NatRes (23-49) 67
Bureau of Forestry, may, and should, in view especially of Act (No. 1148), may decide for itself what portions of the
the provisions of section 4, 8, and 20 of Act No. 1148, "public domain" shall be set aside and reserved as forestry
define what shall be considered forestry lands, to the end or mineral land. (Ramos vs. Director of Lands (39 Phil.
that the people of the Philippine Islands shall be Rep., 175; Jocson vs. Director of Forestry, supra.)
guaranteed in "the future a continued supply of valuable
timber and other forest products." (Sec. 8, Act No. 1148.) If In view of the foregoing we are of the opinion, and so
the Bureau of Forestry should accurately and definitely order and decree, that the judgment of the lower court
define what lands are forestry, occupants in the future should be and is hereby affirmed, with the condition that
would be greatly assisted in their proof and the courts before the final certificate is issued, an accurate survey be
would be greatly aided in determining the question made of the lands to be occupied by the road above
whether the particular land is forestry or other class of mentioned and that a plan of the same be attached to the
lands. original plan upon which the petition herein is based. It is
so ordered, with costs.
In the case of Jocson vs. Director of Forestry(supra), the
Attorney-General admitted in effect that whether the STA. ROSA REALTY DEVELOPMENT CORPORATION,
particular land in question belongs to one class or another petitioner, vs. COURT OF APPEALS, JUAN B. AMANTE
is a question of fact. The mere fact that a tract of land has etc.; G.R. No. 112526 October 12, 2001; PARDO,
trees upon it or has mineral within it is not of itself J.:
sufficient to declare that one is forestry land and the other,
mineral land. There must be some proof of the extent and The case before the Court is a petition for review on
present or future value of the forestry and of the minerals. certiorari of the decision of the Court of Appeals 1 affirming
While, as we have just said, many definitions have been the decision of the Department of Agrarian Reform
given for "agriculture," "forestry," and "mineral" lands, and Adjudication Board2 (hereafter DARAB) ordering the
that in each case it is a question of fact, we think it is safe compulsory acquisition of petitioner's property under the
to say that in order to be forestry or mineral land the proof Comprehensive Agrarian Reform Program (CARP).
must show that it is more valuable for the forestry or the
mineral which it contains than it is for agricultural Petitioner Sta. Rosa Realty Development Corporation
purposes. (Sec. 7, Act No. 1148.) It is not sufficient to (hereafter, SRRDC) was the registered owner of two
show that there exists some trees upon the land or that it parcels of land, situated at Barangay Casile, Cabuyao,
bears some mineral. Land may be classified as forestry or Laguna covered by TCT Nos. 81949 and 84891, with a
mineral today, and, by reason of the exhaustion of the total area of 254.6 hectares. According to petitioner, the
timber or mineral, be classified as agricultural land parcels of land are watersheds, which provide clean
tomorrow. And vice-versa, by reason of the rapid growth of potable water to the Canlubang community, and that
timber or the discovery of valuable minerals, lands ninety (90) light industries are now located in the area.3
classified as agricultural today may be differently classified
tomorrow. Each case must be decided upon the proof in Petitioner alleged that respondents usurped its rights over
that particular case, having regard for its present or future the property, thereby destroying the ecosystem. Sometime
value for one or the other purposes. We believe, however, in December 1985, respondents filed a civil case 4 with the
considering the fact that it is a matter of public knowledge Regional Trial Court, Laguna, seeking an easement of a
that a majority of the lands in the Philippine Islands are right of way to and from Barangay Casile. By way of
agricultural lands, that the courts have a right to presume, counterclaim, however, petitioner sought the ejectment of
in the absence of evidence to the contrary, that in each private respondents.
case the lands are agricultural lands until the contrary is
In October 1986 to August 1987, petitioner filed with the
shown. Whatever the land involved in a particular land
Municipal Trial Court, Cabuyao, Laguna separate
registration case is forestry or mineral land must,
complaints for forcible entry against respondents.5
therefore, be a matter of proof. Its superior value for one
purpose or the other is a question of fact to be settled by After the filing of the ejectment cases, respondents
the proof in each particular case. The fact that the land is a petitioned the Department of Agrarian Reform (DAR) for
manglar [mangrove swamp] is not sufficient for the courts the compulsory acquisition of the SRRDC property under
to decide whether it is agricultural, forestry, or mineral the CARP.
land. It may perchance belong to one or the other of said
classes of land. The Government, in the first instance, On August 11, 1989, the Municipal Agrarian Reform
under the provisions of Act No. 1148, may, by reservation, Officer (MARO) of Cabuyao, Laguna issued a notice of
decide for itself what portions of public land shall be coverage to petitioner and invited its officials or
considered forestry land, unless private interests have representatives to a conference on August 18, 1989. 6
intervened before such reservation is made. In the latter During the meeting, the following were present:
case, whether the land is agricultural, forestry, or mineral, representatives of petitioner, the Land Bank of the
is a question of proof. Until private interests have Philippines, PARCCOM, PARO of Laguna, MARO of
intervened, the Government, by virtue of the terms of said Laguna, the BARC Chairman of Barangay Casile and
Law on NatRes (23-49) 68
some potential farmer beneficiaries, who are residents of On March 23, 1990, the LBP returned the two (2) claim
Barangay Casile, Cabuyao, Laguna. It was the consensus folders previously referred for review and evaluation to the
and recommendation of the assembly that the landholding Director of BLAD mentioning its inability to value the
of SRRDC be placed under compulsory acquisition. SRRDC landholding due to some deficiencies.

On August 17, 1989, petitioner filed with the Municipal On March 28, 1990, Executive Director Emmanuel S.
Agrarian Reform Office (MARO), Cabuyao, Laguna a Galvez wrote Land Bank President Deogracias Vistan to
"Protest and Objection" to the compulsory acquisition of forward the two (2) claim folders involving the property of
the property on the ground that the area was not SRRDC to the DARAB for it to conduct summary
appropriate for agricultural purposes. The area was proceedings to determine the just compensation for the
rugged in terrain with slopes of 18% and above and that land.
the occupants of the land were squatters, who were not
entitled to any land as beneficiaries.7 On April 6, 1990, petitioner sent a letter to the Land Bank
of the Philippines stating that its property under the
On August 29, 1989, the farmer beneficiaries together with aforesaid land titles were exempt from CARP coverage
the BARC chairman answered the protest and objection because they had been classified as watershed area and
stating that the slope of the land is not 18% but only 5- were the subject of a pending petition for land conversion.
10% and that the land is suitable and economically viable
for agricultural purposes, as evidenced by the Certification On May 10, 1990, Director Narciso Villapando of BLAD
of the Department of Agriculture, municipality of Cabuyao, turned over the two (2) claim folders (CACF's) to the
Laguna.8 Executive Director of the DAR Adjudication Board for
proper administrative valuation. Acting on the CACF's, on
On September 8, 1989, MARO Belen dela Torre made a September 10, 1990, the Board promulgated a resolution
summary investigation report and forwarded the asking the office of the Secretary of Agrarian Reform
Compulsory Acquisition Folder Indorsement (CAFI) to the (DAR) to first resolve two (2) issues before it proceeds
Provincial Agrarian Reform Officer (hereafter, PARO).9 with the summary land valuation proceedings.13

On September 21, 1989, PARO Durante Ubeda forwarded The issues that need to be threshed out were as follows:
his endorsement of the compulsory acquisition to the (1) whether the subject parcels of land fall within the
Secretary of Agrarian Reform. coverage of the Compulsory Acquisition Program of the
CARP; and (2) whether the petition for land conversion of
On November 23, 1989, Acting Director Eduardo C. the parcels of land may be granted.
Visperas of the Bureau of Land Acquisition and
Development, DAR forwarded two (2) Compulsory On December 7, 1990, the Office of the Secretary, DAR,
Acquisition Claim Folders covering the landholding of through the Undersecretary for Operations (Assistant
SRRDC, covered by TCT Nos. T-81949 and T-84891 to Secretary for Luzon Operations) and the Regional Director
the President, Land Bank of the Philippines for further of Region IV, submitted a report answering the two issues
review and evaluation.10 raised. According to them, firstly, by virtue of the issuance
of the notice of coverage on August 11, 1989, and notice
On December 12, 1989, Secretary of Agrarian Reform of acquisition on December 12, 1989, the property is
Miriam Defensor Santiago sent two (2) notices of covered under compulsory acquisition. Secondly,
acquisition11 to petitioner, stating that petitioner's Administrative Order No. 1, Series of 1990, Section IV D
landholdings covered by TCT Nos. 81949 and 84891, also supports the DAR position on the coverage of the
containing an area of 188.2858 and 58.5800 hectares, said property. During the consideration of the case by the
valued at P4,417,735.65 and P1,220,229.93, respectively, Board, there was no pending petition for land conversion
had been placed under the Comprehensive Agrarian specifically concerning the parcels of land in question.
Reform Program.
On February 19, 1991, the Board sent a notice of hearing
On February 6, 1990, petitioner SRRDC in two letters12 to all the parties interested, setting the hearing for the
separately addressed to Secretary Florencio B. Abad and administrative valuation of the subject parcels of land on
the Director, Bureau of Land Acquisition and Distribution, March 6, 1991. However, on February 22, 1991, Atty. Ma.
sent its formal protest, protesting not only the amount of Elena P. Hernandez-Cueva, counsel for SRRDC, wrote
compensation offered by DAR for the property but also the the Board requesting for its assistance in the
two (2) notices of acquisition. reconstruction of the records of the case because the
records could not be found as her co-counsel, Atty.
On March 17, 1990, Secretary Abad referred the case to Ricardo Blancaflor, who originally handled the case for
the DARAB for summary proceedings to determine just SRRDC and had possession of all the records of the case
compensation under R. A. No. 6657, Section 16. was on indefinite leave and could not be contacted. The
Board granted counsel's request and moved the hearing to
April 4, 1991.
Law on NatRes (23-49) 69
On March 18, 1991, SRRDC submitted a petition to the in the name of Sta. Rosa Realty Development
Board for the latter to resolve SRRDC's petition for Corporation;
exemption from CARP coverage before any administrative
valuation of their landholding could be had by the Board. "3. The Register of Deeds of the Province of Laguna to
cancel with dispatch Transfer certificate of Title Nos.
On April 4, 1991, the initial DARAB hearing of the case 84891 and 81949 and new one be issued in the name of
was held and subsequently, different dates of hearing the Republic of the Philippines, free from liens and
were set without objection from counsel of SRRDC. During encumbrances;
the April 15, 1991 hearing, the subdivision plan of subject
property at Casile, Cabuyao, Laguna was submitted and "4 The Department of Environment and Natural Resources
marked as Exhibit "5" for SRRDC. At the hearing on April either through its Provincial Office in Laguna or the
23, 1991, the Land Bank asked for a period of one month Regional Office, Region IV, to conduct a final segregation
to value the land in dispute. survey on the lands covered by Transfer certificate of Title
Nos. 84891 and 81949 so the same can be transferred by
At the hearing on April 23, 1991, certification from Deputy the Register of Deeds to the name of the Republic of the
Zoning Administrator Generoso B. Opina was presented. Philippines;
The certification issued on September 8, 1989, stated that
the parcels of land subject of the case were classified as "5. The Regional Office of the Department of Agrarian
"industrial Park" per Sanguniang Bayan Resolution No. Reform through its Municipal and Provincial Agrarian
45-89 dated March 29, 1989.14 Reform Office to take immediate possession on the said
landholding after Title shall have been transferred to the
To avert any opportunity that the DARAB might distribute name of the Republic of the Philippines, and distribute the
the lands to the farmer beneficiaries, on April 30, 1991, same to the immediate issuance of Emancipation Patents
petitioner filed a petition15 with DARAB to disqualify private to the farmer-beneficiaries as determined by the Municipal
respondents as beneficiaries. However, DARAB refused to Agrarian Reform Office of Cabuyao, Laguna."17
address the issue of beneficiaries.
On January 20, 1992, the Regional Trial Court, Laguna,
In the meantime, on January 20, 1992, the Regional Trial Branch 24, rendered a decision in Civil Case No. B-233318
Court, Laguna, Branch 24, rendered a decision,16 finding ruling that respondents were builders in bad faith.
that private respondents illegally entered the SRRDC
property, and ordered them evicted. On February 6, 1992, petitioner filed with the Court of
Appeals a petition for review of the DARAB decision. 19 On
On July 11, 1991, DAR Secretary Benjamin T. Leong November 5, 1993, the Court of Appeals promulgated a
issued a memorandum directing the Land Bank of the decision affirming the decision of DARAB. The decretal
Philippines to open a trust account in favor of SRRDC, for portion of the Court of Appeals decision reads:
P5,637,965.55, as valuation for the SRRDC property.
"WHEREFORE, premises considered, the DARAB
On December 19, 1991, DARAB promulgated a decision, decision dated September 19, 1991 is AFFIRMED, without
the decretal portion of which reads: prejudice to petitioner Sta. Rosa Realty Development
Corporation ventilating its case with the Special Agrarian
"WHEREFORE, based on the foregoing premises, the Court on the issue of just compensation." 20Hence, this
Board hereby orders: petition.21

"1. The dismissal for lack of merit of the protest against the On December 15, 1993, the Court issued a Resolution
compulsory coverage of the landholdings of Sta. Rosa which reads:
Realty Development Corporation (Transfer Certificates of
Title Nos. 81949 and 84891 with an area of 254.766 "G. R. Nos. 112526 (Sta. Rosa Realty Development
hectares) in Barangay Casile, Municipality of Cabuyao, Corporation vs. Court of Appeals, et. al.) Considering the
Province of Laguna under the Comprehensive Agrarian compliance, dated December 13, 1993, filed by counsel
Reform Program is hereby affirmed; for petitioner, with the resolution of December 8, 1993
which required petitioner to post a cash bond or surety
"2. The Land Bank of the Philippines (LBP) to pay Sta. bond in the amount of P1,500,000.00 Pesos before issuing
Rosa Realty Development Corporation the amount of a temporary restraining order prayed for, manifesting that
Seven Million Eight Hundred Forty-One Thousand, Nine it has posted a CASH BOND in the same amount with the
Hundred Ninety Seven Pesos and Sixty-Four centavos Cashier of the Court as evidenced by the attached official
(P7,841,997.64) for its landholdings covered by the two (2) receipt no. 315519, the Court resolved to ISSUE the
Transfer Certificates of Title mentioned above. Should Temporary Retraining Order prayed for.
there be a rejection of the payment tendered, to open, if
none has yet been made, a trust account for said amount "The Court therefore, resolved to restrain: (a) the
Department of Agrarian Reform Adjudication Board from
Law on NatRes (23-49) 70
enforcing its decision dated December 19, 1991 in DARAB the compensation for the land requiring the landowner, the
Case No. JC-R-IV-LAG-0001, which was affirmed by the LBP and other interested parties to submit fifteen (15)
Court of Appeals in a Decision dated November 5, 1993, days from receipt of the notice. After the expiration of the
and which ordered, among others, the Regional Office of above period, the matter is deemed submitted for decision.
the Department of Agrarian Reform through its Municipal The DAR shall decide the case within thirty (30) days after
and Provincial Reform Office to take immediate it is submitted for decision.
possession of the landholding in dispute after title shall
have been transferred to the name of the Republic of the e.) Upon receipt by the landowner of the corresponding
Philippines and to distribute the same through the payment, or, in case of rejection or no response from the
immediate issuance of Emancipation Patents to the landowner, upon the deposit with an accessible bank
farmer-beneficiaries as determined by the Municipal designated by the DAR of the compensation in cash or in
Agrarian Officer of Cabuyao, Laguna, (b) The Department LBP bonds in accordance with this act, the DAR shall
of Agrarian Reform and/or the Department of Agrarian make immediate possession of the land and shall request
Reform Adjudication Board, and all persons acting for and the proper Register of Deeds to issue Transfer Certificate
in their behalf and under their authority from entering the of Titles (TCT) in the name of the Republic of the
properties involved in this case and from introducing Philippines. The DAR shall thereafter proceed with the
permanent infrastructures thereon; and (c) the private redistribution of the land to the qualified beneficiaries.
respondents from further clearing the said properties of
their green cover by the cutting or burning of trees and f.) Any party who disagrees with the decision may bring
other vegetation, effective today until further orders from the matter to the court23 of proper jurisdiction for final
this Court."22 determination of just compensation.

The main issue raised is whether the property in question In compulsory acquisition of private lands, the landholding,
is covered by CARP despite the fact that the entire the landowners and farmer beneficiaries must first be
property formed part of a watershed area prior to the identified. After identification, the DAR shall send a notice
enactment of R. A. No. 6657. of acquisition to the landowner, by personal delivery or
registered mail, and post it in a conspicuous place in the
Under Republic Act No. 6657, there are two modes of municipal building and barangay hall of the place where
acquisition of private land: compulsory and voluntary. In the property is located.
the case at bar, the Department of Agrarian Reform
sought the compulsory acquisition of subject property Within thirty (30) days from receipt of the notice of
under R. A. No. 6657, Section 16, to wit: acquisition, the landowner, his administrator or
representative shall inform the DAR of his acceptance or
"Sec. 16. Procedure for Acquisition of Private Lands. For rejection of the offer.
purposes of acquisition of private lands, the following
procedures shall be followed: If the landowner accepts, he executes and delivers a deed
of transfer in favor of the government and surrenders the
a.) After having identified the land, the landowners and the certificate of title. Within thirty (30) days from the execution
beneficiaries, the DAR shall send its notice to acquire the of the deed of transfer, the Land Bank of the Philippines
land to the owners thereof, by personal delivery or (LBP) pays the owner the purchase price. If the landowner
registered mail, and post the same in a conspicuous place accepts, he executes and delivers a deed of transfer in
in the municipal building and barangay hall of the place favor of the government and surrenders the certificate of
where the property is located. Said notice shall contain the title. Within thirty days from the execution of the deed of
offer of the DAR to pay corresponding value in accordance transfer, the Land Bank of the Philippines (LBP) pays the
with the valuation set forth in Sections 17, 18, and other owner the purchase price. If the landowner rejects the
pertinent provisions hereof. DAR's offer or fails to make a reply, the DAR conducts
summary administrative proceedings to determine just
b.) Within thirty (30) days from the date of the receipt of compensation for the land. The landowner, the LBP
written notice by personal delivery or registered mail, the representative and other interested parties may submit
landowner, his administrator or representative shall inform evidence on just compensation within fifteen days from
the DAR of his acceptance or rejection of the offer. notice. Within thirty days from submission, the DAR shall
decide the case and inform the owner of its decision and
c.) If the landowner accepts the offer of the DAR, the LBP the amount of just compensation.
shall pay the landowner the purchase price of the land
within thirty (30) days after he executes and delivers a Upon receipt by the owner of the corresponding payment,
deed of transfer in favor of the government and other or, in case of rejection or lack of response from the latter,
muniments of title. the DAR shall deposit the compensation in cash or in LBP
bonds with an accessible bank. The DAR shall
d.) In case of rejection or failure to reply, the DAR shall immediately take possession of the land and cause the
conduct summary administrative proceedings to determine issuance of a transfer certificate of title in the name of the
Law on NatRes (23-49) 71
Republic of the Philippines. The land shall then be He shall discuss the MARO/BARC investigation report and
redistributed to the farmer beneficiaries. Any party may solicit the views, objection, agreements or suggestions of
question the decision of the DAR in the special agrarian the participants thereon. The landowner shall also ask to
courts (provisionally the Supreme Court designated indicate his retention area. The minutes of the meeting
branches of the regional trial court as special agrarian shall be signed by all participants in the conference and
courts) for final determination of just compensation. shall form an integral part of the CACF.

The DAR has made compulsory acquisition the priority 4. Submit all completed case folders to the Provincial
mode of land acquisition to hasten the implementation of Agrarian Reform Officer (PARO).
the Comprehensive Agrarian Reform Program (CARP).
Under Sec. 16 of the CARL, the first step in compulsory B. The PARO shall:
acquisition is the identification of the land, the landowners
and the farmer beneficiaries. However, the law is silent on 1. Ensure the individual case folders are forwarded to him
how the identification process shall be made. To fill this by his MAROs.
gap, on July 26, 1989, the DAR issued Administrative
2. Immediately upon receipt of a case folder, compute the
Order No. 12, series of 1989, which set the operating
valuation of the land in accordance with A.O. No. 6, series
procedure in the identification of such lands. The
of 1988. The valuation worksheet and the related CACF
procedure is as follows:
valuation forms shall be duly certified correct by the PARO
A. The Municipal Agrarian Reform Officer (MARO), with and all the personnel who participated in the
the assistance of the pertinent Barangay Agrarian Reform accomplishment of these forms.
Committee (BARC), shall:
3. In all cases, the PARO may validate the report of the
1. Update the masterlist of all agricultural lands covered MARO through ocular inspection and verification of the
under the CARP in his area of responsibility; the masterlist property. This ocular inspection and verification shall be
should include such information as required under the mandatory when the computed value exceeds P500,000
attached CARP masterlist form which shall include the per estate.
name of the landowner, landholding area, TCT/OCT
4. Upon determination of the valuation, forward the case
number, and tax declaration number.
folder, together with the duly accomplished valuation forms
2. Prepare the Compulsory Acquisition Case Folder and his recommendations, to the Central Office.
(CACF) for each title (OCT/TCT) or landholding covered
The LBP representative and the MARO concerned shall
under Phase I and II of the CARP except those for which
be furnished a copy each of his report.
the landowners have already filed applications to avail of
other modes of land acquisition. A case folder shall C. DAR Central Office, specifically through the Bureau of
contain the following duly accomplished forms: Land Acquisition and Distribution (BLAD), shall:
a) CARP CA Form 1MARO investigation report 1. Within three days from receipt of the case folder from
the PARO, review, evaluate and determine the final land
b) CARP CA Form No 2 Summary investigation report
valuation of the property covered by the case folder. A
findings and evaluation
summary review and evaluation report shall be prepared
c) CARP CA Form 3Applicant's Information sheet and duly certified by the BLAD Director and the personnel
directly participating in the review and final valuation.
d) CARP CA Form 4 Beneficiaries undertaking
2. Prepare, for the signature of the Secretary or her duly
e) CARP CA Form 5 Transmittal report to the PARO authorized representative, a notice of acquisition (CARP
Form 8) for the subject property. Serve the notice to the
The MARO/BARC shall certify that all information landowner personally or through registered mail within
contained in the above-mentioned forms have been three days from its approval. The notice shall include
examined and verified by him and that the same are true among others, the area subject of compulsory acquisition,
and correct. and the amount of just compensation offered by DAR.

3. Send notice of coverage and a letter of invitation to a 3. Should the landowner accept the DAR's offered value,
conference/meeting to the landowner covered by the the BLAD shall prepare and submit to the Secretary for
Compulsory Case Acquisition Folder. Invitations to the approval the order of acquisition. However, in case of
said conference meeting shall also be sent to the rejection or non-reply, the DAR Adjudication Board
prospective farmer-beneficiaries, the BARC (DARAB) shall conduct a summary administrative hearing
representatives, the Land Bank of the Philippines (LBP) to determine just compensation, in accordance with the
representative, and the other interested parties to discuss procedures provided under Administrative Order No. 13,
the inputs to the valuation of the property. series of 1989. Immediately upon receipt of the DARAB's
Law on NatRes (23-49) 72
decision on just compensation, the BLAD shall prepare due process. The implementation of the CARL is an
and submit to the Secretary for approval the required order exercise of the State's police power and the power of
of acquisition. eminent domain. To the extent that the CARL prescribes
retention limits to the landowners, there is an exercise of
4. Upon the landowner's receipt of payment, in case of police power for the regulation of private property in
acceptance, or upon deposit of payment in the designated accordance with the Constitution. But where, to carry out
bank, in case of rejection or non-response, the Secretary such regulation, the owners are deprived of lands they
shall immediately direct the pertinent Register of Deeds to own in excess of the maximum area allowed, there is also
issue the corresponding Transfer Certificate of Title (TCT) a taking under the power of eminent domain. The taking
in the name of the Republic of the Philippines. Once the contemplated is not mere limitation of the use of the land.
property is transferred, the DAR, through the PARO, shall What is required is the surrender of the title to and
take possession of the land for redistribution to qualified physical possession of the excess and all beneficial rights
beneficiaries." accruing to the owner in favor of the farmer beneficiary.

Administrative Order No. 12, Series of 1989 requires that In the case at bar, DAR has executed the taking of the
the Municipal Agrarian Reform Officer (MARO) keep an property in question. However, payment of just
updated master list of all agricultural lands under the compensation was not in accordance with the procedural
CARP in his area of responsibility containing all the requirement. The law required payment in cash or LBP
required information. The MARO prepares a Compulsory bonds, not by trust account as was done by DAR.
Acquisition Case Folder (CACF) for each title covered by
CARP. The MARO then sends the landowner a "Notice of In Association of Small Landowners in the Philippines v.
Coverage" and a "letter of invitation" to a "conference/ Secretary of Agrarian Reform, we held that "The CARP
meeting" over the land covered by the CACF. He also Law, for its part, conditions the transfer of possession and
sends invitations to the prospective farmer-beneficiaries, ownership of the land to the government on receipt of the
the representatives of the Barangay Agrarian Reform landowner of the corresponding payment or the deposit by
Committee (BARC), the Land Bank of the Philippines the DAR of the compensation in cash or LBP bonds with
(LBP) and other interested parties to discuss the inputs to an accessible bank. Until then, title also remains with the
the valuation of the property and solicit views, landowner. No outright change of ownership is
suggestions, objections or agreements of the parties. At contemplated either."24
the meeting, the landowner is asked to indicate his
retention area. Consequently, petitioner questioned before the Court of
Appeals DARAB's decision ordering the compulsory
The MARO shall make a report of the case to the acquisition of petitioner's property.25 Here, petitioner
Provincial Agrarian Reform Officer (PARO) who shall pressed the question of whether the property was a
complete the valuation of the land. Ocular inspection and watershed, not covered by CARP.
verification of the property by the PARO shall be
mandatory when the computed value of the estate Article 67 of the Water Code of the Philippines (P. D. No.
exceeds P500,000.00. Upon determination of the 1067) provides:
valuation, the PARO shall forward all papers together with
his recommendation to the Central Office of the DAR. The "Art. 67. Any watershed or any area of land adjacent to
DAR Central Office, specifically, the Bureau of Land any surface water or overlying any ground water may be
Acquisition and Distribution (BLAD) shall prepare, on the declared by the Department of Natural resources as a
signature of the Secretary or his duly authorized protected area. Rules and Regulations may be
representative, a notice of acquisition of the subject promulgated by such Department to prohibit or control
property. From this point, the provisions of R. A. No. 6657, such activities by the owners or occupants thereof within
Section 16 shall apply. the protected area which may damage or cause the
deterioration of the surface water or ground water or
For a valid implementation of the CARP Program, two interfere with the investigation, use, control, protection,
notices are required: (1) the notice of coverage and letter management or administration of such waters."
of invitation to a preliminary conference sent to the
landowner, the representative of the BARC, LBP, farmer Watersheds may be defined as "an area drained by a river
beneficiaries and other interested parties pursuant to DAR and its tributaries and enclosed by a boundary or divide
A. O. No. 12, series of 1989; and (2) the notice of which separates it from adjacent watersheds." Watersheds
acquisition sent to the landowner under Section 16 of the generally are outside the commerce of man, so why was
CARL. the Casile property titled in the name of SRRDC? The
answer is simple. At the time of the titling, the Department
The importance of the first notice, that is, the notice of of Agriculture and Natural Resources had not declared the
coverage and the letter of invitation to a conference, and property as watershed area. The parcels of land in
its actual conduct cannot be understated. They are steps Barangay Casile were declared as "PARK" by a Zoning
designed to comply with the requirements of administrative Ordinance adopted by the municipality of Cabuyao in
Law on NatRes (23-49) 73
1979, as certified by the Housing and Land Use Matang Tubig structures they couldn't care less even if it
Regulatory Board. On January 5, 1994, the Sangguniang would be destroyed.
Bayan of Cabuyao, Laguna issued a Resolution 26 voiding
the zoning classification of the land at Barangay Casile as The Casile and Kabanga-an watersheds can be
Park and declaring that the land is now classified as considered a most vital life support system to thousands of
agricultural land. inhabitants directly and indirectly affected by it. From these
watersheds come the natural God-given precious resource
The authority of the municipality of Cabuyao, Laguna to water. x x x x x
issue zoning classification is an exercise of its police
power, not the power of eminent domain. "A zoning Clearing and tilling of the lands are totally inconsistent with
ordinance is defined as a local city or municipal legislation sound watershed management. More so, the introduction
which logically arranges, prescribes, defines and of earth disturbing activities like road building and erection
apportions a given political subdivision into specific land of permanent infrastructures. Unless the pernicious
uses as present and future projection of needs." 27 agricultural activities of the Casile farmers are immediately
stopped, it would not be long before these watersheds
In Natalia Realty, Inc. v. Department of Agrarian Reform28 would cease to be of value. The impact of watershed
we held that lands classified as non-agricultural prior to the degredation threatens the livelihood of thousands of
effectivity of the CARL may not be compulsorily acquired people dependent upon it. Toward this, we hope that an
for distribution to farmer beneficiaries. acceptable comprehensive watershed development policy
and program be immediately formulated and implemented
However, more than the classification of the subject land before the irreversible damage finally happens.
as PARK is the fact that subsequent studies and survey
showed that the parcels of land in question form a vital Hence, the following are recommended:
part of a watershed area.29
7.2 The Casile farmers should be relocated and given
Now, petitioner has offered to prove that the land in financial assistance.
dispute is a "watershed or part of the protected area for
watershed purposes." Ecological balances and 7.3 Declaration of the two watersheds as critical and in
environmental disasters in our day and age seem to be need of immediate rehabilitation.
interconnected. Property developers and tillers of the land
must be aware of this deadly combination. In the case at 7.4 A comprehensive and detailed watershed
bar, DAR included the disputed parcels of land for management plan and program be formulated and
compulsory acquisition simply because the land was implemented by the Canlubang Estate in coordination with
allegedly devoted to agriculture and was titled to SRRDC, pertinent government agencies."30
hence, private and alienable land that may be subject to
The ERDB report was prepared by a composite team
CARP.
headed by Dr. Emilio Rosario, the ERDB Director, who
However, the scenario has changed, after an in-depth holds a doctorate degree in water resources from U.P. Los
study, survey and reassessment. We cannot ignore the Banos in 1987; Dr. Medel Limsuan, who obtained his
fact that the disputed parcels of land form a vital part of an doctorate degree in watershed management from
area that need to be protected for watershed purposes. In Colorado University (US) in 1989; and Dr. Antonio M.
a report of the Ecosystems Research and Development Dano, who obtained his doctorate degree in Soil and
Bureau (ERDB), a research arm of the DENR, regarding Water management Conservation from U.P. Los Banos in
the environmental assessment of the Casile and Kabanga- 1993.
an river watersheds, they concluded that:
Also, DENR Secretary Angel Alcala submitted a
"The Casile barangay covered by CLOA in question is Memorandum for the President dated September 7, 1993
situated in the heartland of both watersheds. Considering (Subject: PFVR HWI Ref.: 933103 Presidential Instructions
the barangays proximity to the Matangtubig waterworks, on the Protection of Watersheds of the Canlubang Estates
the activities of the farmers which are in conflict with at Barrio Casile, Cabuyao, Laguna) which reads:
proper soil and water conservation practices jeopardize
"It is the opinion of this office that the area in question
and endanger the vital waterworks. Degradation of the
must be maintained for watershed purposes for ecological
land would have double edge detrimental effects. On the
and environmental considerations, among others.
Casile side this would mean direct siltation of the
Although the 88 families who are the proposed CARP
Mangumit river which drains to the water impounding
beneficiaries will be affected, it is important that a larger
reservoir below. On the Kabanga-an side, this would mean
view of the situation be taken as one should also consider
destruction of forest covers which acts as recharged areas
the adverse effect on thousands of residents downstream
of the Matang Tubig springs. Considering that the people
if the watershed will not be protected and maintained for
have little if no direct interest in the protection of the
watershed purposes.
Law on NatRes (23-49) 74
"The foregoing considered, it is recommended that if EDNA COLLADO, BERNARDINA TAWAS, etc. vs.
possible, an alternate area be allocated for the affected COURT OF APPEALS and REPUBLIC OF THE
farmers, and that the Canlubang Estates be mandated to PHILIPPINES, thru the Director of Lands,
protect and maintain the area in question as a permanent BOCKASANJO ISF AWARDEES ASSOCIATION, INC.,
watershed reserved."31 LITA MENDOZA, MORADO PREFIDIGNO, TERESITA
CRUZ and CALOMA MOISES,
The definition does not exactly depict the complexities of a respondents/intervernors.; G. R. No. 107764
watershed. The most important product of a watershed is October 4, 2002; CARPIO, J.:
water which is one of the most important human necessity.
The protection of watersheds ensures an adequate supply The Case
of water for future generations and the control of
flashfloods that not only damage property but cause loss This Petition1 seeks to set aside the Decision of the Court
of lives. Protection of watersheds is an "intergenerational of Appeals,2 dated June 22, 1992, in CA-G.R. SP No.
responsibility" that needs to be answered now. 25597, which declared null and void the Decision 3 dated
January 30, 1991 of the Regional Trial Court of Antipolo,
Another factor that needs to be mentioned is the fact that Rizal, Branch 71, in LRC No. 269-A, LRC Rec. No. N-
during the DARAB hearing, petitioner presented proof that 59179, confirming the imperfect title of petitioners over a
the Casile property has slopes of 18% and over, which parcel of land.
exempted the land from the coverage of CARL. R. A. No.
6657, Section 10, provides: The Facts

"Section 10. Exemptions and Exclusions. Lands actually, On April 25, 1985, petitioner Edna T. Collado filed with the
directly and exclusively used and found to be necessary land registration court an application for registration of a
for parks, wildlife, forest reserves, reforestration, fish parcel of land with an approximate area of 1,200,766
sanctuaries and breeding grounds, watersheds and square meters or 120.0766 hectares ("Lot" for brevity).
mangroves, national defense, school sites and campuses The Lot is situated in Barangay San Isidro (formerly known
including experimental farm stations operated by public or as Boso-boso), Antipolo, Rizal, and covered by Survey
private schools for educational purposes, seeds and Plan Psu-162620. Attached to the application was the
seedlings research and pilot production centers, church technical description of the Lot as Lot Psu-162620 signed
sites and convents appurtenent thereto, communal burial by Robert C. Pangyarihan, Officer-in-Charge of the Survey
grounds and cemeteries, penal colonies and penal farms Division, Bureau of Lands, which stated, "[t]his survey is
actually worked by the inmates, government and private inside IN-12 Mariquina Watershed." On March 24, 1986,
research and quarantine centers, and all lands with petitioner Edna T. Collado filed an Amended Application to
eighteen percent (18%) slope and over, except those include additional co-applicants.4 Subsequently, more
already developed shall be exempt from coverage of this applicants joined (collectively referred to as "petitioners"
Act." for brevity).5

Hence, during the hearing at DARAB, there was proof The Republic of the Philippines, through the Solicitor
showing that the disputed parcels of land may be excluded General, and the Municipality of Antipolo, through its
from the compulsory acquisition coverage of CARP Municipal Attorney and the Provincial Fiscal of Rizal, filed
because of its very high slopes. oppositions to petitioners application. In due course, the
land registration court issued an order of general default
To resolve the issue as to the true nature of the parcels of against the whole world with the exception of the
land involved in the case at bar, the Court directs the oppositors.
DARAB to conduct a re-evaluation of the issue.
Petitioners alleged that they have occupied the Lot since
IN VIEW WHEREOF, the Court SETS ASIDE the decision time immemorial. Their possession has been open, public,
of the Court of Appeals in CA-G. R. SP No. 27234. notorious and in the concept of owners. The Lot was
surveyed in the name of Sesinando Leyva, one of their
In lieu thereof, the Court REMANDS the case to the predecessors-in-interest, as early as March 22, 1902.
DARAB for re-evaluation and determination of the nature Petitioners declared the Lot for taxation purposes and paid
of the parcels of land involved to resolve the issue of its all the corresponding real estate taxes. According to them,
coverage by the Comprehensive Land Reform Program. there are now twenty-five co-owners in pro-indiviso shares
of five hectares each. During the hearings, petitioners
In the meantime, the effects of the CLOAs issued by the submitted evidence to prove that there have been nine
DAR to supposed farmer beneficiaries shall continue to be transfers of rights among them and their predecessors-in-
stayed by the temporary restraining order issued on interest, as follows:
December 15, 1993, which shall remain in effect until final
decision on the case. No costs. SO "1. SESINANDO LEYVA was the earliest known
ORDERED. predecessor-in-interest of the Applicants who was in
Law on NatRes (23-49) 75
actual, open, notorious and continuous possession of the IMELDA CAYLALUAD, NORBERTO CAMILOTE and
property in the concept of owner. He had the property FIDELITO ECO through a Deed of Sale dated 18 January
surveyed in his name on 22 March 1902 (Exhibit "W" and 1987 (Exhibit "T" to "T-9")."6
"W-1" testimonies of J. Torres on 16 December 1987 and
Mariano Leyva on 29 December 1987). During the hearing on January 9, 1991, only the assistant
provincial prosecutor appeared without the Solicitor
2. DIOSDADO LEYVA, is the son of Sesinando Leyva, General. For failure of the oppositors to present their
who inherited the property. He had the property evidence, the land registration court issued an order
resurveyed in his name on May 21-28, 1928 (Exhibit "X" considering the case submitted for decision based on the
and "X-1"; testimony of Mariano Leyva, a son of Diosdado evidence of the petitioners. The court later set aside the
Leyva). order and reset the hearing to January 14, 1991 for the
presentation of the evidence of the oppositors. On this
3. GREGORIO CAMANTIQUE bought the property from date, counsel for oppositors failed to appear again despite
Diosdado Leyva before the Japanese Occupation of the due notice. Hence, the court again issued an order
Philippines during World War II. He owned and possessed submitting the case for decision based on the evidence of
the property until 1958. He declared the property for tax the petitioners.
purposes, the latest of which was under Tax Declaration
No. 7182 issued on 3 February 1957 (Exhibit "I" and The Trial Courts Ruling
testimony of Mariano Leyva, supra).
After appraisal of the evidence submitted by petitioners,
4. ANGELINA REYNOSO, bought the property from the land registration court held that petitioners had
Gregorio Camantique by virtue of a Deed of Sale on 3 adduced sufficient evidence to establish their registrable
February 1958 (Exhibit "H"). During the ownership of the rights over the Lot. Accordingly, the court rendered a
property by Angelina Reynoso, Mariano Leyva the decision confirming the imperfect title of petitioners. We
grandson of Sesinando Leyva, the previous owner, quote the pertinent portions of the courts decision, as
attended to the farm. (Testimony of Mariano Leyva, follows:
supra). Angelina Reynoso declared the property in her
name under Tax Declaration No. 7189 in 4 February 1958, "From the evidence presented, the Court finds that from
under Tax Declaration No. 8775 on 3 August 1965, under the testimony of the witnesses presented by the
Tax Declaration No. 16945 on 15 December 1975, and Applicants, the property applied for is in actual, open,
under Tax Declaration No. 03-06145 on 25 June 1978. public and notorious possession by the applicants and
their predecessor-in-interest since time immemorial and
5. MYRNA TORRES bought the property from Angelina said possession had been testified to by witnesses Jimmy
Reynoso on 16 October 1982 through a Deed of Sale Torres, Mariano Leyva, Sergio Montealegre, Jose Amo
(Exhibit "G"). and one Chona who were all cross-examined by Counsel
for Oppositor Republic of the Philippines.
6. EDNA COLLADO bought the property from Myrna
Torres in a Deed of Sale dated 28 April 1984 (Exhibit "P-1" Evidence was likewise presented that said property was
to "P-3"). declared for taxation purposes in the names of the
previous owners and the corresponding taxes were paid
7. Additional owners BERNARDINA TAWAS, JORETO by the Applicants and the previous owners and said
TORRES, JOSE AMO, VICENTE TORRES and SERGIO property was planted to fruit bearing trees; portions to
MONTEALEGRE who bought portions of the property from palay and portions used for grazing purposes.
Edna Collado through a Deed of Sale on 6 November
1985 (Exhibit "Q" to "Q-3"). To the mind of the Court, Applicants have presented
sufficient evidence to establish registrable title over said
8. And more additional Owners JOSEPH NUNEZ, property applied for by them.
DIOSDADO ARENOS, DANILO FABREGAS, FERNANDO
TORRES, LUZ TUBUNGBANUA, CARIDAD TUTANA, On the claim that the property applied for is within the
JOSE TORRES JR., RODRIGO TUTANA, ROSALIE Marikina Watershed, the Court can only add that all
TUTANA, NORMA ASTORIAS, MYRNA LANCION, Presidential Proclamations like the Proclamation setting
CHONA MARCIANO, CECILIA MACARANAS, PEDRO aside the Marikina Watershed are subject to "private
BRIONES, REMEDIOS BANTIQUE, DANTE rights."
MONTEALEGRE, ARMANDO TORRES, AIDA GADON
and AMELIA M. MALAPAD bought portions of the property In the case of Municipality of Santiago vs. Court of
in a Deed of Sale on 12 May 1986 (Exhibit "S" to "S-3"). Appeals, 120 SCRA 734, 1983 "private rights" is proof of
acquisition through (sic) among means of acquisition of
9. Co-owners DIOSDADO ARENOS, RODRIGO TUTANA, public lands.
CHONA MARCIANO and AMELIA MALAPAD jointly sold
their shares to new OWNERS GLORIA R. SERRANO,
Law on NatRes (23-49) 76
In the case of Director of Lands vs. Reyes, 68 SCRA 193- On November 29, 1991, Bockasanjo ISF Awardees
195, by "private rights" means that applicant should show Association, Inc., an association of holders of certificates
clear and convincing evidence that the property in of stewardship issued by the Department of Environment
question was acquired by applicants or their ancestors and Natural Resources ("DENR" for brevity) under its
either by composition title from the Spanish government or Integrated Social Forestry Program ("ISF" for brevity), filed
by Possessory Information title, or any other means for the with the Court of Appeals a Motion for Leave to Intervene
acquisition of public lands xxx" (underscoring supplied). and to Admit Petition-In-Intervention. They likewise
opposed the registration and asserted that the Lot, which
The Court believes that from the evidence presented as is situated inside the Marikina Watershed Reservation, is
above stated, Applicants have acquired private rights to inalienable. They claimed that they are the actual
which the Presidential Proclamation setting aside the occupants of the Lot pursuant to the certificates of
Marikina Watershed should be subject to such private stewardship issued by the DENR under the ISF for tree
rights. planting purposes.

At any rate, the Court notes that evidence was presented The Court of Appeals granted the motion to intervene
by the applicants that as per Certification issued by the verbally during the preliminary conference held on April 6,
Bureau of Forest Development dated March 18, 1980, the 1992. During the preliminary conference, all the parties as
area applied for was verified to be within the area represented by their respective counsels agreed that the
excluded from the operation of the Marikina Watershed only issue for resolution was whether the Lot in question is
Lands Executive Order No. 33 dated July 26, 1904 per part of the public domain.8
Proclamation No. 1283 promulgated on June 21, 1974
which established the Boso-boso Town Site Reservation, The Court of Appeals Ruling
amended by Proclamation No. 1637 dated April 18, 1977
known as the Lungsod Silangan Townsite Reservation. In a decision dated June 22, 1992, the Court of Appeals
(Exhibit "K")."7 granted the petition and declared null and void the
decision dated January 30, 1991 of the land registration
In a motion dated April 5, 1991, received by the Solicitor court. The Court of Appeals explained thus:
General on April 6, 1991, petitioners alleged that the
decision dated January 30, 1991 confirming their title had "Under the Regalian Doctrine, which is enshrined in the
become final after the Solicitor General received a copy of 1935 (Art. XIII, Sec. 1), 1973 (Art. XIV, Sec. 8), and 1987
the decision on February 18, 1991. Petitioners prayed that Constitution (Art. XII, Sec. 2), all lands of the public
the land registration court order the Land Registration domain belong to the State. An applicant, like the private
Authority to issue the necessary decree in their favor over respondents herein, for registration of a parcel of land
the Lot. bears the burden of overcoming the presumption that the
land sought to be registered forms part of the public
On April 11, 1991, the Solicitor General inquired from the domain (Director of Lands vs. Aquino, 192 SCRA 296).
Provincial Prosecutor of Rizal whether the land registration
court had already rendered a decision and if so, whether A positive Act of government is needed to declassify a
the Provincial Prosecutor would recommend an appeal. public land and to convert it into alienable or disposable
However, the Provincial Prosecutor failed to answer the land for agricultural or other purposes (Republic vs. Bacas,
query. 176 SCRA 376).

According to the Solicitor General, he received on April 23, In the case at bar, the private respondents failed to
1991 a copy of the land registration courts decision dated present any evidence whatsoever that the land applied for
January 30, 1991, and not on February 18, 1991 as as described in Psu-162620 has been segregated from the
alleged by petitioners in their motion. bulk of the public domain and declared by competent
authority to be alienable and disposable. Worse, the
In the meantime, on May 7, 1991, the land registration technical description of Psu-162620 signed by Robert C.
court issued an order directing the Land Regulation Pangyarihan, Officer-in-Charge, Survey Division, Bureau
Authority to issue the corresponding decree of registration of Lands, which was attached to the application of private
in favor of the petitioners. respondents, categorically stated that "This survey is
inside IN-12 Mariquina Watershed.""
On August 6, 1991, the Solicitor General filed with the
Court of Appeals a Petition for Annulment of Judgment That the land in question is within the Marikina Watershed
pursuant to Section 9(2) of BP Blg. 129 on the ground that Reservation is confirmed by the Administrator of the
there had been no clear showing that the Lot had been National Land Titles and Deeds in a Report, dated March
previously classified as alienable and disposable making it 2, 1988, submitted to the respondent Court in LR Case
subject to private appropriation. No. 269-A. These documents readily and effectively
negate the allegation in private respondent Collados
application that "said parcel of land known as Psu-162620
Law on NatRes (23-49) 77
is not covered by any form of title, nor any public land INTERVENTION WHICH WAS FILED OUT OF TIME OR
application and are not within any government reservation LONG AFTER THE DECISION OF THE TRIAL COURT
(Par. 8, Application; Emphasis supplied). The respondent HAD BECOME FINAL.
court could not have missed the import of these vital
documents which are binding upon the courts inasmuch as The Courts Ruling
it is the exclusive prerogative of the Executive Department
to classify public lands. They should have forewarned the The petition is bereft of merit.
respondent judge from assuming jurisdiction over the
First Issue: whether petitioners have registrable title over
case.
the Lot.
"x x x inasmuch as the said properties applied for by
There is no dispute that Executive Order No. 33 ("EO 33"
petitioners are part of the public domain, it is the Director
for brevity) dated July 26, 190410 established the Marikina
of Lands who has jurisdiction in the disposition of the
Watershed Reservation ("MWR" for brevity) situated in the
same (subject to the approval of the Secretary of Natural
Municipality of Antipolo, Rizal. Petitioners even concede
Resources and Environment), and not the courts. x x x
that the Lot, described as Lot Psu-162620, is inside the
Even assuming that petitioners did have the said
technical, literal description of the MWR. However, the
properties surveyed even before the same was declared to
main thrust of petitioners claim over the Lot is that "all
be part of the Busol Forest Reservation, the fact remains
Presidential proclamations like the proclamation setting
that it was so converted into a forest reservation, thus it is
aside the Marikina Watershed Reservation are subject to
with more reason that this action must fail. Forest lands
private rights." They point out that EO 33 contains a saving
are inalienable and possession thereof, no matter how
clause that the reservations are "subject to existing private
long, cannot convert the same into private property. And
rights, if any there be." Petitioners contend that their claim
courts are without jurisdiction to adjudicate lands within the
of ownership goes all the way back to 1902, when their
forest zone. (Heirs of Gumangan vs. Court of Appeals. 172
known predecessor-in-interest, Sesinando Leyva, laid
SCRA 563; Emphasis supplied).
claim and ownership over the Lot. They claim that the
Needless to say, a final judgment may be annulled on the presumption of law then prevailing under the Philippine Bill
ground of lack of jurisdiction, fraud or that it is contrary to of 1902 and Public Land Act No. 926 was that the land
law (Panlilio vs. Garcia, 119 SCRA 387, 391) and a possessed and claimed by individuals as their own are
decision rendered without jurisdiction is a total nullity and agricultural lands and therefore alienable and disposable.
may be struck down at any time (Suarez vs. Court of They conclude that private rights were vested on
Appeals, 186 SCRA 339)."9 Sesinando Leyva before the issuance of EO 33, thus
excluding the Lot from the Marikina Watershed
Hence, the instant petition. Reservation.

The Issues Petitioners arguments find no basis in law.

The issues raised by petitioners are restated as follows: The Regalian Doctrine: An Overview

I Under the Regalian Doctrine, all lands not otherwise


appearing to be clearly within private ownership are
WHETHER THE COURT OF APPEALS ERRED OR presumed to belong to the State.11 The Spaniards first
GRAVELY ABUSED ITS DISCRETION IN REVERSING introduced the doctrine to the Philippines through the Laws
THE DECISION OF THE TRIAL COURT GRANTING THE of the Indies and the Royal Cedulas, specifically, Law 14,
APPLICATION OF THE PETITIONERS FOR Title 12, Book 4 of the Novisima Recopilacion de Leyes de
CONFIRMATION OF TITLE; las Indias12 which laid the foundation that "all lands that
were not acquired from the Government, either by
II purchase or by grant, belong to the public domain." 13 Upon
the Spanish conquest of the Philippines, ownership of all
WHETHER THE COURT OF APPEALS ERRED OR "lands, territories and possessions" in the Philippines
GRAVELY ABUSED ITS DISCRETION IN GIVING DUE passed to the Spanish Crown.14
COURSE TO THE PETITION FOR ANNULMENT OF
JUDGMENT FILED BY THE REPUBLIC LONG AFTER The Laws of the Indies were followed by the Ley
THE DECISION OF THE TRIAL COURT HAD BECOME Hipotecaria or the Mortgage Law of 1893. The Spanish
FINAL; Mortgage Law provided for the systematic registration of
titles and deeds as well as possessory claims. The Royal
III Decree of 1894 or the "Maura Law" partly amended the
Mortgage Law as well as the Law of the Indies. The Maura
WHETHER THE COURT OF APPEALS ERRED OR
Law was the last Spanish land law promulgated in the
GRAVELY ABUSED ITS DISCRETION IN GIVING DUE
Philippines. It required the "adjustment" or registration of
COURSE TO THE INTERVENORS PETITION FOR
Law on NatRes (23-49) 78
all agricultural lands, otherwise the lands would revert to quieted upon issuance of the certificate.19
PD 1529, known
the state.15 as the Property Registration Decree enacted on June 11,
1978,20 amended and updated Act 496.
Four years later, Spain ceded to the government of the
United States all rights, interests and claims over the The 1935, 1973, 1987 Philippine Constitutions
national territory of the Philippine Islands through the
Treaty of Paris of December 10, 1898. In 1903, the United The 1935, 1973 and 1987 Constitutions adopted the
States colonial government, through the Philippine Regalian doctrine substituting, however, the state, in lieu
Commission, passed Act No. 926, the first Public Land of the King, as the owner of all lands and waters of the
Act, which was described as follows: public domain.21 Justice Reynato S. Puno, in his separate
opinion in Cruz vs. Secretary of Environment and Natural
"Act No. 926, the first Public Land Act, was passed in Resources,22 explained thus:
pursuance of the provisions of the Philippine Bill of 1902.
The law governed the disposition of lands of the public "One of the fixed and dominating objectives of the 1935
domain. It prescribed rules and regulations for the Constitutional Convention was the nationalization and
homesteading, selling and leasing of portions of the public conservation of the natural resources of the country. There
domain of the Philippine Islands, and prescribed the terms was an overwhelming sentiment in the Convention in favor
and conditions to enable persons to perfect their titles to of the principle of state ownership of natural resources and
public lands in the Islands. It also provided for the the adoption of the Regalian doctrine. State ownership of
"issuance of patents to certain native settlers upon public natural resources was seen as a necessary starting point
lands," for the establishment of town sites and sale of lots to secure recognition of the states power to control their
therein, for the completion of imperfect titles, and for the disposition, exploitation, development, or utilization. The
cancellation or confirmation of Spanish concessions and delegates to the Constitutional Convention very well knew
grants in the Islands." In short, the Public Land Act that the concept of State ownership of land and natural
operated on the assumption that title to public lands in the resources was introduced by the Spaniards, however, they
Philippine Islands remained in the government; and that were not certain whether it was continued and applied by
the governments title to public land sprung from the the Americans. To remove all doubts, the Convention
Treaty of Paris and other subsequent treaties between approved the provision in the Constitution affirming the
Spain and the United States. The term "public land" Regalian doctrine."
referred to all lands of the public domain whose title still
remained in the government and are thrown open to Thus, Section 1, Article XIII23 of the 1935 Constitution, on
private appropriation and settlement, and excluded the "Conservation and Utilization of Natural Resources" barred
patrimonial property of the government and the friar the alienation of all natural resources except public
lands."16 agricultural lands, which were the only natural resources
the State could alienate. The 1973 Constitution reiterated
Thus, it is plain error for petitioners to argue that under the the Regalian doctrine in Section 8, Article XIV24 on the
Philippine Bill of 1902 and Public Land Act No. 926, mere "National Economy and the Patrimony of the Nation". The
possession by private individuals of lands creates the legal 1987 Constitution reaffirmed the Regalian doctrine in
presumption that the lands are alienable and disposable. Section 2 of Article XII25 on "National Economy and
Patrimony".
Act 2874, the second Public Land Act, superseded Act No.
926 in 1919. After the passage of the 1935 Constitution, Both the 1935 and 1973 Constitutions prohibited the
Commonwealth Act No. 141 ("CA 141" for brevity) alienation of all natural resources except agricultural lands
amended Act 2874 in 1936. CA 141, as amended, remains of the public domain. The 1987 Constitution readopted this
to this day as the existing general law governing the policy. Indeed, all lands of the public domain as well as all
classification and disposition of lands of the public domain natural resources enumerated in the Philippine
other than timber and mineral lands.17 Constitution belong to the State.

In the meantime, in order to establish a system of Watershed Reservation is a Natural Resource


registration by which recorded title becomes absolute,
indefeasible and imprescriptible, the legislature passed Act The term "natural resource" includes "not only timber, gas,
496, otherwise known as the Land Registration Act, which oil coal, minerals, lakes, and submerged lands, but also,
took effect on February 1, 1903. Act 496 placed all features which supply a human need and contribute to the
registered lands in the Philippines under the Torrens health, welfare, and benefit of a community, and are
system.18 The Torrens system requires the government to essential to the well-being thereof and proper enjoyment of
issue a certificate of title stating that the person named in property devoted to park and recreational purposes." 26
the title is the owner of the property described therein,
In Sta. Rosa Realty Development Corp. vs. Court of
subject to liens and encumbrances annotated on the title
Appeals, et al.,27 the Court had occasion to discourse on
or reserved by law. The certificate of title is indefeasible
watershed areas. The Court resolved the issue of whether
and imprescriptible and all claims to the parcel of land are
Law on NatRes (23-49) 79
the parcel of land which the Department of Environment watersheds come the natural God-given precious resource
and Natural Resources had assessed to be a watershed water. x x x
area is exempt from the coverage of RA No. 6657 or the
Comprehensive Agrarian Reform Law ("CARL" for Clearing and tilling of the lands are totally inconsistent with
brevity).28 The Court defined watershed as "an area sound watershed management. More so, the introduction
drained by a river and its tributaries and enclosed by a of earth disturbing activities like road building and erection
boundary or divide which separates it from adjacent of permanent infrastructures. Unless the pernicious
watersheds." However, the Court also recognized that: agricultural activities of the Casile farmers are immediately
stopped, it would not be long before these watersheds
"The definition does not exactly depict the complexities of would cease to be of value. The impact of watershed
a watershed. The most important product of a watershed degradation threatens the livelihood of thousands of
is water which is one of the most important human people dependent upon it. Toward this, we hope that an
necessit(ies). The protection of watershed ensures an acceptable comprehensive watershed development policy
adequate supply of water for future generations and the and program be immediately formulated and implemented
control of flashfloods that not only damage property but before the irreversible damage finally happens."
also cause loss of lives. Protection of watersheds is an
"intergenerational" responsibility that needs to be The Court remanded the case to the Department of
answered now." Agriculture and Adjudication Board or DARAB to re-
evaluate and determine the nature of the parcels of land
Article 67 of the Water Code of the Philippines (PD 1067) involved in order to resolve the issue of its coverage by the
provides: CARL.

"Art. 67. Any watershed or any area of land adjacent to Sta. Rosa Realty gives us a glimpse of the dangers posed
any surface water or overlying any ground water may be by the misuse of natural resources such as watershed
declared by the Department of Natural Resources as a reservations which are akin to forest zones. Population
protected area. Rules and Regulations may be growth and industrialization have taken a heavy toll on the
promulgated by such Department to prohibit or control environment. Environmental degradation from unchecked
such activities by the owners or occupants thereof within human activities could wreak havoc on the lives of present
the protected area which may damage or cause the and future generations. Hence, by constitutional fiat,
deterioration of the surface water or ground water or natural resources remain to this day inalienable properties
interfere with the investigation, use, control, protection, of the State.
management or administration of such waters."
Viewed under this legal and factual backdrop, did
The Court in Sta. Rosa Realty also recognized the need to petitioners acquire, as they vigorously argue, private rights
protect watershed areas and took note of the report of the over the parcel of land prior to the issuance of EO 33
Ecosystems Research and Development Bureau (ERDB), segregating the same as a watershed reservation?
a research arm of the DENR, regarding the environmental
assessment of the Casile and Kabanga-an river The answer is in the negative.
watersheds involved in that case. That report concluded
as follows: First. An applicant for confirmation of imperfect title bears
the burden of proving that he meets the requirements of
"The Casile barangay covered by CLOA in question is Section 48 of CA 141, as amended. He must overcome
situated in the heartland of both watersheds. Considering the presumption that the land he is applying for is part of
the barangays proximity to the Matangtubig waterworks, the public domain and that he has an interest therein
the activities of the farmers which are in conflict with sufficient to warrant registration in his name arising from
proper soil and water conservation practices jeopardize an imperfect title. An imperfect title may have been derived
and endanger the vital waterworks. Degradation of the from old Spanish grants such as a titulo real or royal grant,
land would have double edge detrimental effects. On the a concession especial or special grant, a composicion con
Casile side this would mean direct siltation of the el estado or adjustment title, or a titulo de compra or title
Mangumit river which drains to the water impounding through purchase.29 Or, that he has had continuous, open
reservoir below. On the Kabanga-an side, this would mean and notorious possession and occupation of agricultural
destruction of forest covers which acts as recharged areas lands of the public domain under a bona fide claim of
of the Matangtubig springs. Considering that the people ownership for at least thirty years preceding the filing of his
have little if no direct interest in the protection of the application as provided by Section 48 (b) CA 141.
Matangtubig structures they couldnt care less even if it
would be destroyed. Originally, Section 48(b) of CA 141 provided for
possession and occupation of lands of the public domain
The Casile and Kabanga-an watersheds can be since July 26, 1894. This was superseded by RA 1942
considered a most vital life support system to thousands of which provided for a simple thirty-year prescriptive period
inhabitants directly and indirectly affected by it. From these of occupation by an applicant for judicial confirmation of an
Law on NatRes (23-49) 80
imperfect title. The same, however, has already been The period of occupancy after the issuance of EO 33 in
amended by Presidential Decree No. 1073, approved on 1904 could no longer be counted because as a watershed
January 25, 1977, the law prevailing at the time reservation, the Lot was no longer susceptible of
petitioners application for registration was filed on April occupancy, disposition, conveyance or alienation. Section
25, 1985.30 As amended, Section 48 (b) now reads: 48 (b) of CA 141, as amended, applies exclusively to
alienable and disposable public agricultural land. Forest
"(b) Those who by themselves or through their lands, including watershed reservations, are excluded. It is
predecessors-in-interest have been in open, continuous, axiomatic that the possession of forest lands or other
exclusive and notorious possession and occupation of inalienable public lands cannot ripen into private
agricultural lands of the public domain, under a bona fide ownership. In Municipality of Santiago, Isabela vs. Court of
claim of acquisition or ownership, for at least thirty years Appeals,32 the Court declared that inalienable public lands
immediately preceding the filing of the application for -
confirmation of title, except when prevented by wars or
force majeure. Those shall be conclusively presumed to "x x x cannot be acquired by acquisitive prescription.
have performed all the conditions essential to a Prescription, both acquisitive and extinctive, does not run
Government grant and shall be entitled to a certificate of against the State.
title under the provisions of this chapter."
The possession of public land, however long the period
Interpreting Section 48 (b) of CA 141, the Court stated that may have extended, never confers title thereto upon the
the Public Land Act requires that the applicant must prove possessor because the statute of limitations with regard to
the following: public land does not operate against the State, unless the
occupant can prove possession and occupation of the
"(a) that the land is alienable public land and (b) that his same under claim of ownership for the required number of
open, continuous, exclusive and notorious possession and years to constitute a grant from the State. "
occupation of the same must either be since time
immemorial or for the period prescribed in the Public Land Third, Gordula vs. Court of Appeals33 is in point. In
Act. When the conditions set by law are complied with, the Gordula, petitioners did not contest the nature of the land.
possessor of the land, by operation of law, acquires a right They admitted that the land lies in the heart of the
to a grant, a government grant, without the necessity of a Caliraya-Lumot River Forest Reserve, which Proclamation
certificate of title being issued."31 No. 573 classified as inalienable. The petitioners in
Gordula contended, however, that Proclamation No. 573
Petitioners do not claim to have documentary title over the itself recognizes private rights of landowners prior to the
Lot. Their right to register the Lot is predicated mainly reservation. They claim to have established their private
upon continuous possession since 1902. rights to the subject land. The Court ruled:

Clearly, petitioners were unable to acquire a valid and "We do not agree. No public land can be acquired by
enforceable right or title because of the failure to complete private persons without any grant, express or implied from
the required period of possession, whether under the the government; it is indispensable that there be a
original Section 48 (b) of CA 141 prior to the issuance of showing of a title from the state. The facts show that
EO 33, or under the amendment by RA 1942 and PD petitioner Gordula did not acquire title to the subject land
1073. prior to its reservation under Proclamation No. 573. He
filed his application for free patent only in January, 1973,
There is no proof that prior to the issuance of EO 33 in more than three (3) years after the issuance of
1904, petitioners had acquired ownership or title to the Lot Proclamation No. 573 in June, 1969. At that time, the land,
either by deed or by any other mode of acquisition from as part of the Caliraya-Lumot River Forest Reserve, was
the State, as for instance by acquisitive prescription. As of no longer open to private ownership as it has been
1904, Sesinando Leyva had only been in possession for classified as public forest reserve for the public good.
two years. Verily, petitioners have not possessed the
parcel of land in the manner and for the number of years Nonetheless, petitioners insist that the term, "private
required by law for the confirmation of imperfect title. rights," in Proclamation No. 573, should not be interpreted
as requiring a title. They opine that it suffices if the
Second, assuming that the Lot was alienable and claimant "had occupied and cultivated the property for so
disposable land prior to the issuance of EO 33 in 1904, EO many number of years, declared the land for taxation
33 reserved the Lot as a watershed. Since then, the Lot purposes, [paid] the corresponding real estate taxes
became non-disposable and inalienable public land. At the [which are] accepted by the government, and [his]
time petitioners filed their application on April 25, 1985, the occupancy and possession [is] continuous, open and
Lot has been reserved as a watershed under EO 33 for 81 unmolested and recognized by the government.
years prior to the filing of petitioners application. Prescinding from this premise, petitioners urge that the 25-
year possession by petitioner Gordula from 1944 to 1969,
albeit five (5) years short of the 30-year possession
Law on NatRes (23-49) 81
required under Commonwealth Act (C.A.) No. 141, as me by law, I, FERDINAND E. MARCOS, President of the
amended, is enough to vest upon petitioner Gordula the Philippines, do hereby, exclude from the operation of
"private rights" recognized and respected in Proclamation Executive Order No. 33 dated July 26, 1904, as amended
No. 573. by Executive Orders Nos. 14 and 16, both series of 1915,
which established the Watershed Reservation situated in
The case law does not support this submission. In Director the Municipality of Antipolo, Province of Rizal, Island of
of Lands vs. Reyes, we held that a settler claiming the Luzon, certain portions of land embraced therein and
protection of "private rights" to exclude his land from a reserve the same, together with the adjacent parcel of land
military or forest reservation must show "x x x by clear and of the public domain, for townsite purposes under the
convincing evidence that the property in question was provisions of Chapter XI of the Public Land Act, subject to
acquired by [any] x x x means for the acquisition of public private rights, if any there be, and to future subdivision
lands." survey in accordance with the development plan to be
prepared and approved by the Department of Local
In fine, one claiming "private rights" must prove that he Government and Community Development, which parcels
has complied with C.A. No. 141, as amended, otherwise are more particularly described as follows:
known as the Public Land Act, which prescribes the
substantive as well as the procedural requirements for Lot A (Part of Watershed Reservation)
acquisition of public lands. This law requires at least thirty
(30) years of open, continuous, exclusive and notorious A parcel of land (Lot A of Proposed Poor Mans Baguio,
possession and possession of agricultural lands of the being a portion of the Marikina Watershed, IN-2), situated
public domain, under a bona fide claim of acquisition, in the municipality of Antipolo, Province of Rizal, Island of
immediately preceding the filing of the application for free Luzon, beginning at a point marked "1" on sketch plan,
patent. The rationale for the 30-year period lies in the being N-74-30 E, 8480.00 meters more or less, from
presumption that the land applied for pertains to the State, BLLM 1, Antipolo, Rizal; thence N 33 28 W 1575.00 m. to
and that the occupants and/or possessors claim an point 2; thence N 40 26 W 1538.50 m. to point 3; thence N
interest therein only by virtue of their imperfect title or 30 50W 503.17 m. to point 4; thence N 75 02 W 704.33
continuous, open and notorious possession." m. to point 5; thence N 14 18 W 1399.39 m. to point 6;
thence N 43 25 W 477.04 m. to point 7; thence N 71 38
Next, petitioners argue that assuming no private rights had W 458.36 m. to point 8; thence N 31 05 W 1025.00 m. to
attached to the Lot prior to EO 33 in 1904, the President of point 9; thence Due North 490.38 m. to point 10; thence
the Philippines had subsequently segregated the Lot from Due North 1075.00 m. to point 11; thence Due East
the public domain and made the Lot alienable and 1000.00 m. to point 12; thence Due East 1000.00 m. to
disposable when he issued Proclamation No. 1283 on point 13; thence Due East 1000.00 m. to point 14; thence
June 21, 1974. Petitioners contend that Proclamation No. Due East 1000.00 m. to point 15; thence Due East
1283 expressly excluded an area of 3,780 hectares from 1000.00 m. to point 16; thence Due East 1000.00 m. to
the MWR and made the area part of the Boso-boso point 17; thence Due East 1075.00 m. to point 18; thence
Townsite Reservation. Petitioners assert that Lot Psu- Due South 1000.00 m. to point 19; thence Due South
162620 is a small part of this excluded town site area. 1000.00 m. to point 20; thence Due South 1000.00 m. to
Petitioners further contend that town sites are considered point 21; thence Due South 1000.00 m. to point 22; thence
alienable and disposable under CA 141. Due South 1000.00 m. to point 23; thence Due South
1000.00 m. to point 24; thence Due South 1075.00 m. to
Proclamation No. 1283 reads thus: point 25; thence Due West 1000.00 m. to point 26; thence
Due West 1000.00 m. to point 27; thence Due West
"PROCLAMATION NO. 1283
636.56 m. to point of beginning. Containing an area of
EXCLUDING FROM THE OPERATION EXECUTIVE three thousand seven hundred eighty (3,780) Hectares,
ORDER NO. 33, DATED JULY 26, 1904, AS AMENDED more or less.
BY EXECUTIVE ORDERS NOS. 14 AND 16, BOTH
Lot B (Alienable and Disposable Land)
SERIES OF 1915, WHICH ESTABLISHED THE
WATERSHED RESERVATION SITUATED IN THE A parcel of land (Lot B of Proposed Poor Mans Baguio,
MUNICIPALITY OF ANTIPOLO, PROVINCE OF RIZAL, being a portion of alienable and disposable portion of
ISLAND OF LUZON, A CERTAIN PORTION OF THE public domain) situated in the municipality of Antipolo,
LAND EMBRACED THEREIN AND RESERVING THE Province of Rizal, Island of Luzon. Beginning at a point
SAME, TOGETHER WITH THE ADJACENT PARCEL OF marked "1" on sketch plan being N 74 30 E., 8430.00 m.,
LAND OF THE PUBLIC DOMAIN, FOR TOWNSITE more or less, from BLLM 1. Antipolo, Rizal; thence Due
PURPOSES UNDER THE PROVISIONS OF CHAPTER West 363.44 m. to point 2; thence Due West 1000.00 m. to
XI OF THE PUBLIC LAND ACT. point 3; thence Due West 100.00 m. to point 4; thence Due
West 1000.00 m. to point 5; thence Due West 1075.00 m.
Upon recommendation of the Secretary of Agriculture and
to point 6; thence Due North 1000.00 m. to point 7; thence
Natural Resources and pursuant to the authority vested in
Law on NatRes (23-49) 82
Due North 1000.00 m. to point 8; thence Due North (Proposed Lungsod Silangan Townsite)
1000.00 m. to point 9; thence Due North 1000.00 m. to
point 10; thence Due North 1000.00 m. to point 11; thence A PARCEL OF LAND (Proposed Lungsod Silangan
Due North 509.62 m. to point 12; thence S. 31 05 E Townsite Reservation amending the area under SWO-
1025.00 m. to point 13; thence S 71 38 E 458.36 m. to 41762 establishing the Bagong Silangan Townsite
point 14; thence S 43 25 E 477.04 m. to point 15; thence Reservation) situated in the Municipalities of Antipolo, San
S 14 18 E 1399.39 m. to point 16; thence S 75 02 E Mateo, and Montalban, Province of Rizal, Island of Luzon.
704.33 m. to point 17; thence S. 30 50 E 503.17 m. to Bounded on the E., along lines 1-2-3-4-5-6-7-8-9-10-11-
point 18; thence S 40 26 E 1538.50 m. to point 19; thence 12-13-14-15-16-17-18-19-20-21-22-23 by the Marikina
s 33 23 e 1575.00 m to point of beginning. Containing an Watershed Reservation (IN-12); on the S., along lines 23-
area of one thousand two hundred twenty five (1,225) 24-25 by the portion of Antipolo; on the W., along lines 25-
Hectares, more or less. 26-27-28-29-30 by the Municipalities of Montalban, San
Mateo; and on the N., along lines 30-31-32-33-34-35-36-
Note: All data are approximate and subject to change 37-38-39-40-41-42-43-44 by the Angat Watershed
based on future survey. Reservation. Beginning at a point marked "1" on the
Topographic Maps with the Scale of 1:50,000 which is the
IN WITNESS WHEREOF, I Have hereunto set my hand identical corner 38 IN-12, Marikina Watershed
and caused the seal of the Republic of the Philippines to Reservation.
be affixed.
xxx xxx xxx
Done in the City of Manila, this 21st day of June, in the
year of Our Lord, nineteen hundred and seventy-four. NOTE: All data are approximate and subject to change
based on future survey.
(Sgd.) FERDINAND E. MARCOS
President Proclamation No. 765 dated October 26, 1970, which
Republic of the Philippines" covered areas entirely within the herein Lungsod Silangan
Townsite, is hereby revoked accordingly.
Proclamation No. 1283 has since been amended by
Proclamation No. 1637 issued on April 18, 1977. IN WITNESS WHEREOF, I have hereunto set my hand
Proclamation No. 1637 revised the area and location of and caused the seal of the Republic of the Philippines to
the proposed townsite. According to then DENR Secretary be affixed.
Victor O. Ramos, Proclamation No. 1637 excluded Lot A
(of which the Lot claimed by petitioners is part) for townsite Done in the City of Manila, this 18th day of April, in the
purposes and reverted it to MWR coverage.34 year of Our Lord, nineteen hundred and seventy-seven.
Proclamation No. 1637 reads:
(Sgd.) FERDINAND E. MARCOS
"PROCLAMATION NO. 1637 President of the Philippines"

AMENDING PROCLAMATION NO. 1283, DATED JUNE A positive act (e.g., an official proclamation) of the
21, 1974, WHICH ESTABLISHED THE TOWNSITE Executive Department is needed to declassify land which
RESERVATION IN THE MUNICIPALITIES OF ANTIPOLO had been earlier classified as a watershed reservation and
AND SAN MATEO, PROVINCE OF RIZAL, ISLAND OF to convert it into alienable or disposable land for
LUZON BY INCREASING THE AREA AND REVISING agricultural or other purposes.35 Unless and until the land
THE TECHNICAL DESCRIPTION OF THE LAND classified as such is released in an official proclamation so
EMBRACED THEREIN, AND REVOKING that it may form part of the disposable agricultural lands of
PROCLAMATION NO. 765 DATED OCTOBER 26, 1970 the public domain, the rules on confirmation of imperfect
THAT RESERVED PORTIONS OF THE AREA AS title do not apply.36
RESETTLEMENT SITE.
The principal document presented by petitioners to prove
Upon recommendation of the Secretary of Natural the private character of the Lot is the Certification of the
Resources and pursuant to the authority vested in me by Bureau of Forest Development dated March 18, 1986 that
law, I, FERDINAND E. MARCOS, President of the the Lot is excluded from the Marikina Watershed (Exh. R).
Philippines, do hereby amend Proclamation No. 1283, The Certification reads:
dated June 21, 1974 which established the townsite
reservation in the municipalities of Antipolo and San "Republic of the Philippines
Mateo, Province of Rizal, Island of Luzon, by increasing Ministry of Natural Resources
the area and revising the technical descriptions of the land
BUREAU OF FOREST DEVELOPMENT
embraced therein, subject to private rights, if any there be,
REGION IV
which parcel of land is more particularly described as
EL AL Building
follows:
100 Quezon Avenue, Quezon City
Law on NatRes (23-49) 83
MAR 18 1986 1. A parcel of land described in plan Psu-162620 situated
in the Barrio of San Isidro, Municipality of Antipolo,
VERIFICATION ON THE STATUS OF LAND: Province of Rizal, is applied for registration of title in the
case at bar.
TO WHOM IT MAY CONCERN:
2. After plotting plan Psu-162620 in our Municipal Index
This is to certify that the tract of land situated in Barangay Map it was found that a portion of the SW, described as
San Isidro, Antipolo, Rizal, containing an area of Lot 3 in plan Psu-173790 was previously the subject of
1,269,766 square meters, as shown and described on the registration in Land Reg. Case No. N-9578, LRC Record
reverse side hereof, surveyed by Geodetic Engineer No. N-55948 and was issued Decree No. N-191242 on
Telesforo Cabading for Angelina C. Reynoso, is verified to April 4, 1986 in the name of Apolonia Garcia, et al.,
be within the area excluded from the operation of Marikina pursuant to the Decision and Order for Issuance of the
Watershed Reservation established under Executive Decree dated February 8, 1984 and March 6, 1984,
Order No. 33 dated July 26, 1904 per Proclamation No. respectively, and the remaining portion of plan Psu-
1283, promulgated on June 21, 1974, which established 162620 is inside IN-12, Marikina Watershed. x x x
the Boso-Boso Townsite Reservation, amended by
proclamation No. 1637 dated April 18, 1977 known as "WHEREFORE, this matter is respectfully submitted to the
Lungsod Silangan Townsite Reservation. Honorable Court for its information and guidance with the
recommendation that the application in the instant
Subject area also falls within the bounds of Bagong proceedings be dismissed, after due hearing (Underlining
Lipunan Site under P.D. 1396 dated June 2, 1978 under supplied)."
the sole jurisdiction of the Ministry of Human Settlements,
to the exclusion of any other government agencies. Likewise, in a letter38 dated November 11, 1991, the
Deputy Land Inspector, DENR, Region IV, Community
This verification is made upon the request of the Chief, Environment and Natural Resources Office, Antipolo,
Legal Staff, R-4 as contained in his internal memorandum Rizal, similarly confirmed that the Lot is within the MWR.
dated March 18, 1986. The letter states:

Verified by: "That the land sought to be registered is situated at San


Isidro (Boso-boso), Antipolo, Rizal, with an area of ONE
(Sgd) ROMEO C. PASCUBILLO
HUNDRED TWENTY SIX POINT ZERO SEVEN SIXTY
Cartographer II
SIX (126.0766) hectares, more particularly described in
Checked by: Psu-162620, which is within the Marikina Watershed
Reservation under Executive Order No. 33 dated July 2,
(Sgd) ARMENDO R. CRUZ 1904 which established the Marikina Watershed
Supervising Cartographer Reservation (IN-12) x x x.

ATTESTED: "x x x

(Sgd) LUIS G. DACANAY "That the land sought to be registered is not a private
Chief, Forest Engineering & Infrastructure Section" property of the Registration Applicant but part of the public
domain, not subjected to disposition and is covered by
The above certification on which petitioners rely that a Proclamation No. 585 for Integrated Social Forestry
reclassification had occurred, and that the Lot is covered Program hence, L.R.C. No. 269-A is recommended for
by the reclassification, is contradicted by several rejection (Underlining supplied)." Copy of the letter is
documents submitted by the Solicitor General before the attached herewith as Annex "3" and made an integral part
land registration court. hereof."

The Solicitor General submitted to the land registration Lastly, the Solicitor General pointed out that attached to
court a Report37 dated March 2, 1988, signed by petitioner Edna T. Collados [as original applicant]
Administrator Teodoro G. Bonifacio of the then National application is the technical description39 of the Lot signed
Land Titles and Deeds Registration Administration, by Robert C. Pangyarihan, Officer-in-Charge of the Survey
confirming that the Lot described in Psu-162620 forms part Division of the Bureau of Lands. This technical description
of the MWR. He thus recommended the dismissal of the categorically stated that the Lot "is inside IN-12 Mariquina
application for registration. The Report states: Watershed."

"COMES NOW the Administrator of the National Land The evidence of record thus appears unsatisfactory and
Titles and Deeds Registration Commission and to this insufficient to show clearly and positively that the Lot had
Honorable Court respectfully reports that: been officially released from the Marikina Watershed
Reservation to form part of the alienable and disposable
Law on NatRes (23-49) 84
lands of the public domain. We hold that once a parcel of decision on April 23, 1991.41 Petitioners point out that the
land is included within a watershed reservation duly Solicitor General filed with the Court of Appeals the
established by Executive Proclamation, as in the instant petition for annulment of judgment invoking Section 9(2) of
case, a presumption arises that the land continues to be BP Blg. 12942 only on August 6, 1991, after the decision
part of such Reservation until clear and convincing had supposedly become final and executory. Moreover,
evidence of subsequent declassification is shown. petitioners further point out that the Solicitor General filed
the petition for annulment after the land registration court
It is obvious, based on the facts on record that neither issued its order of May 6, 1991 directing the Land
petitioners nor their predecessors-in-interest have been in Registration Authority to issue the corresponding decree of
open, continuous, exclusive and notorious possession and registration.
occupation of the Lot for at least thirty years immediately
preceding the filing of the application for confirmation of The Solicitor General sought the annulment of the decision
title. Even if they submitted sufficient proof that the Lot had on the ground that the land registration court had no
been excluded from the MWR upon the issuance of jurisdiction over the case, specifically, over the Lot which
Proclamation No. 1283 on June 21, 1974, petitioners was not alienable and disposable. The Solicitor General
possession as of the filing of their application on April 25, maintained that the decision was null and void.
1985 would have been only eleven years counted from the
issuance of the proclamation in 1974. The result will not Petitioners argue that the remedy of annulment of
change even if we tack in the two years Sesinando Leyva judgment is no longer available because it is barred by the
allegedly possessed the Lot from 1902 until the issuance principle of res judicata. They insist that the land
of EO 33 in 1904. Petitioners case falters even more registration court had jurisdiction over the case which
because of the issuance of Proclamation No. 1637 on April involves private land. They also argue that the Republic is
18, 1977. According to then DENR Secretary Victor estopped from questioning the land registration courts
Ramos, Proclamation No. 1637 reverted Lot A or the jurisdiction considering that the Republic participated in
townsite reservation, where petitioners' Lot is supposedly the proceedings before the court.
situated, back to the MWR.
It is now established that the Lot, being a watershed
Finally, it is of no moment if the areas of the MWR are now reservation, is not alienable and disposable public land.
fairly populated and vibrant communities as claimed by The evidence of the petitioners do not clearly and
petitioners. The following ruling may be applied to this convincingly show that the Lot, described as Lot Psu-
case by analogy: 162620, ceased to be a portion of the area classified as a
watershed reservation of the public domain. Any title to the
"A forested area classified as forest land of the public Lot is void ab initio. In view of this, the alleged procedural
domain does not lose such classification simply because infirmities attending the filing of the petition for annulment
loggers or settlers may have stripped it of its forest cover. of judgment are immaterial since the land registration court
Parcels of land classified as forest land may actually be never acquired jurisdiction over the Lot. All proceedings of
covered with grass or planted to crops by kaingin the land registration court involving the Lot are therefore
cultivators or other farmers. "Forest lands" do not have to null and void.
be on mountains or in out of the way places. Swampy
areas covered by mangrove trees, nipa palms and other We apply our ruling in Martinez vs. Court of Appeals, 43 as
trees growing in brackish or sea water may also be follows:
classified as forest land. The classification is descriptive of
its legal nature or status and does not have to be "The Land Registration Court has no jurisdiction over non-
descriptive of what the land actually looks like. Unless and registrable properties, such as public navigable rivers
until the land classified as "forest" is released in an official which are parts of the public domain, and cannot validly
proclamation to that effect so that it may form part of the adjudge the registration of title in favor of private applicant.
disposable agricultural lands of the public domain, the Hence, the judgment of the Court of First Instance of
rules on confirmation of imperfect title do not apply."40 Pampanga as regards the Lot No. 2 of certificate of Title
No. 15856 in the name of petitioners may be attacked at
Second Issue: Whether the petition for annulment of any time, either directly or collaterally, by the State which
judgment is not bound by any prescriptive period provided for by the
Statute of Limitations."
should have been given due course.
We also hold that environmental consequences in this
Petitioners fault the Court of Appeals for giving due course case override concerns over technicalities and rules of
to the Republics petition for annulment of judgment which procedure.
was filed long after the decision of the land registration
court had allegedly become final and executory. The land In Republic vs. De los Angeles,44 which involved the
registration court rendered its decision on January 30, registration of public lands, specifically parts of the sea,
1991 and the Solicitor General received a copy of the the Court rejected the principle of res judicata and
Law on NatRes (23-49) 85
estoppel to silence the Republics claim over public lands. Veterans, Kilingan and Barangay San Joseph and
The Court said: Paenaan, Municipality of Antipolo, Province of Rizal and
place the same under the Integrated Social Forestry
"It should be noted further that the doctrine of estoppel or Program of the Department of Environment and Natural
laches does not apply when the Government sues as a Resources in accordance with existing laws, rules and
sovereign or asserts governmental rights, nor does regulations, which parcel of land is more particularly
estoppel or laches validate an act that contravenes law or described as follows:
public policy, and that res judicata is to be disregarded if
its application would involve the sacrifice of justice to "A PARCEL OF LAND, within the Marikina Watershed
technicality." Reservation situated in the Municipality of Antipolo,
Province of Rizal, beginning at point "1" on plan, being
The Court further held that "the right of reversion or identical to corner 1 of Marikina Watershed Reservation;
reconveyance to the State of the public properties thence
registered and which are not capable of private
appropriation or private acquisition does not prescribe." xxx xxx xxx

Third issue: Whether the petition-in-intervention is proper. Containing an area of One Thousand Four Hundred Thirty
(1,430) Hectares.
The Bockasanjo ISF Awardees Association, Inc., an
association of holders of certificates of stewardship issued All other lands covered and embraced under Executive
by the DENR under its Integrated Social Forestry Order No. 33 as amended, not otherwise affected by this
Program, filed with the Court of Appeals on November 29, Proclamation, shall remain in force and effect.
1991 a Motion for Leave to Intervene and to Admit
Petition-In-Intervention. IN WITNESS WHEREOF, I have hereunto set my hand
and caused the seal of the Republic of the Philippines to
According to intervenors, they are the actual occupants of be affixed.
the Lot which petitioners sought to register. Aware that the
parcels of land which their forefathers had occupied, Done in the City of Manila, this 5th day of June, in the year
developed and tilled belong to the Government, they filed of Our Lord, nineteen hundred and ninety.
a petition with then President Corazon C. Aquino and then
DENR Secretary Fulgencio S. Factoran, to award the (Sgd.) CORAZON C. AQUINO
parcels of land to them. President of the Philippines"

Secretary Factoran directed the Director of Forest Pursuant to Proclamation No. 585, the chief of the ISF
Management Bureau to take steps for the segregation of Unit, acting through the Regional Executive Director of the
the aforementioned area from the MWR for development DENR (Region IV), issued sometime between the years
under the DENRs ISF Programs. Subsequently, then 1989 to 1991 certificates of stewardship contracts to bona
President Aquino issued Proclamation No. 585 dated June fide residents of the barangays mentioned in the
5, 1990 excluding 1,430 hectares from the operation of EO proclamation as qualified recipients of the ISF programs.
33 and placed the same under the DENRs Integrated Among those awarded were intervenors. The certificates
Social Forestry Program. Proclamation No. 585 reads: of stewardship are actually contracts of lease granted by
the DENR to actual occupants of parcels of land under its
PROCLAMATION NO. 585 ISF programs for a period of twenty-five (25) years,
renewable for another twenty-five (25) years.45 The DENR
AMENDING FURTHER EXECUTIVE ORDER NO. 33, awarded contracts of stewardship to ISF participants in
DATED JULY 26, 1904 WHICH ESTABLISHED THE Barangay San Isidro (or Boso-boso) and the other
MARIKINA WATERSHED RESERVATION (IN-12) AS barangays based on the Inventory of Forest Occupants
AMENDED, BY EXCLUDING CERTAIN PORTIONS OF the DENR had conducted.46
LANDS EMBRACED THEREIN SITUATED AT SITIOS
BOSOBOSO, KILINGAN, VETERANS, BARANGAYS According to intervenors, they learned only on July 31,
SAN JOSEPH AND PAENAAN, MUNICIPALITY OF 1991 about the pendency of LRC Case No. 269-A before
ANTIPOLO, PROVINCE OF RIZAL, ISLAND OF LUZON. the Regional Trial Court of Antipolo, Rizal. On August 8,
1991, they filed a Motion for Leave to Intervene and to
Upon recommendation of the Secretary of Environment Admit Opposition in Intervention before the land
and Natural Resources and pursuant to the authority registration court to assert their rights and to protect their
vested in me by law, I, CORAZON C. AQUINO, President interests.
of the Philippines, do hereby exclude from the operation of
Executive Order No. 33, which established the Marikina However, shortly after the filing of their opposition,
Watershed Reservation, certain parcel of land of the public intervenors learned that the land registration court had
domain embraced therein situated in Sitios Bosoboso, already rendered a decision on January 30, 1991
Law on NatRes (23-49) 86
confirming petitioners imperfect title. Intervenors counsel claims of contending parties. It was created not to hinder
received a copy of the decision on August 9, 1991. and delay but to facilitate and promote the administration
of justice. It does not constitute the thing itself which courts
On August 14, 1991, intervenors filed a motion to vacate are always striving to secure to litigants. It is designed as
judgment and for new trial before the land registration the means best adopted to obtain that thing. In other
court. According to intervenors, the land registration court words, it is a means to an end."
could not act on its motions due to the restraining order
issued by the Court of Appeals on August 8, 1991, To be sure, the Court of Appeals did not pass upon the
enjoining the land registration court from executing its actual status of intervenors in relation to the Lot as this
decision, as prayed for by the Solicitor General in its was not in issue. Neither was the validity of the certificates
petition for annulment of judgment. The intervenors were of stewardship contracts which intervenors allegedly
thus constrained to file a petition for intervention before the possessed inquired into considering this too was not in
Court of Appeals which allowed the same. issue. In fact, intervenors did not specifically seek any
relief apart from a declaration that the Lot in question
Rule 19 of the 1997 Rules of Civil Procedure 47 provides in remains inalienable land of the public domain. We cannot
pertinent parts: fault the Court of Appeals for allowing the intervention, if
only to provide the rival groups a peaceful venue for
Section 1. Who may intervene. A person who has a legal ventilating their sides. This case has already claimed at
interest in the matter in litigation, or in the success of least five lives due to the raging dispute between the rival
either of the parties, or an interest against both, or is so camps of the petitioners on one side and those of the
situated as to be adversely affected by a distribution or DENR awardees on the other. It also spawned a number
other disposition of property in the custody of the court, or of criminal cases between the two rival groups including
an officer thereof may, with leave of court, be allowed to malicious mischief, robbery and arson. A strict application
intervene in the action. The Court shall consider whether of the rules would blur this bigger, far more important
or not the intervention will unduly delay or prejudice the picture.
adjudication of the rights of the original parties, and
whether or not the inertvenors rights may be fully WHEREFORE, the Petition is DENIED. The Decision of
protected in a separate proceeding. the Court of Appeals dated June 22, 1992 declaring null
and void the Decision dated January 30, 1991 of Branch
Sec. 2. Time to intervene. The motion to intervene may 71, Regional Trial Court of Antipolo, Rizal, in LRC No.
be filed at any time before rendition of judgment by the trial 269-A, LRC Rec. No. N-59179 is AFFIRMED. SO
court. A copy of the pleading-in-intervention shall be ORDERED.
attached to the motion and served on the original parties.
THE DIRECTOR OF FORESTRY, vs. RUPERTO A.
As a rule, intervention is allowed "before rendition of
VILLAREAL, ; G.R. No. L-32266 February 27, 1989;
judgment by the trial court," as Section 2, Rule 19
CRUZ, J.:
expressly provides. However, the Court has recognized
exceptions to this rule in the interest of substantial justice.
The basic question before the Court is the legal
Mago vs. Court of Appeals48 reiterated the ruling in
classification of mangrove swamps, or manglares, as they
Director of Lands vs. Court of Appeals, where the Court
are commonly known. If they are part of our public forest
allowed the motions for intervention even when the case
lands, they are not alienable under the Constitution. If they
had already reached this Court. Thus, in Mago the Court
are considered public agricultural lands, they may be
held that:
acquired under private ownership. The private
respondent's claim to the land in question must be judged
"It is quite clear and patent that the motions for
by these criteria.
intervention filed by the movants at this stage of the
proceedings where trial had already been concluded x x x
The said land consists of 178,113 square meters of
and on appeal x x x the same affirmed by the Court of
mangrove swamps located in the municipality of Sapian,
Appeals and the instant petition for certiorari to review said
Capiz. Ruperto Villareal applied for its registration on
judgment is already submitted for decision by the Supreme
January 25, 1949, alleging that he and his predecessors-
Court, are obviously and, manifestly late, beyond the
in-interest had been in possession of the land for more
period prescribed under x x x Section 2, Rule 12 of the
than forty years. He was opposed by several persons,
rules of Court.
including the petitioner on behalf of the Republic of the
Philippines. After trial, the application was approved by the
But Rule 12 of the Rules of Court, like all other Rules
Court of First Instance. of Capiz. 1 The decision was
therein promulgated, is simply a rule of procedure, the
affirmed by the Court of Appeals. 2 The Director of
whole purpose and object of which is to make the powers
Forestry then came to this Court in a petition for review on
of the Court fully and completely available for justice. The
certiorari claiming that the land in dispute was forestal in
purpose of procedure is not to thwart justice. Its proper
aim is to facilitate the application of justice to the rival
Law on NatRes (23-49) 87
nature and not subject to private appropriation. He asks manglares and nipa lands into fisheries which became a
that the registration be reversed. common feature of settlement along the coast and at the
same time of the change of sovereignty constituted one of
It should be stressed at the outset that both the petitioner the most productive industries of the Islands, the
and the private respondent agree that the land is abrogation of which would destroy vested interests and
mangrove land. There is no dispute as to this. The bone of prove a public disaster.
contention between the parties is the legal nature of
mangrove swamps or manglares. The petitioner claims, it Mangrove swamps were thus considered agricultural lands
is forestal and therefore not disposable and the private and so susceptible of private ownership.
respondent insists it is alienable as agricultural land. The
issue before us is legal, not factual. Subsequently, the Philippine Legislature categorically
declared, despite the above-cited case, that mangrove
For a proper background of this case, we have to go back swamps form part of the public forests of this country. This
to the Philippine Bill of 1902, one of the earlier American it did in the Administrative Code of 1917, which became
organic acts in the country. By this law, lands of the public effective on October 1 of that year, thus:
domain in the Philippine Islands were classified into three
grand divisions, to wit, agricultural, mineral and timber or Section 1820. Words and phrase defined. - For the
forest lands. This classification was maintained in the purpose of this chapter 'public forest' includes, except as
Constitution of the Commonwealth, promulgated in 1935, otherwise specially indicated, all unreserved public land,
until it was superseded by the Constitution of 1973. That including nipa and mangrove swamps, and all forest
new charter expanded the classification of public lands to reserves of whatever character.
include industrial or commercial, residential, resettlement,
and grazing lands and even permitted the legislature to It is noteworthy, though, that notwithstanding this
provide for other categories. 3 This provision has been definition, the Court maintained the doctrine in the
reproduced, but with substantial modifications, in the Montano case when two years later it held in the case of
present Constitution. 4 Jocson v. Director of Forestry: 7

Under the Commonwealth Constitution, which was the ...the words timber land are always translated in the
charter in force when this case arose, only agricultural Spanish translation of that Act (Act of Congress) as
lands were allowed to be alienated. 5 Their disposition terrenos forestales. We think there is an error in this
was provided for under C.A. No. 141. Mineral and timber translation and that a better translation would be 'terrenos
or forest lands were not subject to private ownership madereros.' Lumber land in English means land with trees
unless they were first reclassified as agricultural lands and growing on it. The mangler plant would never be called a
so released for alienation. tree in English but a bush, and land which has only
bushes, shrubs or aquatic plants growing on it cannot be
In the leading case of Montano v. Insular Government, 6 called 'timber land.
promulgated in 1909, mangrove swamps or manglares
were defined by the Court as: xxx xxx xxx

... mud flats, alternately washed and exposed by the tide, The fact that there are a few trees growing in a manglare
in which grows various kindred plants which will not live or nipa swamps does not change the general character of
except when watered by the sea, extending their roots the land from manglare to timber land.
deep into the mud and casting their seeds, which also
More to the point, addressing itself directly to above-
germinate there. These constitute the mangrove flats of
quoted Section 1820, the Court declared:
the tropics, which exist naturally, but which are also, to
some extent cultivated by man for the sake of the 'In the case of Mapa vs. Insular Government (10 Phil.
combustible wood of the mangrove and like trees as well Rep., 175), this Court said that the phrase agricultural
as for the useful nipa palm propagated thereon. Although lands as used in Act No. 926 means those public lands
these flats are literally tidal lands, yet we are of the opinion acquired from Spain which are not timber or mineral lands.
that they cannot be so regarded in the sense in which that
term is used in the cases cited or in general American Whatever may have been the meaning of the term
jurisprudence. The waters flowing over them are not 'forestry' under the Spanish law, the Act of Congress of
available for purpose of navigation, and they may be July 1st 1902, classifies the public lands in the Philippine
disposed of without impairment of the public interest in Islands as timber, mineral or agricultural lands, and all
what remains. public lands that are not timber or mineral lands are
necessarily agricultural public lands, whether they are
xxx used as nipa swamps, manglares, fisheries or ordinary
farm lands.
Under this uncertain and somewhat unsatisfactory
condition of the law, the custom had grown of converting
Law on NatRes (23-49) 88
The definition of forestry as including manglares found in forested but is a 'mangrove swamps.' Although conceding
the Administrative Code of 1917 cannot affect rights which that 'mangrove swamp' is included in the classification of
vested prior to its enactment. forest land in accordance with Section 1820 of the Revised
Administrative Code, the petitioners argue that no big
These lands being neither timber nor mineral lands, the trees classified in Section 1821 of the said Code as first,
trial court should have considered them agricultural lands. second and third groups are found on the land in question.
If they are agricultural lands, then the rights of appellants Furthermore, they contend that Lot 885, even if it is a
are fully established by Act No. 926. mangrove swamp, is still subject to land registration
proceedings because the property had been in actual
The doctrine was reiterated still later in Garchitorena Vda. possession of private persons for many years, and
de Centenera v. Obias, 8 promulgated on March 4, 1933, therefore, said land was already 'private land' better
more than fifteen years after the effectivity of the adapted and more valuable for agricultural than for forest
Administrative Code of 1917. Justice Ostrand declared for purposes and not required by the public interests to be
a unanimous Court: kept under forest classification.

The opposition rests mainly upon the proposition that the The petition is without merit.
land covered by the application there are mangrove lands
as shown in his opponent's Exh. 1, but we think this A forested area classified as forest land of the public
opposition of the Director of Forestry is untenable, domain does not lose such classification simply because
inasmuch as it has been definitely decided that mangrove loggers or settlers may have stripped it of its forest cover.
lands are not forest lands in the sense in which this phrase Parcels of land classified as forest land may actually be
is used in the Act of Congress. covered with grass or planted to crops by kaingin
cultivators or other farmers. 'Forested lands' do not have
No elaboration was made on this conclusion which was to be on mountains or in out-of-the-way places. Swampy
merely based on the cases of Montano and Jocson. And in areas covered by mangrove trees, nipa palms, and other
1977, the above ruling was reaffirmed in Tongson v. trees growing in brackish or sea water may also be
Director of Forestry, 9 with Justice Fernando declaring that classified as forest land. The classification is descriptive of
the mangrove lands in litis were agricultural in nature. The its legal nature or status and does not have to be
decision even quoted with approval the statement of the descriptive of what the land actually looks like. Unless and
trial court that: until the land classsified as 'forest' is released in an official
proclamation to that effect so that it may form part of the
... Mangrove swamps where only trees of mangrove
disposable agricultural lands of the public domain, the
species grow, where the trees are small and sparse, fit
rules on confirmation of imperfect titles do not apply.'
only for firewood purposes and the trees growing are not
of commercial value as lumber do not convert the land into The view was maintained in Vallarta v. Intermediate
public land. Such lands are not forest in character. They Appellate Court, 14 where this Court agreed with the
do not form part of the public domain. Solicitor General's submission that the land in dispute,
which he described as "swamp mangrove or forestal land,"
Only last year, in Republic v. De Porkan, 10 the Court,
were not private properties and so not registerable. This
citing Krivenko v. Register of Deeds, 11 reiterated the
case was decided only twelve days after the De Porkan
ruling in the Mapa case that "all public lands that are not
case.
timber or mineral lands are necessarily agricultural public
lands, whether they are used as nipa swamps, manglares, Faced with these apparent contradictions, the Court feels
fisheries or ordinary farm lands. there is a need for a categorical pronouncement that
should resolve once and for all the question of whether
But the problem is not all that simple. As it happens, there
mangrove swamps are agricultural lands or forest lands.
is also a line of decisions holding the contrary view.
The determination of this question is a function initially
In Yngson v. Secretary of Agriculture and Natural
belonging to the legislature, which has the authority to
Resources, 12 promulgated in 1983, the Court ruled "that
implement the constitutional provision classifying the lands
the Bureau of Fisheries has no jurisdiction to dispose of
of the public domain (and is now even permitted to provide
swamp lands or mangrove lands forming part of the public
for more categories of public lands). The legislature having
domain while such lands are still classified as forest lands.
made such implementation, the executive officials may
Four months later, in Heirs of Amunategui v. Director of then, in the discharge of their own role, administer our
Forestry, 13 the Court was more positive when it held, public lands pursuant to their constitutional duty " to
again through Justice Gutierrez: ensure that the laws be faithfully executed' and in
accordance with the policy prescribed. For their part, the
The Heirs of Jose Amunategui maintain that Lot No. 885 courts will step into the picture if the rules laid down by the
cannot be classified as forest land because it is not thickly legislature are challenged or, assuming they are valid, it is
claimed that they are not being correctly observed by the
Law on NatRes (23-49) 89
executive. Thus do the three departments, coordinating proclamation declare the establishment of such reserves
with each other, pursue and achieve the objectives of the and the boundaries thereof, and thereafter such forest
Constitution in the conservation and utilization of our reserves shall not be entered, sold, or otherwise disposed
natural resources. of, but shall remain as such for forest uses, and shall be
administered in the same manner as public forest.
In C.A. No. 141, the National Assembly delegated to the
President of the Philippines the function of making periodic The President of the Philippines may in like manner by
classifications of public lands, thus: proclamation alter or modify the boundaries of any forest
reserve from time to time, or revoke any such
Sec. 6. The President, upon the recommendation of the proclamation, and upon such revocation such forest
Secretary of Agriculture and Natural Resources, shall from reserve shall be and become part of the public lands as
time to time classify the lands of the public domain into: though such proclamation had never been made.

(a) Alienable or disposable, Sec. 1827. Assignment of forest land for agricultural
purposes. - Lands in public forest, not including forest
(b) Lumber, and reserves, upon the certification of the Director of Forestry
that said lands are better adapted and more valuable for
(c) Mineral lands,
agricultural than for forest purposes and not required by
and may at any time and in a like manner transfer such the public interests to be kept under forest, shall be
lands from one class to another, for the purposes of their declared by the Department Head to be agricultural lands.
administration and disposition.
With these principles in mind, we reach the following
Sec. 7. For the purposes of the administration and conclusion:
disposition of alienable or disposable lands, the President,
Mangrove swamps or manglares should be understood as
upon recommendation by the Secretary of Agriculture and
comprised within the public forests of the Philippines as
Natural Resources, shall from time to time declare what
defined in the aforecited Section 1820 of the
lands are open to disposition or concession under this Act.
Administrative Code of 1917. The legislature having so
With particular regard to alienable public lands, Section 9 determined, we have no authority to ignore or modify its
of the same law provides: decision, and in effect veto it, in the exercise of our own
discretion. The statutory definition remains unchanged to
For the purpose of their administration and disposition, the date and, no less noteworthy, is accepted and invoked by
lands of the public domain alienable or open to disposition the executive department. More importantly, the said
shall be classified, according to the use or purposes to provision has not been challenged as arbitrary or
which such lands are destined, as follows: unrealistic or unconstitutional assuming the requisite
conditions, to justify our judicial intervention and scrutiny.
(a) Agricultural; The law is thus presumed valid and so must be respected.
We repeat our statement in the Amunategui case that the
(b) Residential, commercial, industrial, or for similar classification of mangrove swamps as forest lands is
productive purposes; descriptive of its legal nature or status and does not have
to be descriptive of what the land actually looks like. That
(c) Educational, charitable, or other similar purposes; and determination having been made and no cogent argument
having been raised to annul it, we have no duty as judges
(d) Reservations for townsites and for public and quasi-
but to apply it. And so we shall.
public uses.
Our previous description of the term in question as
The President, upon recommendation by the Secretary of
pertaining to our agricultural lands should be understood
Agriculture and Natural Resources, shall from time to time
as covering only those lands over which ownership had
make the classifications provided for in this section, and
already vested before the Administrative Code of 1917
may, at any time and in a similar manner, transfer lands
became effective. Such lands could not be retroactively
from one class to another.
legislated as forest lands because this would be violative
of a duly acquired property right protected by the due
As for timber or forest lands, the Revised Administrative
process clause. So we ruled again only two months ago in
Code states as follows:
Republic of the Philippines vs. Court of Appeals, 15 where
Sec. 1826. Regulation setting apart forest reserves- the possession of the land in dispute commenced as early
Revocation of same. - Upon there commendation of the as 1909, before it was much later classified as timberland.
Director of Forestry, with the approval of the Department
It follows from all this that the land under contention being
Head, the President of the Philippines may set apart forest
admittedly a part of the mangrove swamps of Sapian, and
reserves from the public lands and he shall by
for which a minor forest license had in fact been issued by
Law on NatRes (23-49) 90
the Bureau of Forestry from 1920 to 1950, it must be posesoria has been inscribed or registered in the registry
considered forest land. It could therefore not be the of property and that the land has been under the actual
subject of the adverse possession and consequent and adverse possession of the private respondent for
ownership claimed by the private respondent in support of twenty years as required by the Spanish Mortgage Law.
his application for registration. To be so, it had first to be 17 These matters are not presumed but must be
released as forest land and reclassified as agricultural land established with definite proof, which is lacking in this
pursuant to the certification the Director of Forestry may case.
issue under Section 1827 of the Revised Administrative
Code. Significantly, the tax declarations made by the private
respondent were practically the only basis used by the
The private respondent invokes the survey plan of the appellate court in sustaining his claim of possession over
mangrove swamps approved by the Director of Lands, 16 the land in question. Tax declarations are, of course, not
to prove that the land is registerable. It should be plain, sufficient to prove possession and much less vest
however, that the mere existence of such a plan would not ownership in favor of the declarant, as we have held in
have the effect of converting the mangrove swamps, as countless cases. 18
forest land, into agricultural land. Such approval is
ineffectual because it is clearly in officious. The Director of We hold, in sum, that the private respondent has not
Lands was not authorized to act in the premises. Under established his right to the registration of the subject land
the aforecited law, it is the Director of Forestry who has in his name. Accordingly, the petition must be granted.
the authority to determine whether forest land is more
valuable for agricultural rather than forestry uses, as a It is reiterated for emphasis that, conformably to the
basis for its declaration as agricultural land and release for legislative definition embodied in Section 1820 of the
private ownership. Revised Administrative Code of 1917, which remains
unamended up to now, mangrove swamps or manglares
Thus we held in the Yngson case: form part of the public forests of the Philippines. As such,
they are not alienable under the Constitution and may not
It is elementary in the law governing the disposition of be the subject of private ownership until and unless they
lands of the public domain that until timber or forest lands are first released as forest land and classified as alienable
are released as disposable and alienable neither the agricultural land.
Bureau of Lands nor the Bureau of Fisheries has authority
to lease, grant, sell or otherwise dispose of these lands for WHEREFORE, the decision of the Court of Appeals is
homesteads, sales patents, leases for grazing or other SET ASIDE and the application for registration of title of
purposes, fishpond leases and other modes of utilization. private respondent is DISMISSED, with cost against him.
This decision is immediately executory.SO ORDERED.
The Bureau of Fisheries has no jurisdiction to administer
and dispose of swamp lands or mangrove lands forming ATOK-BIG WEDGE MINING COMPANY, INC., petitioner,
part of the public domain while such lands are still vs. COURT OF APPEALS, and LIWAN CONSI,
classified as forest land or timber land and not released for respondents.; G.R. No. 88883 January 18, 1991;
fishery or other purposes. PARAS, J.:

The same rule was echoed in the Vallarta case, thus: This is a petition for review on certiorari which seeks to
annul and set aside; (a) the decision* of the Court of
It is elementary in the law governing natural resources that Appeals dated March 13, 1989 in CA-G.R. No. SP No.
forest land cannot be owned by private persons. It is not 13528 entitled "Liwan Consi vs. Hon. Judge Ruben C.
registerable. The adverse possession which can be the Ayson, et al." declaring that both the petitioner and private
basis of a grant of title in confirmation of imperfect title respondent hold possessory titles to the land in question,
cases cannot commence until after the forest land has and (b) the resolution denying the motion for
been declared alienable and disposable. Possession of reconsideration.
forest land, no matter bow long cannot convert it into
private property.' The facts of the case are as follows:

We find in fact that even if the land in dispute were Fredia Mineral claim of about nine (9) hectares situated in
agricultural in nature, the proof the private respondent Tuding, Itogon, Benguet, was located sometime between
offers of prescriptive possession thereof is remarkably December 25, 1930 and December 31, 1930, a period of
meager and of dubious persuasiveness. The record six (6) days, by A.I. Reynolds in accordance with the
contains no convincing evidence of the existence of the provisions of the Act of Congress of July 1, 1902, better
informacion posesoria allegedly obtained by the original known as the Philippine Bill of 1902, in a so-called
transferor of the property, let alone the fact that the Declaration of Location. The said Declaration of Location
conditions for acquiring title thereunder have been of mineral claim was duly recorded in the Office of the
satisfied. Nowhere has it been shown that the informacion Mining Recorder sometime on January 2, 1931. Fredia
Law on NatRes (23-49) 91
mineral claim, together with other mineral claims, was sold WHEREFORE, in view of all the foregoing the decision of
by A.I. Reynolds to Big Wedge Mining Company, the the Municipal Trial Court of Itogon dated January 29, 1987
earlier corporate name of Atok Big Wedge Mining appealed from is hereby reversed and set aside and a new
Company, Inc. (Atok for short; herein petitioner) in a Deed one entered in its place ordering the defendant Liwan
of Sale executed on November 2, 1931. Since then Consi and all those claiming under him to vacate the
petitioner Atok has been in continuous and exclusive premises of the Fredia Mineral claim at Tuding, Itogon,
ownership and possession of said claim up to the present Benguet immediately, and to restore possession thereof to
(Rollo, Annex "B", p. 21). the plaintiff Atok Big Wedge Mining Company.

Atok has paid the realty taxes and occupation fees for the The defendant, Liwan Consi, is further ordered to remove
Fredia mineral claim. The Fredia mineral claim together and demolish his house constructed in the premises of the
with other mineral claims owned by Atok has been land of Fredia mineral claim at Tuding, Benguet, and to
declared under Tax Declaration No. 9535 and that in view pay the costs.
of Presidential Decree No. 1214 an application for lease
was filed by Atok covering the Fredia mineral claim (Rollo, SO ORDERED. (Rollo, p. 30).
Ibid., p. 22).
From said decision, Liwan Consi filed with the Court of
On the other hand, private respondent Liwan Consi has a Appeals a petition for review (Rollo, Petition, p. 4). On
lot below the land of a certain Mr. Acay at Tuding Slide, March 13, 1989, the Court of Appeals rendered its
Itogon, Benguet. He constructed a house thereon decision, the dispositive portion of which reads:
sometime in 1964. The lot is covered by Tax Declaration
No. 9462. When he first constructed his house below the WHEREFORE, judgment is hereby rendered dismissing
lot of Mr. Acay he was told that it was not necessary for the subject forcible entry action. Costs against private
him to obtain a building permit as it was only a nipa hut. respondent.
And no one prohibited him from entering the land so he
SO ORDERED. (Rollo, Annex "C" p. 48).
was constructing a house thereon. It was only in January
1984 when private respondent Consi repaired the said The Court of Appeals further ruled in part to wit:
house that people came to take pictures and told him that
the lot belongs to Atok. Private respondent Consi has The determination of whether the subject lot is mineral
been paying taxes on said land which his father before him land or agricultural awaits the decision of the Secretary of
had occupied (Rollo, Ibid., p. 22). Natural Resources in a proceeding called for that purpose.
Thus, there is a chance that the subject property may be
On January 1984, the security guards of Atok informed classified as alienable agricultural land. At any rate, the
Feliciano Reyes, Security Officer of Atok, that a mining company may not so readily describe Liwan Consi
construction was being undertaken at the area of the as a "squatter" he also has possessory rights over the
Fredia mineral claim by private respondent Liwan Consi. property. Such rights may mature into ownership on the
Feliciano Reyes instructed the cashier to go and take basis of long-term possession under the Public Land Law,
pictures of the construction. Feliciano Reyes himself and
other security guards went to the place of the construction Thus it is Our holding, that both Consi and ATOK are of
to verify and then to the police to report the matter (Rollo, equal legal footing with regards the subject lot. Both hold
Ibid.). possessory titles to the land in question the petitioner
through his long term occupancy of the same; the
On March 1, 1984, Atok filed a complaint for forcible entry respondent mining firm by virtue of its being the claim
and detainer against Liwan Consi (Rollo, Annex "C", p. locator and applicant for a lease on the mineral claim
32). within which the subject lot is found. But it was established
that the petitioner has been in actual and beneficial
On January 29, 1987, after due hearing, the Municipal
possession of the subject lot since before the Second
Trial Court of Itogon, presided over by Judge Irving
World War in the concept of owner and in good faith.
rendered a decision, the dispositive portion of which reads:
(Rollo, Annex "C", pp. 47-48).
WHEREFORE, this case against Liwan Consi is hereby
On June 16, 1989, the Court of Appeals denied the motion
ordered dismissed. (Rollo, Annex "A", p. 20).
for reconsideration filed by petitioner ATOK (Rollo, Annex
Petitioner ATOK appealed the decision to the Regional "D", p. 50).
Trial Court (RTC) of Baguio and Benguet, Branch VI,
Hence, the petition.
presided over by Judge Ruben Ayson (Rollo, Petition, p.
3). On December 5, 1987, the RTC rendered its decision, The main issue in this case is whether or not an
the dispositive portion of which reads: individual's long term occupation of land of the public
domain vests him with such rights over the same as to
defeat the rights of the owner of that claim.
Law on NatRes (23-49) 92
The petition is impressed with merit. them and the mining companies for agricultural and
mineral purposes (Ibid).
It is of no importance whether Benguet and Atok had
secured a patent for as held in the Gold Creek Mining On the matter of possession, private respondent contends
Corporation case, for all physical purposes of ownership, that his predecessor-in-interest has been in possession of
the owner is not required to secure a patent as long as he said lot even before the war and has in fact cultivated the
complies with the provisions of the mining laws; his same.
possessory right, for all practical purposes of ownership, is
as good as though secured by patent (Republic v. Court of In the case of Republic v. Court of Appeals, 160 SCRA
Appeals, 160 SCRA 228 [1988]). 288 1988, this Court held:

In the case at bar, the evidence on record pointed that the . . . even if it be assumed that the predecessor-in-interest
petitioner Atok has faithfully complied with all the of the de la Rosas had already been in possession of the
requirements of the law regarding the maintenance of the subject property, their possession was not in the concept
said Fredia Mineral Claim. of owner of the mining claim but of the property as
agricultural land, which it was not. The property was
The perfection of the mining claim converted the property mineral land, and they are claiming it as agricultural land.
to mineral land and under the laws then in force removed it They were not disputing the rights of the mining locators
from the public domain. By such act, the locators acquired nor where they seeking to oust them as such and to
exclusive rights over the land, against even the replace them in the mining of the land. . . .
government, without need of any further act such as the
purchase of the land or the obtention of a patent over it. As Since the subject lot is mineral land, private respondent's
the land had become the private property of the locators, possession of the subject lot no matter how long did not
they had the right to transfer the same, as they did, to confer upon him possessory rights over the same.
Benguet and Atok (Ibid.).
Furthermore, Article 538 of the New Civil Code provides:
As in the instant petition, the record shows that the lot in
question was acquired through a Deed of Sale executed Art. 538. Possession as a fact cannot be recognized at the
between Atok and Fredia Mineral Claim. same time in two different personalities except in the
cases of co-possession. Should a question arise regarding
The legal effect of a valid location of a mining claim is not the fact of possession, the present possessor shall be
only to segregate the area from the public domain, but to preferred; if there are two possessors, the one longer in
grant to the locator the beneficial ownership of the claim possession; if the dates of the possession are the same,
and the right to a patent therefor upon compliance with the the one who presents a title; and if all these conditions are
terms and conditions prescribed by law. Where there is a equal, the thing shall be placed in judicial deposit pending
valid location of mining claim, the area becomes determination of its possession or ownership through
segregated from the public and the property of the locator. proper proceedings.
When a location of a mining claim is perfected it has the
effect of a grant by the United States of the right of present Since 1931 up to the present, petitioner ATOK has been in
and exclusive possession, with the right to the exclusive continuous and exclusive possession of the Fredia mineral
enjoyment of all the surface ground as well as of all the claim while private respondent's possession started only
minerals within the lines of the claim, except as limited by sometime in 1964 when he constructed a house thereon.
the extralateral right of adjoining locators; and this is the Clearly, ATOK has superior possessory rights than private
locator's right before as well as after the issuance of the respondent, Liwan Consi, the former being "the one longer
patent. While a lode locator acquires a vested right by in possession."
virtue of his location made in compliance with the mining
It is therefore clear that from the legal viewpoint it was
laws, the fee remains in the government until patent
really petitioner who was in actual physical possession of
issues. (St. Louis Mining & Mineral Co. v. Montana Mining
the property. Having been deprived of this possession by
Co., 171 U.S. 605, 655; 43 Law ed., 320, 322)
the private respondent, petitioner has every right to sue for
It is, therefore, evident that Benguet and Atok have ejFectment.
exclusive rights to the property in question by virtue of
With this ruling enunciated by the Court, it can further be
their respective mining claims which they validly acquired
declared and held that petitioner Atok has the exclusive
before the Constitution of 1935 prohibited the alienation of
right to the property in question.
all lands of the public domain except agricultural lands,
subject to vested rights existing at the time of its adoption. PREMISES CONSIDERED, the petition is GRANTED and
The land was not and could not have been transferred to the questioned decision of the Court of Appeals dated
the private respondents by virtue of acquisitive March 13, 1989 is REVERSED and SET ASIDE and the
prescription, nor could its use be shared simultaneously by decision of the Regional Trial Court of Baguio and
Law on NatRes (23-49) 93
Benguet dated June 16, 1989 is REINSTATED. SO the main petition even before the issue of ownership
ORDERED. thereof is finally resolved by the Court.

REPUBLIC OF THE PHILIPPINES, petitioner, After the private respondent SHAI had filed its Comment2
vs. SOUTHSIDE HOMEOWNERS ASSOCIATION, INC. to the petition in G.R. No. 156951, the Bases Conversion
and the REGISTER OF DEEDS OF PASIG, RIZAL, Development Authority (BCDA), followed by the
respondents.; G.R. No. 156951 September 22, Department of National Defense (DND) and the AFP,
2006 joined causes with the petitioner Republic and thus sought
leave to intervene. The Court, per its Resolutions dated
x-------------------------------------------x September 3, 2003,3 and September 29, 2003,4
respectively, allowed the intervention and admitted the
BASES CONVERSION DEVELOPMENT AUTHORITY, corresponding petitions-for-intervention.
intervenor
Per Resolution of the Court dated August 09, 2006, both
x-------------------------------------------x petitions were ordered consolidated.

DEPARTMENT OF NATIONAL DEFENSE, represented The Republics recourse in G.R. No. 156951 is cast
by HON. SECRETARY ANGELO T. REYES, and the against the following backdrop:
ARMED FORCES OF THE PHILIPPINES, represented
by CHIEF OF STAFF, AFP, GENERAL NARCISO L. On July 12, 1957, then President Carlos P. Garcia issued
ABAYA, intervenors Proclamation No. 4235 establishing a military reservation
known as Fort William McKinley later renamed Fort
x-------------------------------------------x Andres Bonifacio Military Reservation (FBMR). The
proclamation "withdr[ew] from sale or settlement and
G.R. No. 173408 September 22, 2006 reserve[d] for military purposes, under the administration
of the Chief of Staff of the [AFP] the [certain] parcels of
RENE A.V. SAGUISAG, MGEN. MARCIANO ILAGAN the public domain [indicated in plan Psu-2031]" situated in
(Ret.), MGEN. PONCIANO MILLENA (Ret.), BGEN.
the several towns and a city of what was once the
JUANITO MALTO (Ret.), BGEN. RAYMUNDO JARQUE
Province of Rizal. On its face, the proclamation covers
(Ret.) and COL. DOMINADOR P. AMADOR (Ret.),
three (3) large parcels of land, to wit: Parcel No. 2
petitioners, vs. L/T. GEN. HERMOGENES C. ESPERON,
(portion), Parcel No. 3 (or 3-A) and Parcel No. 4 (or 4-A).
JR., respondent.
Parcel No. 3 with an area of 15,912,684 square meters
and Parcel No. 4 with an area of 7,660,128 square meters
DECISION
are described in the proclamation as situated inside Fort
GARCIA, J.: McKinley, Rizal. Specifically mentioned as excluded from
Parcel No. 4 albeit within its boundaries are the American
Before the Court are these two petitions having, as Battle Monument Cemetery (606,985 sq. m.), the Traffic
common denominator, the issue of ownership of a large Circle (7,093 sq. m.) and the Diplomatic and Consular
tract of land. area (100,000 sq.m.).

In the first, a petition for review under Rule 45 of the Rules Several presidential proclamations would later issue
of Court and docketed as G.R. No. 156951, the petitioner excluding certain defined areas from the operation of
Republic of the Philippines seeks to nullify and set aside Proclamation No. 423 and declaring them open for
the Decision1 dated January 28, 2003 of the Court of disposition. These are Proclamation No. 4616 and
Appeals (CA) in CA-G.R. CV No. 59454, affirming the Proclamation No. 462,7 both series of 1965, excluding
dismissal by the Regional Trial Court (RTC) of Pasig City, portions of the reservation and declaring them the AFP
Branch 71, of the Republics complaint for declaration of Officers Village and the AFP EMs Village, respectively, to
nullity and cancellation of a land title against the herein be disposed of under Republic Act (R.A.) 274 8 and R.A.
private respondent, the Southside Homeowners 7309 in relation to the Public Land Act (C.A. 141, as
Association, Inc. (SHAI). amended). Excluded, too, under Proclamation No. 172
dated October 16, 1987 and to be disposed pursuant to
In the second, docketed as G.R. No. 173408, petitioners the same laws aforementioned, save those used or
Rene Saguisag and five (5) retired military officers pray earmarked for public/quasi-public purposes, are portions
that Lt. Gen. Hermogenes C. Esperon, Jr., the present of the reservation known as Lower and Upper Bicutan,
Chief of Staff of the Armed Forces of the Philippines Western Bicutan and the Signal Village, all in Taguig,
(AFP), be asked to show cause why he should not be cited Metro Manila.
for contempt for having announced time and again that the
military officers and their families in the contempt action In 1992, Congress enacted the Bases Conversion and
would be ousted and evicted from the property subject of Development Act (R.A. 7227, as amended), investing the
BCDA the power to own, hold and administer portions of
Law on NatRes (23-49) 94
Metro Manila military camps that may be transferred to it 2. Original copy of the Deed of Sale issued in favor of
by the President10 and to dispose, after the lapse of a SHAI containing the signature of "ABELARDO G.
number of months, portions of Fort Bonifacio.11 PALAD, JR." ... designated as "Q-962.

At the core of the instant proceedings for declaration of xxx xxx xxx
nullity of title are parcels of land with a total area of 39.99
hectares, more or less, known as or are situated in what PURPOSE OF EXAMINATION:
is referred to as the JUSMAG housing area in Fort
Bonifacio. As may be gathered from the pleadings, military To determine whether or not the questioned and
officers, both in the active and retired services, and their sample/specimen signatures "ABELARDO G. PALAD,
respective families, have been occupying housing units JR." were written by one and the same person.
and facilities originally constructed by the AFP on the
FINDINGS:
JUSMAG area.
Scientific comparative examination and analysis of the
Private respondent SHAI is a non-stock corporation
specimens, submitted, under stereoscopic microscope and
organized mostly by wives of AFP military officers.
magnifying lens, with the aid of photographic enlargement
Records show that SHAI was able to secure from the
reveals that there exist fundamental, significant
Registry of Deeds of the Province of Rizal a title Transfer
differences in writing characteristics between the
Certificate of Title (TCT) No. 1508412 - in its name to the
questioned and the standard/sample signatures
bulk of, if not the entire, JUSMAG area. TCT No. 15084
"ABELARDO G. PALAD, JR." such as in:
particularly describes the property covered thereby as
follows: - The questioned signatures show slow, drawn,
painstaking laborious manner in execution of strokes; that
A parcel of land (Lot 3-Y-1, Psd-76057, being a portion of
of the standard/sample signatures show free, rapid
Parcel 3 of plan Psu-2031) situated in Jusmang (sic) Area,
coordinated and spontaneous strokes in the manner of
Fort Bonifacio, Province of Rizal. containing an area of
execution of letters/elements.
(398,602) SQUARE METERS. xxx.
xxx xxx xxx
A parcel of land (Lot 3-Y-2, Psd-76057 as shown on
subdivision Plan Psd 76057, being a portion of parcel 3 of Furthermore, the questioned signature "ABELARDO G.
plan Psu-2031, LRC Rec. No.) situated in Jusmang (sic) PALAD, JR." marked "Q-961" is a product of TRACING
Area, Fort Bonifacio, Province of Rizal. containing an PROCESS by CARBON-OUTLINE METHOD.
area of (1,320) SQUARE METERS xxx.. (Underscoring
added.) CONCLUSION:

The Rizal Registry issued TCT No. 15084 on October 30, Based on the above FINDINGS, the questioned and the
1991 on the basis of a notarized Deed of Sale13 standard/sample signatures "ABELARDO G. PALAD, JR."
purportedly executed on the same date by then Director were not written by one and the same person.
Abelardo G. Palad, Jr. (Palad, for brevity) of the Lands
Management Bureau (LMB) in favor of SHAI. The total The questioned signature "ABELARDO G. PALAD, JR."
purchase price as written in the conveying deed was marked "Q-961" is a TRACED FORGERY by carbon
P11,997,660.00 or P30.00 per square meter. process.

It appears that in the process of the investigation REMARKS:


conducted by the Department of Justice on reported land
scams at the FBMR, a copy of the aforesaid October 30, The other questioned Deeds of Sale containing the
1991 deed of sale surfaced and eventually referred to the signatures of "ABELARDO G. PALAD, JR." are still in the
National Bureau of Investigation (NBI) for examination. process of examination.15
The results of the examination undertaken by NBI
Document Examiner Eliodoro Constantino are embodied On October 16, 1993, then President Fidel V. Ramos
in his Questioned Documents Report (QDR) No. 815- issued Memorandum Order No. 17316 directing the Office
1093.14 Its highlights: of the Solicitor General (OSG) to institute action towards
the cancellation of TCT No. 15084 and the title acquired
QUESTIONED SPECIMENS: by the Navy Officers Village Association (NOVA) over a
bigger parcel within the reservation. A month later, the
1. Original copy of the Deed of Sale issued in favor of OSG, in behalf of the petitioner Republic, filed with the
the Navy Officers Village Association (NOVA) RTC of Pasig City the corresponding nullification and
containing the signature of "ABELARDO G. PALAD, cancellation of title suit against the private respondent
JR." designated as "Q-961" . SHAI. In its complaint, docketed as Civil Case No. 63883
and eventually raffled to Branch 71 of the court, the
Law on NatRes (23-49) 95
Republic alleged that fraud attended SHAIs procurement Garcia, is to the effect that TCT No. 15084 must be
of TCT No. 15084. In paragraph No. 5 of the complaint, genuine as it emanated from the Registrys office on the
the Republic alleged that TCT No. 15084 is void owing, basis of the October 30, 1991 Deed of Sale.22
inter alia, to the following circumstances: a) the conveying
deed is spurious as the purported signature thereon of On rebuttal, Palad would deny authorship of Exhibit "10"
Palad is a forgery; b) there are no records with the LMB of and an LMB official would disclaim transmitting the same
(i) the application to purchase and (ii) the alleged payment to Atty. Garcia.
of the purchase price; and c) the property in question is
inalienable, being part of a military reservation established Eventually, in a decision23 dated October 7, 1997, the trial
under Proclamation No. 423.17 court rendered judgment dismissing the Republics
complaint, to wit:
In its ANSWER with counterclaim, respondent SHAI
denied the material allegations of the complaint and WHEREFORE, in view of the foregoing, the Complaint
countered that the impugned title as well as the October dated November 15, 1991 is hereby DISMISSED without
30, 1991 Deed of Sale are valid documents which the pronouncement as to costs.
Republic is estopped to deny.18 SHAI also alleged paying
The counterclaims are also DISMISSED.
in full the purchase price indicated in the deed as
evidenced by Official Receipt No. 6030203-C dated SO ORDERED.
October 29, 1991.
In not so many words, the trial court considered the
On October 19, 1994, the case was heard on pre-trial in parcels covered by the deed in question as no longer part
the course of which the Republic, as plaintiff therein, of the FBMR.
marked (and later offered in evidence) the Deed of Sale
dated October 30, 1991 as its Exhibit "A," and TCT No. Therefrom, the Republic went on appeal to the CA
15084 as Exhibit "B." Respondent, then defendant SHAI whereat its appellate recourse was docketed as CA-G.R.
adopted Exhibits "A" and "B" as its Exhibits "1" and "2," CV No. 59454.
respectively. As the pre-trial order was written, it would
appear that the parties agreed to limit the issue to the due In the herein assailed Decision24 dated January 28, 2003,
execution and genuineness of Exhs. "A" and "B."19 the appellate court affirmed in toto that of the trial court.

During the trial, the Republic presented as expert witness Hence, this petition of the Republic on the threshold
NBI Document Examiner Eliodoro Constantino who abstract submission that the CA "completely ignored,
testified on NBI QDR No. 815-1093 and asserted that the overlooked and/or grossly misappreciated facts of
signature of Palad in Exhibit "A" is a forgery. For his part, substance which, if duly considered, will materially affect
Palad dismissed as forged his signature appearing in the the outcome of this case."
same document and denied ever signing the same, let
alone in front of a notary public holding office outside of In its COMMENT To Petition,private respondent SHAI
the LMB premises. Pressing the point, Palad stated that parlays the "what-can-be-raised" line. It urges the
he could not have had signed the conveying deed dismissal of the petition on the ground that the issues
involving as it did a reservation area which, apart from its raised therein, particularly those bearing on the
being outside of the LMBs jurisdiction, is inalienable in the authenticity of Exhibit "A"/"1," are mainly questions of
first place. The testimony of other witnesses revolved fact, adding that the matter of the inalienability of the area
around the absence of bureau records respecting SHAIs purportedly sold is outside the issue agreed upon during
application to acquire, payment of the purchase price and the pre-trial stage.
Psd-76057, the plan described in TCT No. 15084. 20
The desired dismissal cannot be granted on the bases of
For its part, then defendant SHAI presented an opposing the reasons proffered above.
expert witness in the person of Police Inspector Redencion
Caimbon who brought with him PNP QDR No. 001-96 and While the Court, in a petition for review of CA decisions
testified that Palads signature in Exhibit "A" (same as under Rule 45 of the Rules of Court, usually limits its
Exh. "1") is genuine. Mrs. Virginia Santos, then SHAI inquiry only to questions of law, this rule is far from
president, likewise testified, saying that applications to absolute. Reyes v. Court of Appeals,25 citing Floro v.
purchase were signed and then filed with the LMB by one Llenado,26 for one, suggests as much. In Floro, we wrote:
Engr. Eugenia Balis,21 followed by the payment in full of
xxx There are, however, exceptional circumstances that
the contract price. Atty. Vicente Garcia, the then Register
would compel the Court to review the finding of facts of the
of Deeds of Rizal, also testified about his having endorsed
[CA], summarized in and subsequent cases as follows:
to Palad a letter-inquiry he received from SHAI respecting
1) when the inference made is manifestly mistaken,
the authenticity of TCT No. 15084. Palads response-letter
absurd or impossible; 2) when there is grave abuse of
dated January 23, 1992 (Exh. "10"), according to Atty.
discretion; 3) when the finding is grounded entirely on
Law on NatRes (23-49) 96
speculations, surmises or conjectures; 4) when the At any rate, Palads testimony drew nary an objection from
judgment of the [CA] are based on misapprehension of private respondent SHAI. It even cross-examined said
facts; 5) when the findings of facts are conflicting; 6) ; 7) witness.33 The rule obtains that the introduction of
; 8) ; 9) when the [CA] manifestly overlooked certain evidence bearing on an issue not otherwise included in the
relevant facts not disputed by the parties and which if pre-trial order amounts to implied consent conferring
properly considered would justify a different conclusion; jurisdiction on the court to try such issue.34
and 10) when the findings of facts are premised on the
absence of evidence and are contradicted by the evidence Digressing from the procedural aspects of this case, we
on record. (Words in bracket, added.) now consider the clashing assertions regarding the
JUSMAG area. Was it, during the period material,
To the mind of the Court, the instant case is within the alienable or inalienable, as the case may be, and,
purview of at least three of the exceptions listed above, therefore, can or cannot be subject of a lawful private
foremost of which is item #9. conveyance?

Private respondent SHAIs stance about the petitioner Petitioner Republic, as do the intervenors, asserts the
Republic being barred from raising the issue of inalienable character of the JUSMAG area, the same
inalienability since it failed to plead or assert the same at having not effectively been separated from the military
the pre-trial proceedings is, to a degree, correct. For the reservation and declared as alienable and disposable.
general rule, as articulated in Permanent Concrete
Products, Inc. v. Teodoro,27 is that the determination of The Republics and the intervenors parallel assertions are
issues at a pre-trial conference bars the consideration of correct.
others on appeal. It should be pointed out, however, that
the rationale for such preliminary, albeit mandatory, The President, upon the recommendation of the Secretary
conference is to isolate as far as possible the trial out of of Environment and Natural Resources, may designate by
the realm of surprises and back-handed maneuverings. proclamation any tract or tracts of land of the public
And lest it be overlooked, the adverted rule on the domain as reservations for the use of the Republic or any
procedure to be observed in pre-trials is, as Bergano v. of its branches, or for quasi-public uses or purposes.35
Court of Appeals28 teaches, citing Gicano v. Gegato,29 Such tract or tracts of land thus reserved shall be non-
subject to exceptions. And without meaning to diminish the alienable and shall not be subject to sale or other
importance of the same rule, the Court is possessed with disposition until again declared alienable.36 Consistent with
inherent power to suspend its own rules or to except a the foregoing postulates, jurisprudence teaches that a
particular case from its operations whenever the demands military reservation, like the FBMR, or a part thereof is not
of justice so require.30 open to private appropriation or disposition and, therefore,
not registrable,37 unless it is in the meantime reclassified
Given the foregoing considerations, the rule to be and declared as disposable and alienable public land.38
generally observed in pre-trial conferences hardly poses And until a given parcel of land is released from its
an insurmountable obstacle to tackling the question of classification as part of the military reservation zone and
inalienability which, under the premises, is an issue more reclassified by law or by presidential proclamation as
legal than factual. As it were, the element of surprise is not disposable and alienable, its status as part of a military
really present here. For the issue of inalienability, which is reservation remains,39 even if incidentally it is devoted for
central to the Republics cause of action, was raised in its a purpose other than as a military camp or for defense. So
basic complaint, passed upon by the CA and, before it, by it must be here.
the trial court31 and of which at least one witness (Palad)
was examined as follows: There can be no quibbling that the JUSMAG area subject
of the questioned October 30, 1991 sale formed part of the
Q: Mr. Witness you stated that the parcel of land in FBMR as originally established under Proclamation No.
question at the time of the land alleged sale was part of 423. And while private respondent SHAI would
the [FBMR]. Now as part of the [FBRM] do you know categorically say that the petitioner Republic had not
whether the said parcel of land can be the subject of presented evidence that "subject land is within military
disposition? reservation,"40 and even dared to state that the JUSMAG
area is the private property of the government and
A: If it is part of the reservation it cannot be sold and it is therefore removed from the concept of public domain per
already part of those government lands that has been se,41 its own evidence themselves belie its posture. We
assigned to other government agencies that is no longer start with its Exhibit "2" (petitioners Exh. "B"), a copy of
within my jurisdiction. Meaning to say I have no more say TCT No. 15084, which described the area covered thereby
on that because the proclamation to the effect was measuring 399,922 square meters as a "portion of Parcel
reserving this for particular purpose under the DND . 32 3 of plan Psu-2031 situated in Jusmang (sic) area Fort
(Words in bracket added.) Bonifacio." Complementing its Exhibit "2" is its Exhibit "1" -
the deed of sale - which technically described the property
Law on NatRes (23-49) 97
purportedly being conveyed to private respondent SHAI as 2. It specifically denies the allegations in paragraphs
follows: 5 of the complaint, the truth of the matter being that
in the Deed of Sale , the Director of Lands Certificate
A PARCEL OF LAND (Lot 3-Y-1, Psd-76067, being a (sic) that he is "authorized under the law to sell" the
portion of Parcel 3 of plan Psu-2031) situated in Jusmag subject property and that the "lots were duly awarded by
(sic) area, Fort Bonifacio, Province of Rizal. Xxx the [LBM] to the vendee.44 ( Emphasis and word in bracket
(Emphasis added) added.)

As the Court distinctly notes, the disputed property, as In net effect, private respondent SHAI admitted what the
described in private respondents Exhibits "1" and "2," petitioner Republic alleged in par. 5 (e) and (g) of the
formed part of that wide expanse under Proclamation No. complaint, the formers denial to such allegations on the
423 which lists, as earlier stated, three (3) parcels of land inalienable nature of the property covered by TCT No.
of the public domain as falling within its coverage. These 15084 being in the nature of a general denial. Under the
include, inter alia, the entire 15,912,684-square meter area rules on pleadings, a specific, not a general, denial is
constituting Parcel No. 3 of Plan Psu 2031 located inside required; a denial is not specific because it is so qualified
the now renamed Fort Mckinley which, to a redundant or termed "specific" by the pleader.45 The defendant must
point, was declared a military reservation. specify each material factual allegation the truth of which
he absolutely denies and, whenever practicable, shall set
The Court has, on the issue of inalienability, taken stock of forth the substance of the matters upon which he will rely
the Compilation Map of Approved Surveys Plan inside to support his denial.46 Else, the denial will be regarded as
Parcels 1, 2, 3 and 4, of plan Psu 2031 42 prepared in general and will, therefore, be regarded as an admission
September 1995 and certified by the Department of of a given material fact/s stated in the complaint.
Environment and Natural Resources (DENR). It indicates
in colored ink the outlines of Parcels 2, 3 and 4 covered by What private respondent SHAI did under the premises was
Proclamation No. 423. As there also shown, the 399,992- to enter what, under the Rules, is tantamount to a general
square meter area embraced by SHAIs TCT No. 15084, denial of the Republics averments that what SHAIs TCT
defined in the legend by red-colored stripes, is within the No. 15084 covers is part of the military reservation. In the
violet-colored borders of Parcel No. 3 and Parcel No. 4 of process, private respondent SHAI is deemed to admit the
Proclamation No. 423. reality of such averment.

Indubitably, the area covered by SHAIs TCT No. 15084 To be sure, the petitioner Republic, as plaintiff below, had
was and is still part of the FBMR, more particularly within more than sufficiently established its claim on the
the 15,912,684- square meter Parcel No. 3 of the inalienability of the parcels of land covered by TCT No.
reservation. The petitioner Republic, joined by the 15084. In fine, it had discharged the burden of proof on the
intervenors BCDA, DND and AFP in this appellate issue of inalienability. Be that as it may, the burden of
proceedings, has maintained all along this thesis. Towards evidence to disprove inalienability or, to be precise, that
discharging its burden of proving that the disputed said parcels of land had, for settlement purposes,
property is part of the reservation, the petitioner Republic effectively been withdrawn from the reservation or
need only to demonstrate that all of the 15,912,684 square excluded from the coverage of Proclamation No. 423,
meters of Parcel No. 3 of Plan Psu 2031 have been devolves upon the private respondent. This is as it should
reserved for military purposes. The evidence, however, of be for the cogency of SHAIs claim respecting the validity
the fact of reservation is the law or, to be more precise, of both the underlying deed of sale (Exh. "A"/"1") and its
Proclamation No. 423 itself, the contents and issuance of TCT No. 15084 (Exh. "B"/"2") rests on the postulate that
which courts can and should take judicial notice of under what it purportedly bought from the LMB had ceased to be
Section 1, Rule 129 of the Rules of Court.43 part of the reserved lands of the public domain. Elsewise
put, SHAI must prove that the JUSMAG area had been
The Republic has, since the filing of its underlying withdrawn from the reservation and declared open for
complaint, invoked Proclamation No. 423. In the process, disposition, failing which it has no enforceable right over
it has invariably invited attention to the proclamations the area as against the State.
specific area coverage to prove the nullity of TCT No.
15084, inasmuch as the title embraced a reserved area Private respondent SHAI has definitely not met its burden
considered inalienable, and hence, beyond the commerce by reason of lack of evidence. To be sure, it has not,
of man. In this regard, the appellate court seemed to have because it cannot even if it wanted to, pointed to any
glossed over, if not entirely turned a blind eye on, certain presidential act specifically withdrawing the disputed
admissions made by the private respondent, the most parcels from the coverage of Proclamation No. 423. Worse
basic being those made in its answer to the Republics still, its own Exhibit "5,"47 a letter dated March 19, 1991 of
allegations in paragraph 5 (e) and (g) of its complaint. To then PA Commanding General, M/Gen Lisandro Abadia,
the Republics allegations that the property covered by to one Mrs. Gabon, then President of the SHAI, cannot but
TCT No. 15084 was and remains part the FBMR, SHAIs be viewed as a partys judicial admission that the disputed
answer thereto reads:
Law on NatRes (23-49) 98
land has yet to be excluded from the military reservation. roughly a total area of 25,875,000 square meters, not
The Abadia letter, with its feature dis-serving to private 7,053,143. On the other hand, Proclamation Nos. 461 and
respondent SHAI, reads in part as follows: 172 excluded a combined area of 6,892,338 square
meters. Now then, the jump from an acknowledgment of
Dear Mrs. Gabon: the disputed parcels of land having been reserved for
military purposes to a rationalization that they must have
This is in connection with your move to make a petition to been excluded from the reservation because 6,892,338
President Aquino regarding the possible exclusion of square meters had already been withdrawn from
Southside Housing Area from the military reservation and Proclamation 423 is simply speculative. Needless to
for its eventual allotment to the military officers stress, factual speculations do not make for proof.
presently residing thereat. Allow me to state that I
interpose no objection . I find it helpful to our officers Corollary to the first reason is the fact that private
to be provided a portion of the Fort Bonifacio military respondent SHAI - and quite understandably, the appellate
reservation . (Underscoring added.) court - had not pointed to any proclamation, or legislative
act for that matter, segregating the property covered by
Owing to the foregoing considerations, the Court is hard TCT No. 15084 from the reservation and classifying the
put to understand how the CA could still have found for same as alienable and disposable lands of the public
SHAI.. The appellate court, apparently swayed by what domain. To reiterate what we earlier said, lands of the
SHAI said in its Brief for the Appellees48 that: public domain classified as a military reservation remains
as such until, by presidential fiat or congressional act, the
Appellant [petitioner Republic] is probably unaware that ,
same is released from such classification and declared
then President Diosdado Macapagal issued
open to disposition.50 The October 30, 1991 Deed of Sale
Proclamation 461 when he excluded from the operation of
purportedly executed by Palad, assuming for the nonce its
Proclamation No. 423 an area of 2,455,810 square
authenticity, could not plausibly be the requisite classifying
meters more or less. Likewise on October 16, 1987,
medium converting the JUSMAG area into a disposable
then President Corazon Aquino issued Proclamation No.
parcel. And private respondent SHAIs unyielding stance
172 excluding five (5) parcels of land from the operation of
that would have the Republic in estoppel to question the
Proclamation No. 423 also located at Fort Bonifacio
transfer to it by the LMB Director of the JUSMAG area is
containing an area of 4,436, 478 . So if we deduct the
unavailing. It should have realized that the Republic is not
6,892,288 [2,455,810 + 4,436,478 = 6,892,288] square
usually estopped by the mistake or error on the part of its
meters covered by Proclamation Nos. 461 and 172 of the
officials or agents.51
areas reserved for military purposes of 7,053,143 square
meters, what is only left is 160,857 square meters or more Since the parcels of land in question allegedly sold to the
or less 16 hectares .49 private respondent are, or at least at the time of the
supposed transaction were, still part of the FBMR, the
justified its holding on the alienability of the disputed land
purported sale is necessarily void ab initio.
with the following disquisition:
The Court can hypothetically concede, as a matter of fact,
The foregoing admission aside, appellants [now
the withdrawal of the JUSMAG area from the ambit of
petitioners] reliance on Proclamation No. 493 [should be
Proclamation No. 423 and its reclassification as alienable
423] in insisting that the land in litigation is inalienable
and disposable lands of the public domain. Still, such
because it is part of the [FBMR] is too general to merit
hypothesis would not carry the day for private respondent
serous consideration. While it is true that, under the said
SHAI. The reason therefor is basic: Article XII, Section 352
July 12, 1957 Proclamation, then President Carlos P.
of the 1987 Constitution forbids private corporations from
Garcia reserved the area now known as Fort Bonifacio for
acquiring any kind of alienable land of the public domain,
military purposes, appellee [now respondent] correctly
except through lease for a limited period. While Fr. Bernas
calls our attention to the fact, among other matters, that
had stated the observation that the reason for the ban is
numerous exceptions thereto had already been declared
not very clear under existing jurisprudence,53 the fact
through the years. The excluded areas under
remains that private corporations, like SHAI, are prohibited
Proclamation No. 461, dated September 29, 1965 and
from purchasing or otherwise acquiring alienable public
Proclamation No. 172, dated October 16, 1987 alone
lands.
already total 6,892,338 square meters. (Figures in bracket
added.) Even if on the foregoing score alone, the Court could write
finis to this disposition. An appropriate closure to this case
The CAs justifying line does not commend itself for
could not be had, however, without delving to an extent on
concurrence.
the issue of the validity of the October 30, 1991 Deed of
For one, it utilizes SHAIs misleading assertion as a Sale which necessarily involves the question of the
springboard to justify speculative inferences. Per our authenticity of what appears to be Palads signature
count, Proclamation 423 reserved for military purposes thereon.
Law on NatRes (23-49) 99
With the view we take of the case, the interplay of Office."60A copy of the OR receipt is not appended to any
compelling circumstances and inferences deducible of the pleadings filed before the Court. We can thus validly
therefrom, would, as a package, cast doubt on the presume that no such OR exists or, if it does, that its
authenticity of such deed, if not support a conclusion that presentation would be adverse to SHAI.
the deed is spurious. Consider:
A contract of sale is void where the price, which appears in
1. Palad categorically declared that his said signature on the document as paid has, in fact, never been paid.61
the deed is a forgery. The Court perceives no reason why
he should lie, albeit respondent states, without elaboration, 5. The purchase price was, according to the witnesses for
that Palads declaration is aimed at avoiding "criminal SHAI, paid in full in cash to the cashier of the LMB the
prosecution".54 The NBI signature expert corroborated corresponding amount apparently coming in a mix of P500
Palads allegation on forgery.55 Respondent SHAIs expert and P100 denominations. Albeit plausible, SHAIs
witness from the PNP, however, disputes the NBIs witnesses account taxes credulity to the limit.
findings. In net effect, both experts from the NBI and the
PNP cancel each other out. A final consideration in G.R. No. 156951. This case could
not have come to pass without the participation of a cabal
2. Palad signed the supposed deed of sale in Manila, of cheats out to make a dishonest buck at the expense of
possibly at the LMB office at Plaza Cervantes, Binondo. the government and most likely the members of SHAI. No
Even if he acted in an official capacity, Palad nonetheless less than its former president (Ms. Virginia Santos)
proceeded on the same day to Pasig City to appear before testified that a "facilitator" did, for a fee, the necessary
the notarizing officer. The deed was then brought to the paper and leg work before the LMB and the Registry of
Rizal Registry and there stamped "Received" by the entry Deeds that led to the execution of the Deed of Sale and
clerk. That same afternoon, or at 3:14 p.m. of October 30, issuance of the certificate of title in question. 62 Ms. Santos
1991 to be precise, TCT No. 15084 was issued. In other identified Eugenia Balis, a geodetic engineer, as the
words, the whole conveyance and registration process "facilitator"63 who "facilitated all these presentation" of
was done in less than a day. The very unusual dispatch is documents,64 and most of the time, "directly transacted"
quite surprising. Stranger still is why a bureau head, while with the LMB and the Register of Deeds leading to
in the exercise of his functions as the bureaus authorized acquisition of title.65 Engr. Balis was, in the course of Ms.
contracting officer, has to repair to another city just to have Santos testimony, directly mentioned by name for at least
a deed notarized. fifteen (15) times. Not surprisingly, Engr. Balis did not
appear in court, despite SHAIs stated intention to present
3. There is absolutely no record of the requisite public land her as witness.66
application to purchase required under Section 89 of the
Public Land Act.56 There is also no record of the deed of The extent of the misappropriation of the Fort Bonifacio
sale and of documents usually accompanying an land involved in this and the NOVA area litigations is, as
application to purchase, inclusive of the investigation described in the Report of the FactFinding Commission,67
report and the property valuation. The Certification under "so epic in scale as to make the overpricing of land
the seal of the LMB bearing date November 24, 1994 and complained of in the two hundred AFP [Retirement and
issued/signed by Alberto Recalde, OIC, Records Separation Benefits System] RSBS cases (P703 million)
Management Division of the LMB pursuant to a subpoena seem like petty shoplifting in comparison."68 The members
issued by the trial court57 attest to this fact of absence of of private respondent SHAI may very well have paid for
records. Atty. Alice B. Dayrit, then Chief, Land Utilization what they might have been led to believe as the purchase
and Disposition Division, LMB, testified having personally price of the JUSMAG housing area. The sad reality,
looked at the bureau record book, but found no entry however, is that the over P11 Million they paid, if that be
pertaining to SHAI.58 the case, for a piece of real estate contextually outside the
commerce of man apparently fell into the wrong hands and
4. In its Answer as defendant a quo, respondent SHAI did not enter the government coffers. Else, there must be
states that the "deed of sale specifically meritorious Official some memorials of such payment.
Receipt No. 6030203C dated 29 October 1991, (sic) as
evidence of full payment of the agreed purchase At bottom, this disposition is nothing more than restoring
price.." An official receipt (O.R.) is doubtless the best the petitioner Republic, and eventually the BCDA, to what
evidence to prove payment. While it kept referring to O.R. rightfully belongs to it in law and in fact. There is nothing
No. 6030203 as its evidence of the required payment,59 it unjust to this approach.
failed to present and offer the receipt in evidence. A
Certification under date September 15, 1993 of the OIC With the foregoing disquisitions, the petition for contempt
Cash Division, LMB, states that "OR # 6030203 in the in G.R. No. 173408 need not detain us long. As it were,
amount of P11,977,000.00 supposedly paid by [SHAI] is the question raised by the petitioners therein respecting
not among the series of [ORs] issued at any time by the the ownership of the JUSMAG area and, accordingly, of
National Printing Office to the Cashier, LMB, Central the right of the petitioning retired military officers to remain
in the housing units each may be occupying is now moot
Law on NatRes (23-49) 100
and academic. However, contempt petitioners expressed situated at Linga, Pila, Laguna, with an area of 8.1263
revulsion over the efforts of the military establishment, hectares, reflected in survey plan Psu-116971, which was
particularly the AFP Chief of Staff, to oust them from their amended after the land was divided into two parcels,
respective dwellings, if that really be the case, even before namely, Lot 1 with an area of 5.2476 hectares and Lot 2
G.R. No. 156951 could be resolved, is understandable as with an area of 2.8421 hectares, reflected in survey plan
it is justified. We thus end this ponencia with a reminder to Psu-226971, amd. 2.
all and sundry that might is not always right; that ours is
still a government of laws and not of men, be they in the The Republic opposed the application on the stereo-typed
civilian or military sector. Accordingly, the Court will not ground that applicants and their predecessors have not
treat lightly any attempt to trifle, intended or otherwise, been in possession of the land openly, continuously,
with its processes and proceedings. A becoming respect publicly and adversely under a bona fide claim of
to the majesty of the law and the prerogatives of the Court ownership since July 26, 1894 and the land has not
is a must for the orderly administration of justice to ceased to be a part of the public domain. It appears that
triumph. barrio folk also opposed the application. (LRC Case No.
189. G.L.R.O. Rec. No. 4922 of the Court of First Instance
WHEREFORE, the petition in G.R. No. 156951 is of Laguna).
GRANTED and the appealed CA Decision is REVERSED
and SET ASIDE. Accordingly, the Deed of Sale dated By virtue of a final judgment in said case, promulgated
October 30, 1991 (Exh. "A"/"1") purportedly executed in January 16, 1956, supplemented by orders issued on
favor of private respondent SHAI and TCT No. 15084 March 21, 1956 and August 13, 1956, defendants were
(Exh. "B"/"2") of the Registry of Deeds of Rizal issued on declared owners of Lot 1 and the remaining portion, or Lot
the basis of such deed are declared VOID. The Register of 2, was declared public land. Decree No. N-51479 was
Deeds of Pasig or Taguig, as the case may be, is hereby entered and Original Certificate of Title No. 0- 40 1, dated
ordered to CANCEL TCT No. 15084 in the name of SHAI October 18, 1956, was issued in the names of defendants.
and the area covered thereby is DECLARED part of the
Fort Bonifacio Military Reservation, unless the same has, In August, 1966, Civil Case No. 52 of the Municipal Court
in the interim, been duly excluded by law or proclamation of Pila, Laguna, was filed by defendants to evict the barrio
from such reservation. Private respondent SHAI, its folk occupying portions of Lot 1. On August 8, 1968,
members, representatives and/or their assigns shall judgment was rendered in the eviction case ordering the
vacate the subject parcels of land immediately upon the defendants therein to return possession of the premises to
finality of this decision, subject to the provisions of herein defendants, as plaintiffs therein. The defendants
Republic Act No. 7227, otherwise known as the Bases therein did not appeal.
Conversion and Development Act.
The foregoing anterior proceedings triggered the filing of
Cost against the private respondent SHAI. the instant case. On October 6, 1970, as prayed for in the
complaint, a writ of preliminary injunction was issued
Having said our piece in G.R. No. 173408, we need not enjoining the Provincial Sheriff of Laguna or his deputies
speak any further thereon other than to deny as we hereby from enforcing the writ of execution issued in Civil Case
similarly DENY the same. SO ORDERED. No. 52, and the defendants from selling, mortgaging,
disposing or otherwise entering into any transaction
REPUBLIC OF THE PHILIPPINES, represented by the affecting the area.
DIRECTOR OF LANDS, petitioner,vs. MELITONA
This case was set for pre-trial on July 6, 1971. Despite
ALAGAD, SPOUSES CARMEN ALAGAD AND
notice of the pre-trial, Atty. Alejandro A. Ponferada,
ESPIRIDION KOLIMLIM, JUSTO ALAGAD, CARLOS
Special Attorney, Bureau of Lands, representing plaintiff
ALAGAD, SPOUSES LIBRADA ALAGAD AND
Republic, did not appear. On July 16, 1971, the court a
EMERSON ABANO, DEMETRIO ALAGAD, ANTONIO
quo dismissed the complaint. The Republic filed a motion
ALAGAD, REGISTER OF DEEDS OF LAGUNA, and the
for reconsideration, was set for hearing, and finally denied
INTERMEDIATE APPELLATE COURT (Fourth Civil
by the court a quo, hence, this appeal.
Cases Division), respondents.; G.R. No. L-66807
January 26, 1989; SARMIENTO, J.:
Plaintiff filed its record on appeal on March 13, 1972. It
appears that the appeal was dismissed by this Court for
The Republic appeals from the decision of the Court of
failure to show in the record on appeal that the appeal was
Appeals 1 affirming two orders of the defunct Court of First
perfected on time. Plaintiff went to the Supreme Court on a
Instance of Laguna 2 dismissing its petition for "annulment
petition for review on the action of this Court. On
of title and reversion. 3 The facts appear in the decision
November 19, 1982, the Supreme Court set aside the
appealed from:
dismissal resolution of this Court and ordered Us to
On or about October 11, 1951, defendants filed an reinstate and give due course to plaintiffs appeal.
application for registration of their title over a parcel of land
Law on NatRes (23-49) 101
In commencing proceedings below, the Republic claims well-established that the State cannot be bound by, or
that the decree and title [rendered and issued in LRC estopped from, the mistakes or negligent acts of its official
Case No. 189, G.L.R.O. Rec. No. L-4922] insofar as the or agents, 7 much more, non-suited as a result thereof.
1.42 hectare northwestern portion on end of Lot 1, Psu-
116971, Amd. 2, is concerned, are void ab initio, 5 for the This is so because:
following reasons:
... [T]he state as a persona in law is the judicial entity,
(a) That said l.42 hectare northwestern portion or end of which is the source of any asserted right to ownership in
Lot l, Psu-116971, Amd. 2, like the adjoining Lot 2 of the land under the basic doctrine embodied in the 1935
same survey plan containing 2.8421 hectares, had since Constitution as well as the present charter. It is charged
time immemorial, been foreshore land reached and moreover with the conservation of such patrimony. There
covered by the waters of the Laguna de Bay (Republic vs. is need therefore of the most rigorous scrutiny before
Ayala y Cia, L-20950, May 31, 1965; Antonio Dizon, et al., private claims to portions thereof are judicially accorded
vs. Juan de G. Rodriguez, et al., L-20355- 56, April 30, recognition, especially so where the matter is sought to be
1965); raked up anew after almost fifty years. Such primordial
consideration, not the apparent carelessness, much less
(b) That moreover said 1.42 hectare portion is actually the acquiescense of public officials, is the controlling norm
now the site of Barrio Aplaya, formerly a sitio of Linga, ...
Pila, Laguna, having been occupied by the barrio people
since the American occupation of the country in the early The cases of Ramos v. Centra l Bank of the Philippines 9
1900's where they established their houses; and Nilo v. Romero, 10 cited by the Court of Appeals in
support of its decision, are not applicable. In Ramos, we
(c) That the barrio people of Aplaya thru the years since applied estoppel upon finding of bad faith on the part of
the early 1900's have filled up and elevated the land to its the State (the Central Bank) in deliberately reneging on its
present condition of being some feet above the level of the promises. In Nilo, we denied efforts to impugn the
adjoining Lot 2 of plan Psu-116971 and the rest of Lot 1 of jurisdiction of the court on the ground that the defendant
the same survey plan so much so that this barrio site of had been "erroneously' represented in the complaint by
Aplaya where there are now sixty-eight (68) houses the City Attorney when it should have been the City Mayor,
occupied by more than one hundred (100) families is no on a holding that the City Attorney, in any event, could
longer reached and covered by the waters of the Laguna have ably defended the City (Davao City). In both cases, it
de Bay; and is seen that the acts that gave rise to estoppel were
voluntary and intentional in character, in which cases, it
(d) That were it not for the fillings made by the barrio could not be said that the Government had been
people, the land in question would not have been fit for prejudiced by some negligent act or omission.
human habitation, so much so that defendants and their
predecessors-in-interest could not have acquired an There is no merit either, in claims that res judicata is an
imperfect title to the property which could be judicially impediment to reversion of property. In Republic v. Court
confirmed in a registration case, as in fact said defendants of Appeals, 11 this Court stated:
and their predecessors-in-interest have never been in
actual possession of the land in question, the actual ... [a] certificate of title may be ordered cancelled (Republic
occupants thereof being the barrio people of Aplaya; 6 v Animas, et al., . supra), and the cancellation may be
pursued through an ordinary action therefor. This action
In sustaining the trial court, the Court of Appeals held that cannot be barred by the prior judgment of the land
under Section 20, of Rule 20, of the Rules of Court, registration court, since the said court had no jurisdiction
dismissal was proper upon failure of the Republic to over the subject matter. And if there was no such
appear for pre-trial. It likewise ruled that the judgment, jurisdiction, then the principle of res judicata does not
dated January 16, 1956, in the said LRC No. 189 has long apply. For it is a well-settled rule that for a prior judgment
become final, titles to the properties had been issued (in to constitute a bar to a subsequent case, the following
favor of the private respondents), and that res judicata, requisites must concur; (1) it must be a final judgment; (2)
consequently, was a bar. it must have been rendered by a court having jurisdiction
over the subject matter and over the parties; (3) it must be
In its petition, the Republic assails the decision insofar as a judgment on the merits; and (4) there must be, between
it sustained the lower court: (1) in dismissing the petition the first and second actions, identity of parties, identity of
for failure of the Republic to appear for pre-trial; and (2) in subject matter and identity of cause of action (Municipality
holding that res judicata is an obstacle to the suit. of Daet vs. CA, 93 SCRA 503; Mendoza vs. Arrieta, et al.,
91 SCRA 113)... 12
I.
In the case at bar, if the parcel registered in the names of
With respect to the first question, we hold that the Court of the private respondents were foreshore land, the land
Appeals has been guilty of grave abuse of discretion. It is registration court could not have validly awarded title
Law on NatRes (23-49) 102
thereto. It would have been without the authority to do so. (9) The waste waters of fountains, sewers and public
The fact that the Bureau of Lands had failed to appeal establishments. 20
from the decree of registration could not have validated
the court's decision, rendered without jurisdiction. So also is it ordained by the Spanish Law of Waters of
August 3, 1866:
II.
Art. 44. Natural ponds and lakes existing upon public lands
"Property, according to the Civil Code, is either of public and fed by public waters, belong to the public domain.
dominion or of private ownership ." 13 Property is of public
dominion if it is: Lakes, ponds, and pools existing upon the lands of private
individuals, or the State or provinces, belong to the
(1) ... intended for public use, such as roads, canals, respective owners of such lands, and those situated upon
rivers, torrents, ports and bridges constructed by the State, lands of communal use belong to their respective pueblos.
banks, shores, roadsteads and others of similar character;
14 or if it: Assuming, therefore, for purposes of this petition, that the
lands subject of the Republic's reversion efforts are
(2) . . . belong[s] to the State, without being for public use, foreshore in nature, the Republic has legitimate reason to
and are intended for some public service or for the demand reconveyance. In that case, res judicata or
development of the national wealth. 15 estoppel is no defense.

All other property of the State, it is provided further, which Of course, whether or not the properties in question are,
is not of the character mentioned in ... article [4201, is indeed, foreshore lands is the core of controversy.
patrimonial property, 16 meaning to say, property 'open to According to the trial court, the aforementioned parcel of
disposition 17 by the Government, or otherwise, property land is a portion of the public domain belonging to the
pertaining to the national domain, or public lands. 18 Republic of the Philippines, 23 and hence, available
Property of the public dominion, on the other hand, refers disposition and registration. As we have pointed out, the
to things held by the State by regalian right. They are Government holds otherwise, and that as foreshore laud, it
things res publicae in nature and hence, incapable of is not registerable.
private appropriation. Thus, under the present
Constitution, [w]ith the exception of agricultural lands, all The question, so it follows, is one of fact: Is the parcel
other natural resources shall not be alienated.' 19 foreshore or is it part and parcel of the public domain?

Specifically: Laguna de Bay has long been recognized as a lake .


Thus:
ART. 502. The following are of public dominion:
Laguna de Bay is a body of water formed in depressions
(1) Rivers and their natural beds; of the earth; it contains fresh water coming from rivers and
brooks or springs, and is connected with Manila Bay by
(2) Continuous or intermittent waters of springs and brooks the Pasig River. According to the definition just quoted,
running in their natural beds and the beds themselves; Laguna de Bay is a lake.

(3) Waters rising continuously or intermittently on lands of And, "[i]nasmuch as Laguna de Bay is a lake, so Colegio
public dominion; de San Jose further tells us, "we must resort to the legal
provisions governing the ownership and use of lakes and
(4) Lakes and lagoons formed by Nature on public lands, their beds and shores, in order to determine the character
and their beds; and ownership of the parcels of land in question. 26 The
recourse to legal provisions is necessary, for under Article
(5) Rain waters running through ravines or sand beds,
74 of the Law of Waters, [T]he natural bed or basin of
which are also of public dominion;
lakes ... is the ground covered by their waters when at
(6) Subterranean waters on public lands; their highest ordinary depth. 27 and in which case, it forms
part of the national dominion. When Laguna de Bay's
(7) Waters found within the zone of operation of public waters are at their highest ordinary depth has been
works, even if constructed by a contractor; defined as:

(8) Waters rising continuously or intermittently on lands ... the highest depth of the waters of Laguna de Bay during
belonging to private persons, to the State, to a province, or the dry season, such depth being the regular, common,
to a city or municipality from the moment they leave such natural, which occurs always or most of the time during the
lands; year . . . 28

Otherwise, where the rise in water level is due to the


extraordinary action of nature, rainfall for instance, the
Law on NatRes (23-49) 103
portions inundated thereby are not considered part of the personally visited and examined the land applied for
bed or basin of the body of water in question. It cannot recommended that the application be given due course.
therefore be said to be foreshore land but land outside of
the public dominion, and land capable of registration as District Land Officer Norberto Bernas thereafter submitted
private property. to the Director of Lands a report, 4 together with relevant
documents including the preliminary investigation report.
A foreshore land, on the other hand, has been defined as The pertinent portion of Bernas report reads:
follows:
. . . I personally visited the area applied for by the herein
. . . that part of (the land) which is between high and low applicant and found that the same is actually occupied and
water and left dry by the flux and reflux of the tides... used by them as a sanctuary of their marine equipment
which they are using in their construction work of the Iloilo
The strip of land that lies between the high and low water Port. The applicant has also introduced some facilities on
marks and that is alternatively wet and dry according to the area applied for in the repair and maintenance of said
the flow of the tide. equipment. A portion of the land applied for has
already been filled up by the applicantas they are in
If the submergence, however, of the land is due to need of a land area for the repair and maintenance of their
precipitation, it does not become foreshore, despite its equipment and in the loading and unloading of materials
proximity to the waters. that they use in the construction of the Iloilo City Port.

The case, then, has to be decided alongside these x x x x 5 (Emphasis and underscoring supplied)
principles and regretfully, the Court cannot make a ruling,
in the first place, because it is not a trier of facts, and in Petitioner Siain Enterprises Inc. (SIAIN), who purchased
the second, it is in possession of no evidence to assist it in from the DBP the properties previously owned by
arriving at a conclusive disposition 31 We therefore WESVICO including Lot 3309, 6 filed on September 29,
remand the case to the court a quo to determine whether 1986 a foreshore lease application 7 over the foreshore
or not the property subject of controversy is foreshore. We, land adjacent to the properties it bought from DBP.
consequently, reverse both the Court of Appeals and the
trial court and reinstate the Republic's complaint. Upon learning that 130 linear meters of the foreshore land
subject of F.F. Cruzs foreshore lease application
WHEREFORE, this case is hereby REMANDED to the trial overlapped that covered by its foreshore lease application,
court for further proceedings. SIAIN filed on January 9, 1987 a protest 8 alleging that it
being the owner of the property adjoining the overlapping
SIAIN ENTERPRISES, INC.,Petitioner, area, it should be given preference in its lease.
vs. F.F. CRUZ & CO., INC.,Respondent.; G.R. No.
146616 August 31, 2006; CARPIO MORALES, J.: On March 6, 1987, the Sangguniang Panglungsod of Iloilo
City, by Resolution No. 174, 9 approved the
Western Visayas Industrial Corporation (WESVICO) filed recommendation of its Committee on Finance that "for the
on September 18, 1973 a foreshore lease application over mutual interest" of F.F. Cruz and SIAIN, SIAIN would get
the foreshore land adjacent to certain lots registered in its 70 linear meters and F.F. Cruz would get 60 linear meters
name, located in Loboc, Lapuz, La Paz, Iloilo City, of the disputed area, in light of its finding that, among other
including Lot 3309. It eventually withdrew the application things, both SIAIN and F.F. Cruz would "contribute
and filed on March 1976 a petition for registration over the substantially to the economic growth of the City of Iloilo."
same foreshore land with the then Court of First Instance
of Iloilo. The case was, however, archived as WESVICOs Concurring with the Sangguniang Panglungsod, the Land
representative could no longer be contacted. Management Bureau (LMB) through its Director, by Order
10 of July 15, 1989, dismissed SIAINs protest in this wise:

It appears that WESVICO ceased to hold operations and


its properties including Lot 3309 were foreclosed by the . . . While it cannot be denied that protestant is now the
Development Bank of the Philippines (DBP) which later registered owner of the property adjoining the foreshore in
consolidated its ownership thereon. 1 question, the disputed foreshore cannot be considered to
have been built or formed by means of accretion but is a
On July 7, 1983, F.F. Cruz & Co. (F.F. Cruz) filed with the reclaimed land made by respondent F.F. Cruz and
Bureau of Lands, Iloilo City, District Land Office VI-1 a Company for the purpose of utilizing the same in the
foreshore lease application 2 over a foreshore land, a loading and unloading of their equipment and materials
portion of which is adjacent to Lot 3309. The application and for the repair and maintenance of said equipment
was docketed as FLA (VI-1) 176. which respondents use in the reclamation of the Iloilo City
Port. This is supported by the findings of the District Land
In the preliminary investigation report 3 on F.F. Cruz FLA Officer Norberto Bernas who, in his letter dated February
(VI-1) 176, Senior Special Investigator Ramon Torre who 18, 1984 to this Office, reported that he personally visited
Law on NatRes (23-49) 104
the foreshore in question and found that the same is now had allegedly filled up the area undeniably shows
actually occupied and used by the respondent CRUZs admission that it is a foreshore and not
company as a sanctuaryof its marine equipment which it something else.
is using in its construction work of the Iloilo City Port and
that a portion of the land applied for has already been The assumption that the contested area is a reclaimed
filled up by the applicant to be utilized in the repair and land runs smack against the provision of Article 5 of the
maintenance of its equipment and in the loading and Spanish Law on Waters of August 3, 1866 stating that:
unloading of materials it uses in the construction of the
Iloilo City Port. It is therefore clear that the foreshore in "Lands reclaimed from the sea in consequence of works
question is neither an accretion nor an accessory to constructed by the State, or by provinces, pueblos or
protestants property. While protestant SEI appears to be private persons, with proper permission,shall become the
owner of the property adjacent to the disputed foreshore, it property of the party constructing such works, unless
cannot be considered as a riparian owner within the otherwise provided by the terms of the grant of authority."
contemplation of the aforementioned law. 11 (Emphasis
We cannot find in the records anything to show that a
and underscoring supplied)
"permission" was ever sought by or granted to, CRUZ for
Accordingly, the LMB disposed: the alleged reclamation of the land in question.

WHEREFORE, it is ordered that the protest of SIAIN xxxx


Enterprises, Inc. be, as it hereby it is, dismissed and this
It is by reason of the Director of Lands erroneous
case, dropped from the records. Both Foreshore Lease
classification of the contested area as "reclaimed"
Application Nos. (VI-5) 220 and (VI-1) 176 of SIAIN
that he awarded 60 linear meters thereof to CRUZ.
Enterprises, Inc. and F.F. Cruz and Co., Inc. respectively,
However, as heretofore discussed, the said area in
shall be amended in such a way that SIAINs application
question is clearly a natural foreshore and SIAIN is
shall cover SEVENTY (70) linear meters of the disputed
correct in claiming it to be so. Hence, the law that
foreshore adjoining Lot 3309 while F.F. Cruzs application
applies in this case is Section 32 of Lands Administrative
shall cover SIXTY (60) linear meters thereof. Accordingly,
Order No. 7-1 which was issued by the Secretary of the
both applications shall be give due course in accordance
then Department of Agriculture and Natural Resources . . .
with the provisions of the Public Land Law, otherwise
known as Commonwealth Act No. 141, as amended. 12 xxxx
(Underscoring supplied)
It is an undisputed fact that SIAIN is the registered owner
SIAIN appealed to the Secretary of the Department of of the land adjoining the foreshore area in controversy.
Environment and Natural Resources (DENR), arguing that Hence SIAIN is the riparian/littoral owner insofar as the
the LMB: contested foreshore area is concerned and should enjoy
the preferential right to lease the entire one hundred thirty
1. . . . made [a] false assumption of fact when it
(130) linear meters of said area adjoining its property,
considered the foreshore area under . . . controversy as
which includes the sixty (60) linear meters thereof
reclaimed land;
awarded to CRUZ in the questioned Order.
2. . . . committed a grave error in not considering the
x x x x 15 (Emphasis supplied; underscoring partly in the
preferential right of the riparian owner/littoral owner, . . . to
original and partly supplied)
apply for a lease over the foreshore under controversy;
[and] The DENR Acting Secretary thus ordered that the
application of F.F. Cruz be amended to exclude the
3. . . . erred in awarding sixty (60) linear meters of the
disputed foreshore area adjacent to Lot 3309 and that
foreshore under controversy to [F.F. Cruz]. 13
SIANs application be given due course.
By Decision 14 of May 6, 1997, then DENR Acting
F.F. Cruz appealed to the Office of the President,
Secretary Antonio G.M. La Via set aside the LMB Order,
contending that the DENR Acting Secretary acted with
the pertinent portions of which decision read:
grave abuse of discretion:
It is blatant error to consider the contested area as
I. IN DISREGARDING THE FINDINGS OF THE
reclaimed land as it has no basis in fact, in law and
DIRECTOR OF LANDS MANAGEMENT BUREAU THAT
jurisprudence.
THE CONTROVERTED AREA IS A RECLAIMED LAND
The area in question is unquestionably a natural UNDERTAKEN BY APPELLANT F.F. CRUZ . . .
foreshore for which various applicants prior to the
II. IN RULING THAT [SIAIN] HAS A PREFERENTIAL
herein parties have applied. CRUZs F.L.A. No. (VI-1)
RIGHT OVER THE PROPERTY IN DISPUTE; [and]
176 itself which was filed on July 7, 1983, long after it
Law on NatRes (23-49) 105
III. IN ISSUING THE SUBJECT DECISION Not being the product of accretion, the disputed strip of
CONSIDERING THAT HE IS NOT EMPOWERED BY foreshore land cannot be the proper subject of a riparian
LAW OR RULE TO ISSUE THE SAME. 16 or littoral claim.

By Decision 17 of March 12, 1999, the Office of the xxxx


President, through then Executive Secretary Ronaldo B.
Zamora, reversed the decision of the DENR Acting The actuality of the DENR not formally granting CRUZ a
Secretary and reinstated that of the LMB in this wise: permit to undertake reclamation works on the disputed
area can be conceded. But in the light of the Bernas
Records reveal that WESVICO, who may be considered report, . . . there can be no quibbling that CRUZ occupied
as the real riparian owner, had previously availed itself and raised, thru filling, the area to its present level, with
of the preferential right to apply for the foreshore area the implicit consent, if not approval, of lands authorities.
adjacent to its property. However, it withdrew its That consent and/or approval have been given may be
application, and instead sought the titling of said deduced from the fact that the Bureau of Lands required
property via a petition for registration filed with the the payment of, and received from appellant, the amount
court, which eventually archived the case for of P40,032.00 as occupation fee. Any suggestion that
petitioners lack of interest. In net effect, WESVICOs CRUZ, after paying the occupational fee, merely planted
preferential right adverted to, albeit initially pursued, itself on the disputed area without as much as dredging
was thereafter abandoned due to its voluntary and filling the same is unacceptable. In a very real sense,
withdrawal of the corresponding application and its therefore, the reclamation work undertaken by CRUZ was
erroneous resort to some other mode of acquisition, with the proper permission, or at least the acquiescence of
i.e., the filing of a petition for registration. Consequent the Bureau of Lands, the agency which, following Insular
to such abandonment, it may be said that WESVICO Government v. Aldecoa (19 Phil. 505), is empowered to
had already waived its preferential right over the grant such permit in behalf of the DENR Secretary. 18
controverted area at the time SIAIN purchased the (Emphasis and underscoring supplied)
adjacent property. As vendee, SIAIN was subrogated
not only to the rights and actions of its predecessor- In its petition for review before the Court of Appeals, SIAIN
in-interest, WESVICO, but also to the absence/lack of raised the issues of 1) whether the disputed area is
those. reclaimed land or foreshore land and if found to be
foreshore land, 2) whether SIAIN has preferential right to
Also decidedly going for CRUZ is the fact that it lease the same. 19
applied for the disputed area, occupied the same and
introduced improvements thereon long before SIAIN By Decision of July 3, 2000, 20 the appellate court
filed its own lease application. Subject to certain dismissed SIAINs petition, ruling that there is no
exceptions, it is axiomatic in public land grant that he justification to digress from the findings and conclusions of
who is first in time is preferred or stronger in law the Office of the President and the LMB and that
Priore in tempore, potior jure. administrative matters within the executive jurisdiction can
only be set aside on proof of gross abuse of discretion,
It may be, as stated by the DENR, that the contested area fraud or error of law.
abuts upon the titled property of SIAIN, a circumstance
which ordinarily would accord that firm a preferential right Hence, the present petition for review filed by SIAIN.
to lease the property in question, the rule being that a
riparian/littoral owner enjoys preference over the abutting SIAIN contends that the evidence overwhelmingly proves
foreshore lands formed by accretion or alluvial deposits. that the disputed area is foreshore land and not reclaimed
land as found by the Office of the President. It invites
xxxx attention to F.F. Cruzs own declaration in its foreshore
lease application that the disputed area is a "parcel of
. . . The principle thus enunciated in Santulanproperly foreshore land." To SIAIN, this declaration is equivalent to
applies where the adjoining lot is a natural foreshore, a judicial admission which does not require proof and is
meaning that the foreshore was formed by what may conclusive as to it.
rightfully be considered as accretion, or the settling down,
by natural causes, of soil, earth and other deposits. But Further, SIAIN argues that the records reveal that the only
such is not what it obtains in this case, contrary to the bare evidence relied upon by the Office of the President is the
assertion of the DENR Acting Secretary that the "area in Bernas report which speaks of a portion allegedly filled-up
question is unquestionably a natural foreshore." . . . by F.F. Cruz, the identity, location and size of which were
never established; and that there is no evidence to prove
xxxx that the filled-up portion is one and the same as the
disputed area, but that even assuming that it is, F.F. Cruz
cannot have a better right over it as the reclamation was
Law on NatRes (23-49) 106
made without the necessary permit, hence, it cannot be washed by the waters of the sea are not necessary for
allowed to benefit from its own wrongdoing. purposes of public utility, or for the establishment of
special industries, or for the coast guard service," shall be
Furthermore, SIAIN contends that there can be no waiver declared by the Government "to be the property of the
of preferential right over the disputed property, no advice owners of the estates adjacent thereto and as increment
from the Director of Lands having been communicated to thereof."
WESVICO, DBP or SIAIN of their preferential right to lease
the adjacent foreshore land, and therefore, the 60 days In other words, article 4 recognizes the preferential
within which they are supposed to apply 21 has not begun right of the littoral (riparian according to paragraph 32)
to run. to the foreshore land formed by accretions or alluvial
deposits due to the action of the sea.
The key to the present controversy lies in the classification
of the disputed area. The reason for the preferential right is the same as the
justification for giving accretions to the riparian owner for
The DENR Secretary found that the disputed area is a the diminutions which his land suffers by reason of the
"natural foreshore," hence, it concluded that SIAIN, being destructive force of the waters. So, in the case of littoral
a littoral owner (owner of land bordering the sea or lake or lands, he who loses by the encroachments of the sea
other tidal waters 22), has preferential right to lease it as should gain by its recession. 25 (Emphasis and
provided in paragraph 32 of Lands Administrative Order underscoring supplied)
No. 7-1 dated April 30, 1936 which reads:
Furthermore, as reflected above, the Office of the
32. Preference of Riparian Owner. The owner of the President, finding that F.F. Cruzs occupation and
property adjoining foreshore lands or lands covered introduction of improvements on the contested area long
with water bordering upon shores or banks of before SIAIN filed its lease application, held that "it is
navigable lakes or rivers, shall be given preference to axiomatic in public land grant that he who is first in time is
apply for such lands adjoining his property as may not preferred or stronger in law."
be needed for the public service, subject to the laws and
regulations governing lands of this nature, provided that he The petition is impressed with merit.
applies therefore within sixty (60) days from the date he
receives a communication from the Director of Lands That the foreshore area had been reclaimed does not
advising him of his preferential right. 23 (Emphasis remove it from its classification of foreshore area subject
supplied) to the preferential right to lease of the littoral owner.

The DENR Secretary found the LMBs classification of the It bears noting that it was not the reclamation that brought
disputed area as "reclaimed" erroneous for lack of basis in the disputed foreshore area into existence. Such foreshore
fact, law and jurisprudence. area existed even before F.F. Cruz undertook its
reclamation. It was "formed by accretions or alluvial
On the other hand, while the Office of the President deposits due to the action of the sea." Following Santulan,
recognized the preferential right of littoral owner the littoral owner has preferential right to lease the same.
WESVICO, it held that it had waived its preferential right
and SIAIN, as successor-in-interest, was subrogated to Contrary to the ruling of the Office of the President, as
WESVICOs right or lack of it. affirmed by the appellate court, littoral owner WESVICO
cannot be considered to have waived or abandoned its
The Office of the President went on to hold that since the preferential right to lease the disputed area when it
disputed area is already reclaimed land, it cannot be subsequently filed an application for registration thereover.
subject to littoral claim, SIAIN, not being the littoral owner For being a part of the public domain, ownership of the
within the contemplation of the law, citing Santulan v. The area could not be acquired by WESVICO. Its preferential
Executive Secretary 24 which elucidated on the principal right remained, however. Its move to have the contested
reason for giving a riparian or littoral owner preferential land titled in its name, albeit a faux pas, in fact more than
right, thus: proves its interest to utilize it.

Now, then, is there any justification for giving to the littoral As correctly argued by SIAIN, were WESVICOs petition
owner the preferential right to lease the foreshore land for registration which, as stated earlier, was archived by
abutting on his land? the trial court, pursued but eventually denied, WESVICO
would not have been barred from filing anew a foreshore
That rule in paragraph 32 is in consonance with article 4 of lease application. Parenthetically, the petition for
the Spanish Law of Waters of 1866 which provides that, registration of WESVICO was archived not on account of
while lands added to the shores by accretions and alluvial lack of interest but because it ceased operations due to
deposits caused by the action of the sea form part of the financial reasons.
public domain, such lands, when they are no longer
Law on NatRes (23-49) 107
WHEREFORE, the Court of Appeals Decision dated July the lots above enumerated in the table. Three witnesses
3, 2000 is REVERSED and SET ASIDE. were called to the stand, but each time, before any
pertinent testimony could be secured from them, an
The May 6, 1997 Decision of then Acting Secretary objection was made by counsel for the oppositors that the
Antonio G.M. La Via of the Department of Environment proof related to the evidence in chief of the Church, and
and Natural Resources is REINSTATED. SO this was sustained by the court.
ORDERED.
To resolve the facts into their simplest terms, it is evident
THE DIRECTOR OF LANDS, applicant-appelle, vs. that when an admission was made of the royal title, the
THE ROMAN CATHOLIC ARCHBISHOP OF MANILA, Church had shown that it was the legitimate owner of the
objector-appellant. JUSTA DE GUZMAN, ET AL., land to which it refers. The most perfect title could,
objectors-appellees.; G.R. No. L-14869 October however, be lost by abandonments. When, therefore, the
27, 1920; MALCOLM, J.: private oppositors showed possession for the prescriptive
period, they had made their case, and the burden of proof
In 1913, cadastral proceedings were begun to settle the had shifted. To overcome this burden, it was then
title to a considerable tract of land situated in the Province incumbent upon the Church to demonstrate that such
of Rizal. Thirteen of the cadastral lots were claimed by the possession had been interrupted, or that it was merely
municipality of Cainta, Province of Rizal, the Roman possession through the tolerance of the Church.
Catholic Archbishop of Manila, and various private
individuals. The judgment of the trial court adjudicated the This brings us to the specific consideration of assignment
parcels in question to the private claimants. From this of error No. 2 of the appellant to the effect that the court
judgment both the Roman Catholic Archbishop of Manila erred in refusing to admit evidence tendered by this
and the municipality of Cainta appealed, but subsequently claimant and appellant in answer to rival claims. A correct
the appeal of the latter wad dismissed for failure to ruling can most appropriately be arrived at by a
prosecute. The contest has thus narrowed down to one consideration of the nature of cadastral proceedings, with
between the Church as appellant and various individuals reference to the usual rules of trial practice and evidence.
as appellees.
The object of a cadastral petition, as all know, is that the
A ruling as to four of the lots can quickly be made. By title to the various lots embraced in the survey may be
stipulation during the trial, and by admission of counsel for settled and adjudicated. It is in the nature of a proceeding
appellant, Policarpio Buenviaje is conceded to have title to in rem, promoted by the Director of Lands, somewhat, akin
lot 2187. As to lot 2186, the only evidence before us, to a judicial inquiry and investigation leading to a judicial
confirmed by the findings of the trial court, is, that Mamerta decree. In one sense, there is no plaintiff and there is no
Roxas went into possession of the same in 1895; and defendant. In another sense, the Government is the
when counsel for the Church made his offer to present plaintiff and all the claimants are defendants. (Act No.
rebuttal testimony, he failed to mention this lot. Likewise, 2259, sec. 10.) The trial is conducted in the same manner
as to lots 2213 and 2214, the only evidence before us, as ordinary trials and proceedings in the Court of Land
confirmed by the findings of the trial court, is, that Antonio, Registration. (Sec. 11.) As to this court, now abolished, the
Benito, and Gervasio dela Paz went into possession of the Land Registration Act provides that it "shall conform, as
same in 1896; the record states that "ambas partes dan near as may be, to the practice in special proceedings in
por terminadas sus pruebas," while counsel for the Church courts of first instance." (Act No. 496, sec. 2) The Code of
in making his offer of rebuttal testimony again failed to Civil Procedure, which is thus brought into relation with the
include these two lots. Cadastral Act, prescribes the order in which the trial must
proceed. (Secs. 56, 132). The usual rules of practice,
A more difficult situation has arisen with reference to the procedure, and evidence govern registration proceedings.
nine remaining cadastral lots. To understand it, a brief
narration of the course of the proceedings in the trial court Obviously, orderly procedure must be followed if injurious
will have to be made. surprises and annoying delays in the administration of
justice are to be avoided. Evidence cannot be given
The trial as to the land now before us opened with a piecemeal. The strict rule is that the plaintiff must try his
stipulation to the effect that the composition title of the case out when he commences. Nevertheless, a relaxation
Church with the spanish Government included this land. of the rule is permitted in the sound discretion of the court.
The Church presented one witness and rested. The "The proper rule for the exercise of this discretion," it has
private oppositors then called their respective witnesses. been said by an eminent author, "is, that material
Each endeavored to prove title by possession, best testimony should not be excluded because offered by the
understood by the following table: plaintiff after the defendant has rested, although not in
rebuttal, unless it has been kept back by a trick, and for
Counsel for the Church, thereupon, made an offer to the purpose of deceiving the defendant and affecting his
present additional testimony with reference to lots 2176,
2178, 2180, 2182, 2184, 2185, 2190, 2191, and 2192, or
Law on NatRes (23-49) 108
case injuriously." (1 Thompson on Trials, sec. him had not been formally accepted according to Article
346.)1awph!l.net 633 of the Civil Code. Upon appeal to this Court, the
judgment of the trial court was affirmed on the second
These principles find their echo in Philippine remedial law. ground aforementioned (Abellera vs. Balanag G.R. No.
While the general rule is rightly recognized, the Code of 11970, promulgated March 22, 1918, and reported in 37
Civil Procedure authorizes the judge "for special reasons," Phil. 865).
to change the order of the trial, and "for good reason, in
the furtherance of justice," to permit the parties "to offer It appears in that decision of this Court that after the
evidence upon their original case." (Sec. 132.) These perfection of the appeal, Abellera executed a public
exceptions are made stronger when one considers the document formally accepting the donation of the land, and
character of registration proceedings and the fact that presented and deed of acceptance together with proofs of
where so many parties are involved, and action is taken notification of acceptance to the donor, as ground for new
quickly and abruptly, conformity with precise legal rules trial. This Court held that this was not newly-discovered
should not always be expected. Even at the risk of evidence, and that Abellera had not acquired title to the
violating legal formul, an opportunity should be given to hacienda until the execution of the deed of acceptance
parties to submit additional corroborative evidence in and the notification thereof to donor. This Court added:
support of their claims of title, if the ends of justice so
require. (Rodriquez vs. Director of Lands [1915], 31 Phil., So that whether rights he may have to institute and
272; Government of the Philippine Islands vs. Abural maintain a new action of ejectment in reliance upon his
[1919], 39 Phil., 996.) claim that he has acquired title to the hacienda, since the
date of the dismissal of this action, it is clear that the
We believe that the offer of counsel for the Church could present action was properly dismissed on the ground of
property be classified as evidence in denial of an failure of proof of title in the plaintiff at the time when the
affirmative fact; but that even if not technically rebuttal action was instituted and later when judgment of dismissal
evidence, yet in the interest of justice and the was entered by the trial court.
ascertainment of the truth it should be received. Whether
such evidence would be sufficient to overcome the case In July of 1918, or four months after the above-mentioned
which exists in favor of the claimants of the nine lots decision of this Court, petitioner herein brought another
cannot now be determined. action for recovery of the land against the same
defendants in the previous case. The second suit was later
In so far as the judgment relates to lots No. 2186, 2187, dismissed by the Court of First Instance and transferred to
2213, and 2214, it is affirmed, and in so far as it relates to cadastral case No. 5 which included the haciendain
lots Nos. 2176, 2178, 2180, 2182, 2184, 2185, 2190, question that had in the meantime been subdivided into
2191, and 2192, it is reversed, and the record shall be lots. When the cadastral case came up before the Hon.
returned to the lower court for the taking of additional Meynardo M. Farol at Aringay, La Union, in July 1941,
evidence under the offer of counsel for the Church found Fabian B.S. Abellera appeared as claimant while Narciso
on page 83 of the stenographic notes. No finding as to de Guzman and others appeared as adverse claimant.
costs is made in this instance. So ordered. The latter through counsel moved that Abellera's claim
over the lots concerned be dismissed on the grounds of
FABIAN B. S. ABELLERA, petitioner, res judicataand prescription.
vs. MEYNARDO M. FAROL, ET AL., respondents.
BOCOBO, J.:; G.R. No. L-48480 July 30, 1943 A careful examination of the decision of this Court in the
previous case (37 Phil., 865) convinces us that there is no
Whether in a cadastral case, the judge may upon motion res judicata. We merely held that Abellera had not
of adverse claimants order the cancellation of the acquired title to the hacienda until the execution of the
claimant's answer and keep the latter from introducing deed of acceptance and the notification thereof, and we
evidence to prove his ownership because the case is clearly refused to prevent Abellera from instituting a new
barred by a prior judgment, is the legal question at issue in action based upon his assertion that he had acquired title
this case. An order to that effect issued by the Court of to the estate since the dismissal of his original action.
First Instance of La Union, is impugned by Fabian B.S.
Abellera in a petition for a writ of certiorari. The other ground for the motion for dismissal, prescription,
is not involved in the present proceedings.
Abellera, in a previous case concerning the same real
estate involved herein, sued Hermegildo Balanag and The next question is: Did the cadastral court, on the
others who are either the same parties in this case or the ground of res judicata, have any power to entertain the
latter's predecessors in interest, alleging ownership of the motion to dismiss Abellera's claim and bar him from
land. But his complaint was dismissed by the Court of First presenting evidence to prove his ownership of these lots?
Instance on two grounds: (1) prescription in favor of
Rule 132 of the Rules of Court provides:
defendants; and (2) the deed of donation of these lands to
Law on NatRes (23-49) 109
These rules shall not apply to land registration, cadastral and SIMEONA GAYAPANAO-NOVENARIO,
and election cases, naturalization and insolvency respondents. G.R. No. L-68109 July 17, 1991; FERNAN,
proceedings, and other cases not herein provided for, C.J.:p
except by analogy or in a suppletory character and
whenever practicable and convenient. In this Petition for Review on Certiorari, petitioners
Severino Gayapanao and his siblings question the
The Rules of Court may be applied in cadastral cases decision of the then Intermediate Appellate Court (IAC) 1
when two conditions are present: (1) analogy or need to in AC-G.R. No. CV-59589, entitled "Severino Gayapanao,
supplement the cadastral law, and (2) practicability and et al. vs. Simeona Gayapanao-Novenario" upholding the
convenience. validity of the sale of two (2) hectares of the ten-hectare
homestead land by their father in favor of their sister
If the nature and objective of the cadastral scheme are Simeona Gayapanao-Novenario.
kept in view, a motion to dismiss in a cadastral case on the
ground of prior judgment would seem to be out of place. Briefly, the facts of the case as found by the Appellate
The Government initiates a cadastral case, compelling all Court are as follows:
claimants in a municipality to litigate against one another
regarding their respective claims of ownership. By this . . . the 2-hectare land subject of this case is part and
plan, all the private lands in a town are registered in one parcel of a homestead lot registered in the name of
single collective proceeding. Thus, the piece-meal and Constantino Gayapanao under Original Certificate of Title
isolated registration of lands, so inadequate in more ways No. 3625 (Exhibits B and 2, Plaintiffs' Folder of Exhibits, p.
than one, is avoided. The principal aim is to settle as much 2). The homestead application of the late Constantino
as possible all disputes over land and to remove all clouds Gayapanao over the said lot was approved on September
over land titles, as far a practicable, in a community. To 7, 1931 (Exhibit C, Ibid., p. 5) and the final order of the
attain this purpose, the cadastral court should allow all Director of Lands for the issuance of patent was issued on
claimants ample freedom to ventilate whatever right they December 10, 1937, (Exhibit A, Ibid., p. 1). On July 13,
may assert over real estate, permitting them, in keeping 1939, the Homestead Patent Title was issued in the name
with the law of evidence, to offer proofs in support of their of Constantino Gayapanao married to Aurelia Maamo
allegations. To countenance the contrary opinion, by (Exhibit D, Ibid., p. 2). On November 15, 1938, the late
suppressing the presentation of evidence in support of Constantino Gayapanao executed a private deed entitled
claims, would but serve to perpetuate conflicts over land, Kasulatan ng Bilihan in favor of Serafin Novenario and his
for such stifled affirmations of ownership will fester like wife, Simeona Gayapanao over 20,000 square meters of
wounds unskillfully treated. No sufficient leeway having the homestead land (Exhibit 4, Defendants' Folder of
been give all claimants to demonstrate the strength and Exhibits, p. 8).
consistently of their alleged rights, the stability of decrees
of title is jeopardized. Constantino Gayapanao and his wife Aurelia Maamo died
intestate on December 22, 1942 and September 29, 1966,
In Haw Pia vs. Roman A. Cruz(G.R. No. 48506), we respectively (Record on Appeal, p. 70) . . .
declared that the Court of First Instance in a cadastral
proceeding cannot appoint a receiver because its On January 2, 1974, Severino, Teodoro, Roberto,
jurisdiction is special and limited. We declined in that case Salvador, and Lauro, all surnamed Gayapanao,
to apply the new Rules of Court by analogy. Raymunda Gayapanao-Ramos and the heirs of Eleuterio
Gayapanao filed before the then Court of First Instance of
We are, therefore, of the opinion that while in a cadastral Oriental Mindoro Civil Case No. R-317, a Complaint for
case res judicatais available to a claimant in order to Partition and Accounting with Prayer for Appointment of
defeat the alleged rights of another claimant, nevertheless Receiver against their sisters Gloria Gayapanao-Saet and
prior judgment can not set up in a motion to dismiss. Simeona Gayapanao-Novenario (private respondent
herein) who were then occupying the subject homestead
The order appealed from is hereby reversed. Petitioner lot. Both defendants filed their Answer to the complaint.
herein shall in the cadastral proceedings be allowed to
present evidence to prove his claim over the lots in On March 11, 1975, the court a quo rendered a decision 3
question. With costs against the adverse claimants who declaring as null and void Exhibit 4, the contract of sale
are respondents herein. So ordered. between Simeona Gayapanao-Novenario and her father
Constantino Gayapanao for having been executed within
SEVERINO GAYAPANAO, TEODORO GAYAPANAO, the five-year prohibitory period provided under Section 118
LAURO GAYAPANAO, SALVADOR GAYAPANAO, of the Public Land Law, 4 and at the same time declaring
RAYMUNDA GAYAPANAO-RAMOS, HEIRS OF as valid Exhibit 5, the deed of sale executed by Teodoro
ELEUTERIO GAYAPANAO and HEIRS OF ROBERTO Gayapanao in favor of his sister Gloria Gayapanao-Saet
GAYAPANAO, petitioners, vs. THE covering the former's hereditary share in the homestead
HONORABLE INTERMEDIATE APPELLATE COURT lot. Accordingly, the lower court ruled as follows:
Law on NatRes (23-49) 110
WHEREFORE, in view of the above findings, judgment is We rule otherwise. The pertinent portion of Section 118 of
hereby rendered as follows: the Public Land Law provides:

A. Adjudicating unto plaintiffs SEVERINO, ROBERTO, Except in favor of the government or any of its branches,
SALVADOR, LAURO, RAYMUNDA and the heirs of units or institutions land acquired under free patent or
ELEUTERIO, all surnamed Gayapanao and Simeona homestead provisions shall not be subject to encumbrance
Gayapanao-Novenario one-ninth 1/9 each of the intestate or alienation from the date of the approval of the
estate of the deceased Constantino and Aurelia Maamo application and for a term of five years from and after the
covered in and embraced by Original Certificate of Title date of issuance of the patent or grant nor shall they
No. 3625 and the remaining two-ninths (2/9) to Gloria become liable to the satisfaction of any debt contracted
Gayapanao-Saet, she having purchased the one-ninth prior to the expiration of said period, but the improvements
(1/9) hereditary share of plaintiff Teodoro Gayapanao; or crops on the land may be mortgaged or pledged to
qualified persons, associations or corporations.
B. Authorizing the herein parties to agree among
themselves to cause the relocated survey of the entire The provision of law which prohibits the sale or
land in question covered by Original Certificate of Title No. encumbrance of the homestead within five years after the
3625 so that their respective shares may be properly grant is mandatory. From the date of the approval of the
delineated; application and for a term of five (5) years from and after
the date of issuance of the patent or grant, lands acquired
C. Ordering the parties to submit the necessary project of under free patent or homestead provisions cannot be
partition after the relocation survey has been executed not subject to encumbrance or alienation, nor shall they
later than three (3) months after entry of this judgment in become liable to the satisfaction of any debt contracted
order to terminate tills proceeding; prior to the expiration of said period. The only exception
mentioned by the law is the sale or encumbrance in favor
D. Declaring as null and void document marked Exhibit of the government or any of its branches, units or
"4", the same having been executed one year prior to the institutions.
issuance of the patent, but Exhibit "5" is confirmed and
declared valid. In a number of cases, we have consistently ruled that a
sale of homestead within the five (5) year prohibitive
WITHOUT special findings as to costs. period is void ab initio and the same cannot be ratified nor
can it acquire validity through the passage of time.
IT IS SO ORDERED.
In the case of Arsenal vs. IAC, 10 we said:
Simeona Gayapanao-Novenario moved to reconsider, but
failed. She then sought relief before the IAC which, finding The above provisions of law are clear and explicit. A
the sale in her favor to be perfectly valid, set aside the contract which purports to alienate, transfer, convey or
decision of the lower court and entered a new one encumber any homestead within the prohibitory period of
dismissing the complaint against her. five (5) years from the date of issuance of the patent is
void from its execution. In a number of cases, this Court
Hence, this appeal filed by Severino Gayapanao, et al.
has held that such provision is mandatory (De los Santos
after their motion for reconsideration was denied 7 by the
vs. Roman Catholic Church of Midsayap, 94 Phil. 405).
respondent appellate court.
It is dangerous precedent to allow the sale of a homestead
In upholding the sale of a portion of the homestead lot by
during the five-year prohibition to anyone, even to the
Constantino Gayapanao to his daughter, herein private
homesteader's own son or daughter. As aptly put by the
respondent Simeona Gayapanao-Novenario, the
petitioners, a clever homesteader who wants to circumvent
respondent court interpreted the prohibition against the
the ban may simply sell the lot to his descendant and the
alienation or encumbrance of the homestead land under
latter after registering the same in his name would sell it to
Section 118 of the Public Land Law as referring to an
a third person. This way, public policy would not be
alienation or encumbrance in favor of a third person
subserved.
outside the family circle of the original homesteader.
Since, according to the appellate court, the conveyance Moreover, the sale to a descendant is not one of the
involved herein was made in favor of Simeona exceptions contemplated by law. Only the government or
Gayapanao-Novenario, who is one of the nine (9) children any of its branches, units or institutions is given the right to
of the original homesteader and who is the "continuity of acquire homestead by purchase at any time and even
the personality of her father for all legal intents and during the five-year prohibitory period. To hold valid the
purposes," such sale is "not in contravention of the sale at bar would be to throw the door open to schemes
avowed policy of the State, which is to preserve and keep and subterfuges which would defeat the law prohibiting the
to the homesteader and his family the land granted to him alienation of homestead within five (5) years from the
by the State." 8 issuance of the patent.
Law on NatRes (23-49) 111
The respondent Court cited the case of Lasud v. Lasud, 11 parcel of homestead land situated at Mabay, Kiamba,
in support of its decision. Said case is not applicable to the Cotabato, containing an area of 23.21 hectares
case at bar, considering that the plaintiff, Sigbe Lasud sold (Homestead Application No. 182259 [E-90722]). The
the inherited homestead to his brother Santay Lasud and transfer was approved by the Secretary of Agriculture and
the latter's wife twenty-one (21) years after the patent was Natural Resources upon recommendation of the Director
issued to his father the homesteader. On the other hand, of Lands and Patent No. V-532 was issued in favor of the
the questioned conveyance in the case at bar was done appellant Isabelo Garcia and his wife. Three years and
within the five year prohibitory period. Furthermore, what three months after the issuance of the homestead patent,
was involved in the Lasud case is the right of therein or on 14 April 1950, for and in consideration of the sum of
plaintiff under Section 119 of the Public Land Law to P11,000, by an instrument executed and acknowledged
redeem the portion sold. In contrast, the case at bar before a notary public the appellant Isabelo Garcia and his
centers on Section 118 of the same law wife sold and conveyed to Domingo Colorado, Raymundo
de Guzman, Inocencio Padama, Ignacio Ramos and Leon
WHEREFORE, the assailed decision of the then de Guzman 19 hectares of the homestead land (Exhibit A).
Intermediate Appellate Court, now Court of Appeals, is The vendees took possession of the part sold to them. The
REVERSED and SET ASIDE. The decision in Civil Case deed of sale was not submitted to the Secretary of
No. R-317 of the then Court of First Instance of Oriental Agriculture and Natural Resources for approval nor
Mindoro is REINSTATED. Costs against private presented to the Registrar of Deeds in and for the province
respondent. SO ORDERED. of Cotabato for registration.

REPUBLIC OF THE PHILIPPINES, plaintiff-appellee, Section 118 of Commonwealth Act No. 141 partly
vs. ISABELO GARCIA, DOMINGO COLORADO, provides:
RAYMUNDO DE GUZMAN, INOCENCIO PADAMA,
IGNACIO RAMOS, LEON DE GUZMAN and THE Except in favor of the Government or any its branches,
REGISTER OF DEEDS OF THE PROVINCE OF units, or institutions, lands acquired under free patent or
COTABATO, defendants-appellants.; G.R. No. L-11597 homestead provision shall not be subject to encumbrance
May 27, 1959; PADILLA, J.: of alienation from the date of the approval of the
application and for a term of five years from and after the
Appeal from a judgment rendered by the Court of First date of issuance of the patent or grant, nor shall they
Instance of Cotabato decreeing the reversion to the State become liable to the satisfaction of any debt contracted
of a homestead land covered by Patent No. V-532 and prior to the expiration of said period, but the improvements
original certificate of title No. V-17 issued by the Registrar or crops on the land may be mortgaged or pledged to
of Deeds in and for the province of Cotabato. The qualified persons, association, or corporations.
dispositive part of the judgment is:
Section 124 of the same Act provides:
IN VIEW THEREFORE, judgment is hereby rendered in
favor of the plaintiff: Any acquisition, conveyance, alienation, transfer, or other
contract made or executed in violation of any of the
(a) ordering defendant Isabela Garcia to return the owner's provisions of section one hundred and eighteen, one
Certificate of Title No. V-17 to the Register of Deeds; hundred and twenty, one hundred and twenty-one, one
hundred and twenty-two, and one hundred and twenty-
(b) ordering the Register of Deeds of Cotabato that three of this Act shall be unlawful and null and void from its
homestead patent No. V-532 be returned to the Bureau of execution and shall produce the effect of annulling and
Lands for cancellation; cancelling the grant, title, patent, or permit originally
issued, recognized or confirmed, actually or presumptively,
(c) ordering the Register of Deeds of Cotabato to cancel and cause the reversion of the property and its
Certificate of Title No. V-17; and improvements to the State.

(d) declaring hereby reverted to the public domain as As the sale of the 19 hectares of the homestead land was
belonging to the Republic of the Philippines the land made within the prohibitive period of five years three
covered by said Certificate of Title No. V-17, with costs years and three months after the issuance of the
against the defendants. homestead patent is null and void,1 and is a cause for
reversion of the homestead to the State.
Only questions of law are raised.
Appellants' defense set up in the court below was that the
From the pleadings, stipulation of facts and documentary document Exhibit A was intended merely as a mortgage
evidence submitted by the parties, the following facts are on the improvements and crops existing on the 19
gathered: Sometime before the last war appellant Isabelo hectares of the homestead land to secure the payment by
Garcia and his wife Tagumpay Dumaguindin acquired by instalment of a loan, only that it was drawn up by mistake
purchase the homestead rights of Lingasa Bapanialag to a as an absolute sale upon the insistence of the vendees. If
Law on NatRes (23-49) 112
it was drawn up as an absolute sale upon the insistence of
the vendees, then there was no mistake committed. The
document Exhibit A is also so clear worded as to preclude
an interpretation other than what the parties had intended
it to be a deed of absolute sale of the 19 hectares of the
homestead land. Moreover, as found and held by the trial
court:

. . . as stated in Annex "A", the improvements on the land


only consist of 50 coconut, 5 mango trees, 1 nangka tree,
bananas and other fruit trees. It is unbelievable that
defendants Domingo Colorado, Raymundo de Guzman,
Inocencio Padama, Ignacio Ramos and Leon de Guzman
would grant a loan of P11,000.00 for such a small security
as those improvements specified above. Then if it is true
that the transaction had between them is only a loan, to be
paid by instalment every year, the defendants failed to
produce any evidence that any instalment has ever been
paid, taking into consideration that already elapsed more
than five (5) years from that time up to the hearing of this
case.

The fact that the appellant Isabelo Garcia moved to the


municipality of Bislig, province of Surigao, and abandoned
his homestead in Kiamba, Cotabato, is proof that he sold
19 hectares of his homestead.

Appellants contended that, under section 50, Act No. 496,


the operative act to convey and affect lands registered
thereunder is the act of registration, that inasmuch as the
deed of sale Exhibit A was never registered there was
actually no conveyance made of the 19 hectares of the
homestead land, and that for that reason there was no
infringement of section 118 of Commonwealth Act No.
141. To constitute a violation of the section just referred to,
it is enough that the homestead be encumbered or
alienated within the prohibitive period of five years; it is not
necessary that the encumbrance or alienation be
registered in the Office of the Register of Deeds. To
uphold the appellants' contention would defeat the very
prohibition established by law, for no party to a prohibited
sale or conveyance would register such an illegal
transaction. Besides, the vendees already, had taken
possession of the part sold to them.

Even if only 19 out of the 23.21 hectares of the homestead


land had been sold or alienated within the prohibitive
period of five years from date of issuance of the patent to
the grantee, such alienation is a sufficient cause for
reversion to the State of the whole grant. In granting a
homestead to an applicant, the law imposes as a condition
that the land should not be encumbered, sold or alienated
within five years from the issuance of the patent. The sale
or alienation of part of the homestead violates that
condition.

The judgment appealed from is affirmed, with costs


against the appellants.

S-ar putea să vă placă și